You are on page 1of 322

Mathematics Magazine, Vol.

82, December 2008


1806. Proposed by C. Pohoata (Romania). Let M be a point on the circumcircle of triangle ABC and lying on the arc BC that does not contain A. Let I be the incenter of ABC , and let E and F be the feet of the perpendiculars from I to lines M B and M C , respectively. Prove that the value of IE + IF AM is independent of the position of M . Solution proposed by G.R.A.20 Problem Solving Group, Roma, Italy. Let , , and be respectively the angles CAB , ABC , BCA and BCM and let a, b, c be the sides CB , AC , AB . Since IB bisects and CM B = (ABM C is cyclic), then IBE = /2 + and sin(/2 + ) . IE = IB sin(/2 + ) = r sin(/2) Similarly IF = IC sin(/2 + ) = r Moreover, CM BM a b c = = = = = 2R sin( ) sin() sin() sin( ) sin( ) and therefore, since ABM C is cyclic, we have that AM = CM Hence c b 2R + MB = (sin( ) sin( ) + sin() sin( )) . a a sin() IE + IF r sin() sin( ) sin( ) = f ( ) AM 2R sin(/2) sin(/2) f () = sin(/2 + ) sin(/2) + sin(/2 + ) sin(/2) . sin( ) sin( ) + sin() sin( ) sin(/2 + ) . sin(/2)

where

Reminding that = , after some tedious computations we veried that f () = f (0) for [0, ] that is (IE + IF )/AM is independent of the position of M .

Mathematics Magazine, Vol. 81, October 2008


1803. Proposed by M.W. Botsko (USA). Let (X, ) be a real inner product space, and let B = {x X : ||x|| 1} be the unit ball in X , where ||x|| = x, x . Let f : B B be a function satisfying ||f (x) f (y )|| ||x y || for all x, y B . Prove that the set of xed points of f is convex. Solution proposed by G.R.A.20 Problem Solving Group, Roma, Italy. We will show that the set of xed points of f is convex in the more general setting that X is a strictly convex normed space, that is x = y , x 1, y 1 implies that
x+ y 2

< 1.

Note that a real inner product space is strictly convex because if x = y , ||x|| 1, ||y || 1 then x+y 2
2

1 1 1 xy = x + y, x + y = ||x||2 + ||y ||2 4 2 2 2

xy 1 2

< 1.

If the set of xed points is empty or it contains just one point there is nothing to prove. Assume that x, y are two dierent xed points of f and let z = x + (1 )y with [0, 1] then x f (z ) = f (x) f (z ) x z (1 ) x y . If f (z ) = z then, since X is strictly convex, x z (1 ) x y implies that x x f (z ) (1 ) x y and

z + f (z ) < (1 ) x y . 2

In a similar way we can show that y z + f (z ) < xy . 2

Hence we get the following contradiction xy x z + f (z ) z + f (z ) + y < (1 ) x y + x y = x y 2 2

and therefore f (z ) = z , that is any convex combination of x and y is a xed point of f. Note that if X is not strictly convex than the statement can be false: let X = R2 with the norm (x1 , x2 ) = max(|x1 |, |x2 |) and consider the non-expanding map f : B B given by f (x1 , x2 ) = (x1 , |x1 |). Then the set of xed points of f is not convex: x = (1, 1) and y = (1, 1) are xed points but (x + y )/2 = (0, 1) is not.

Mathematics Magazine, Vol. 81, June 2008


1797. Proposed by O. Furdui (USA). Let a, b and c be nonnegative real numbers. Find the value of n n2 + kn + a lim . 2 + kn + b n2 + kn + c n n k=1

Solution proposed by G.R.A.20 Problem Solving Group, Roma, Italy. Since k n a + kn

n2 + kn + a = n

1+

1+

n2

then, for k = 1, . . . , n (remember that a is nonnegative) n 1+ k n 1+ a n2 + kn + a n 2n2 1+ k n 1+ a . n2

Hence, we have an upper bound n n2 + kn + a 2 + kn + b n2 + kn + c n k=1 and a lower bound n n2 + kn + a n2 + kn + b n2 + kn + c k=1

1+ 1+
b n2

a 2n2

c n2

1+

1 n

1 1+
k n

k=1

1+ 1+
b 2n2

a n2

1+

c 2n2

1 n

1 1+
k n

k=1

Since both the upper bound and the lower bound, as n goes to innity, tend to
1

1 dx = 2( 2 1) 1+x

this is also the value of our limit.

Mathematics Magazine, Vol. 81, June 2008


1796. Proposed by M. McMullen (USA). A point is selected at random from the region inside of a regular n-gon. What is the probability that the point is closer to the center of the n-gon than it is to n-gon itself? Solution proposed by G.R.A.20 Problem Solving Group, Roma, Italy. Consider the right triangle with vertices: the center of the n-gon, the midpoint of a side, a vertex of that side. We can assume that its cartesian coordinates are respectively: (0, 2), (0, 0), (2 tan(/n), 0). The locus of points which have the same distance from the center and from the side is a parabola whose equation is x2 + (y 2)2 = y 2 , that is x2 y= + 1. 4 The arc of this parabola, which goes from (0, 1) to (a, a2 /4 + 1), separates the right triangle in two parts and the fraction of the area of the triangle that contains the center is the probability that we are looking for: p= 1 2 tan(/n) a 2 2+ a2 +1 4
a

x2 +1 4

dx

a 4 tan(/n)

a2 +1 . 12

a can be found by intersecting the parabola with the line which contains the hypotenuse. Therefore a is the positive solution of the equation a2 a +1=2 4 tan(/n) which yields a = 2 tan(/(2n)). Finally, we nd that p= a 4 tan(/n) a2 +1 12 = 1 3 1 1 4 cos4 (/(2n)) .

Note that, as one can expect, the limit as n goes to innity is 1/4.

Mathematics Magazine, Vol. 81, April 2008


1782. Proposed by H. A. ShahAli, Iran. Let N be a positive integer. Prove that there is a positive integer n such that n2 + 3 is divisible by at least N distinct primes. Solution proposed by G.R.A.20 Problem Solving Group, Roma, Italy. It suces to nd an increasing sequence of positive numbers {nN }N >0 such 2 2 that: n2 N +3 divides nN +1 +3 with quotient QN > 1 and gcd(nN +3, QN ) = 1. 2 Then, by induction, n2 1 + 3 > 3 has at least one prime factor and nN +1 + 3 has at least one more prime factor than n2 N + 3 (given by QN ). Let P (s) = s2 + s + 1 then P (s2 ) = P (s)P (s 1) and for any integer s > 0, since P (s 1) is odd, gcd(2m P (s), P (s 1)) = gcd(2m (P (s) P (s 1)), P (s 1)) = gcd(2m+1 s, P (s 1)) = gcd(s, s2 s + 1) = gcd(s, 1) = 1. Take nN = 2k 2
N 1

2 + 1 for k > 1 then n2 N + 3 = 4P (k


N N 1

N 1

),
N 1

2 n2 ) = 4P (k 2 N +1 + 3 = 4P (k

)P (k 2

N 1

2 1) = (n2 N + 3)P (k

1)

and
2 gcd(n2 N + 3, P (k
N 1

1)) = gcd(4P (k 2

N 1

), P (k 2

N 1

1)) = 1.

Mathematics Magazine, Vol. 81, April 2008


1791. Proposed by Mowaaq Hajja, Jordan. Let ABC be a triangle with circumcenter O, perimeter P , and area K . Prove that if BC [OBC ] 1 = = , P K 3 then ABC is equilateral. Solution proposed by G.R.A.20 Problem Solving Group, Roma, Italy. Let a = BC > 0, b = CA > 0, c = AB > 0, and p = P/2. Since 3[OBC ] = K , then, by Herons formula, 9 R+ a 2 R a 2 a 2 a = p(p a)(p b)(p c) 2

Since R = abc/4K , then 9a4 b2 c2 = 64p2 (p a)2 (p b)2 (p c)2 + 36a4 p(p a)(p b)(p c). Since a = BC = P/3 then p = 3a/2 and a = (b + c)/2. Therefore 4b2 c2 = 9(3a 2b)2 (3a 2c)2 + 3a2 (3a 2b)(3a 2c), and, after expanding and simplifying, we nd 2bc = b2 + c2 that is b = c = a and we have proved that ABC is equilateral.

Mathematics Magazine, Vol. 81, February 2008


1786. Proposed by Marian Tetiva, Romania. Let n 2 be a positive integer and let On = {1, 3, . . . , 2n 1} be the set of odd positive integers less than or equal to 2n 1. a. Prove that if m is a positive integer with 3 m n2 and m = n2 2, then m can be written as a sum of distinct elements from On . b. Prove that n2 2 cannot be written as a sum of distinct elements of On . Solution proposed by G.R.A.20 Problem Solving Group, Roma, Italy. We will prove that an integer m [0, n2 ] can be written as a sum of distinct elements of On (possibly empty sum) i m {2, n2 2}. Since k = n2
k On

then m [0, n2 ] can be written as a sum of distinct elements of On i n2 m has this property. Moreover 2, and therefore n2 2, cannot be written as a sum of distinct elements of On because since 2 > 0 and 2 On we need at least two elements of On . But if S On with |S | 2 then k 1 + 3 > 2.
k On

We will prove the rest of the statement by induction: For n = 2 the property holds trivially. Now we assume that the property holds for n 2, and we prove that it holds for n + 1. If m [0, n2 ] \ {n2 2, 2} then m can be written as a sum of distinct elements from On On+1 . If m [2n + 1, (n + 1)2 ] \ {(n + 1)2 2, 2n + 3} then m (2n + 1) [0, n2 ] \ {n2 2, 2} and therefore m can be written as a sum of distinct elements from On On+1 plus the odd number (2n + 1) On+1 \ On . Now, since n 2 then [0, n2 ]\{n2 2, 2}[2n+1, (n+1)2 ]\{(n+1)2 2, 2n+3} = [0, (n+1)2 ]\{(n+1)2 2, 2} and the property holds also for (n + 1).

The College Mathematics Journal, November 2008


887. Proposed by J.L. Diaz-Barrero (Spain). Evaluate
n n

k 1+ n

n/k3

lim

k=1

Solution proposed by G.R.A.20 Problem Solving Group, Roma, Italy. We rst transform the product in a sum by applying the logarithm 3 n n n/k 3/2 k k k = 1 log log 1 + 1+ n n k=1 n n k=1

Since such sum is actually a Riemann sum, its limit is the following integral
1 1

x
0

3/2

log(1 + x) dx = 2[x

1/2

log(1 +
1

x)]1 0

+2
0

x1/2 dx 1+x

= 2 log 2 + 2

2 dt 2 0 1+t = log 4 + 4[arctan t)]1 0 = log 4 +


n/k3

Therefore
n n

lim

k=1

k 1+ n

= exp( log 4 + ) =

e . 4

The College Mathematics Journal, November 2008


887. Proposed by J.L. Diaz-Barrero (Spain). Evaluate
2n+1

k=1

2 zk 2 + zk

where z1 , z2 , . . . , z2n+1 are the (2n + 1)st roots of unity, and z denotes the complex conjugate of z . Solution proposed by G.R.A.20 Problem Solving Group, Roma, Italy. Let N > 1 and let zk be one of the N -roots of unity. If a > 1 then |zk /a| < 1 and 1 a zk = a + zk 1+ Hence, since 1 N we have that
N N m zk = [N |m] k=1 zk a zk a

zk a2 + 1 1 zk a2 + 1 zk = + = + z k 2 2 a a 1+ a a a a m=0

k=1

a zk = a + zk

k=1 N

zk zk a2 + 1 + a a2 m=0 a

1 = a
2

a2 + 1 1 zk + 2 a a m=0 k=1
m

m N m zk k=1

a +1 1 2 a a m=0

N [N |m]
Nj

a2 + 1 = N a2
2

j =0

1 a
N

a +1 1 = N a2 1 1 a Hence, for N = 2n + 1 and a = 2 we have that


2n+1

aN + aN 2 =N N . a (1)N

k=1

2 zk 10(2n + 1)4n1 = . 2 + zk 2 4n + 1

The College Mathematics Journal, September 2008


885. Proposed by O. Furdui (USA). Evaluate
0 0

log(1 + x2 + y 2 ) dx dy. (1 + x2 )(1 + y 2 )

Solution proposed by G.R.A.20 Problem Solving Group, Roma, Italy. We rst use the polar coordinates
0

I =
0

log(1 + x2 + y 2 ) dx dy (1 + x2 )(1 + y 2 ) log(1 + 2 ) d d (1 + 2 cos2 )(1 + 2 sin2 )


/2 =0

/2 =0

=
=0

=
=0

log(1 + 2 ) (2 + 2 )

1 d + 2 1 + cos2 d.

/2 =0

1 1+ 2 sin2

d d

=
0

log(1 + 2 ) (2 + 2 ) 1 + 2

Let u = 1/ 1 + 2 . Then
1

I = = 4
0

(log u)2 du 1 + u2
t

= 4 lim +
t1

(log u)2
0 1 k=0

(1)k u2k du

= 4
k=0

(1)k
0

u2k (log u)2 du

= 8
k=0

(1)k 3 4 = 8 = . (2k + 1)3 32 4

The College Mathematics Journal, September 2008


883. Proposed by B. Bradie (USA). Evaluate (a)
0 1

log(1 + x) dx and (b) 1 + x2

1 0

arctan x dx 1+x

Solution proposed by G.R.A.20 Problem Solving Group, Roma, Italy. As regards (a), let x = tan s then
1 0

log(1 + x) dx = 1 + x2 =

/4

log(1 + tan s) ds
0 /4

log(1 + tan(/4 s)) ds


0 /4

=
0

log 1 + log 2 4
/4

1 tan s 1 + tan s

ds

log (1 + tan s) ds.


0

Hence (note that the integral (a) is nite)


1 0

log 2 log(1 + x) dx = 2 1+x 8

As regards (b), by integrating by parts, we obtain


1 0

arctan x dx = 1+x

arctan x d (log(1 + x))


0 1

= [arctan x log(1 + = log 2 4


1 0

x)]1 0

log(1 + x) d (arctan x)

log(1 + x) log 2 = . 2 1+x 8

The College Mathematics Journal, September 2008


881. Proposed by K. McIntur (USA). Prove that for every positive integer n
n1

k=0

2k k

=
k=1

2 sin(2k/3) 2n . n+k 3

Solution proposed by G.R.A.20 Problem Solving Group, Roma, Italy. We prove the identity by induction. It holds for n = 1. Assume that it holds for n > 1, then it suces to prove that 2n n
n

=
k=0

2k 2k k k k=0

n1

n+1

=
k=1

k 3 k 3

2n + 2 n+1+k k=1

k 3

2n n+k

=
k=1

2n + 2 2n n+1+k n+k

where (k/3) = 2 sin(2k/3)/ 3 is the Legendre symbol. Note that (k/3) is equal to 0, 1, 1 whether k is equal to 0, 1, 1 mod 3. Since 2n + 2 n+1+k then

2n + 1 2n + 1 + n+k n+1+k

2n 2n 2n +2 + n+k1 n+k n+k+1

k=1

k 3

2n + 2 2n n+1+k n+k

=
k=1

k 3

2n 2n 2n + + n+k1 n+k n+k+1 k1 3 + k 3 + k+1 3 2n n+k

= = because for any integer k k1 3 + k 3

2n + n k=2 2n n

k+1 3

=0

The College Mathematics Journal, March 2008


874. Proposed by M. S. Becker (USA). Let f (x) = 1/(1 + x2 ) and let f (n) (x) denote its nth derivative. Show that

n=0

f (4n) (1) f (2n) (1) + (4n)! (2n)!

=1

Solution proposed by G.R.A.20 Problem Solving Group, Roma, Italy. Lets consider the expansion of the complex rational function f (z ) = 1/(1 + z 2 ) at z0 = 1 f (z ) = an (z 1)n for z D = {|z 1| < |i 1| = 2}
n=0

Let Ak =

n=0

a4n+k for k = 0, 1, 2, 3. Since an = f (n) (1)/n!, we have to prove that f (4n) (1) f (2n) (1) + (4n)! (2n)! = A0 + (A0 + A2 ) = 2A0 + A2 = 1.

n=0

Since 0, 2, 1 + i D then f (0) = A0 A1 + A2 A3 = 1 1 f (2) = A0 + A1 + A2 + A3 = 5 f (1 + i) = A0 + iA1 A2 iA3 = 1 2i 5

and by solving this linear system of equations we nd that A0 = 2 5 A1 = 2 5 A2 = 1 5 A3 = 0

and nally we verify that 2A0 + A2 = 1.

The College Mathematics Journal, March 2008


873. Proposed by O. Furdui (USA). Find (a)
n=1 n1 ln n 2 n1 ln

(1)

and (b)
n=1

(1)

Solution proposed by G.R.A.20 Problem Solving Group, Roma, Italy. The zeta function (s) =
n=1

1 ns

has a unique analytic continuation to C \ {1}. At s = 1 it has a simple pole and its expansion there gives (1)n 1 + n (s 1)n (s) = s 1 n=0 n! where n is the n-th Stieltjes constant (0 = is the Eulero-Mascheronis constant). Therefore around 1 we can use the following approximations 1 + + o(1), s1 1 (s) = 1 + o(1), (s 1)2 2 (s) = + 2 + o(1). (s 1)3 (s) = Moreover, for Re(s) > 0 and s = 1, the following identity holds

(1 21s ) (s) =
n=1

(1)n1 ns

(a) By deriving this identity with respect to s, we obtain

(1 2 Since

1 s

) (s) + 2

1 s

(s) ln 2 =
n=1

(1)n1

ln n . ns

21s = exp((s1) ln 2) = 1(ln 2)(s1)+(ln 2)(s1)2 /2(ln 2)3 (s1)2 /6+o(s1)3 then, by using the above approximations of (s) and its derivatives around s0 = 1, and by taking the limit as s 1, we nd that

(1)n1
n=1

ln2 n (ln 2)2 = log 2 0.15986890. n 2

(b) By deriving the above identity two times with respect to s, we obtain

(1 2

1 s

) (s) + 2 2

1s

(s) ln 2 2

1 s

(ln 2) (s) =
n=1

(1)n1

ln2 n . ns

and, in similar way as in case (a), we nd that

(1)n1
n=1

(ln 2)3 ln2 n = (log 2)2 1 (2 ln 2) 0.06537259. n 3

The College Mathematics Journal, January 2008


870. Proposed by M. K. Azarian (USA). Prove that
1

(1)i
i=0 0

16x3 (1 + x8i ) dx = . (1 + x4 )2i+2

Solution proposed by G.R.A.20 Problem Solving Group, Roma, Italy. Letting f (x) = 1 (1 + x4 )2 and g (x) = x8 (1 + x4 )2

we have that (note that |f (x)| < 1 and |g (x)| < 1 for x (0, 1])
1

(1)i
i=0 0

16x3 (1 + x8i ) dx = (1 + x4 )2i+2 = = = =

1 0 1

16x3 (1 + x4 )2

(1)i (f i (x) + g i (x)) dx


i=0 3

1 16x3 16x dx + dx 4 2 4 2 0 (1 + g (x))(1 + x ) 0 (1 + f (x))(1 + x ) 1 1 4x3 4x3 4 dx + 4 dx 4 2 4 2 8 0 (1 + x ) + 1 0 (1 + x ) + x 4 1 4[arctan(1 + x4 )]1 0 + 4[arctan(1 + 2x )]0 4(arctan 2 + arctan 3) 2 = 4( arctan 1) 2 =

where, in the last equality, we used the known fact that arctan 1 + arctan 2 + arctan 3 = .

The College Mathematics Journal, January 2008


869. Proposed by V. Radulescu (Romania). Prove that there does not exist a positive twice-dierentiable function f dened on [0, +) such that f (x)f (x) 1 for all x 0.

Solution proposed by G.R.A.20 Problem Solving Group, Roma, Italy. We will prove a more general result: if a > 0 then there is no positive twicedierentiable function f dened on [0, +) such that f (x)f (x) a for all x 0. Note that the bound a cant be replaced by 0 because f (x) = 1 + log(1 + x) is a positive twice-dierentiable function dened on [0, +) such that f (x)f (x) = 1 + log(1 + x) 0 for all x 0. (1 + x)2

Proof. By contradiction, we assume that such function f exists. 1) f has no upper bound: if f (x) L for some L > 0 then for x 0 a a f (x) f (x) L and by integrating this inequality two times over the interval [0, x] we obtain that a 2 f (x) f (0) + f (0)x x. 2L Since the RHS goes to as x + and the LHS is positive we have a contradiction. 2) f is positive: if there is x0 0 such that f (x0 ) 0 then, since f is concave (f (x) is negative by the hypothesis), f (x) f (x0 ) + f (x0 )(x x0 ) f (x0 ) for x x0 . which implies that f (x) has max[0,x0 ] f (x) as upper bound, contradicting 1). 3) Final step: by 2) and 1) f is strictly increasing and f (x) + as x +. By hypothesis, for x 0 D[(f (x))2 ] = 2f (x)f (x) 2a f (x) = 2aD[log(f (x))] f (x)

and by integrating this inequality over the interval [0, x] we have (f (x))2 (f (0))2 2a(log f (x) log f (0)), that is (f (x))2 (f (0))2 + 2a log f (0) 2a log f (x) Since the RHS goes to as x + and the LHS is positive we have a contradiction.

The College Mathematics Journal, January 2008


868. Proposed by A. R. Miller (USA). Evaluate the double innite sum
m+n m+n+1 , ( )( ) 2 2 m=0 n=0

where (x) is the Gamma function. Solution proposed by G.R.A.20 Problem Solving Group, Roma, Italy. Since m + n = k for m, n 0 has k + 1 solutions then

S=
m=0 n=0

1 m+n n+1 = ( 2 )( m+2 )

k=0

k+1 = k ( 2 )( k +1 ) 2 2

( k )( k 2 2 k=1

k+1 1 +2 )

where we dropped the rst term because the function has a (simple) pole at 0. Note that we can alter the summation order because the terms are all non-negative. Since the duplication formula holds (z )(z + 1/2) = 212z (2z ) we have that 1 S = 1 = 4 =

k=1

(k + 1)2k1 1 = (k ) (k + 2)2 2 = k! 2 4e = . k!
k 2 k

k=1 k 1

(k + 1)2k1 (k 1)!

k=0

k=1

2 2 + (k 1)!

k=0

2k k!

k=0

Mathematics Magazine, Vol. 80, December 2007


1781. Proposed by Paul Bracken, USA. Let be Eulers constant and for positive integer n dene
n

n =
k=1

1 log n and n = 2n(n ). k

Prove that the sequence {n } is monotonically increasing and bounded above. In addition determine lim n .
n

Solution proposed by G.R.A.20 Problem Solving Group, Roma, Italy. It is well known that
n

Hn =
k=1

1 1 1 = + log n + +o k 2n 12n2

1 n2

therefore n = 2n(n ) = 2n(Hn log n ) = 1 1 +o 6n


n

1 n

and it follows that the sequence {n } is bounded and lim n = 1. Lets prove the last property: the sequence {n } is increasing that is n := (n+1 n )/2 > 0 for n 1. Since n 0 then
N 1

n = lim (n N ) = lim
N

(k k+1 )
k =n

so it suces to show that n > n+1 for n 1. Since n = Hn + 1 [(n + 1) log(n + 1) n log n] then letting f (x) = we have that
+

1 (x + 1) log(x + 1) + 2x log x (x 1) log(x 1) x

n n+1 = f (n + 1) =
n+1

f (x) dx > 0

because lim f (x) = 0 and for x > 1


x+

f (x) =

1 log(x+1)+2 log xlog(x1) = x2

1 1 log 1 + 2 2 x x

>0

(remember that t > log(1 + t) for t (1, 0)).

Mathematics Magazine, Vol. 80, October 2007


1780. Proposed by Yiu Tung Poon, USA. For positive integer n, dene odd(2n ) to be the number of odd digits in the (base-ten) expansion of 2n . Prove that

n=1

odd(2n ) 1 = . n 2 9

Solution proposed by G.R.A.20 Problem Solving Group, Roma, Italy. For any k 1, 1/10k has a unique (base-2) representation (avoid a tail of ones) 1 aj = with aj {0, 1}. k 10 2j j =1 So for any n 1 2n = 10k and
n1

2nj aj + an + with [0, 1)


j =1

2n 10k

mod 2 = an = oddk+1 (2n )

where oddk+1 (n) is 1 if the (k + 1)-th digit of 2n is odd and 0 otherwise. Therefore we proved that for k 1 then

n=1

oddk+1 (2n ) 1 = k. n 2 10

Hence (note that the series has non negative terms and it is convergent)

n=1

odd(2n ) = 2n = =

n=1

1 2n 1 2n

oddk+1 (2n )
k=0

oddk+1 (2n ) oddk+1 (2n ) 2n

n=1 k=1

k=1 n=1

=
k=1

1 1 = . k 10 9

Mathematics Magazine, Vol. 80, June 2007


1771. Proposed by Mowaaq Hajja, Jordan. Let P be a point inside a triangle ABC , and let AA , BB , and CC be the cevians through P . Prove that if A B = A C and BC = CB , then the triangle ABC is isosceles. Solution proposed by G.R.A.20 Problem Solving Group, Roma, Italy. Let x1 = BA , x2 = CA , y1 = CB , y2 = AB , z1 = AC , z2 = BC . They are all positive numbers because the point P is inside the triangle. Since A B = A C then
2 2 2 x2 2 + y1 2x2 y1 cos(BCA) = x1 + z2 2x1 z2 cos(ABC ).

By eliminating the cosines we have


2 x2 2 + y1 2x2 y1

x2 + y 2 z 2 2xy

2 = x2 1 + z2 2x1 z2

x2 + z 2 y 2 2xz

where x = x1 + x2 , y = y1 + y2 , z = z1 + z2 . Then
2 2 2 2 2 2 xyz (x2 2 + y1 x1 z2 ) + x (yx1 z2 zx2 y1 ) + (yx1 z2 + zx2 y1 )(z y ) = 0

We know that z2 = y1 and since by Cevas theorem x1 y1 z1 = x2 y2 z2 then x2 = z1 x1 /y2 . By eliminating z2 and x2 , and by factoring we nd x1 (z y2 )Q = 0 2 1 y2 where
4 3 2 3 2 3 2 2 2 2 Q = y1 y2 + 3y2 y1 + y2 z1 y1 + 2y2 y1 + 2y2 y1 z1 + y2 x1 z1 3 3 3 2 2 2 2 2 2 y1 + x2 +y2 z1 y1 + 3y2 y1 z1 + 2y2 z1 x1 + 2y2 z1 y1 + y2 z1 1 z1 .

Since Q is positive it follows that z1 = y2 . Since z2 = y1 then z = z1 + z2 = y1 + y2 = y and the triangle is isosceles.

Mathematics Magazine, Vol. 80, April 2007


1769. Proposed by Michel Bataille, France. For positive integer n, let
n

Pn (x, y ) =
k=0

2n + 1 nk x (x + y )k . 2k + 1

Find a closed form expression for the coecient of xi y j when Pn is expanded. Solution proposed by G.R.A.20 Problem Solving Group, Roma, Italy. First note that
n

Pn (x, y ) =
k=0

2n + 1 nk x 2k + 1

j =0

k k j j x y = j

j =0 k=j

2n + 1 2k + 1

k nj j x y . j

Now we prove that


n

k =j

2n + 1 2k + 1

k j

= 4nj

2n j j

and therefore for integers i, j [0, n] we have [xi y j ]Pn (x, y ) = Since [z k ](1 + z )2n+1 = then (1 + z )2n+1 (z 2 )j [z ] = (1 z 2 )j +1
2n n

4nj 0

2nj j

if i = n j otherwise 0
k/2 j

2n + 1 k

and [z k ]

(z 2 )j = (1 z 2 )j +1
n

if k is odd if k is even

k=0

2n + 1 2n 2k

k j

=
k =j

2n + 1 2k + 1

k . j

On the other hand (1 + z )2n+1 (z 2 )j [z ] (1 z 2 )j +1


2n

= [z

2nj

(1 + z )2nj z j ] = (1 z )j +1 2n j j

2nj

k=0

2n j k

2n j k j

2nj

=
k=0

2n 2j k 2n j j

2n j nj 4 . j

because

2n j k

2n j k j

2n 2j . k

Mathematics Magazine, Vol. 80, April 2007


1767. Proposed by Mowaaq Hajja, Jordan. Let G be the centroid of ABC . Prove that if B AC = 60 and B GC = 120 , then the triangle is equilateral. Solution proposed by G.R.A.20 Problem Solving Group, Roma, Italy. Let a, b, c be the sides and let ma , mb , mc be the medians of ABC . Since Area(BAC ) = 3Area(BGC ) then 1 3 bc sin(60 ) = 2 2 that is 4mb mc = 3bc. Moreover, it is well known that
2 2 2 4m2 b = 2a + 2c b 2 2 2 and 4m2 c = 2a + 2b c .

2mb 3

2mc 3

sin(120 )

By cosine rule applied to BGC we have a = then 9a2 = (2a2 + 2c2 b2 ) + (2a2 + 2b2 c2 ) + 3bc that is 5a2 (c2 + b2 + 3bc) = 0. By cosine rule applied to BAC we have a2 = b2 + c2 2bc cos(60 ). Therefore 5(b2 + c2 bc) (c2 + b2 + 3bc) = 4(b c)2 = 0 or b = c. Since a2 = b2 + c2 bc then a = b = c and ABC is equilateral.
2

2mb 3

2mc 3

2mb 2mc cos(120 ) 3 3

Mathematics Magazine, Vol. 80, February 2007


1764. Proposed by Ovidiu Furdui, USA. For positive integer n, let gn = 1 + lim 1 1 + + log(n). Prove that 2 n
gn gn 2n

where is the Euler-Mascheroni constant. Solution proposed by G.R.A.20 Problem Solving Group, Roma, Italy. Since gn = Hn log(n) = + then log
gn gn

1 + o(1/n) 2n

= log(gn ) gn log( ) 1 1 + o(1/n)) ( + + o(1/n)) log( ) 2n 2n 1 1 = log( ) + log(1 + + o(1/n)) ( + + o(1/n)) log( ) 2n 2n 1 log( ) = + o(1/n). 2n = log( +

Therefore
gn gn 2n

= exp 2n log

gn gn

= exp (1 log( ) + o(1))

e .

Mathematics Magazine, Vol. 80, February 2007


1762. Proposed by Erwin Just, USA. Let n be an integer with n 2. Prove that for any even integer k , there exist odd primes p and q such that p + q = k (mod n). Solution proposed by G.R.A.20 Problem Solving Group, Roma, Italy. We have to prove that given two integers n 2 and m then we can nd two odd primes p and q and an integer x such that p = (2m q ) + nx. By Dirichlets Theorem, the linear progression x (2m q ) + nx for x N contains innite primes as soon as gcd((2m q ), n) = 1 therefore it suces to nd a prime q such that 2m q and n are relatively prime. s 1 2 If n 1 n2 ns is the prime factorization of n then the above condition is equivalent to the following: gcd(2m q, ni ) = 1 for i = 1, . . . , s. For any i = 1, . . . , s take an integer ri such that 0 < ri < ni and ri = 2m (mod ni ) and solve the system of congruences q = 2m r1 (mod n1 ) . . . q = 2m r (mod n ) s s By the Chinese Remainder Theorem there exists a solution 0 < q0 < n1 ns (note that gcd(ni , nj ) = 1 for i = j ). Hence y q0 + (n1 ns )y for y N is a linear progression whose elements q solve the system and therefore gcd(2m q, ni ) = 1 for i = 1, . . . , s. Moreover, since by construction gcd(q0 , n1 ns ) = 1, then, again by Dirichlets Theorem, this progression contains innite primes.

Mathematics Magazine, Vol. 80, February 2007


1761. Proposed by Steve Butler, USA. For integer n 2 dene the sets A(n) = {(k, j ) : 1 k j n, k + j n, and gcd(k, j ) = 1} B (n) = {(k, j ) : 1 k j n, k + j > n, and gcd(k, j ) = 1}. Prove that A(n) and B (n) have the same cardinality. Solution proposed by G.R.A.20 Problem Solving Group, Roma, Italy. We denote by [k, j ] = We rst note that for n 2
n j n

1 if gcd(k, j ) = 1 . 0 otherwise

|A(n)| + |B (n)| =
j =1 k=1

[k, j ] =
j =1

(j ).

Moreover for n > 2 1 |A(n)| |A(n 1)| = 2


n1

1 [k, n k ] = 2 k=1

n1

[k, n] =
k=1

(n) 2

because gcd(k, n k ) = gcd(k, n). Therefore, since A(2) = ((1) + (2))/2 = 1, we have that 1 |A(n)| = 2 that is |A(n)| = |B (n)|. 1 (j ) = (|A(n)| + |B (n)|) 2 j =1
n

The College Mathematics Journal, November 2007


861. Proposed by DiDomenico (USA). Show that if P is an even perfect number, then there exist positive integers a < b < c such that P = a + b + c and (a + b)2 + (a + c)2 = (b + c)2 ; that is, (a + b, a + c, b + c) is a Pythagorean triple. Solution proposed by G.R.A.20 Problem Solving Group, Roma, Italy. Any even perfect number has the form P = 2n (2n+1 1) for n 1. Let a = 2n 1 , then 0 < a < b and for n 1 c b = 2n (2n+1 1) (2n 1) 2 2n = 2(2n )2 4 2n + 1 > 2 2n (2n 2) 0. Moreover (a + b)2 + (a + c)2 = (b + c)2 is equivalent to aP = b(P a b), that is b(a + b) = P (b a) and b(a + b) P (b a) = 2n (2 2n 1) 2n (2n+1 1) = 0. b = 2n , c=P ab

The College Mathematics Journal, May 2007


855. Proposed by Michel Bataille, France. Show that for |x| < 1

n=1

n xn 2

x
n=1

n 3

=
n=1

n xn 3

x
n=1

n 2

where u denotes the oor function. Solution proposed by G.R.A.20 Problem Solving Group, Roma, Italy. It suces to prove that for |x| < 1

n=1

n n x

x = 1x

x
n=1

=
n=1

n n=1

where is a positive irrational number. All power series are clearly convergent for |x| < 1. Moreover the coecient of xn of the product power series on the right is

[n k 1]
k=1

where [u] denotes the indicator function. Since n/ is never an integer then [n k 1] = [ k n 1] = [ k < n] = [k < n] = [k < n/] = [k n/ ] . Therefore

[n k 1] =
k=1 n k=1

[k n/ ] =

that is the coecient of x of the power series on the left.

The College Mathematics Journal, May 2007


854. Proposed by Ovidiu Furdui, USA. Prove that for every natural number n 3

k=1

Hk 1 = k (k + 1)(k + 2) (k + n) n!
k j =1

2 1 1 1 + 2 + + 6 2 (n 1)2 1/j .

where Hk is the k -th harmonic number

Solution proposed by G.R.A.20 Problem Solving Group, Roma, Italy. Since


k=1

1/k 2 = 2 /6, it suces to prove that

F (n) := n!
k=1

Hk = k (k + 1)(k + 2) (k + n)

k =n

1 . k2

We rst consider the case n = 1:

F (1) =
k=1

Hk = k (k + 1)

j =1

1 j

k =j

1 1 k k+1

=
j =1

1 . j2

Assume now that n > 1, then

F (n) = n!
k=1

Hk n

1 1 k (k + 1) (k + n 1) k (k + 1) (k + n)

= F (n 1) (n 1)!
k=1

Hk . (k + 1) (k + n)

Note that

k=1

Hk = (k + 1) (k + n)

j =1

1 j

k =j

1 (k + 1) (k + n)

=
j =1

1 j (n 1)

k =j

1 1 (k + 1) (k + n 1) (k + 2) (k + n)

=
j =1

1 1 j (n 1) (j + 1) (j + n 1)

= = =

1 (n 1) 1 (n 1)2 (n

j =1

1 j (j + 1) (j + n 1) 1 1 j (j + n 2) (j + 1) (j + n 1) .

j =1

1 1)2 (n 1)!

Therefore F (n) = F (n 1) (n 1)! 1 = F (n 1) 2 (n 1) (n 1)! (n 1)2

and the statement is proved by induction.

The College Mathematics Journal, May 2007


851. Proposed by Jos e Luis D az-Barrero, Spain. Let a, b, and c be positive real numbers such that ab + bc + ca = 1. Prove that a b2 + c2 + bc + b c2 + a2 + ca + c a2 + b2 + ab 3.

Solution proposed by G.R.A.20 Problem Solving Group, Roma, Italy. The inequality is equivalent to abf (c/b) + bcf (a/c) + caf (b/a) 3

where f (x) = x2 + x + 1. Since f (x) is convex on the real line and ab + bc + ca = 1 with ab, bc, and ca positive real numbers then abf (c/b)+bcf (a/c)+caf (b/a) f (ab(c/b) + bc(a/c) + ca(b/a)) = f (1) = 3

The College Mathematics Journal, March 2007


847. Proposed by Juan-Bosco Romero M arquez, Spain. Show that if r and R are the radii of the inscribed and circumscribed circles of ABC and s is its semiperimeter, then 3 r2 s 4Rr + r2 s . 3 3

Solution proposed by G.R.A.20 Problem Solving Group, Roma, Italy. We will use Feuerbachs equations. The rst group involve the radii r1 , r2 , r3 of the ex-circles of ABC r1 + r2 + r3 = 4R + r r1 r2 + r 2 r3 + r3 r1 = s2 . r1 r2 r 3 = s2 r By Symmetric Mean Inequality (AGM), r1 + r2 + r 3 (r1 r2 r3 )1/3 3 that is 4Rr + r2 3 r2 s . 3 The second group involve the sides a1 , a2 , a3 of ABC . a1 + a2 + a3 = 2s a1 a2 + a2 a3 + a3 a1 = s2 + r(4R + r) . a1 a2 a3 = 4sRr By Symmetric Mean Inequality, a1 + a2 + a3 3 that is 4Rr + r2 s . 3 3 a1 a2 + a2 a3 + a3 a1 3
1 /2

The College Mathematics Journal, March 2007


846. Proposed by Jos e Luis D az-Barrero, Spain. Prove that for every positive integer n 1 n
n

1
n k k

k=1

n+1 2n

n+1 2

Solution proposed by G.R.A.20 Problem Solving Group, Roma, Italy. By AGM 1 n


n

1
n k k

k=1

1
n k k

1 n

k=1

Therefore, it suces to prove that 2 n+1 Since


n n n

k=1

n k

2 n(n+1)

2 =22

n1

=2
k=1

n1 k1

=2
k=1

k n n k

2 = n

k
k=1

n k

then, after taking the logarithm, the inequality becomes log 2n n+1 = log 2 n(n + 1)
n

k
k=1

n k

2 n(n + 1)

k log
k=1

n k

which holds because the logarithm is a concave function and 2 n(n + 1)


n

k = 1.
k=1

The College Mathematics Journal, January 2007


844. Proposed by Ovidiu Furdui, USA. Let L = lim Prove that
n 1 k=1 k n n

k=1

ln k ln2 (n) k 2

n=1

ln n 2 2 = + L n 2 12

where is Eulers constant. Solution proposed by G.R.A.20 Problem Solving Group, Roma, Italy. The partial sum is
N

SN =
n=1

n 1 k=1 k

ln n = n

n=1

Hn ln n HN n n n=1

We note that
N

n=1

Hn = n

n=1

1 n

k=1

1 1 = k 2

n=1

1 n

1 2

n=1

2 1 HN 1 = + 2 n 2 2

n=1

1 . n2

Since = HN ln N + O(1/N ) then SN


2 1 HN + = 2 2 N

n=1

ln n 1 (HN ln N )HN + O(1/N )HN 2 n n n=1


N

(HN ln N )2 1 = + 2 2

n=1

1 n2

n=1

ln n ln2 N n 2

+ O(1/N )HN .

Hence, by taking the limit we nd


N

lim SN =

2 2 + L. 2 12

The College Mathematics Journal, January 2007


841. Proposed Mohammad K. Azarian, USA. Assume that the quadratic polynomial f (x) = ax2 + bx + c, a = 0, has two xed points x1 and x2 , x1 = x2 . If 1 and 1 are two xed points of the function f (f (x)), but not of f (x), then nd the exact values of x1 and x2 . Solution proposed by G.R.A.20 Problem Solving Group, Roma, Italy. The numbers x1 and x2 satisfy the following equations f (x) x = ax2 + (b 1)x + c = 0 and f (f (x)) x = a(ax2 + bx + c)2 + b(ax2 + bx + c) + c x = a3 x4 + 2a2 bx3 + C (a, b, c)x2 + D(a, b, c)x + ac2 + bc + c = 0 Since 1 and 1 are the other two solutions of the last equation then 2a2 b b1 = x1 + x2 + 1 1 = x1 + x2 = 3 a a 2 ac + bc + c = x1 x2 1 (1) = x1 x2 = c a3 a Therefore b = 1 and c = 0 or b = 1 and c = a. If b = 1 and c = 0 then f (x) = ax2 x and f (f (1)) = f (a 1) = a(a 1)2 (a 1) = a3 2a2 + 1 = 1, f (f (1)) = f (a + 1) = a(a + 1)2 (a + 1) = a3 + 2a2 1 = 1 that is a2 (a 2) = 0 and a2 (a + 2) = 0 which do not hold if a = 0. If b = 1 and c = a then f (x) = ax2 x a = a(x2 1) x and f (f (1)) = f (1) = 1, By solving f (x) x = a(x2 1) x x = ax2 2x a = 0 we nd the two xed points of f 1 + 1 + a2 x1 = a f (f (1)) = f (1) = 1.

and x2 =

1 + a2 . a

Mathematics Magazine, Vol. 79, December 2006


1760. Proposed by P. Iv ady, Hungary. Prove that for 0 < a < b < a2 + b 2 a+b ba + ab > . 2 2 log(b) log(a)

Solution proposed by G.R.A.20 Problem Solving Group, Roma, Italy. We rst divide by ab: (a/b) + (b/a) ( 4 b/a 2 2
4

a/b)2

>

b/a a/b . log(b/a)

Letting t = log(b/a) > 0, we can introduce the hyperbolic functions cosh(x) = (ex + ex )/2 and sinh(x) = (ex ex )/2 cosh(t) 2 (sinh(t/4))2 > or equivalently cosh(t) > 2 (sinh(t/4))2 + sinh(t/2) (t/2)
2

sinh(t/2) (t/2)

We will prove this inequality by comparing the series expansions at 0 of the two sides (which converge for t > 0). The expansion of the left hand side gives

LHS = cosh(t) = 1 +
n=1

t2n . (2n)!

The expansion of the right hand side gives RHS = = 2 (sinh(t/4))2 + sinh(t/2) (t/2)
2

sinh(t/2) (cosh(t/2) 1) + (t/2)


2

sinh(t/2) 2 sinh(t/2) cosh(t/2) 2 sinh(t/2) = (cosh(t/2)) 2 cosh(t/2) + 1 + + (t/2) (t/2) (t/2) cosh(t) + 1 cosh(t) 1 sinh(t) 2 sinh(t/2) = 2 cosh(t/2) + 1 + 2 +2 2 2 t t (t/2)

= 1+
n=1

t2n 1 2 2 2 2 2n + + 2n (2n)! 2 2 (2n + 2)(2n + 1) 2n + 1 2 (2n + 1)

where we used the following identities to reduce the powers and the products of the hyperbolic functions 2 (sinh(x/2))2 = cosh(x)1, 2 (cosh(x/2))2 = cosh(x)+1, sinh(2x) = 2 sinh(x) cosh(x). Therefore the inequality is veried as soon as we show that for n 1 1> that is 1 2 2 2 2 2n + + 2n 2 2 (2n + 2)(2n + 1) 2n + 1 2 (2n + 1) 2n + 3 2n + 2 1 > 2n . 4 (2n + 2)(2n + 1) 2 (2n + 1)

The last inequality holds for n = 1 and for n > 1 it easy to prove that 2n + 3 1 > . 4 (2n + 2)(2n + 1)

Mathematics Magazine, Vol. 79, December 2006


1759. Proposed by L. Cusick and M. Nogin, USA. Let ABC be an equilateral triangle with three external triangles CDB , AEC and AF B . Assume that = F AB = DCB , = EAC = DBC , and = F BA = ECA. Prove that segments AD, BE , and CF are concurrent. Solution proposed by G.R.A.20 Problem Solving Group, Roma, Italy. Let P , Q, and R respectively the intersection between AD and CB , EB and AC , and CF and AB . It is easy to see that |CP | tan (tan + 3) . = |P B | tan (tan + 3) Similarly tan (tan + 3) |BR| = |RA| tan (tan + 3) Therefore |CP | |BR| |AQ| =1 |P B | |RA| |QC | and concurrency follows from Cevas Theorem. and |AQ| tan (tan + 3) . = |QC | tan (tan + 3)

Mathematics Magazine, Vol. 79, December 2006


1758. Proposed by M. Goldenberg and M. Kaplan, USA. Let Fn be the nth Fibonacci number, that is, F0 = 0, F1 = 1, and Fn = Fn1 + Fn2 for n 2. Prove that F2n + 1 = 3. F 1 2 n n=2 Solution proposed by G.R.A.20 Problem Solving Group, Roma, Italy. First we note that for N 2 SN := log where arctanh(x) = F2n + 1 F 1 n=2 2n 1 log 2
N N

=2
n=2

arctanh(1/F2n )

1+x 1x

for |x| < 1.

We have to show that limN SN = log 3. Since arctanh(x) arctanh(y ) = 1 log 2 1 = log 2 (1 + x)(1 y ) (1 x)(1 + y ) (1 xy ) + (x y ) (1 xy ) (x y )

= arctanh

xy 1 xy

then arctanh(1/F2n1 )arctanh(1/F2n+1 ) = arctanh F2n+1 F2n1 F2n+1 F2n1 1 = arctanh(1/F2n )

2 because F2n+1 F2n1 = F2n and F2n+1 F2n1 1 = F2 n . Therefore N

SN = 2
n=2

(arctanh(1/F2n1 ) arctanh(1/F2n+1 )) = 2 (arctanh(1/F3 ) arctanh(1/F2N +1 ))

and nally
N

lim SN = 2 arctanh(1/F3 ) = 2 arctanh(1/2) = log 3.

Mathematics Magazine, Vol. 79, December 2006


1756. Proposed by C. Moen and W. Wardlaw, USA. For which non negative integers n do there exist nonnegative integers a and b such that n! = 2a (2b 1)? Solution proposed by G.R.A.20 Problem Solving Group, Roma, Italy. Any factorial n! is the product of a power of 2 by an odd number f (n). The question is for which n, f (n) has the form 2b 1 (note b has to be positive). By direct inspection we have that for n < 10 the answer is yes for n {0, 1, 2, 3, 4, 5}: 0! = 1! = 20 (21 1), 2! = 21 (21 1), 3! = 21 (22 1), 4! = 23 (22 1), 5! = 23 (24 1). For n 10, if f (n) = 2b 1 then b > 13 because f (n) is an increasing function (although not strictly), and f (10) = 10!/28 = 14175 > 8191 = 213 1. By a theorem of A. Schinzel (see On primitive prime factors of an bn , Proc. Cambridge Philos. Soc. 58 (1962), 555-562) for b 13 there is a prime p 2b + 1 such that p | 2b 1. Hence if n! = 2a (2b 1) then p | n! which means that p | n and n p 2b + 1. So we have a bound for b : b (n 1)/2. We can nd also a bound for a : since a is the power of 2 in the factorization of n! then n n a= < = n. k k 2 2 k=1 k=1 Therefore for n 10 n! = 2a (2b 1) < 2a+b < 2n+(n1)/2 < 23n/2 On the other hand, for n 10 n! > 3! 4n3 > 4n2 = 22n4 and we have a contradiction because 22n4 < n! < 23n/2 means that 2n/2 < 16 which is evidently false for n 10.

Mathematics Magazine, Vol. 79, October 2006


1754. Proposed by M. Benzce, Romania. Let a1 , a2 , . . . , an be positive real numbers. Prove that
n n
n

ak log 1 +
k=1

(eak
k=1

1 1) n

ak .
k=1

Solution proposed by G.R.A.20 Problem Solving Group, Roma, Italy. We introduce the following notations:
n

G ({xk }) =

xk
k=1

1 and A ({xk }) = n

xk .
k=1

Note that for all positive real numbers x1 , x2 , . . . , xn then log (G ({ak })) = A ({log (ak )}) . Letting f (x) = ex 1, we have to show that G ({xk }) f 1 (G ({f (ak )})) A ({ak }) or, since f 1 (x) = log(1 + x) is an increasing function, f (G ({ak })) G ({f (ak )}) f (A ({ak })) . The second one si equivalent to A ({log (f (ak ))}) = log (G ({f (ak )})) log (f (A ({ak }))) which holds because the function log (f (x)) is concave in (0, +): D2 (log (f (x))) = The rst one is equivalent to log (f (exp (log (G ({ak }))))) log (G ({f (exp (log (ak )))})) that is log (f (exp (A ({log (ak )})))) A ({log (f (exp (log (ak ))))}) which holds because the function log (f (exp (x))) is convex in R: D (log (f (exp(x)))) =
2
x +x x

ex < 0 for x > 0. (ex 1)2

ee

ee (1 + ex ) > 0 for x R (eex 1)2

(note that ey > 1 + y for any real y = 0).

Mathematics Magazine, Vol. 79, October 2006


1753. Proposed by J.C. George, New Mexico, USA. Let n be a positive integer and let Sn be the set of all positive integers whose (base ten) digit sum is n. Determine the convergence or divergence of the series 1 . k

k Sn

Solution proposed by G.R.A.20 Problem Solving Group, Roma, Italy. We will show that the series converge. Since the terms are positive we can rearrange them and it suces to prove that the sum is bounded. First solution. Let Sn,d,p be the set of all positive integers whose digit sum is n, the number of digits is d and the number of non-zero digits is p. Note that 1 p min(n, d) and |Sn,d,p | 9p d1 . p1

because there are 9 ways to choose one of the p non-zero digits and they have to be placed one at the most signicant position and the other p 1 among the remaining d 1 positions. Therefore 1 = k
n

k Sn

p=1 d=p kSn,d,p

1 k

p=1 d=p

|Sn,d,p | min(Sn,d,p )

9p
p=1 d=p

d1 1 d1 . p1 10

Since

d=p

d 1 d1 xp1 . x = (1 x)p p1
n

then
k Sn

1 k

1/10p1 9 = (1 1/10)p p=1


p

10 = 10n
p=1

Second solution. Let T1 be the set of all positive integers whose digits are dierent from 1 then by a 1 well known result due to Kempner the series kT1 k converge: 1 = k

kT1

d=1 kT1,d

1 k

d=1

|T1,d | min(T1,d )

d=1

8 9d1 1 = 8 = 80 10d1 1 9/10

where T1,d is the set of the numbers of T1 with d digits.

Now we prove the statement by induction with respect to n. For n = 1 1 = k

k S1

d=1

1 10 1 = . = d 1 10 1 1/10 9 1 80 + k k S 1 k
n1

For n > 1
k Sn

1 = k k S

n T1

1 + k

kSn \T1

because any number k Sn \ T1 has a digit equal to 1 and therefore if we replace that 1 with a 0 we obtain a nonzero number k Sn1 which is less than k . Hence 1 10 (n 1)80 + . k 9

k Sn

Mathematics Magazine, Vol. 79, October 2006


1751. Proposed by I. Bluskov, Canada. Let k1 , k2 , . . . , kn be integers with ki > 2, i = 1, 2, . . . , n, and let N = Prove that n ki kj ki + 1 N +3 = 2 2 4 2 1=1 1i<j n Solution proposed by G.R.A.20 Problem Solving Group, Roma, Italy. We rst note that for any k > 2 then 3 because 3 k+1 4 = 1 k k2 k 2 1 k (k + 1)k (k 1)(k 2) = = 8 2 2 2 2 2 k 1 . 2 k+1 4 = 1 2 k 2
2 n i=1 ki 2

k 2

Therefore N 2 1 N2 N = = 2 2 2 1 = 2 =
1i<j n n n

i=1

ki 2 ki 2
n

1 2 kj 2 ki 2

i=1

ki 2
n

1=1

ki 2 ki 2 ki 2

+
1i<j n

1 2
2

i=1

ki 2

kj 2 kj 2

1 2

i=1 n

ki 2

=
1i<j n

+3
1=1

ki + 1 4

Mathematics Magazine, Vol. 79, April 2006


1744. Proposed by M. Goldenberg and M. Kaplan, The Ingenuity Project, Baltimore Polytechnic Institute, Baltimore, MD. For real number x 1, dene a1 = 2x and an+1 = a2 n 2, n = 1, 2, 3, . . .. Find a closed form expression for n 1 S (x) = . a k n=1 k=1 Solution proposed by G.R.A.20 Problem Solving Group, Roma, Italy. We will show that S (x) = x Since x= then an = z 2 because a1 = z + 1/z , and an+1 = a2 n2 = Moreover 1 z z
n
n1

x2 1 := z 1 z

for x 1.

1 2

z+

for 0 < z 1 1 z 2n1 1 1 2n 2 = z . + n z2 z 2n

z2

n1

1 z
2n1

2 = z2 + 2 +
n

k=1

ak = z 2

1 . z 2n

This identity holds for n = 1 and it can be easily veried by induction for n > 1 z Therefore

1 z

k=1

ak =

z2

n1

1 z
2n1

an = z 2

1 . z 2n

S (x) =
n=1

z
2

1 z

z2

1 z 2n
k

= z
2

1 z

1z n z2 z n=1

z2
k=0

n+1

1z n1 (z 2 )(2k+1)2 . z n=1 k=0

n=1

z2 1 z 2n+1

Since any positive integer m can be written in a unique way as the product of an odd number, 2k + 1, by a power of 2, 2n1 , then for 0 < z < 1 1 z2 1 z2 z2 S (z ) = (z 2 )m = = z = x x2 1. 2 z z 1 z m=1 If z = 1 then x = 1, an = 2 for all n 1 and S (x) = 1 = z .

Mathematics Magazine, Vol. 79, February 2006


1739. Proposed by Emeric Deutsch, Polytechnic University, Brooklyn, NY. An object moves in the plane, starting from the origin, and at each step moving one unit up, down, to the right, or to the left. Find the number of such paths that stay in the quadrant {(x, y ) : x, y 0}, and consist of a total of n steps, exactly k of which are vertical (up or down). Solution proposed by G.R.A.20 Problem Solving Group, Roma, Italy. We rst consider the one-dimensional problem: count the number of paths tn in {x : x 0} of n steps, starting from the origin, and at each step moving one unit to the right, or to the left. If the number of steps n is odd then the objects position x is positive and we can choose both directions: tn+1 = 2 tn if n is odd If the number of steps n is even then the objects position x can be positive and again we can choose both directions, or it can be the origin and in this case we must move to the right. Since the number of paths which arrive in the origin after n steps is precisely the catalan number Cn/2 = then tn+1 = 2 (tn Cn/2 ) + 1 Cn/2 = 2 tn Cn/2 Since t0 = 1 and t1 = 1 it suces to verify that the formula tn = n n/2 if n is even. n 1 n/2 + 1 n/2

solves the two previous recurrences: if n is odd tn+1 = if n is even tn+1 = n+1 n/2 = n+1 n/2 + 1 = n+1 n n/2 + 1 n/2 = 2 tn Cn/2 . n+1 (n + 1)/2 = n+1 (n + 1)/2 n n/2 = 2 tn ,

Finally we note that a path in the quadrant can be decomposed in two paths: one of k steps along the half-line {x : x 0} and one of n k steps along the half-line {y : y 0}. Therefore the number of paths is n tk tnk = k n k k k/2 nk (n k )/2 .

The College Mathematics Journal, November 2006


838. Proposed by Arkady Alt, San Jose, CA. Show that in any acute triangle ABC with sides a, b and c, the following inequality is true: 27 (a + b + c)2 a2 1 1 1 + 2 + 2 2 2 2 2 +b c b +c a c + a2 b 2

Solution proposed by G.R.A.20 Problem Solving Group, Roma, Italy. Since c2 = a2 + b2 2ab cos then 1 tan 1 = = a2 + b 2 c 2 2ab cos 4A where A is the triangles area. Simmetrically b2 1 tan = 2 2 +c a 4A and c2 1 tan = . 2 2 +a b 4A

After transforming and rearranging the terms, the inequality becomes 27 tan + tan + tan A 2 s 27

where s is the triangles semiperimeter. Now it suces to prove that A 1 2 s 27 and 27 tan + tan + tan . s(s a)(s b)(s c) and by

First inequality: by Herons formula A = applying AGM inequality we obtain A = s2 a 1 s b 1 s c 1 s 1 3

a b c 1 +1 +1 s s s

1 = . 27

Second inequality: we note that tan( + ) = tan + tan 1 tan tan

therefore, since tan = tan( + ), we nd that tan + tan + tan = tan tan tan . Since the triangle is acute then tan , tan and tan are positive and by applying AGM inequality we obtain tan + tan + tan = tan tan tan which means that tan + tan + tan 3
3

27 tan + tan + tan .

The College Mathematics Journal, September 2006


835. Proposed by Juan-Bosco Romero M arquez, Universidad de Valladolid, Vallalodid, Spain. Show that for any positive real numbers a, b, and c, abc 1 (a + b + c)3 (a3 + b3 + c3 ) (a + b + c)3 . 24 27

Solution proposed by G.R.A.20 Problem Solving Group, Roma, Italy. First inequality gives (a + b + c)3 (a3 + b3 + c3 ) 24 3a2 b + 3a2 c + 3b2 a + 3b2 c + 3c2 a + 3c2 b + 6abc a2 b + a2 c + b 2 a + b 2 c + c 2 a + c 2 b a2 b + a2 c + b 2 a + b 2 c + c 2 a + c 2 b 6 which holds by the AM-GM inequality. The second inequality gives (a + b + c)3 (a3 + b3 + c3 ) 24 (a + b + c)3 (a3 + b3 + c3 ) 8 (a + b + c)3 9 3 a+b+c 3 1 (a + b + c)3 27 1 (a + b + c)3 9 abc 24abc 6abc abc

a3 + b 3 + c 3 a3 + b 3 + c 3 3

which holds by the Power Means inequality (or by the convexity of the function x3 for x 0).

The College Mathematics Journal, September 2006


834. Proposed by Jos e Luis D az-Barrero, Barcelona, Spain. Let n be a positive integer. Prove that 1 2 Fn
3

Fn+1 + Fn+1 Fn

Fn+2 + Fn+2 Fn+1

Fn Fn+2

< Fn+2

where Fn is the nth Fibonacci number (F1 = 1, F2 = 1, Fn = Fn1 + Fn2 ). Solution proposed by G.R.A.20 Problem Solving Group, Roma, Italy. First Proof. By A-G-M inequality
2/3 Fn Fn+1 1/ 3

2Fn + Fn+1 2Fn+1 + Fn+2 2Fn+2 + Fn 2/3 1/3 2/3 1 /3 , Fn+1 Fn+2 < , Fn+2 Fn < . 3 3 3

Note that in the last two inequalities the left terms are strictly less than the right terms because Fn+1 = Fn+2 and Fn = Fn+2 for any positive integer n. Finally, since Fn + Fn+1 = Fn+2 , we have that 1 Fn + Fn+1 + Fn+2 2/3 1/3 2/3 1 /3 2/3 1/3 Fn Fn+1 + Fn+1 Fn+2 + Fn+2 Fn < = Fn+2 . 2 2 Second Proof. We rst show that Fn
3

Fn+1 = Fn Fn

1+

Fn1 1 Fn + Fn1 Fn 3 Fn 1 Fn+1 + Fn Fn+1 3

for n 1

Fn+1

Fn+2 = Fn+1 Fn+1

1+

for n 1

where we used the inequality

1+x1+ 1 x for x 0. 3

Moreover, since Fn+2 > 2Fn then Fn+2


3

Fn Fn+2 Fn+2

Fn = Fn+2 2Fn

1 4 < Fn+2 2 5

for n 1.

Now we sum the upper bounds and by the recurrence formula we obtain: 1 1 1 4 9 1 (Fn + Fn1 + Fn+1 + Fn + Fn+2 ) = Fn+2 + Fn+1 . 2 3 3 5 10 6 So, it suces to prove that 9 1 Fn+2 + Fn+1 < Fn+2 10 6 that is 5Fn+1 < 3Fn+2 which is true because 5Fn+1 = 5Fn1 + 5Fn < 6Fn1 + 6Fn = 3Fn + 3Fn+1 = 3Fn+2 .

The College Mathematics Journal, September 2006


831. Proposed by Stanley Rabinowitz, MathPro Press, Chelmsford, MA. In Problem 218 of this Journal (14 (1983) p.358), it was shown that tan Prove the related identities tan and tan 3 + 4 sin = 11 11 11 4 + 4 sin = 11. 11 11 3 2 + 4 sin = 11 11 11

Solution proposed by G.R.A.20 Problem Solving Group, Roma, Italy. Let S (z ) := z + z 3 + z 4 + z 5 + z 9 and S (z ) := z 2 + z 6 + z 7 + z 8 + z 10 where z = 1 is a complex number not such that z 11 = 1. Since S (z )+ S (z ) = 1 and S (z ) S (z ) = 3 (see Problem 218) then the numbers S (z ) and S (z ) are the roots of the quadratic equation u2 + u + 3 = 0 and therefore S (z ) S (z ) = = i 11. Let x = e2i/11 then for any integer number k i tan xk/2 xk/2 xk 1 xk (1 x10k ) k = k/2 = = 11 x + xk/2 xk + 1 xk + 1 = xk x2k + x3k x4k + x5k x6k + x7k x8k + x9k x10k = S (xk ) S (xk ) 2(x4k x7k )

Moreover, since x11 = 1, 2(x4k x7k ) = 2(x4k x4k ) = 4i sin 8k . 11

Therefore

k 8k + 4i sin = S (xk ) S (xk ) 11 11 and we found a general identity: for any integer number k not multiple of 11 i tan tan 8k k + 4 sin = 11 11 11

(when k is divisible by 11 the sum is trivially zero). Now, changing k we show all the requested identities: for k = 1 8 3 + 4 sin = tan + 4 sin = 11, tan 11 11 11 11 for k = 3 24 3 2 3 + 4 sin = tan + 4 sin = 11, tan 11 11 11 11 for k = 4 4 32 4 tan + 4 sin = tan + 4 sin = 11. 11 11 11 11 Note that in these cases it was easy to choose the sign of 11 because the value of the functions tan and sin were positive. This choice is not so trivial for k = 5: tan 40 5 4 5 + 4 sin = tan 4 sin = 11. 11 11 11 11

The College Mathematics Journal, Vol. 37, May 2006


827. Proposed by Syrous Marivani,Louisiana State University at Alexandria, Alexandria, LA. Evaluate the sums
n

k=0

n F4ak+b k

and
k=0

n L4ak+b k

where Fk and Lk are elements of Fibonacci and Lucas sequences, respectively, a and b are integers, and n is a positive integer. (F1 = 1, F2 = 1, Fn = Fn1 +Fn2 and L1 = 1, L2 = 3, Ln = Ln1 + Ln2 ) Solution proposed by G.R.A.20 Problem Solving Group, Roma, Italy. We will prove that
n

k=0

n G4ak+b = G2an+b k

F4a F2a

where Gn is the General Fibonacci sequence: Gn = Gn1 + Gn2 , but the starting values of G0 and G1 can be specied. If G0 = 0 and G1 = 1 we have the Fibonacci sequence, G0 = 2 and G1 = 1 gives the Lucas sequence. Solving the characteristic polynomial z 2 z 1 we have that where = (1 + 5)/2, = 1/ and the constants C1 and C2 are uniquely determined by the initial terms G0 and G1 .
n

Gn = C1 n + C2 n

k=0

n n n G4ak+b = C1 b (4a )k + C2 b ( 4a )k k k k k=0 k=0 = C1 b 4a + 1 2a F4a = C1 b F2a + C2 b 4a + 1 n 2a F4a + C2 b F2a n F4a 2an+b 2an+b = C1 + C2 F2a n F4a . = G2an+b F2a
n n n

The identities
4a

2a F4a +1= F2a

and

4a

2a F4a +1= F2a

can be easily veried: since 2 = 2 and by the Binets formula Fn = (n n )/ 5 then


4a

+1=

2a

2a

2a

)=

2a

4a 4a 4a 4a 2a F4a 2a = 2a = , 2a 2a 2a F2a

and 4a + 1 = 2a ( 2a + 2a ) = 2a
4a 4a 4a 4a 2a F4a 2a = = . 2a 2a 2a 2a F2a

The College Mathematics Journal, Vol. 37, March 2006


824. Proposed by Jeremy Leach (student), Prescott, AZ. Prove that the value of the sum
n n

Sn :=
j =1 k=1

(1)j +k

1 n1 j !k ! j 1

n1 (j + k )! k1

is independent of n Solution proposed by G.R.A.20 Problem Solving Group, Roma, Italy. We will show that the sum is always equal to S1 := (1)1+1 We note that
n n

1 11 1!1! 1 1

11 (1 + 1)! = 2 11

Sn =
j =1 k=1 n

(1)j +k (1)
j =1 j

n1 j1
n

n1 k1

j+k k j+k k

= Since j+k k =

n1 j1

(1)k
k=1

n1 k1

(j + k )k j 1 (j + 1)k (j 1)k = = (1)k = (1)k k k! k! k!

then we can apply the Vandermondes convolution formula to the internal sum
n

(1)
k=1

n1 k1

j+k k

=
k=1

n1 nk

j 1 k

nj2 n

Therefore for n 2
n

Sn =
j =1

(1)

n1 j1

nj2 n

=
j =n1

(1)j 2 n

n1 j1

nj2 n

= (1)n1

n 1 1 n1 + (1)n n2 n n1 = (n 1) + (n + 1) = 2

because for n 3 and 1 j n 2 the binomial coecient

nj 2 n

is zero.

The College Mathematics Journal, Vol. 37, March 2006


821. Proposed by Andrew Cusumano, Great Neck, NY. Find
n n+2 j =1 n+1 j =1

lim

jj jj

n+1 j j =1 j n j j =1 j

Solution proposed by G.R.A.20 Problem Solving Group, Roma, Italy. Let sn :=


n j =1

j j . Since for n > 1

nn < sn = sn1 + nn = (n 1) (n 1)n1 + nn = (n 1)n + n2 < 2 nn then sn /nn = O(1). We can rene this result sn /nn = 1 + sn1 (n 1)n1 = 1 + O(1) nn nn n 1 1 1+ O(1) = 1 + O(1/n) = 1+ n1 n

and sn /nn = 1 + sn1 (n 1)n1 = 1 + (1 + O(1/n)) nn nn n 1 1 = 1+ 1 (1 + O(1/n)) n1 n 1 + O(1/n2 ). = 1+ en

Moreover sn+1 /sn


1 2 (n + 1)n+1 1 + e(n+1) + O(1/n ) = 1 nn 1 + en + O(1/n2 ) 1 1 = (n + 1) (1 + 1/n)n 1 + + O(1/n2 ) 1 + O(1/n2 ) e(n + 1) en 1 = (n + 1) e 1 + O(1/n2 ) 1 + O(1/n2 ) 2n 1 = e n + + O(1/n) . 2

Therefore sn+2 sn+1 1 1 = e (n + 1) + + O(1/n) e n + + O(1/n) sn+1 sn 2 2 and the limit is e. = e + O(1/n).

The College Mathematics Journal, Vol. 37, January 2006


819. Proposed by Michael Andreoli, Miami, FL. For n 1, evaluate

k=1

1 k (k + 1)(k + 2) (k + n)

Solution proposed by G.R.A.20 Problem Solving Group, Roma, Italy. We will prove that

an :=
k=1

1 1 = . k (k + 1)(k + 2) (k + n) n n!
k=1

The series converges by the comparison test: an Moreover

1/k 2 < +.

a1 =
k=1

1 = lim k (k + 1) N k=1

1 1 k k+1

= lim

1 N +1

= 1.

Since for n, k 1 1 1 = k (k + 1)(k + 2) (k + n) n then for n > 1 an = = 1 n 1 n

1 1 k (k + n 1) (k + 1) (k + n)

an1
k=1

1 (k + 1) (k + n) 1 k (k + n 1) = 1 n n!

an1
k=2

1 1 = an1 an1 n n!

Mathematics Magazine, Vol. 78, December 2005


1734. Proposed by H. A. ShahAli, Teheran, Iran. Let be a xed irrational number and let P be a polynomial with integer coecients and with deg(P ) 1. Prove that there are innitely many pairs (m, n) of integers such that P (m) = n . Solution proposed by G.R.A.20 Math Problems Group, Roma, Italy. The equation P (m) = n is equivalent to P (m) n < P (m) + 1. We can assume that > 0 (otherwise we change the sign of n) we have that P (m) P (m) 1 n< + . If P (m) = 0 (there are a nite number of such m) then P (m)/ is irrational and since n is integer the previous equation is equivalent to P (m) P (m) <n< + P (m) + 1

where {x} = x x is the fractional part of x. These inequalities will be satised by at least an integer n if the length of the interval is greater than 1 that is if 1 P (m) + > 1. If 0 < < 1 then this condition is trivially satised for all m but for > 1 the inequality becomes harder: we need innite integers m such that P (m) 1 1 ,1 .

This is provided by the following theorem due to H. Weyl where we take Q(x) = P (x)/: Let Q(x) be a nonconstant polynomial with real coecients such that at least one of its nonconstant coecients is irrational. Then the sequence of fractional parts {Q(m)} is dense in [0, 1).

Mathematics Magazine, Vol. 78, October 2005


1729. Proposed by Brian T. Gill, Seattle Pacic University, Seattle, WA. For positive integer k , let ck denote the product of the rst k odd positive integers, and let c0 = 1. Prove that for each nonnegative integer n
n

(1)k
k=0

n n! nk 2 ck = cn . k k!

Solution proposed by G.R.A.20 Math Problems Group, Roma, Italy. First we note that (2k )! . 2k k ! Then the identity we are going to prove is equivalent to this one ck = 1 3 (2k 1) = n 2k (4)nk k k
n

= (1)n

k=0

2n . n

Since the binomial coecient 2kk is the constant term of (x + 1/x)2k , then the sum on the left side is the constant term of the rational function 1 x+ x On the other hand 1 x+ x
2 n 2 n n

=
k=0

n 1 (4)nk x + k x
n 2 n

2k

1 x + 2 +24 x
2 2n n

1 x x

1 x x

2n

whose constant term (1)n equation.

which is just the right side of the previous

We remark that if dk is the product of the rst k even positive integers dk = 2 4 (2k ) = 2k k ! then the analogous sum is identically zero
n

(1)k
k=0

n n! nk 2 dk = k k!

(1)k
k=0

n n!2n = n!2n (1 1)n = 0. k

Mathematics Magazine, Vol. 78, October 2005


1727. Proposed by Jody M. Lockhart and Willaim P. Wardlaw, U.S. Naval Academy, Annapolis, MD. Chain addition is a technique used in cryptography to extend a short sequence of digits, called seed, to a longer sequence of pseudorandom digits. If the seed sequence of digits is a1 , a2 , . . . , an , then for positive integer k , an+k = ak + ak+1 mod 10, that is, an+k is the units digit in the sum ak + ak+1 . Suppose that the seed sequence is 3, 9, 6, 4. Prove that the sequence is periodic and nd, without the use of calculator or computer, the number of digits in the sequence before the rst repetition of 3, 9, 6, 4. Solution proposed by G.R.A.20 Math Problems Group, Roma, Italy. The sequence with seed 3, 9, 6, 4 can be generated in this way: vk+1 = M vk mod 10 for k 1 and v1 = [3, 9, 6, 4]t 1 0 0 1 0 1 0 0 0 0 . 1 0 where vk = [ak , ak+1 , ak+2 , ak+2 ]t and 0 0 M = 0 1 We will rst prove that M 1560 = I M 15 = I mod 10 mod 5. that is, by the Chinese Remainder Theorem, mod 2 and M 312 = I Note that M 4 = M + I mod 10 and if p is a prime then (A + I )p = A + I mod p. Using these two facts we get M 16 = (M 4 )4 = (M + I )4 = M 4 + I = M + 2I = M M 100 = = = = mod 2 and since M is invertible mod 2 we have that M 15 = I mod 2. Moreover, (M 4 )25 = (M + I )25 = M 25 + I = M (M 4 )6 + I M (M + I )6 + I = M (M + I )(M + I )5 + I = (M 2 + M )(M 5 + I ) + I (M 2 + M )(M (M 4 ) + I ) + I = (M 2 + M )(M 2 + M + I ) + I M 4 + 2M 3 + 2M 2 + M + I = 2(M 3 + M 2 + M + I ) mod 5

and M 60 = = = = (M 4 )15 = (M + I )53 = (M 5 + I )3 M 15 + 3M 10 + 3M 5 + I M 3 (M + I )3 + 3M 2 (M + I )2 + 3M (M + I ) + I M 2 (M 4 ) + 3M (M 4 ) + 3(M 4 ) + M 3 + 3(M 4 ) +6M 3 + 3M 2 + 3M 2 + 3M + I = 3M 3 + 2M + 2I mod 5.

Therefore M 160 = 2(M 3 + M 2 + M + I ) (3M 3 + 2M + 2I ) = M I = M 4 mod 5

and since M is invertible mod 5 we have that M 156 = I mod 5 and M 312 = I mod 5. Finally we prove that the period of the sequence is just 1560 = lcm(15, 312) by showing that M d v1 = v1 mod 10 for any proper divisor d of 1560 = 23 3 5 13.

For d divisor of 520 = 8 5 13 M 520 v1 = M 5 v1 = [0, 1, 0, 1]t = v1 = [1, 1, 0, 0]t For d divisor of 312 = 8 3 13 M 312 v1 = M 3 v1 = [0, 1, 1, 1]t = v1 = [1, 1, 0, 0]t For d divisor of 120 = 8 3 5 M 120 v1 = (3M 3 + 2M + 2I )2 v1 = [0, 3, 2, 4]t = v1 = [3, 4, 1, 4]t For d divisor of 780 = 4 3 5 13 M 780 v1 = (M 156 )5 v1 = v1 = [2, 1, 4, 1]t = v1 = [3, 4, 1, 4]t mod 5. mod 5. mod 2. mod 2.

Mathematics Magazine, Vol. 78, June 2005


1721. Proposed by Emerich Deutsch, Polytechnic University, Brooklyn, NY. Let k and n be positive integers with k n. Find the number of permutations of {1, 2, . . . , n} in which 1, 2, . . . , k appears as a subsequence but 1, 2, . . . , k, k + 1 does not. Solution proposed by G.R.A.20 Math Problems Group, Roma, Italy. The number of permutations of {1, 2, . . . , n} in which 1, 2, . . . , k appears as a subsequence is determined by the number of dierent positions of the numbers 1, 2, . . . , k , that is n , multiplied by the number of ways that the k remaining places can be lled, that is (n k )!. Therefore for k < n the required number of permutations is n n k n! (n k )! (n (k + 1))! = . k k+1 (k + 1)! Of course for k = n there is only one permutation with such property.

Mathematics Magazine, Vol. 78, June 2005


1721. Proposed by Donald Knuth, Stanford University, Stanford, CA. The Fibonacci graphs are dened by successively replacing the edge with maximum label n by two edges n and n + 1, in series if n is even, and in parallel if n is odd. Prove that the Fibonacci graph with n edges has exactly Fn+1 spanning trees, where F1 = F2 = 1 and Fn+1 = Fn + Fn1 . Show also that these spanning trees can be listed in such a way that some edge k is replaced by k 1 as we pass from one tree to the next. For example, for n = 5 the eight spanning trees can be listed as 125, 124, 134, 135, 145, 245, 235, 234. Solution proposed by G.R.A.20 Math Problems Group, Roma, Italy. Let Gn be the nth Fibonacci graphs and denote by T (Gn ) the set of all its spanning trees. In order to count these spanning trees we consider rst the trees that contain the edge n and then the trees that do not contain the edge n. If we remove the edge n from a spanning tree of the rst kind then it becomes a spanning tree of the graph obtained by contracting the edge n in Gn that is Gn1 if n is odd or Gn2 if n is even. A spanning tree of the second kind is a spanning tree of Gn2 plus the edge n 1 if n is odd or it is a spanning tree of Gn1 if n is even. Therefore in both cases for n > 2 |T (Gn )| = |T (Gn1 )| + |T (Gn2 )| and since |T (G1 )| = 1 and |T (G2 )| = 2 we have that |T (Gn )| = Fn+1 . Now we show a recursive algorithm that lists the trees of T (Gn ) in such a way that two consecutive trees dier only for some edge k that is replaced by k 1. Let T (G1 ) = 1 and T (G2 ) = 2, 1 , and, by the remark we made at the beginning, we are able to list T (Gn ) for n > 2 in following way: take the trees of T (Gn2 ) and add them the edge n 1 if n is odd or the edge n if n is even, take the trees of T (Gn1 ) and add them the edge n if n is odd, reverse the order of the two list and merge them. Below there are the lists for n = 3, 4, 5, 6: T (G3 ) = 12, 13, 23

T (G4 ) = 14, 24, 23, 13, 12 T (G5 ) = 234, 134, 124, 125, 135, 235, 245, 145 T (G6 ) = 126, 136, 236, 246, 146, 145, 245, 235, 135, 125, 124, 134, 234 . For a generic n if
2 2 , . . . , tn T (Gn2 ) = tn 1 Fn1 1 1 , . . . , tn and T (Gn1 ) = tn 1 Fn

then the list of the spanning trees of Gn is T (Gn ) =


2 n2 n1 1 tn (n 1), tF n, . . . , tn n, 1 Fn1 (n 1), . . . , t1 n n1 n2 n2 n1 tFn1 n, . . . , t1 n, tFn , . . . , t1

for n odd . for n even

The required property holds because it holds for T (Gn2 ), T (Gn1 ) and the trees in the middle are
n2 2 n1 n = t1 n for n odd tn (n 1) and tF 1 n . n2 n1 n2 t1 n and tFn = t1 (n 1) for n even

Mathematics Magazine, Vol. 78, April 2005


1718. Proposed by David Callan, Madison, WI. Let k, n be integers with 1 k n. Prove the identity.
k 1

i=0

k1 i

n (k 1) ki1 2 = ki

k 1

i=0

ki i

ni . k

Solution proposed by G.R.A.20 Math Problems Group, Roma, Italy. We consider the polynomial P (x) = (2 + x)k1 (1 + x)n(k1)
k 1 n(k1)

=
i=0

k 1 (k1)i i 2 x i

j =0

n (k 1) j x. j

Then the coecient of xk of P (x) is


k 1

i=0

k 1 (k1)i n (k 1) 2 i ki

which is the left side of the identity. On the other hand P (x) = (1 + (1 + x))
k 1 k 1

(1 + x)n = (1 + x)k1

1 +1 1+x
k 1

k 1

(1 + x)n

=
i=0 k 1

k1 1 (1 + x)n = i (1 + x)i k1 i
ni

i=0

k1 (1 + x)ni i

=
i=0

j =0

ni j x. j

Therefore the coecient of xk of P (x) is


k 1

i=0

k1 i

ni k

which is the right side of the identity.

Mathematics Magazine, Vol. 78, April 2005


1716. Proposed by Caf e Dalat Problem Solving Group, Washington D.C. Let k, n be integers with n 1 and 0 k n. Prove that there is an n n matrix A of 0s and 1s with per(A) = k. (Here per(A) denotes the permanent of A.) Solution proposed by G.R.A.20 Math Problems Group, Roma, Italy. The permanent of a n n matrix A = [aij ] is given by the formula per(A) =

a1(1) a2(2) an(n)

where the sum runs over all permutations of {1, 2, . . . n}. We rst dene the following n n matrix An,n = 1 1T n1 1n1 In1

By induction, per(An,n ) = n: per(A1,1 ) = per(1) = 1 and for n > 1, expanding the computation of the permanent along the last row, we have that per(An,n ) = per 1 1T 1T n2 n2 1 + per 1n2 In2 In2 0n2 = per (In2 ) + per(An1,n1 ) = 1 + (n 1) = n.

Now we dene An,0 = 0n,n Therefore per(An,0 ) = 0 and per(An,k ) = per(Ak,k ) per(Ink ) = k 1 = k. and An,k = Ak,k 0k,nk 0nk,k Ik for 0 < k < n.

Mathematics Magazine, Vol. 78, February 2005


1713. Proposed by Shawn Hedman and David Rose, Florida Southern College, Lakeland, FL. Prove that
n2

1
n k

n=4 k=2

3 = . 2

Solution proposed by G.R.A.20 Math Problems Group, Roma, Italy. We will use the following expession for the binomial coecient n k = nk k!

where nk = n(n 1) (n k + 1). For n k we compute the rst dierence (with respect to n) n 1 (n 1)k1 = 1 n k 1 1 (n k + 1) n k1 = = k . k 1 (n 1) n(n 1) (n k + 1) n

Therefore, changing order of summation (the terms are non negative), we obtain that
n2

1
n k

= =

n=4 k=2

k! nk k=2 n=k+2

k=2

k! k1 k! k1

n
n=k+2

1 (n 1)k1

=
k=2

1 (k + 1)k1

=
k=2

2 (k 1)(k + 1) 1 1 k1 k+1 k 1 1 + k k1 = 1 3 +1= . 2 2

=
k=2

=
k=2

Mathematics Magazine, Vol. 78, February 2005


1712. Proposed by William P. Wardlaw, U.S. Naval Academy, Annapolis, MD. For each integer m > 1, let Zm = {0, 1, . . . , m 1} be the ring of integers modulo m, and let Z m be the (multiplicative) group of units in Zm . Find the sum S (m) = uZ u and the product P (m) = uZ u of elements in Z m.
m m

Solution proposed by G.R.A.20 Math Problems Group, Roma, Italy.


st 1 s2 Let m = ps 1 p2 pt the prime factorization of m. We note that t

Z m = Zm \
i=1

{n Zm : pi | gcd(n, m)} .

By the inclusion-exclusion principle we have that S (m) =


m m pi pi + pi pj 2 2 i=1 1i<j t t m pi pj

+(1)t p1 pt

m p1 pt

Using the M obius function (n), the Euler function (n) and the identities (m) =
d|m

(d)

m = d

(
d|m

m )d and d 1 2

(d) = 0 for m 1
d|m

then S (m) =
d|m

(d)d

m d

(d)d
d|m

m m 1 = d d

(d)
d|m

= Similarly for P (m): P (m) =


d|m

m 2

m d

d|m

m(m) (d) = . 2

m m !d d d

(d)

=
d|m

m d! d

m d ( d )

=m

(m) d|m

d! dd

( m ) d

The College Mathematics Journal, Vol. 36, November 2005


814. Proposed by Jos e Luis D az-Barrero, Universitat Polit` ecnica de Catalu na, Barcelona, Spain. Suppose that a, b, and c are positive real numbers such that a+b+c = ab+bc+ca. Find the minimum value of a3 b3 c3 + + . (b c)2 + bc (c a)2 + ca (a b)2 + ab Solution proposed by G.R.A.20 Math Problems Group, Roma, Italy. We will prove a that for all integers n 3 then an bn cn an Fn (a, b, c) = + + = (b c)2 + bc (c a)2 + ca (a b)2 + ab cyc (b c)2 + bc has minimum value 3 which is attained for a = b = c = 1. Note that it can be proved that for n = 1 and n = 2 the minimum is attained for a = b = 2, c = 0 and its value is respectively 1 and 2. Letting 1 = a + b + c 2 = ab + bc + ca 3 = abc
2 we rst note that since by a mean inequality 1 32 and s = 1 = 2 > 0 then s = 1 = 2 3. Moreover, for k 2, by Muirheads inequality ((n, 0, 0) (n 1, 1, 0)) then

2
cyc

ak =
sym

ak
sym

ak 1 b =
cyc

ak1 (b + c)

1 ak (a + b + c) ak 1 ak 1 a2 . 3 cyc cyc cyc cyc Now we distinguish two cases: when n is odd and when n is even. If n = 2k + 1 with k 1 then, since the map 1/x is convex for x > 0, 2 ak+1
cyc

that is

Fn (a, b, c) =

b2 + c2 bc ak


cyc cyc

ak+1

a(b2 + c2 bc)

and by the previous inequality Fn (a, b, c)


cyc cyc

2 a2 =
2 (1 22 )2 . 1 2 63

a(b2 + c2 bc)

We nally verify that the last term is 3, that is (s2 2s)2 3(s2 63 )
3 + 93 41 2 then we have that is 0: since by Schurs inequality 1

(s4 6s3 + 9s2 ) + 2(s3 + 93 4s2 ) s2 (s 3)2 0. If n = 2k with k 2 then, since the map 1/x is convex for x > 0, 2 ak
cyc

Fn (a, b, c) =

b + c bc ak


cyc cyc

ak

(b2 + c2 bc)

and by the previous inequality Fn (a, b, c)


cyc cyc

2 a2 =
2 (1 22 )2 . 2 21 52

(b2 + c2 bc)

We nally verify that the last term is 3, that is (s2 2s)2 3(2s2 5s) is 0: for s 3 s4 4s3 2s2 + 15s = (s 3)(s2 s 5) 0 because s 3. 2

The College Mathematics Journal, Vol. 36, November 2005


812. Proposed by Ovidui Furdui, Western Michigan University, Kalamazoo, MI. Calculate the limit
n
n+1 k=1

lim e

sin(1/k)

n k=1

sin(1/k)

Solution proposed by G.R.A.20 Math Problems Group, Roma, Italy. The function is esin(1/(n+1)) 1 eHn e Since Hn = log(n) + + o(1), then the limit
n
n k=1 (1/k sin(1/k ))

lim esin(1/(n+1)) 1 eHn = lim (1/(n + 1)) elog(n)+ = e .


n

Moreover since 0 x sin(x) x3 /3 for x 0 then the following sum converges as n goes to innity because it has non negative terms and it is bounded 1 0 < c := (1/k sin(1/k )) 6 k=1

k=1

1 1 < 3 k 6

1+
1

1 dx x3

1 = . 4

Therefore the required limit exists: it is equal to e c (e 1/4 , e ) (1, 2) and it is approximately equal to 1.470079645.

The College Mathematics Journal, Vol. 36, September 2005


809. Proposed by Ovidui Furdui, Western Michigan University, Kalamazoo, MI. Find
n 2n

lim

2
k=n+1

2k

2k

k n .

Solution proposed by G.R.A.20 Math Problems Group, Roma, Italy. We rst note that
2n n

lim

2
k=n+1

2k

2k

2n

k n

= lim

2
k=n+1

2k

2k

k1 .

Since |ex 1 x|

k=2

|x|k e|x|2 k!

for x (1, 1)

then
2k

2k 1

log(2k ) log(2k ) e 2k 2k

and

k1

log(k ) log(k ) e k k

and the absolute value of the dierence


2n

2
k=n+1

2k

2k

2n

k1
k=n+1

2 1+

log(2k ) 2k
2n

1+

log(k ) k

is less than 2e

2n

k=n+1

log(2k ) 2k

+e
k=n+1

log(k ) k

which goes to zero as n goes to innity because

k=1

log(k ) k

< +.

Therefore the problem is equivalent to nd the limit as n goes to innity of


2n

k=n+1

log(2k ) 2 1+ 2k
n k=1

log(k ) 1+ k

= log(2)

1 k k=n+1

2n

that is, since

1/k = Hn = log(n) + + o(1),

log(2)(H2n Hn ) = log(2)(log(2n) + log(n) + o(1)) = (log(2))2 +o(1). So the required limit is equal to (log(2))2 .

The College Mathematics Journal, Vol. 36, September 2005


807. Proposed by Angelo S. DiDomenico, Milford, MA. Let a, b, c and d be positive integers such that gcd(a, c) = gcd(b, d) = 1 and b, d have opposite parity. (a) Find necessary and sucient conditions under which bc = ab + ad + cd. (b) In part (a), let x = bc ab, y = bc cd, and z = bc ad. Show that (x, y, z ) generates all primitive Pythagorean triples. Solution proposed by G.R.A.20 Math Problems Group, Roma, Italy. We prove that b d = a and b + d = a are necessary and sucient conditions under which bc = ab + ad + cd. Clearly they are sucient conditions. We can write bc = ab + ad + cd as a(b + d) = c(b d). Since (a, c) = 1, then b d = ah and b + d = ch for some h Z+ . Otherwise we can write bc = ab + ad + cd as b(c a) = d(c + a) and since (b, d) = 1, then c a = dk and c + a = bk for some k Z+ . Comparing these last equalities, we get hk = 2 and since b d 1 mod 2, then h = 1. Now x = bc ab = 2bd, y = bc cd = ac = b2 d2 and z = bc ad = b(b + d) (b d)d = b2 + d2 and since b, d are coprime and have opposite parity, then they generate all primitive Pythagorean triples.

The College Mathematics Journal, Vol. 36, May 2005


805. Proposed by Jos e Luis D az-Barrero, Universitat Polit` ecnica de Catalu na, Barcelona, Spain. Let n be a positive integer. Prove that
4 4 8 8 8 Fn Fn+1 n3 (F1 + F2 + . . . + Fn )

where Fn is the nth Fibonacci number dened by F0 = 0, F1 = 1, and for n 2, Fn = Fn1 + Fn2 . Solution proposed by G.R.A.20 Math Problems Group, Roma, Italy. By the Power Mean Inequality we have that
2 2 2 + . . . + Fn + F2 F1 n 1/2

8 8 8 F1 + F2 + . . . + Fn n

1/ 8

Therefore
2 2 2 4 8 8 8 (F1 + F2 + . . . + Fn ) n3 (F1 + F2 + . . . + Fn ),

and since the following identity holds


2 2 2 F1 + F2 + . . . + Fn = Fn Fn+1

then
4 4 8 8 8 Fn+1 n3 (F1 + F2 + . . . + Fn ). Fn

The College Mathematics Journal, Vol. 36, May 2005


802. Proposed by William P. Wardlaw, U. S. Naval Academy, Annapolis, MD. Let n and k be xed positive integers and let Mn (Zk ) be the algebra of all n n matrices over the ring Zk of integers modulo k . Randomly choose matrices A, B Mn (Zk ). What is the probability that AB = I ? Solution proposed by G.R.A.20 Math Problems Group, Roma, Italy. The required probability is the product of the probability to choose in Mn (Zk ) an invertible matrix A and the probability to choose the corresponding unique inverse B P =
2

u(n, k ) k n2

1 k n2

1 = n2 k

1
p|k j =1

1 pj

where k n = |Mn (Zk )| and u(n, k ) is the number of invertible matrices in Mn (Zk ):
n

u(n, k ) = k

n2 p|k j =1

1 pj

We now show that the formula for u(n, k ) holds (see also problem AMM10767). A matrix A is invertible in Mn (Zk ) i det(A) is invertible in Zk and, by the Chinese Remainder Theorem, this happens i det(A) 0 mod p for each prime p which divides k . The construction of a generic invertible matrix in Mn (Zp ) when p is prime goes as follows. We can select the rst row in pn 1 ways. The j th row have to be linearly independent with respect to the rst j 1 rows, therefore it can be selected in a number of ways given by pn , the number of vectors in (Zp )n , minus pj 1 , the number of linear combinations of the rst j 1 rows. Hence
n n

u(n, p) =
j =1

p p

j 1

=p

n2 j =1

1 pj

In order to determine a generic invertible matrix in Mn (Zp ) we can modify each entry of an invertible matrix in Mn (Zp ) in p1 ways. Therefore u(n, p ) = p1
n2

u(n, p).

t Finally for k = T t=1 pt , a generic invertible matrix in Mn (Zk ) can be obtained by choosing an invertible matrix in Mn (Zp t ) for each prime pt and solving the system t of congruences. Therefore, by the Chinese Remainder Theorem, T T
t u(n, p t ) t 1 p t

u(n, k ) =
t=1 T

=
t=1

n2

u(n, pt )
T n

=
t=1

n2 t 1 p t

2 pn t

j =1

1 1 j pt

=k

n2 t=1 j =1

1 pj t

The College Mathematics Journal, Vol. 36, March 2005


800. Proposed by Hojoo Lee, Seoul National University, Seoul, Korea. Let f (x) be a real cubic polynomial with positive leading coecient such that f () f () 2f () for some > 0. Suppose that both f (0) and f () are non-negative. Show that f (x) 0 for all x 0. Solution proposed by G.R.A.20 Math Problems Group, Roma, Italy. Let f (x) = ax3 + bx2 + cx + d with a > 0. Since f () f () then a3 + b2 + c + d (3a2 + 2b + c) that is d 2a3 + b2 and this implies that ( is positive) b Since f () 2f () then (3a2 + 2b + c) 2(a3 + b2 + c + d) that is a3 c + 2d and this implies that ( is positive) c a2 2d . d 2a. 2

Now, for x 0, using the two lower bounds established above, we have that f (x) = ax3 + bx2 + cx + d ax3 + d 2d 2a x2 + a2 x+d 2 x 2 x ax x2 2x + 2 + d 2 +1 2 x = ax (x )2 + d 1 0.

where the last inequality holds because a > 0 and d = f (0) 0.

The College Mathematics Journal, Vol. 36, March 2005


797. Proposed by Jos e Luis D az-Barrero, Universitat Polit` ecnica de Catalunya, Barcelona, Spain. Let n be a positive integer. Prove that
n

1 (n k ) cos 4n
2

k=1

4n

1 k cos 4n
2

k=1

Solution proposed by G.R.A.20 Math Problems Group, Roma, Italy. Consider the function f (x) = 1/ cos2 x. Since it is increasing in the interval [0, /4] then
n1 /4 n1

f (xk )(xk+1 xk )
k=0 0

f (x) dx
k=0

f (xk+1 )(xk+1 xk )

where xk = (k/n)(/4) for k = 1, . . . , n. Therefore


n1

1 cos k 4n
2

k=0

4 [tan x]/ 0 4n

n1

1 cos (k + 1) 4n
2

k=0

, 4n

that is 4n
n

1 cos (n k ) 4n
2

k=1

1 4n

1 cos k 4n
2

k=1

which is equivalent to the required inequality.

The College Mathematics Journal, Vol. 36, March 2005


796. Proposed by Andrew Cusamano, Great Neck, NY. Let the Fibonacci sequence be dened as f1 = 1, f2 = 1, fn = fn1 +fn2 , n 3. 1+ 5 Let = 2 . (a) Show that (1)k+1 1+ fk fk+1 k=1 converges to . (b) Let N Z+ . Express each ak in terms of Fibonacci numbers so that the series ak 1+ (fk fk+1 )N k=1 converges to N . Solution proposed by G.R.A.20 Math Problems Group, Roma, Italy.
2 (a) By the Cassinis identity (1)k+1 = fk +1 + fk+2 fk , we have that

1+
k=1

2 fk fk+1 fk+2 (1)k+1 +1 + fk+2 fk = 1+ =1+ + fk fk+1 f f fk+1 k fk+1 k k=1 k=1

= =

lim 1 + 1 + 2 2 +

3 fn+1 fn+2 + + 2 fn fn+1

fn+2 = . n fn+1 lim

2N N N (b) Let ak = fk +1 + fk+2 fk for k 1. Hence we nd out that

1+
k=1

ak = 1+ (fk fk+1 )N k=1

fk+1 fk

fk+2 fk+1

= lim

fn+2 fn+1

= N .

The College Mathematics Journal, Vol. 35, November 2004


792. Proposed by Jos e Luis D az-Barrero, Universitat Polit` ecnica de Catalunya, Barcelona, Spain. Suppose that x1 , x2 , . . . , xn and are positive real numbers. Prove that 1 n
n n 2 1/n xk k=1

(xk + )
k=1

k 2

(n+1)/2

Solution proposed by G.R.A.20 Math Problems Group, Roma, Italy. Squaring each member, the inequality becomes 1 n
n

x2 k +
k=1

2 n

xk k +
k=1

1 n

2k
k=1 k=1

xk

2/n

+ 2(n+1)/2
k=1

xk

1/n

+ n+1 .

It holds because by AGM inequality we have that 1 n 2 n


n n n 1/n n

x2 k
k=1

k=1 n

x2 k
1/n

=
k=1

xk ,
n

2/n

xk 2
k=1 k=1

xk

= 2
1/n

(n+1)/2 k=1

xk ,

1/n

1 n

k=1

2k

k=1

2k

= n+1 .

Mathematics Magazine, Vol.77, December 2004


1709.Proposed by Mih aly Bencze, Sa cele-N egyfalu, Romania. Let x1 , x2 , . . . , x3n 0. Prove that 1 + x2 k 2 1 + x k k=1
n 3n 3n n 1/n xk k=1

1+

Solution proposed by G.R.A.20 Math Problems Group, Roma, Italy. First we prove the two following facts: 1. Let x, y, z 0 then 1 + x2 1 + y 2 1 + z 2 1 + xyz . 1+x 1+y 1+z 2 The above inequality is equivalent to (1+x2 )(y z )2 +(1+y 2 )(z x)2 +(1+z 2 )(xy )2 +(xyz 1)(x1)(y 1)(z 1) 0, and therefore it holds trivially when 0 x, y, z 1 and when x, y, z 1. By symmetry we have two remaining cases to check: x 1 y z and x y 1 z . Letting f (x) = (1 + x2 )/(1 + x) then xf (1/x) = f (x) and in the rst case: f (x)f (y )f (z ) = xf (1/x)f (y )f (z ) x 1 + yz/x x + yz 1 + xyz = 2 2 2

where the last inequality holds because (x 1)(1 yz ) 0. On the other hand, in the second case: f (x)f (y )f (z ) = xf (1/x)yf (1/y )f (z ) xy 1 + z/(xy ) xy + z 1 + xyz = 2 2 2

where the last inequality holds because (z 1)(1 xy ) 0. 2. Let a0 , a1 , . . . , an1 0 then
n1

(1 + ai ) 1 +

n1

1/n

ai
i=0

i=0

be the symmetric mean of k th order of the sequence a0 , a1 , . . . , an1 Let k / n k then by the AGM inequality k / n k (n )k/n ,

and therefore
n1 n n

(1 + ai ) =
i=0 k=0

k
k=0 n

n (n )k/n k = 1 +

n1

1/n

n .

1 + (n )1/n

ai
i=0

Now we prove the main inequality using rst 1. and then 2. 1 + x2 k 2 1 + x k k=1
n 3n n1

=
i=0

1 + x2 3i+j 2 1 + x3i+j j =1
n1 3

n1

i=0 1/n n

1+
j =1

x3i+j
3n n 1/n xk k=0

1 +

x3i+j
i=0 j =1

1+

It is interesting to note that the more general inequality 1 + x2 k 2 1 + xk k=1


n an an n 1/n xk k=1

1+

where a is a positive integer and x1 , x2 , . . . , xan 0 holds if and only if a = 1, 2, 3 (for a 4, take xk = 2 and note that 2 5a < 3a + 6a ). This motivates the choice of the number 3 in the statement.

Mathematics Magazine, Vol.77, December 2004


1708.Proposed by Stephen J. Herschkorn, Highland Park, NJ. It is well known that the area of a square is half of the length of its diagonal. Show that if the area of a parallelogram is half the square of one of its diagonals, and if the area and each side have rational measure, then the parallelogram is a square. Solution proposed by G.R.A.20 Math Problems Group, Roma, Italy. We can assume that the sides of the parallelogram have integer lengths a b > 0 and that 0 < /2 is the angle between these sides. Hence
2 2 A = ab sin() and d2 = a + b 2ab cos()

where A is the area and d d+ are the lengths of its two diagonals. Since 2A = 2ab sin() 2ab a2 + b2 a2 + b2 + 2ab cos() = d2 +, if 2A = d2 + then a = b and = /2 that is the parallelogram is a square. Now assume that 2A = d2 then cos() = x sin() where x is the rational 2 2 number (a + b )/2ab 1. We have that 1 = cos2 () + sin2 () = (x sin())2 + sin2 () = x2 2x sin() + 2 sin2 () and 2 sin2 () 2x sin() + (x2 1) = 0. It follows that sin() is equal to one of these numbers 1 x 2 x2 2 and since A is rational then sin() is rational too and 2 x2 = y 2 for some rational number 0 y 1. Therefore c = 2aby is a non-negative integer such that c2 = 2(2ab)2 (a2 + b2 )2 = (2ab)2 (a2 b2 )2 . Moreover (a2 + b2 )2 = (2ab)2 + (a2 b2 )2 and by multiplying these two equations we obtain that X = a2 b2 , Y = c(a2 + b2 ) and Z = 2ab is a non-negative solution of the diophantine equation X 4 + Y 2 = Z 4. By the following lemma, X = 0 or Y = 0. In the rst case a = b, x = 1, y = 1, sin() = 1 which means that the parallelogram is a square. In the second case c = 0, y = 0 and x2 = 2 contradicting the fact that x is rational.

Lemma. There are no positive integers X, Y, Z which solve the diophantine equation X 4 + Y 2 = Z 4. Proof. We will use the method of the innite descent. Let X, Y, Z be positive integers which satisfy the equation and assume that Z is the minimum one. (X 2 , Y, Z 2 ) is a pythagorean triple and we have two cases: i) X 2 = P 2 Q2 , Y = 2P Q and Z 2 = P 2 + Q2 for some integers P > Q > 0. Then (XZ )2 = (P 2 Q2 )(P 2 + Q2 ) = P 4 Q4 that is Q4 + (XZ )2 = P 4 < Z 4 which contradicts the minimality of Z . ii) X 2 = 2P Q, Y = P 2 Q2 and Z 2 = P 2 + Q2 for some integers P > Q > 0. Since 2P Q is a square and P and Q are relatively prime (otherwise X, Y, Z have a common factor contradicting the minimality of Z ) then P = 2U 2 , Q = V 2 and Z 2 = 4U 4 + V 4 . (2U 2 , V 2 , Z ) is a pythagorean triple with 2U 2 = 2ST (Q is odd because P is even) and V 2 = S 2 T 2 for some integers S > T > 0. Since ST is a square and S and T are relatively prime then S = A2 , T = B 2 and V 2 = A4 B 4 that is B 4 + V 2 = A4 < Z 4 which contradicts the minimality of Z .

Mathematics Magazine, Vol.77, December 2004


1706.Proposed by Steve Edwards and James Whitenton, Southern Polytechnic State University, GA. Let 0 < a, b < 1. Evaluate 1 + b2 . 1 + a2 n n= Solution proposed by G.R.A.20 Math Problems Group, Roma, Italy. Let PN =
n=N n=+N +
n

1 + b2 + n = PN PN 2 1+a

where
n=N + PN

=
n=0

1 + b2 = 1 + a2 n

n=N

n=0

1 b2 1 b 2n

n+1

n=N

n=0

1 a2 1 b2 = 1b 1 a2n+1

N +1

1a 1 a2N +1

and
n=N PN = n=1

n n 1 n=N n=N n N 1+ 2 b 1 2 b 1 2 a 1b 1 2 a = . = n n 1 N 2n 1a 1+ 2 a b n=1 1 2 a 1 2 b n=1 1

Hence

N +1 N 1 b2 1 2 a PN = . 1 a2N +1 1 2N b Since 0 < a, b < 1 then log a, log b < 0 and N N elog a/2 1 log a 1 2 a lim PN = lim = . = lim log b/2N N 2 N + N + 1 log b 1 b N + e

Mathematics Magazine, Vol.77, October 2004


1705.Proposed by Michel Bataille, Rouen, France. Let be n a positive integer. Find the minimum value of (a b)2n+1 + (b c)2n+1 + (c a)2n+1 (a b)(b c)(c a) for distinct real numbers a, b, c with bc + ca 1 + ab + c2 . Solution proposed by G.R.A.20 Math Problems Group, Roma, Italy. Well show that the minimum is 4n 1. Letting x = a b, y = b c, z = c a then x = y z and the problem is equivalent to nd the minimum value of F (y, z ) = (y + z )2n+1 z 2n+1 y 2n+1 yz (y + z )

for y = 0, z = 0, y + z = 0 and yz 1. Since yz 1 and F (y, z ) = F (y, z ) we can assume that y, z > 0. First proof.
2n

F (y, z ) = Since yz 1 then


2n

k=1

2n + 1 2nk k1 y z k y+z

k=1

2n + 1 2nk k1 y z = k

k=1 n

2n + 1 (y 2nk z k1 + y k1 z 2nk ) k 2n + 1 (y 2(nk)+1 + z 2(nk)+1 ). k

k=1

Moreover for 1, t is convex for t 0 and y +1 + z +1 y z = y + z y+z y+z y+z y2 + z2 y+z

where in the last step we use the fact that yz 1. Hence


n

F (y, z )
k=1 n

2n + 1 y 2(nk)+1 + z 2(nk)+1 k y+z 2n + 1 k = (22n+1 2)/2 = 4n 1 = F (1, 1).

k=1

Second proof.
2n1

F (y, z ) =
k=1

2n (1)k y 2nk z k k

We have that
2n1

k=1

2n (1)k k

2n1

= 4 1 and
k=1

2n (1)k k

= n(4n 1).

Then by AGM inequality


2n1 1/(4n 1)

F (y, z ) (4n 1)
k=1
n

y 2nk z k

((2kn)(1)k )
1/(4n 1)

(4n 1) y n(4 1) z n(4 4n 1 = F (1, 1)

n 1)

= (4n 1)(yz )n

where in the last step we use the fact that yz 1.

Mathematics Magazine, Vol.77, October 2004


1701.Proposed by Murray Klamkin, University of Alberta, Edmunton, AB. Prove that for all positive numbers a, b, c, d, a4 b + b4 c + c4 d + d4 a abcd(a + b + c + d).

Solution proposed by G.R.A.20 Math Problems Group, Roma, Italy. We will prove the following more general result: let n 2 be an integer then
n n n

an k
k=1

ak+1
k=1

ak

k=1

ak

for all positive numbers a1 , a2 , . . . , an , an+1 = a1 . For n = 2 the inequality is trivial. Assume that n 3 and let 1 xk = ak
1/(n2)

aj
j =k,k+1

for k = 1, . . . , n.

Since the function f (x) = 1/xn2 is convex in (0, +) then n n n an k ak+1


k=1 n n

ak f (xk ) = ak
k=1 n

ak
k=1

k=1

ak
k=1

ak x k k=1 f n ak
k=1

n2 ak 1/(n2) n2 ak /(n 2) =1

k=1 n

aj
k=1 j =k,k+1 n

k=1 n

aj
k=1 j =k,k+1

where in the last step we applied the AGM inequality.

Mathematics Magazine, Vol.77, June 2004


1699.Proposed by Zhang Yun, First Middle School of Jinchung City, Gan Su, China. Let A1 A2 A3 A4 be a nondegenerate tetrahedron, let hk , 1 k 4, be the lenght of the altitude from Ak , and let r be the radius of the inscribed sphere. Prove that h1 h2 h3 h4 16 + + + . h1 + 3r h2 + 3r h3 + 3r h4 + 3r 7 Solution proposed by G.R.A.20 Problems Group, Roma, Italy. Let Sk be the area of the face opposite Ak , and let V be the volume of the tetrahedron. Since 1 1 hk Sk = V = r(S1 + S2 + S3 + S4 ) for 1 k 4 3 3 then
4

k=1

r = 1. hk

Now we consider the function f (x) = 1/(1 + 3x) which satisfy by convexity the following inequality f (x) f So we have that
4

1 4

1 4

+f

1 4

1 x ( , +). 3

f
k=1

r hk

k=1

1 4

1 r hk 4

+f

1 4

= 4f

1 4

16 . 7

Mathematics Magazine, Vol.77, June 2004


1696. Proposed by Albert F. S. Wong, Temasek Polytechnic, Singapore. For which positive integers k does the equation x2k1 + y 2k = z 2k+1 have a solution in positive integers x, y , and z ? Solution proposed by G.R.A.20 Problems Group, Roma, Italy. We will show that for any integer k 1 the equation has a solution in positive integers x, y , and z of this form x = 2 , y = 2 , z = 2 where , , and are positive integers such that (2k 1) = (2k ) = (2k + 1) 1. These conditions are equivalent to nd positive integers , m, and n such that (2k + 1) 1 2 1 = =+ =+m 2k 1 2k 1 . = (2k + 1) 1 = + 1 = + n 2k 2k Therefore we have that m and n satisfy the following diophantine linear equation (2k 1)m (4k )n = 1 which has innite positive solutions for any integer k 1 because gcd(2k 1, 4k ) = 1 and the product of the coecients is negative. Letting m = 2k 1 and n = k 1 we get the simplest solution and we have that = 2k 2 , Thus (2 )2k1 + (2 )2k = 24k
3 2k 2

= 2k 2 k + 24k

= 2k 2 2k + 1.
3 2k 2

= 24k

3 2k 2 +1

= (2 )2k+1 .

Here are the numerical values of , , and for k = 1, 2, 3. k=1 2 1 1 k=2 8 6 5 k=3 18 15 13

Mathematics Magazine, Vol.77, April 2004


1693. Proposed by Erwin Just and Norman Schaumberger, Bronx Community College of the City of New York, Bronx, NY. Let A = (x, y ), B = (x2 , y 2 ) e C = (x3 , y 3 ) be the vertices of a non degenerate triangle. 1) For how many pairs (x, y ) is triangle ABC equilateral? 2) If x or y is rational, can triangle ABC be equilateral? Solution proposed by G.R.A.20 Problems Group, Roma, Italy. The solution we present need two awful computations which we made by using Mathematica. However these algorithms have to be considered exact: they perform a nite number of elementary operations between polynomials with integer coecients. The triangle ABC is equilateral i |AB | = |BC | = |AC | that is (x2 x)2 + (y 2 y )2 = (x3 x2 )2 + (y 3 y 2 )2 = (x3 x)2 + (y 3 y )2 . These equations give the following symmetric polynomial system: F (x, y ) = (x6 + y 6 ) 3(x4 + y 4 ) + 2(x3 + y 3 ) = 0 G(x, y ) = 2(x5 + y 5 ) 3(x4 + y 4 ) + (x2 + y 2 ) = 0 Considering these polynomials with respect to the variable x then they have a common root i the coecent y is a root of the resultant Rx (F, G) which can be calculated using Mathematica: Rx (F, G) = 4y 10 (y 1)10 Q(y ) where P (y ) = 32y 10 + 32y 9 40y 8 64y 7 10y 6 + 26y 5 + 5y 4 76y 3 175y 2 162y 54. Since the triangle is non degenerate then y = 0, 1 and it suces to consider P (y ). We rst note that if there is an equilateral triangle ABC with x or y is rational, by symmetry, we can assume that y is rational. But Q(y ) has no rational roots: one can easily check there is no roots among the nite set of fractions p/q with p a divisor of the last coecient 54 and q a divisor of the rst coecient 32. So the answer to the second question is no. In order to nd the number distinct real roots of P (y ) we generate from P0 (y ) = P (y ) and the derivative P1 (y ) = P (y ) by the Euclidean Algorithm (changing the sign of the remainder) a Sturm sequence of polynomials with decreasing degree (Mathematica si very helpful also for these computations). Then we determine the number of sign changes of these polynomials for y that goes to and +. The dierence of these two numbers is just the number of distinct real roots of P (y ). P0 + + + P1 + P2 + + P3 + P4 + + P5 + P6 + + P7 + P8 P9 + P10 + + 6 4

Hence there are 6 4 = 2 real roots. Here are the approximate values: y1 = 1.48459 and y2 = 0.87791.

Finally, by symmetry, x can assume only these two values and therefore all the possible solutions (x, y ) are: (y1 , y1 ) (y2 , y2 ) (y1 , y2 ) (y2 , y1 ). The rst two pair do not work because when x = y F (x, x) = 2x3 (x 1)2 (2x + 1) = 0 G(x, x) = 2x3 (x 1)2 (2 + x) = 0 which is solved by x = 0 and x = 1 (degenerate triangles). The two remaining symmetric pairs have to be solutions because the resultant Rx (F, G) is zero. So the answer to the rst question is: there are exactly two non degenerate equilateral triangles.

Mathematics Magazine, Vol.77, February 2004


1687. Proposed by Sung Soo Kim, Hanyang University, Ansan Kyunggi, Korea. A two-player game starts with two sticks, one of length n and one of length n + 1, where n is a positive integer. Players alternate turns. A turn consists of breaking a stick into two sticks of positive integer lengths, or removing k sticks of length k for some positive integer k . The player who makes the last move wins. Which player can force a win? Solution proposed by G.R.A.20 Problems Group, Roma, Italy. The second player can always force a win using this strategy: whenever there is a stick of positive even length then divide it in two sticks of odd lengths. Let E (m) and T (m) be respectively the number of sticks of even lengths and the total number of sticks before the m-th move, then in order to prove that the stategy works it suces to show that: a) E (m) 1 when m is odd and E (m) 2 when m is even. b) T (m) is even when m is odd and T (m) is odd when m is even. When the game start E (1) = 1 and T (1) = 2 so a) and b) hold. Now assume that a) and b) are satised up to the m-th move. Here is a table which shows the variations of E and T when a certain move is made Move E T 1 2 3 4 +1 1 +1 0 +1 +1 +1 k 5 k k

where: 1. Divide a positive even stick in two even sticks. 2. Divide a positive even stick in two odd sticks. 3. Divide an odd stick in two sticks one even and the other odd. 4. Remove k sticks of length k with k odd. 5. Remove k sticks of length k with k even. Suppose rst that it is second player turn: m is even and if E (m) is 1 or 2 then he plays 2 and E (m + 1) 1. Otherwise E (m) = 0 and since T (m) = 0 he can play 3 or 4 then E (m +1) 1 in both cases. T (m +1) is even because T (m) is odd and he does not play 5. Finally assume that it is rst player turn: m is odd and if T (m) = 0 then the game is nished and the second player wins. Otherwise, since he can not play 5, then whatever move he chooses E (m + 1) 2 and T (m + 1) is odd.

The College Mathematics Journal, Vol. 35, November 2004


789. Proposed by Zengxiang Tong and Zhen Huang, Westerville, OH. 1. Show the convergence of the sequence given by a1 = 2 2 and an = 2
n+1 2

2n

4n a2 n1

for any n > 1, and nd the limit limn an . 2. Show the convergence of the sequence given by b1 = 2 and bn = 2 2
n

2n1

4n1 b2 n1

for any n > 1, and nd the limit limn bn . Solution proposed by G.R.A.20 Math Problems Group, Roma, Italy. For n 2 let An be the area of the regular 2n -polygon inscribed in the circumference of radius 1. Then An = 2n1 sin(/2n1 ). Morever, for n > 3 we have that cos(/2n2 ) 0 and An = 2n1 = 2n 2 = 2n 2 = 2
n1 2 3 3

1 cos(/2n2 ) 2 1 1 1 sin2 (/2n2 ) 1 (An1 /2n2 )2 4n2 A2 n1 .

2n2

Since a1 = b2 = A3 = 2 2 then an2 = bn1 = An and for n > 3. Therefore in both cases the limit is the area of the circle of radius 1, that is .

The College Mathematics Journal, Vol. 35, September 2004


785. Proposed by Jos e Luis D az-Barrero, Univesitat Polit` ecnica de Catalunya, Barcelona, Spain. Prove that in any acute triangle ABC , with the usual notations, the following inequality holds: a2 b2 c2 9 2r cos(A/2) + cos(B/2) + cos(C/2) 2 b2 + c 2 c 2 + a2 a2 + b 2

(r is the inradius of the triangle). Solution proposed by G.R.A.20 Math Problems Group, Roma, Italy. Let I be the left side of the inequality then by the AGM inequality I3 a2 b2 c2 cos(A/2) cos(B/2) cos(C/2) b 2 + c 2 c 2 + a2 a2 + b 2
1/3

Since the triangle ABC is acute then cos(A) 0 and by Carnots theorem a = b2 + c 2 1 2bc cos(A) + c2 1 cos(A) = sin(A) . 2 cos(A/2)

b2

Hence by repeating the same argument for the other angles we obtain 3 3 I (a b c sin(A) sin(B ) sin(C ))1/3 = 2 2 8 S3 abc
1 /3

=3 2

S (abc)1/3

where S = ab sin(C )/2 = bc sin(A)/2 = ca sin(B )/2 is the area of the triangle. Finally we apply AGM inequality again and we use S = r(a + b + c)/2 I3 2 S 3S 9 3 2 = 2 r. (abc)1/3 a+b+c 2

The College Mathematics Journal, Vol. 35, May 2004


780. Proposed by Jos e Luis D az-Barrero, Univesitat Polit` ecnica de Catalunya, Barcelona, Spain. Let n be a positive integer. Prove that 1 n
n

k=1

1 2 Fk

enFn Fn+1

where Fn is the nth Fibonacci number; that is, F0 = 0, F1 = 1, and for n 2, Fn = Fn1 + Fn2 . Solution proposed by G.R.A.20 Problems Group, Roma, Italy. We present two solutions. In both of them we use the identity
n 2 Fk = Fn Fn+1 . k=1

1. Since 1/x is convex in (0, +) and ln x x 1 for x > 0 1 n


n

k=1

n 1 1 ln =e = n 2 Fk Fn Fn+1 1 2 Fk n k=1

Fn Fn+1 n

e1

Fn Fn+1 n

which is equivalent to the required inequality. 2. By the AGM inequality and since ln x x 1 for x > 0 then 1 n
n

k=1

1 2 Fk

1 = e 2 F k=1 k

n k=1

2) ln(Fk

n 2 k=1 (Fk 1)

= enFn Fn+1 .

The College Mathematics Journal, Vol.35, March 2004


779. Proposed by Kedar Hardikar, San Jose, CA. Let A be a positive denite linear operator on Rn (i.e., Ax x > 0 for all x Rn with x = 0). Evaluate I= R where dx denotes an innitesimal element in Rn (i.e., dx = dx1 dx2 . . . dxn ). Solution proposed by G.R.A.20 Problems Group, Roma, Italy. First we consider the matrix A as the sum of the symmetric matrix As and the antisymmetric Aa where 1 As = (A + At ), 2 1 Aa = (A At ). 2
n

eAxx dx

Since As is symmetric, then there will be an orthogonal matrix U such that As = U t DU , where 1 0 . . . 0 0 2 . . . 0 D=. . . . . . . . . . . . 0 . . . 0 n with k > 0 for k = 1, . . . , n. Since Aa x x = 0 for all x Rn , letting y = U x, we have that Ax x = As x x = U t DU x x = Dy y Therefore
n

and dy = | det(U )|dx = dx.

eAxx dx =
n

eDyy dy = R

e(1 y1 ++n yn ) dy1 dyn k y k

1 2 e(k yk ) d k R k=1 n n = = . det(As ) k k=1 =

The College Mathematics Journal, Vol.35, March 2004


775. Proposed by John H. Webb, University of Cape Town, Rondebosch, South Africa Let a, b and c be the lengths of the sides of a triangle. Show that 13 (a2 + b2 + c2 )(a + b + c) + 4abc 1 < . 3 27 (a + b + c) 2 Solution proposed by G.R.A.20 Problems Group, Roma, Italy. By the triangular inequality, we know that a + b c > 0, b + c a > 0 and a + c b > 0. Now we apply the arithmetic-geometric mean inequality to these three real positive numbers: 0 < (a + b c)(b + c a)(a + c b) Dividing by 2(a + b + c)3 and then adding
1 2

a+b+c 3

we get

1 1 1 (a + b c)(b + c a)(a + c b) 1 < 3 2 54 2 2(a + b + c) 2 and therefore the inequality becomes (a + b + c)3 (a + b c)(b + c a)(a + c b) 1 13 < 3 27 2(a + b + c) 2 that is 13 (a2 + b2 + c2 )(a + b + c) + 4abc 1 < . 3 27 (a + b + c) 2

The College Mathematics Journal, Vol.35, March 2004


771. Proposed by Yi-chuan Pan, Jackson State University, Jackson, MS Show that there are innitely many rational numbers x such that both x2 + 5 and x2 5 shall each be squares of rational numbers. Solution proposed by G.R.A.20 Problems Group, Roma, Italy. The following sequence gives innitely many rational numbers with the required property: x0 = 41/12 and xn = (x4 n1 + 25)/(2xn1 We dene for n 1 4 pn = p4 n1 + 25qn1 qn = 2pn1 qn1 rn1 sn1 2 2 2 rn = 10p2 n1 qn1 + rn1 sn1 2 2 2 sn = 10p2 n1 qn1 rn1 sn1 with p0 = 41, q0 = 12, r0 = 49 and s0 = 31. Now we show by induction that
2 2 rn = p2 n + 5qn 2 2 and s2 n = pn 5qn

x4 n1 25) for n 1.

for n 1.

These equalities are satised for n = 0 and if n 1 then


2 p2 n 5qn = = = = 4 2 2 (p4 n1 + 25qn1 ) 5(2pn1 qn1 rn1 sn1 ) 4 4 4 2 2 100p4 n1 qn1 + (pn1 25qn1 ) 20(pn1 qn1 rn1 sn1 ) 2 2 2 2 2 2 (10p2 n1 qn1 ) + (rn1 sn1 ) 20(pn1 qn1 rn1 sn1 ) 2 2 2 2 (10p2 n1 qn1 rn1 sn1 )

2 if the sign is + and it is equal to s2 which is equal to rn n if the sign is . Hence xn = pn /qn is a rational number and both 2 2 2 2 x2 n + 5 = (pn + 5qn )/qn = (rn /qn ) 2 2 2 2 and x2 n 5 = (pn 5qn )/qn = (sn /qn )

are squares of rational numbers. Since the sequence pn is increasing, in order to prove that the rational numbers xn are all dierent (and therefore innite), it suces to show that pn and qn are relatively prime. Note that pn is odd (because qn is even) and it is not a multiple of 5. Then (pn , qn ) = (pn , 2pn1 qn1 rn1 sn1 ) = 1 if and only if 2 2 (pn , pn1 ) = (p4 n1 + 25qn1 , pn1 ) = (qn1 , pn1 ) = 1 2 4 (pn , qn1 ) = (p4 . n1 + 25qn1 , qn1 ) = (pn1 , qn1 ) = 1 2 2 4 4 4 4 4 4 (pn , rn1 sn1 ) = (pn1 + 25qn1 , pn1 25qn1 ) = (pn1 , qn1 ) = 1 which are all true by the inductive hypothesis.

College Mathematics Journal, Vol.34, January 2004


769. Proposed by Wenchang Chu and Leontina Veliana Di Claudio, Universit` a di Lecce, Lecce, Italy. For a natural number n, show that n := (2k )!(2k )! = 2n . k !(n + k )! k=0
n

Solution proposed by G.R.A.20 Problems Group, Roma, Italy. First Proof: by induction on n. 0 = 0!/0! = 20 and if n1 = 2n1 then n = 2
n1

n (2n)!(2n)! = 2n1 n1 n!(2n)! k=0


n1

n1

(2k )!(2k )! k !(n 1 + k )! k !(n + k )! (2k )!(2k )!

1 (2n 1)! (2n 1)! = 2n = 2n = 2 1 (n 1)! k=0 (n + k ) ( n + k ) (n 1)! n k=0


n

Second Proof: note that


n n n n 2n

Pn :=
k=0

(2k )!(2k )! =
k=1

(2k )
k=1

(2k 1)!
k=0

(2k )!

= 2 n!
k=0

k!

and
n n n n 2n 2n

Qn :=
k=0

k !(n + k )! =
k=0

k!
k=0

(n + k )! =
k=0

k!
k =n

k ! = n!
k=0

k!

therefore n = Pn /Qn = 2n .

Mathematics Magazine, Vol.76, December 2003


1684. Proposed by Ethan S. Brown, Massachusetts Istitute of Technology, Cambridge, MA., and Christopher J. Hillar, University of California, Berkeley, CA. Let S be the set of all words of length n in two letters, say a and b. Dene an equivalence relation on S as follows: given a word w, the reverse of w, the complement of w (that is , change all as to bs and all bs to as) and the reverse of the complement are all equivalent to w. Find the number of equivalence classes of S that do not contain any palindromes. Solution proposed by G.R.A.20 Problems Group, Roma, Italy. Let G be the group transformations on the set of all words S : the identity i, the reverse r, the complement c and the reverse of the complement r c. Note that r r = c c = i and c r = r c. Now [w] = {w, r(w), c(w), r c(w)} is the equivalence class represented by w and its cardinality |[w]| can be equal to 2 or 4 because c(w) = w for all w S . If w P = {w S : r(w) = w}, the set of palindromes, then [w] = {w = r(w), c(w) = r c(w)} P and |[w]| = 2.

If w A = {w S : r c(w) = w}, the set of anti-palindromes, then [w] = {w = r c(w), r(w) = c(w)} A and |[w]| = 2.

Hence if w S \ (A P ) then [w] S \ (A P ) and |[w]| = 4. The sets P , A and S \ (A P ) give a partition of S therefore the total number of equivalence classes N of S that do not contain any palindromes is equal to the number of equivalence classes in A, that is |A|/2, plus the number of equivalence classes in S \ (A P ), that is (|S | |A| |P |)/4. Since |S | = 2n , |P | = 2n/2 if n is even, (n+1)/2 2 if n is odd and |A| = 2n/2 if n is even, , 0 if n is odd

then we have that N = |A|/2 + (|S | |A| |P |)/4 = 2n2 if n is even, . (2n 2(n+1)/2 )/4 if n is odd

Mathematics Magazine, Vol.76, December 2003


1682. Proposed by Paul Bracken, University of Texas, Edinburg, TX. Let n be a positive integer. Prove that
n

1
n j

j =1

n+1 2n

n1

j =0

2j j+1

Solution proposed by G.R.A.20 Problems Group, Roma, Italy. We will prove the identity by induction on n. If n = 1 then
1

1
1 j

j =1

1+1 =1= 2

j =0

2j . j+1

Now take n 2 and assume that the identity holds for smaller n. Note that for j = 1, . . . , n 1 1
n j +1

1
n j

n j

n j +1

n j +1 n j

n+1 j +1 n j +1

n j

n+1 n j +1 j n1 n j +1 j

n j

n+1 n

1
n1 j

Then summing over j and using the induction hypothesis we get


n1

1
n j +1

n1

+
j =1

1
n j

j =1

n+1 n

n1

1
n1 j

j =1

n n+1 n1 n 2

n2

j =0

2j j+1

that is
n

1
n j

n1

+
j =1

1
n j

=2
j =1

1
n j

j =2

n+1 1 1 = n1 n 2

n2

j =0

2j j+1

and nally
n

1
n j

j =1

n+1 = 2n

n2

j =0

2j n+1 n+1 + = j+1 2n 2n

n1

j =0

2j . j+1

Mathematics Magazine, Vol. 76, December 2003


1681. Proposed by Mihai Manea, Princeton University, NJ. Let p be a prime number. Prove that the polynomial xp1 + 2xp2 + 3xp3 + + (p 1)x + p is irreducible in Z[x]. Solution proposed by G.R.A.20 Problems Group, Roma, Italy. We will use the following variation of Eisensteins criterion: let f (x) = xn + an1 xn1 + a1 x + a0 Z[x] with n 2 and assume that f (x) has no integer roots. If there exists a prime number p such that p divides all ai for i = 0, . . . , n 2 and p2 does not divide a0 , then f (x) is irreducible in Z[x]. Proof: assume that f (x) is the product of two polynomials in Z[x] say bs xs + + b1 x + b0 and cns xns + + c1 x + c0 where 2 s n 2 because f (x) has no integer roots. By hypothesis p divides a0 = b0 c0 but not both of b0 and c0 , so we can suppose that p | b0 and p c0 . Since f is monic then bs = cns = 1 and therefore not all the bj are divisible by p . Let k be the smallest index such that p bk . Note that 1 k n 2 and therefore p divides ak = bk c0 + bk1 c1 + + b0 ck and every term on the right side except bk c0 which yields a contradiction. In our case f (x) = xp1 + 2xp2 + 3xp3 + + (p 1)x + p
p

=
k=1

xk 1 = x1

p k=0

xk 1 p xp+1 1 (p + 1)(x 1) = . x1 (x 1)2

Letting x = y + 1 we obtain a new polynomial (y ) = f (y + 1) = f y k (p + 1)y 1 y2 p + 1 p3 p+1 p+1 = y p1 + (p + 1)y p2 + y + + y+ . p1 3 2


p+1 k=0 p+1 k

(y ) satises the criterions hypothesis just for the prime p and therefore Now f (x 1) is irreducible in Z[x]. f (x) = f

Mathematics Magazine, Vol.76, October 2003


1679. Proposed by Jos e Luis D az-Barreo and Juan Jos e Egozcue, Univesitat Polit` ecnica de Catalunya, Barcelona, Spain. Let fk denote the k th Fibonacci number, that is, f0 = 0, f1 = 1, and fk+2 = fk+1 + fk , k 0. Prove that for any positive integer n,
n

k=1

n n fk f 2n ). ) (2n 1) log(fk log(fk k k k=1

Solution proposed by G.R.A.20 Problems Group, Roma, Italy. The inequality can be written in this way 1 n 2 1
n

k=1

n 1 log(fk ) f2n k

k=1

n fk log(fk ). k

We will use the following two identities that can be easily proved by induction
n

k=1

n k

= 2 1 and
k=1

n fk = f2n . k

Since the function log(x) is concave then 1 2n 1


n

k=1

n log(fk ) log k

1 2n 1

k=1

n fk k

= log

f2n 2n 1

On the other hand, the function log(x) is convex and therefore 1 f2n
n

k=1

n 1 fk log(fk ) = k f2n

k=1

n fk log k 1 f2n
n

1 fk = log f2n 2n 1 .

log Hence 1 n 2 1
n

k=1

n k
n

k=1

n log(fk ) log k

f2n n 2 1

1 f2n

k=1

n fk log(fk ). k

Mathematics Magazine, Vol.76, June 2003


1674. Proposed by H. A. Shah Ali, Teheran, Iran. Given that 0 x1 x2 xn , and xn+1 = x1 , prove that
n

k=1

xk xk+1 0. 1 + xk xk+1

Solution proposed by G.R.A.20 Problems Group, Roma, Italy. The inequality can be written in this way
n1

k=1

xk+1 xk xn x1 . 1 + xk+1 xk 1 + xn x1

Since tan( ) =
n1

tan tan , then 1 + tan tan

tan(arctan xk+1 arctan xk ) tan(arctan xn arctan x1 ),


k=1

and letting k = arctan xk+1 arctan xk 0 the inequality becomes


n1 n1

tan k tan
k=1 with 1 + + n1 < 2 . Now If n = 2 then, since 0 1 < 2 k=1

we prove it by induction on n 2. 2 , we have that 2 0 tan 1 tan 2 < tan( 2 ) tan 2 = 1 2 tan 1 + tan 2 tan 1 + tan 2 . 1 tan 1 tan 2
n2

and therefore tan(1 + 2 ) = Finally, for n > 2


n1 n2

tan k =
k=1 k=1

tan k + tan n1 tan


n2 k=1 n1

k k
k=1

+ tan n1 .

tan
k=1

k + n1

tan

Mathematics Magazine, Vol.76, February 2003


1662. Proposed by Erwin Just (Emeritus) and Norman Schaumberger (Emeritus), Bronx Community College, Bronx, NY. Let xk , 1 k n, be positive real numbers with 2 Prove that n k=1 (2k 1)xk n .
n k=1 k 1 x2 n. k

Solution proposed by G.R.A.20 Problems Group, Roma, Italy. The function f (x) = x2k1 is convex for k 1 and x > 0 and therefore its graphic stays above the line y = (2k 1)(x 1) + 1 which is tangent to f at the point (1, 1): (2k 1)(x 1) + 1 x2k1 x > 0.

Now letting x = xk and summing for k = 1, . . . , n, we obtain


n n

[(2k 1)(xk 1) + 1]
k=1 k=1

k 1 x2 k

that is

n k 1 x2 k

(2k 1)xk
k=1 k=1 n

+
k=1

(2k 1)
k=1 n

1.

Since n=

1 and n =
k=1 k=1

(2k 1)

then the inequality becomes


n n

(2k 1)xk
k=1 k=1

2k 1 xk + n2 n.

By hypothesis

n k=1 n

k 1 n and therefore x2 k n

(2k 1)xk
k=1 k=1

2k 1 + n2 n n2 . xk

College Mathematics Journal, Vol.34, November 2003


761. Proposed by Charles L. Cooper and Michael Scott McClendon, University of Central Oklahoma, Edmond, OK. Let C (a, b) denote the circle centered at point a that passes through point b, and let D(a, b) denote the closed disk with boundary C (a, b). Suppose p, q , and w are three distinct points and that x is a point on the line segment connecting p and q . (a) Must it be the case that C (x, w) is contained in D(p, w) D(q, w)? (b) Must it be the case that D(x, w) is contained in D(p, w) D(q, w)? Solution proposed by G.R.A.20 Problems Group, Roma, Italy. We will show that the following inclusion holds D(x, w) D(p, w) D(q, w), therefore the answer is always yes. Lets consider our problem in the complex plane C. We can assume without lack of generality that p, q and x are on the real axis with p x q . Now it suces to take z D(x, w) and prove that z D(p, w) D(q, w). The point z belongs to D(x, w) if and only if |z x|2 |w x|2 : |z |2 2Re( x z ) + |x|2 |w|2 2Re( x w) + |x|2 that is |z |2 |w|2 + 2x Re(z w). We distinguish two cases. If Re(z w) < 0 then, since x p, |z |2 |w|2 + 2x Re(z w) |w|2 + 2p Re(z w) and this means that z D(p, w). On the other hand, if Re(z w) 0 then, since x q , |z |2 |w|2 + 2x Re(z w) |w|2 + 2q Re(z w) that is z D(q, w).

College Mathematics Journal, Vol.34, September 2003


757. Proposed by W. V. Grounds, SUNY New Paltz, New Paltz, NY. For x and y integers, let, f (x, y ) = 4x4 + y 4 and g (x, y ) = 4x4 y 4

Find all primes assumed by either f (x, y ) or g (x, y ). Solution proposed by G.R.A.20 Problems Group, Roma, Italy. The polynomial f (x, y ) can be factored over Z in this way: f (x, y ) = 4x4 + y 4 = (2x2 + 2xy + y 2 )(2x2 2xy + y 2 ) If f (x, y ) is a prime number then the smaller factor 2x2 2xy + y 2 is necessarly equal to 1 2x2 2xy + y 2 = x2 + (x y )2 = 1 and, since x and y are positive integers, this equation is solved only for x = 1 and x y = 0, that is x = 1 and y = 1. Therefore the polynomial f (x, y ) can assume only one prime number: f (1, 1) = 5. The polynomial g (x, y ) can be factored over Z in this way: g (x, y ) = 4x4 y 4 = (2x2 + y 2 )(2x2 y 2 ) If g (x, y ) is a prime number then the smaller factor 2x2 y 2 is necessarly equal to 1. Hence y 2 = 2x2 1 and g (x, y ) = (2x2 + y 2 )(2x2 y 2 ) = 4x2 1 = (2x + 1)(2x 1). Again, assuming that g (x, y ) is prime, we have that the smaller factor 2x 1 is equal to 1, that is x = 1 and y = 1. Therefore the polynomial g (x, y ) can assume only one prime number: g (1, 1) = 3.

College Mathematics Journal, Vol.34, September 2003


756. Proposed by Michel Bataille, Rouen, France Prove that
6

sin sin

k=1

k 7 (3k+7) 21

= 0.

Solution proposed by G.R.A.20 Problems Group, Roma, Italy. The sum can be written in this way
6

k=1

sin( k ) 7 = k sin( 7 + ) 3

6 1 k=1 2

sin( k ) 7 sin( k )+ 7
3 2

cos( k ) 7

=2
k=1

) tan( k 7 . k tan( 7 ) + 3

Since tan( x) = tan(x), then tan


3

(7k) 7

= tan( k ) and 7
3

2
k=1

) tan( k ) tan( k 7 7 + k k tan( 7 ) + 3 tan( 7 ) 3

=4
k=1

) tan2 ( k 7 2 k tan ( 7 ) 3

Letting tk = tan2 ( k ), the sum becomes 7


3

4
k=1

tk 9(t1 + t2 + t3 ) 6(t1 t2 + t1 t3 + t2 t3 ) + 3(t1 t2 t3 ) =4 . tk 3 (t1 3)(t2 3)(t3 3) tan6 21 tan4 + 35 tan2 7 . 7 tan6 35 tan4 + 21 tan2 1 for k = 1 . . . 6, then t1 , t2 and t3 are the roots

By De Moivres formula, it is easy to show that tan(7) = tan Since tan(7) = 0 when = of the polynomial Therefore t1 + t2 + t3 = 21 , t1 t2 + t1 t3 + t2 t3 = 35 , t1 t2 t3 = 7 and
6

k 7

x3 21x2 + 35x 7.

sin sin

k=1

k 7 (3k+7) 21

=4

9(21) 6(35) + 3(7) = 0. (t1 3)(t2 3)(t3 3)

Junior problems

J163. Let a, b, c be nonzero real numbers such that ab + bc + ca 0. Prove that bc ca 1 ab + + . a2 + b2 b2 + c2 c2 + a2 2 Proposed by Titu Andreescu, University of Texas at Dallas, USA

Solution by Ercole Suppa, Teramo, Italy We have ab = a2 + b2


cyc

cyc

cyc

ab 1 + a2 + b2 2 b)2

3 = 2

cyc

(a + b)2 3 2 (a2 + b2 ) 2

2 a2 + b2 + c2 + 2(ab + bc + ca) 3 (a + 3 = 2 (a2 + b2 + c2 ) 2 2 (a2 + b2 + c2 ) 2 ab + bc + ca 3 ab + bc + ca 1 1 = 2 2 2 2 2 2 a +b +c 2 a +b +c 2 2

=1+

where in the last step we have used the fact that ab + bc + ca 0. Also solved by Arkady Alt, San Jose, California, USA; Daniel Lasaosa, Universidad P ublica de Navarra, Spain; Perfetti Paolo, Dipartimento di Matematica, Universit` a degli studi di Tor Vergata Roma, Italy; Prithwijit De, HBCSE, India; Andrea Ligori, Universit` a di Roma Tor Vergata, Italy; Piriyathumwong P., Bangkok, Thailand.

Mathematical Reflections 4 (2010)

J164. If x and y are positive real numbers such that x + minimum possible value of x + y.

x2 + 1

y+

y 2 + 1 = 2011, nd the

Proposed by Neculai Stanciu, George Emil Palade, Buzau, Romania


2010 . First solution by Michel Bataille, France The required minimum value is 2011 Write x = sinh(a) and y = sinh(b) where a = ln(x + x2 + 1) > 0 and b = ln(y + y 2 + 1) > 0. From the hypothesis, we have a + b = ln(2011) and using a known formula,

x + y = sinh(a) + sinh(b) = 2 sinh

a+b 2

cosh

ab 2

2 sinh

a+b 2

= 2 sinh(ln( 2011)

where the inequality follows t) 1 for all t and sinh(u) > 0 for u > 0. from cosh( 1 2010 Since 2 sinh(ln( 2011) = 2011 2011 = , we obtain 2011 2010 x+y . 2011 Clearly equality holds when a = b (since cosh(0) = 1), that is, when x = y . The result follows. Second solution by the authors Let z = x + x2 + 1. We have z > 0 and (1) x = 2 z 2 we get (2) y = 2011 22011z . From (1) and (2), x+y =

z 2 1 2z .

From hypothesis y +

y2 + 1 =

2011 z ,

z 2 1 20112 z 2 2010 + = 2z 2 2011 z 2 2011

z+

2011 z

2010 2011

2011 . z

2 The equality occurs for z 2011 z or equivalently z = 2011. Then from (1) and

(2) we obtain 2010 1005 x=y= = . 2 2011 2011


2010 . 2011

So min(x + y ) =

Also solved by Arkady Alt, San Jose, California, USA; Francisco Javier Garcia Capitan, Spain; Ercole Suppa, Teramo, Italy; Daniel Lasaosa, Universidad P ublica de Navarra, Spain; Perfetti Paolo, Dipartimento di Matematica, Universit` a degli studi di Tor Vergata Roma, Italy.

Mathematical Reflections 4 (2010)

J165. Find all triples (x, y, z ) of integers satisfying the system of equations x2 + 1 y2 + 1 +
z2 10

= 2010

(x + y )(xy 1) + 14z = 1985. Proposed by Titu Andreescu, University of Texas at Dallas, USA

Solution by Arkady Alt, San Jose, California, USA z2 Note that z = 10k for some integer k because = 2010 x2 + 1 10 p = x + y and q = xy 1. Then x2 + 1

y 2 + 1 is an integer. Let

y 2 + 1 = x2 y 2 + x2 + y 2 + 1 = (xy 1)2 + (x + y )2 = p2 + q 2

and the system becomes p2 + q 2 + 10k 2 = 2010 pq + 140k = 1985 p2 + q 2 = 2010 10k 2 pq = 1985 140k (1)

Since (p q )2 = 2010 10k 2 2 (1985 140k ) = 10 (k 14)2 then only k = 14 can provide p2 + q 2 = 50 p = q = 5. solvability to (1). And for k = 14, (1) becomes pq = 25 Hence, x+y =5 xy = 4 x=4 or y=1 x=1 y=4 and triples (5, 1, 140) , (1, 5, 140) are all

integer solutions of the original system in integers. Also solved by Daniel Lasaosa, Universidad P ublica de Navarra, Spain; Piriyathumwong P., Bangkok, Thailand.

Mathematical Reflections 4 (2010)

J166. Let P be a point inside triangle ABC and let da , db , dc be the distances from point P to the sides of the triangle. Prove that K s da db dc Rr where K is the area of the pedal triangle of P and s, R, r are the semiperimeter, circumradius, and inradius of triangle ABC. Proposed by Andrei Razvan Baleanu, George Cosbuc, Motru, Romania Remark: The problem contains a typ and the inequality that needs to be proven is s K . da db dc 2Rr Many readers have solved the correct inequality. First solution by Daniel Lasaosa, Universidad P ublica de Navarra, Spain The proposed inequality is not true, since if P = I is the incenter of equilateral triangle ABC , then K = rs 4 is one quarter the area of ABC , while da = db = dc = r, and the proposed inequality would be equivalent to R 4r = 2R, absurd. We show that the correct inequality that always holds is 2K s = . da db dc Rr Now, denoting by PA , PB , PC the respective projections of P on sides BC, CA, AB , we have CPA P = CPB P = 90 , or PA P PB = 180 C , and the area of P PA PB is da db sin C da db dc c = . 2 4R dc Adding the analogous expressions for the areas of P PB PC and P PC PA , we nd K= da db dc 4R a c b + + da db dc ,

or the proposed inequality is equivalent to a b c 2s + + . da db dc r Now,


1 x

is convex for positive x because

d2 dx2

1 x

2 x3

> 0, or by Jensens inequality,

a b c a+b+c 4s2 2s + + (a + b + c) = = , da db dc ada + bdb + cdc S r where 2S = ada + bdb + cdc = 2rs is twice the area of ABC because ada is twice the area of BP C , and similarly for its cyclic permutations, and equality is reached i da = db = dc , ie i 2 2s B +sin C ) P is the incenter of ABC , in which case we easily nd K = r (sin A+sin =r 2 2R .

Mathematical Reflections 4 (2010)

Second solution by G.R.A.20 Math Problems Group, Roma, Italy Since 4KR = cda db + adb dc + bdc da it follows that the inequality becomes b a c + + da db dc which holds by Cauchy-Schwarz. Also solved by Arkady Alt, San Jose, California, USA; Michel Bataille, France; Ercole Suppa, Teramo, Italy. (ada + bdb + cdc ) (a + b + c)2 and 2sr = ada + bdb + cdc ,

Mathematical Reflections 4 (2010)

J167. Let a, b, c be real numbers greater than 1 such that b+c c+a a+b + 2 + 2 1. 2 a 1 b 1 c 1 Prove that bc + 1 a2 1
2

ca + 1 b2 1

ab + 1 c2 1

10 . 3

Proposed by Titu Andreescu, University of Texas at Dallas, USA First solution by Prithwijit De, HBCSE, India Observe that bc + 1 a2 1 ca + 1 b2 1 ab + 1 c2 1 Therefore bc + 1 a2 1
2 2 2

b+c a2 1 c+a b2 1 a+b c2 1


2

(b2 1)(c2 1) ; (a2 1)2 (c2 1)(a2 1) ; = (b2 1)2 (a2 1)(b2 1) = . (c2 1)2 =

b+c a2 1

+ b+c a2 1 3

(b2 1)(c2 1) .. . . (1) (a2 1)2


2

Now observe that

b+c 2 a2 1 and by A.M-G.M inequality we get

1 . . . (2) 3

2 2 2 2 2 2 (b2 1)(c2 1) 3 (a 1) (b 1) (c 1) 3 = 3. . . . (3) (a2 1)2 (a2 1)2 (b2 1)2 (c2 1)2

By virtue of (1),(2) and (3) we obtain

bc + 1 a2 1

3+

1 10 = . 3 3

Second solution by Perfetti Paolo, Dipartimento di Matematica, Universit` a degli studi di Tor Vergata Roma, Italy We observe that bc + 1 = b + c + (b 1)(c 1) and then (bc + 1)2 = (b + c)2 + 2(b + c)(b 1)(c 1) + (b 1)2 (c 1)2 . By powermeansinequality and the constraint on a, b, c we have (b + c)2 1 2 2 (a 1) 3 b+c a2 1
2

cyc

cyc

1 3

thus the inequality becomes 2


cyc

(b + c)(b 1)(c 1) (b 1)2 (c 1)2 + (a2 1)2 (a2 1)2

Mathematical Reflections 4 (2010)

or
cyc

(b 1)(c 1)(bc + b + c + 1) 3 (a2 1)2

Since the inequality in the statement is symmetric, we can set a b c. Then we observe that (b 1)(c 1) (c 1)(a 1) (a 1)(b 1) , , (a 1)2 (b 1)2 (c 1)2 and bc + b + c + 1 ca + c + a + 1 ab + a + b + 1 , , (a + 1)2 (b + 1)2 (c + 1)2

are equally sorted. This allows us to employ Chebyshevinequality 1 (b 1)(c 1)(bc + b + c + 1) 2 2 (a 1) 3 (b 1)(c 1) (a 1)2 bc + b + c + 1 (a + 1)2

cyc

cyc

cyc

Moreover by AGM we have


cyc

(b 1)(c 1) 3 (a 1)2 (b + 1)(c + 1) 3 (a + 1)2

and
cyc

bc + b + c + 1 = (a + 1)2

cyc

Bringing together the last three inequalities we obtain (b 1)(c 1)(bc + b + c + 1) 1 33=3 2 2 (a 1) 3

cyc

and we are done. Also solved by Daniel Lasaosa, Universidad P ublica de Navarra, Spain; Prithwijit De, HBCSE, India.

Mathematical Reflections 4 (2010)

J168. Let n be a positive integer. Find the least positive integer a such that the system x1 + x2 + + xn = a 2 2 x2 1 + x2 + + xn = a has no integer solutions. Proposed by Dorin Andrica, Babe s-Bolyai University, Cluj-Napoca, Romania

Solution by Lorenzo Pascali, Universita di Roma La Sapienza, Italy First, we notice that if xi = 0, 1 for an integer component xi then x2 i > xi and we have a contradiction 2 2 a = x2 1 + x2 + + xn > x1 + x2 + + xn = a. Hence any component xi is 0 or 1 and the system has integer solutions for a = 1, . . . , n: take x1 = = xa = 1 and xa+1 = = xn = 0. Therefore the least positive integer a such that the system has no integer solutions is n + 1:
2 2 x1 + x2 + + xn x2 1 + x2 + + xn n < a = n + 1 .

Also solved by Arkady Alt, San Jose, California, USA; Daniel Lasaosa, Universidad P ublica de Navarra, Spain; Ercole Suppa, Teramo, Italy.

Mathematical Reflections 4 (2010)

Senior problems S163. (a) Prove that for each positive integer n there is a unique positive integer an such that (1 + 5)n = an + an + 4n . (b) When n is even, prove that an is divisible by 5 4n1 and nd the quotient. Proposed by Dorin Andrica, Babe s-Bolyai University, Cluj-Napoca, Romania

First solution by G. C. Greubel, Newport News, VA Let 2 = 1 + 5. With this we have 2n n = an + an + 4n . Squaring both sides leads to 4n (2n 1) = 2an + 2 an (an + 4n ).

(1)

(2)

Subtracting 2an from both sides and squaring the resulting value leads to [4n (2n 1) 2an ]2 = 4an (an + 4n ). This is reduced to an = 4n1 2n 1 n
2

(3)

= 4n1 (n (1)n n )2 = 4n1 2n + 2n 2 = 4n1 (L2n 2) (4)

where Lm is the mth Lucas number. Hence it has been shown that an is a positive integer and is given by an = 4n1 (L2n 2).

B) If n is an even value, say n = 2m, then a2m = 42m1 (L4m 2)


2 = 42m1 5F2 n

= 5 4m1 (2m F2m )2 .

(5)

From this relation it is shown that a2m is divisible by 5 4m1 and has the quotient value (2m F2m )2 .
Mathematical Reflections 4 (2010)

Second solution by the authors (a) Let (1 + 5)n = xn + yn 5, where xn , yn are positive integers, n = 1, 2, . . . Then (1 5)n = xn yn 5, n = 1, 2, . . . , hence
2 n x2 n 5yn = (4) , n = 1, 2, . . . n If n is even, consider an = x2 n 4 and we have n an + an + 4n = x2 x2 n4 + n = n = yn 5 + xn = (1 + 5) . 2 4n and we have If n is odd, consider an = 5yn 2 4n + 2 = an + an + 4n = 5yn 5yn = xn + yn 5 = (1 + 5)n . n 2 (b) If n is even, then we have an = x2 n 4 = 5yn , where

(1)

2 + 5yn

x2 n

x2 n+

2 5yn

1 yn = [(1 + 5)n (1 5)n ] 2 5 2n = 2 5 1+ 5 2


n

1 5 2

= 2n1 Fn ,

2 , hence 5 4n1 |a where Fn is the nth Fibonacci number. In this case we get an = 5 4n1 Fn n 2. and the quotient is Fn

Also solved by Arkady Alt, San Jose, California, USA; Daniel Lasaosa, Universidad P ublica de Navarra, Spain.

Mathematical Reflections 4 (2010)

10

S164. Let ABCD be a cyclic quadrilateral whose diagonals are perpendicular to each other. For a point P on its circumscribed circle denote by P the line tangent to the circle at P. Let U = A B , V = B C , W = C D , K = D A . Prove that U V W K is a cyclic quadrilateral. Proposed by Ivan Borsenco, Massachusetts Institute of Technology, USA

Solution by Michel Bataille, France Let U1 , V1 , W1 , K1 be the midpoints of AB, BC, CD, DA, respectively. The Varignon parallelogram U1 V1 W1 K1 of the quadrilateral ABCD is a rectangle (because AC BD), hence U1 , V1 , W1 , K1 lie on a circle centered at the centre of the rectangle. Note that O, U1 , U are collinear (on the perpendicular bisector of AB ) and that AB is the polar of U with respect to . Similar results hold for V1 , W1 , K1 and it follows that the inverses of U1 , V1 , W1 , K1 in the circle are U, V, W, K , respectively, so that U, V, W, K all lie on the inverse of the circle . Since U, V, W, K clearly cannot be collinear, the inverse of is a circle and so U V W K is a cyclic quadrilateral.

A U U1 K1

B V1 V C

D W 1

Also solved by Ercole Suppa, Teramo, Italy; Daniel Lasaosa, Universidad P ublica de Navarra, Spain; Prithwijit De, HBCSE, India.

Mathematical Reflections 4 (2010)

11

S165. Let I be the incenter of triangle ABC. Prove that AI BI CI 8r3 , where r is the inradius of triangle ABC. Proposed by Dorin Andrica, Babe s-Bolyai University, Cluj-Napoca, Romania

Solution by Piriyathumwong P.,Bangkok, Thailand Since r r r , BI = , CI = , AI = A B sin 2 sin 2 sin C 2


B C the inequality above is equivalent to sin A 2 sin 2 sin 2 of the two well-known facts below: 1 8

,which is immediately true because

r A B C = 4 sin sin sin , R 2r R 2 2 2 ,where R is the circumradius of triangle ABC .

Also solved by Arkady Alt, San Jose, California, USA; Michel Bataille, France; Scott H. Brown, Auburn University Montgomery, USA; Ercole Suppa, Teramo, Italy; Daniel Lasaosa, Universidad P ublica de Navarra, Spain; G.R.A.20 Math Problems Group, Roma, Italy.

Mathematical Reflections 4 (2010)

12

S166. If a1 , a2 , . . . , ak (0, 1), and k, n are integers such that k > n 1, prove that the following inequality holds min{a1 (1 a2 )n , a2 (1 a3 )n , . . . , ak (1 a1 )n } nn . (n + 1)n+1

Proposed by Marin Bancos, North University of Baia Mare, Romania First solution by Arkady Alt, San Jose, California, USA Let M = min {a1 (1 a2 )n , a2 (1 a3 )n , . . . , ak (1 a1 )n } and for any function f (x, y ) let
k

f (a1 , a2 ) = f (a1 , a2 ) + f (a2 , a3 ) + + f (ak , a1 ) .


cyc

Since for any x, y (0, 1) by the AMGM inequality


n+1

nx (1 y )n

nx + n ny n (x y ) + n = . n+1 n+1

Then
n+1

M = min 1 k =
k

n+1

a1 (1 a2 )n , a1 (1 a2 )n
n+1

n+1

a2 (1 a3 )n , . . . ,

n+1

ak (1 a1 )n

n+1

cyc

1 n+1

na1 (1 a2 )n
k

cyc

1 k (n + 1)

n+1

(n (a1 a2 ) + n)
cyc
n+1

nk = k (n + 1) n+1 nn = . (n + 1) Then M nn . (n + 1)n+1

n+1

n = n (n + 1)

Second solution by the author Reductio ad absurdam Lets suppose that the inequality doesnt hold. Therefore a1 (1 a2 )n >
Mathematical Reflections 4 (2010)

nn (n + 1)n+1
13

a2 (1 a3 )n > ..... ak (1 a1 )n > Multiplying these relations up, we get

nn (n + 1)n+1 nn (n + 1)n+1

a1 a2 ... ak (1 a1 )n (1 a2 )n ... (1 ak )n > But, for a (0, 1), we have a(1 a)n Lets prove this inequality. 1 na(1a)n n
AM GM

nn (n + 1)n+1

()

nn (n + 1)n+1

a(1a)n =

1 na+ (1 a) + + (1 a) n n+1
1 n+1

n times

n+1 = 1 n n n+1

n+1

nn (n + 1)n+1

The equality holds for: na = 1 a a =

(0, 1)

Using the proved inequality for a1 , a2 , . . . , ak , we get: a1 (1 a1 )n a2 (1 a2 )n ..... ak (1 ak )n Multiplying these relations up, we get a1 a2 ak (1 a1 )n (1 a2 )n (1 ak )n nn (n + 1)n+1
k

nn (n + 1)n+1 nn (n + 1)n+1 nn (n + 1)n+1

This inequality contradicts (), which follows from the initial assumption. Therefore, that assumption is false. Also solved by Michel Bataille, France; Daniel Lasaosa, Universidad P ublica de Navarra, Spain; Perfetti Paolo, Dipartimento di Matematica, Universit` a degli studi di Tor Vergata Roma, Italy.

Mathematical Reflections 4 (2010)

14

S167. Let Ia be the excenter corresnponding to the side BC of triangle ABC. Denote by A , B , C the tangency points of the excircle of center Ia with the sides BC, CA, AB, respectively. Prove that the circumcircles of triangles AIa A , BIa B , CIa C have a common point, dierent from Ia , situated on the line Ga Ia , where Ga is the centroid of triangle A B C . Proposed by Dorin Andrica, Babe s-Bolyai University, Cluj-Napoca, Romania

First solution by Michel Bataille, France

B B1 A1

A' G1 C 1

C B'

C'

I1

For typographical reasons, Ia and Ga are denoted by I1 and G1 on the gure above. Let be the excircle. Since Ia A = Ia C an BA = BC , the line Ia B is the perpendicular bisector of A C and intersects A C in its midpoint B1 . Since A C is the polar of B with respect to , the inversion in the circle exchanges B1 and B . Since B is invariant under this inversion, the circumcircle of Ia BB inverts into the median B B1 of triangle A B C . Similarly, the circumcircles of Ia AA , Ia CC invert into the medians A A1 , C C1 . As a result, the three circumcircles all pass through Ia and through the inverse of Ga (because Ga lies on the three medians A A1 , B B1 , C C1 ). The second result follows from the fact that the inverse of Ga is on the line through Ia and Ga . Second solution by Daniel Lasaosa, Universidad P ublica de Navarra, Spain Let D be the midpoint of B C . Now AB Ia B , while AIa B C where B D = C D by symmetry around the internal bisector of angle A. Thus, triangles AB D and B Ia D are similar, hence B D C D = B D 2 = Ia D AD , and the power of D with respect to the circumcircles of A B C and AIa A is the same, or D lies on the radical axis of both circles, which is median AD.
Mathematical Reflections 4 (2010)

15

Let E be the midpoint of C A . BA Ia A , while BIa A C where A E = C E symmetry around the external bisector of angle B . Thus, triangles BE A and A E Ia similar, hence A E C E = A E 2 = BE Ia E , and median B E is the radical axis of circumcircles of A B C and BIa B . Similarly, median C F (F is the midpoint of A B ) is radical axis of the circumcircles of A B C and CIa C .

by are the the

Clearly, the point Ga where the medians A D , B E and C F meet, has the same power with respect to the four circumcircles; consider now the second point P where Ia Ga meets the circumcircle of AIa A . Since Ia Ga is the radical axis of the circumcircles of AIa A and BIa B because Ia , Ga have the same power with respect to both, then P also has the same power with respect to both, but since it is on the circumcircle of AIa A , it is also on the circumcircle of BIa B . Similarly, it is also on the circumcircle of CIA C . The conclusion follows.

Mathematical Reflections 4 (2010)

16

S168. Let a0 2 and an+1 = a2 n an + 1, n 0. Prove that loga0 (an 1) loga1 (an 1) logan1 (an 1) nn , for all n 1. Proposed by Titu Andreescu, University of Texas at Dallas, USA

Solution by Perfetti Paolo, Dipartimento di Matematica, Universit` a degli studi di Tor Vergata Roma, Italy Proof Induction loga0 (an+1 1) loga1 (an+1 1) logan1 (an+1 1) logan (an+1 1) (n + 1)n+1 an+1 1 = an (an 1) (an 1)2 This implies loga0 (an+1 1) loga1 (an+1 1) logan1 (an+1 1) logan (an+1 1) 2n loga0 (an 1) loga1 (an 1) logan1 (an 1) logan (an+1 1) 2n nn logan (an+1 1) = 2n nn logan (an (an 1)) = (2n)n (1 + logan (an 1)) (n + 1)n+1 For n 4 we have (2n)n (n + 1)n+1 . Indeed 2n (n + 1)e (n + 1) 1 + 1 n
n

logx (an+1 1) 2 logx (an 1)

(1)

n4

We need to show yet the validity of our inequality for n = 1, 2, 3. For n = 1 the inequality is loga0 (a0 (a0 1)) = 1 + loga0 (a0 1) 1 being a0 1 1. For n = 2 we have loga0 (a2 1) loga1 (a2 1) 4 or loga0 a1 (a1 1) loga1 a1 (a1 1) 4 namely loga0 a1 a0 (a0 1) loga1 a1 a0 (a0 1) 4 We rewrite (2) as 1 + loga0 a1 + loga0 (a0 1)
Mathematical Reflections 4 (2010)

(2)

1 + loga1 a0 + loga1 (a0 1) 4


17

which is implied by, use again a0 1 1, 1 + loga0 a1 1 + loga1 a0 4

and this holds true since it may be written as (1 + x)(1 + 1/x) 4, and x + 1/x 2, x > 0. The last integer still remaining is n = 3 and by (1) we need to show 63 (1 + lna3 (a3 1)) 44 The rst step is: lna3 (a3 1) increases with a3 7. To prove this lets write loga3 (a3 1) = ln(a3 1) so that ln a3 d ln(a3 1) 1 = da3 ln a3 ln a3 1 ln(a3 1) a3 1 a3 ln a3 >0 for a3 7 (3)

The monotonicity of lna3 (a3 1) for a3 7 implies that it is greater than or equal to ln7 6 and this in turn implies that it suces to show 63 (1 + ln7 6) 44 which evidently holds true and we are done. Also solved by Michel Bataille, France; Daniel Lasaosa, Universidad P ublica de Navarra, Spain; Prithwijit De, HBCSE, India; Lorenzo Pascali, Universit` a di Roma La Sapienza, Roma, Italy; Piriyathumwong P., Bangkok, Thailand.

Mathematical Reflections 4 (2010)

18

Undergraduate problems

U163. Find the minimum of f (x, y, z ) = x2 + y 2 + z 2 xy yz zx over all triples (x, y, z ) of positive integers for which 2010 divides f (x, y, z ). Proposed by Titu Andreescu, University of Texas at Dallas, USA

Solution by Daniel Lasaosa, Universidad P ublica de Navarra, Spain If wlog z is odd and x, y are even, then z 2 is the only odd term, f (x, y, z ) is odd, hence not a multiple of 2010, while if wlog x, y are odd and z is even, then x2 , y 2 , xy are the only odd terms, y f (x, y, z ) is again odd. Therefore, x, y, z have the same parity, and we may dene u = x 2 , y z v = 2 , or 3s2 + d2 = 4(u2 + v 2 + uv ) = (x y )2 + (y z )2 + (x y )(y z ) = f (x, y, z ), where s = u + v and d = u v , and if 2010 divides f (x, y, z ), then 4020 = 22 3 5 67 divides f (x, y, z ). Now, any perfect square leaves a remainder equal to 1, 0, 1 modulus 5, hence if d, s are not both multiples of 5, then 3s2 + d2 cannot be a multiple of 5, hence 52 divides s 3s2 + d2 = f (x, y, z ), and 20100 divides f (x, y, z ). Dening s = 5 and d = d 5 , we nd that 3s 2 + d 2 = f (x, y, z ) 20100k = = 804k. 25 25

But taking s = 16, d = 6, we nd 3s 2 + d 2 = 768 + 36 = 804, or f (x, y, z ) 25 804 = 20100, with equality for example for s = 80 and d = 30, ie u = 55 and v = 25, or f (z +160, z +50, z ) = 20100 for all positive integer z as it is easily checked by direct calculation. Note: We have restricted ourselves to positive values of f (x, y, z ), since clearly f (x, x, x) = 0 is a multiple of 2010 for all positive integer x, making the problem trivial.

Mathematical Reflections 4 (2010)

19

U164. Prove that 22010! 1 ends in at least 499 zeros. Proposed by Dorin Andrica, Babe s-Bolyai University, Cluj-Napoca, Romania

Solution by G.R.A.20 Math Problems Group, Roma, Italy We will prove that (22010! 1) ends with 501 zeros by showing that it is divisible by 5501 and 2501 . Since

k=1

2010 = 501 5k 5501 a= 22002

and
k=1

2010 = 2002, 2k

it follows that 2010! = 4 By Eulers Theorem

b for some positive integers a and b.

22010! 1 = 245

501 a

1 = (2a )(5

502

)10

(mod 5502 )

which implies that 5501 divides (22010! 1). Now we show that 2k divides (22 b 1) for all k 1. For k = 1 we have that 22b 1 is odd k1 k1 and 2 divides (22b 1). Moreover, since gcd(22 b 1, 22 b + 1) = 1, (22
k1 k b k

1) = (22

k1 b

1) (22

k1 b

+ 1)
k1 b

and 2k1 divides (22 b 1) by inductive hypothesis and 2 divides (22 k1 22 b + 1 is odd. Hence 22002 divides (22010! 1).

+ 1) because

Also solved by Daniel Lasaosa, Universidad P ublica de Navarra, Spain; Neacsu Adrian, Pitesti, Romania.

Mathematical Reflections 4 (2010)

20

U165. Let G = {A1 , A2 , . . . , Am } Mn (R) such that (G, ) is a group. Prove that T r(A1 + A2 + + Am ) is an integers divisible by m. Proposed by Mihai Piticari, Dragos Voda National College, Campulung Moldovenesc, Romania

Solution by Michel Bataille, France Let B = A1 + A2 + + Am . Since (G, ) is a group, for any xed j {1, 2, . . . , m} the mapping A AAj is a bijection from G onto G. It follows that BAj = B , and since this is true for j = 1, 2, . . . , m, we have B 2 = B (A1 + +A2 + + Am ) = BA1 + BA2 + + BAm = mB. Now, the matrix C =
1 m

B is idempotent since C2 = 1 2 1 B = 2 (mB ) = C, 2 m m

hence T r(C ) = rank(C ). Thus, T r(B ) = m (rank(C )), a multiple of m. Also solved by Moubinool Omarjee, Paris, France; Carlo Pagano, Universit` a di Roma Tor Vergata, Roma, Italy

Mathematical Reflections 4 (2010)

21

U166. Find all functions f : [0, ) [0, ) such that (a) f is multiplicative (b) limx f (x) exists, is nite, and dierent from 0. Proposed by Mihai Piticari, Dragos Voda National College, Campulung Moldovenesc, Romania

First solution by Emanuele Natale, Universit` a di Roma Tor Vergata, Roma, Italy limt f (t) = c = 0 and let a > 0, then c = lim f (at) = lim f (a)f (t) = f (a) lim f (t) = f (a)c
t t t

Let

which implies that f (a) = 1. Hence f is identically equal to 1 in (0, +), whereas f (0) can assume any nonnegative real number. Its trivial to check that such functions verify the assumptions. Second solution by Daniel Lasaosa, Universidad P ublica de Navarra, Spain Since f is multiplicative, for any non-negative real x we have f (x)f (1) = f (x 1) = f (x), f (x) (f (1) 1) = 0.

If f (1) = 1, then f (x) = 0 for all non-negative real x, in contradiction with condition (b), hence f (1) = 1. Since f (xy ) = f (x)f (y ) for all non-negative reals x, y , after trivial induction we nd that, for any positive integer n and any non-negative integer x, we have f (xn ) = (f (x))n . Take any y > 1, and assume that f (y ) = a > 1. Then,
x

lim f (x) = lim f (y n ) = lim an = ,


n n

in contradiction with condition (b). Similarly, if f (y ) = a < 1, we nd limx f (x) = 0, again in contradiction with condition (b), or f (y ) = 1 for all y > 1, and limx f (x) = 1, nite and nonzero.
1 For any 0 < z < 1, y = z > 1, or f (z ) = f (y )f (z ) = f (yz ) = f (1) = 1. We conclude that f (x) = 1 for all positive real x. Now, f (0) = f (02 ) = (f (0))2 , and either f (0) = 0 or f (0) = 1. In the second case, f (x) is trivially multiplicative since f (xy ) = 1 = 12 = f (x)f (y ) for all non-negative reals x, y , while in the rst case, if wlog y = 0, we nd for any non-negative real x that f (x 0) = f (0) = 0 = f (x) 0 = f (x)f (0). There are therefore two functions that satisfy simultaneously both conditions, f (x) = 1 for any non-negative real x, and f (x) = 1 for all positive x and f (0) = 0.

Mathematical Reflections 4 (2010)

22

U167. Let f : [0, 1] R be a continuously dierentiable function such that f (1) = 0. Prove that
1

xf (x)dx
0

1 max |f (x)|. 6 x[0,1]

Proposed by Duong Viet Thong, National Economics University, Ha Noi, Vietnam

Solution by Arkady Alt, San Jose, California, USA Using integration by parts we obtain 1 2
1 2 0 x f 1 0 xf

(x) dx =

x2 f (x) 2

1 0

x2 f (x) dx = 2

(x) dx. Since by condition f (x) is continuously dierentiable then


1 0 xf

M := max |f (x)| and, therefore,


x[0,1]

(x) dx =
1 2 0 x dx

1 2

1 2 0 x f

(x) dx =

1 2

1 2 0 x f

(x) dx

1 2

1 2 0 x |f

(x)| dx

M 2

1 max |f (x)| .. 6 x[0,1]

Also solved by Michel Bataille, France; Daniel Lasaosa, Universidad P ublica de Navarra, Spain; Carlo Pagano, Universit` a di Roma Tor Vergata, Roma, Italy; Perfetti Paolo, Dipartimento di Matematica, Universit` a degli studi di Tor Vergata Roma, Italy.

Mathematical Reflections 4 (2010)

23

U168. Let f : [a, b] R be a twice dierentiable function on (a, b) and let maxx[a,b] |f (x)| = M. Prove that
b

f (x)dx f
a

a+b 2

(b a)

(b a)3 M. 24

Proposed by Duong Viet Thong, National Economics University, Ha Noi, Vietnam

Solution by Michel Bataille, France b b b b Let = a f (x) dx f a+ (b a). From the easily veried result a x a+ dx = 0, we 2 2 deduce b a+b a+b a+b f (x) f = x f dx. 2 2 2 a Now, for any x [a, b], we have f (x) = f a+b 2 + x
a+b 2 ,

a+b 2

a+b 2

1 2

a+b 2

f ()

for some real between x and f (x) f It follows that || M 2


b

hence a+b 2 f a+b 2 M 2 x a+b 2


2

a+b 2

x
a

a+b 2

dx =

M 6

ba 2

ab 2

(b a)3 M. 24

Also solved by Arkady Alt, San Jose, California, USA; Daniel Lasaosa, Universidad P ublica de Navarra, Spain; G.R.A.20 Math Problems Group, Roma, Italy; Perfetti Paolo, Dipartimento di Matematica, Universit` a degli studi di Tor Vergata Roma, Italy.

Mathematical Reflections 4 (2010)

24

Olympiad problems O163. Prove that the equation x3 + y 3 = 2010 xy is not solvable in positive integers. Proposed by Titu Andreescu, University of Texas at Dallas, USA and Dorin Andrica, Babe s-Bolyai University, Cluj-Napoca, Romania First solution by Daniel Lasaosa, Universidad P ublica de Navarra, Spain Assume that x, y have opposite parity, then x3 + y 3 is odd, and 2010 is odd, contradiction, hence x, y have the 3 +y 3 same parity and x y is even. If x, y are both odd, x2 xy + y 2 = xx+ y is odd, hence 2 divides x + y with multiplicity a + 1, where a is the multiplicity with which 2 divides x y . Now, (x + y ) + (x y ) = 2x is divided by 2 with multiplicity 1, hence a = 1, or if x, y are both odd, x y is divisible by 2 but not by 4, and x + y is divisible by 4 but not by 8. If x, y are both even, and both are divided by 2 with dierent multiplicity, then x + y and x y are (x+y ) both divided by the lowest of both multiplicities, hence xyx = 2010 x2 + y 2 is a multiple y of 4, or since x2 , y 2 are also multiples of 4, 2010 is a multiple of 4, absurd. Therefore, x, y are divisible by 2 with the same multiplicity a, and x + y , x y are divisible by 2 with multiplicity at least a + 1, hence x y is divisible by 2 with multiplicity at least 3a 3. It is well known that any perfect cube leaves remainder 1, 0, 1 modulus 9, or since x3 + y 3 is a multiple of 3 because 3 divides 2010, then x3 + y 3 is a multiple of 9, and x y is a multiple of 3 because 2010 is not a multiple of 9. If x, y are not multiples of 3, since x y divides xy (x + y ), then x + y must be a multiple of 3, or 2x = (x + y ) + (x y ) is a multiple of 3, contradiction, hence x, y are multiples of 3, hence x3 + y 3 is a multiple of 27, and x y must be a multiple of 9.
y 2 Since 452 = 2025 > 2010 = xx+ y > x , we have x 44, or since y 1, then x y 43 must be an even multiple of 9, divisible by 2 if x, y are both odd, or divisible by at least 23 = 8 if x, y are both even. We conclude that x y = 18, and x, y are both odd. Writing x as a function of y , the proposed equation becomes
3 3

y3 + y 2 + 18y = 562, 27

z 3 + 9z 2 + 54z = 562,

where we have dened z = y 3 because y is clearly divisible by 3. Note now that the LHS increases strictly with z , and if z = 5 then z 3 + 9z 2 + 54z = 125 + 225 + 270 = 620 > 562, while if z = 4 then z 3 + 9z 2 + 54z = 64 + 144 + 216 = 424 < 562. It follows that no positive integral solutions exist for the proposed equation. Second solution by the authors Assume that the equation is solvable in positive integers. It is clear that x > y . We can write 2010 = x3 + y 3 x3 y 3 > = x2 + xy + y 2 = (x y )2 + 3xy > (x y )2 , xy xy
25

Mathematical Reflections 4 (2010)

and get x y <

2010. It follows x y 44.

On the other hand, we have 2010 = x3 + y 3 3 x3 + y 3 3 x+y 3 x3 + y 3 = ( ) ( ) , xy 44 44 3 44 3

hence we obtain x + y 96. The equation is equivalent to (x + y )(x2 xy + y 2 ) = 2 3 5 67 (x y ). If x + y is divisible by 67, then necessarily x + y = 67, since x + y 96 and 67 is a prime number. In this case we get x2 xy + y 2 = 30(x y ) and x + y = 67, hence (x + y )2 3xy = 30(x y ). That is 672 = 30(x y )+3x(67 x), equation with no integer solutions, since 67 is not divisible by 3. If x2 xy + y 2 is divisible by 67, then x2 xy + y 2 = 67k for some positive integer k . The equation is equivalent to k (x + y ) = 30(x y ), that is (30 k )x = (30 + k )y . It follows y =
30k 30+k x,

hence

x2 [(30 + k )2 (30 k )(30 + k ) + (30 k )2 ] = 67k (30 + k )2 . and we get x2 (3k 2 + 302 ) = 67k (30 + k )2 . It is clear that k is divisible by 3, hence we have k = 3a for some positive integer 1 a 9. Then x2 (3a2 + 100) = 67 a (a + 10)2 . Because x2 cant be divisible by 67 it follows that 3a2 + 100 is divisible by 67. Replacing a = 1, 2, ..., 9 is easy to see that 3a2 + 100 has no this property, hence the equation is not solvable. Also solved by Raul A. Simon, Chile.

Mathematical Reflections 4 (2010)

26

O164. Let ABC be a triangle and let A1 be a point on the side BC. Starting with A1 construct reections in one of the angle bisectors of triangle such that the next point lies on the other side of the triangle. The process is done in one direction: either clockwise or counterclockwise. Thus at the rst step we construct an isosceles triangle A1 CB1 with point B1 lying on AC. At the second step we construct an isosceles triangle B1 AC1 with point C1 on AB. In fact we get a sequence of points A1 , B1 , C1 , A2 , . . . . (a) Prove that the process terminates in six steps, that is A1 A3 (b) Prove that A1 , A2 , B1 , B2 , C1 , C2 lie on the same circle. Proposed by Ivan Borsenco, Massachusetts Institute of Technology, USA

Solution by Michel Bataille, France

A C1 B1 B 2

C2

A 1

A 2

Let RM N denote the reection in the line M N and let I be the incentre of ABC . (a) As the product of three opposite isometries (reversing the orientation), the isometry R = RBI RAI RCI is opposite as well and since R(I ) = I , R must be a reection in a line . Since R(A1 ) = A2 , must be the perpendicular bisector of the line segment A1 A2 in the general case when A1 = A2 (and IA1 if A1 = A2 ). Thus, R = R and RBI RAI RCI RBI RAI RCI = R R = Id where Id denotes the identity of the plane. As a result, A3 = Id(A1 ) = A1 . (b) Since is the the perpendicular bisector of A1 A2 , we have IB1 = IA1 = IA2 = IC1 . Similarly, if denote the perpendicular bisector of B1 B2 , we have R = RCI RBI RAI and so IC2 = IB2 = IB1 . In conclusion, IA1 = IA2 = IB1 = IB2 = IC1 = IC2 and the six points A1 , A2 , B1 , B2 , C1 , C2 all lie on a circle with centre I . Also solved by Daniel Lasaosa, Universidad P ublica de Navarra, Spain; Raul A. Simon, Chile.
Mathematical Reflections 4 (2010)

27

O165. Let R and r be the circumradius and the inradius of a triangle ABC with the lengths of sides a, b, c. Prove that 2 a r 22 . b+c R cyc Proposed by Dorin Andrica, Babe s-Bolyai University, Cluj-Napoca, Romania

Solution by Arkady Alt, San Jose, California, USA 2 a r a Note that 2 2 6 2 R cyc b + c cyc b + c 2 3
cyc

4+

r R

a b+c

4+

r r (b + c)2 a2 4+ . 2 2 R R (b + c) cyc 1 (b + c)2 a2 r 1 and then = 1 + cos A R 4bc 2bc (b + c)2

Since cos A + cos B + cos C = 1 + (b + c)2 a2 (b + c)2 cyc Remark. 2

r (b + c)2 a2 (1 + cos A) = 4 + . = 2bc R cyc cyc


2 ala (b + c)2 a2 = we can abc (b + c)2

Let la , lb , lc be angle bisectors of a triangle ABC.Noting that rewrite original inequality in such form 2
cyc 2 ala

2 2 ala ala r r 4+ 2 4+ abc R 4 Rrs R cyc

+ + a+b+c

2 blb

2 clc

r (4R + r) .

Also solved by Daniel Lasaosa, Universidad P ublica de Navarra, Spain; Michel Bataille, France.

Mathematical Reflections 4 (2010)

28

O166. The incircle of triangle ABC with incenter I is tangent to sides BC and AC at points A1 and B1 , respectively. Points A2 and B2 are diametrically opposite to A1 and B1 in . Let A3 and B3 be the intersection points of AA2 with BC and BB2 with AC, respectively. Let M be the midpoint of side AC and let N be the midpoint of A1 A3 . Line M I meets BB1 in T and line AT meets BC in P. Let Q (BC ), R be the intersection of lines AB and QB1 and N R AC = {S }. Prove that [AS ] = 2[SM ] if and only if [BP ] = [P Q]. Proposed by Andrei Razvan Baleanu, George Cosbuc, Motru, Romania

Solution by Daniel Lasaosa, Universidad P ublica de Navarra, Spain Consider the parallel to BC through A2 , clearly the incircle of ABC is the excircle of the triangle formed by this line, AB and AC , and this triangle is homothetic to ABC , with center of homothety in A, or A3 = AA2 BC is the contact point of the excircle of ABC tangent to segment BC and to lines AB and AC . It is well known that the contact points of the incircle and the excircle on a given triangle side are symmetric with respect to its midpoint, so N is AS also the midpoint of BC . Note that AS = 2SM i 3AS = 2AM = AC , or SC =1 2 . Therefore, CN since N B = 1, and R, S, N are collinear, by Menelaus theorem, AS = 2SM i BR = 2AR. AB1 bc b+ca a and CB1 = a+2 , or B = a Now, it is well known that AB1 = b+c 2 +bc , and again by 1C Menelaus theorem, and using that Q, B1 , R are collinear, AS = 2SM i equivalently, i BQ =
2a(b+ca) 3b+ca . BQ CQ

2(b+ca) a+bc ,

or

hc a Using exact trillinear coordinates, M h 2 , 0, 2 , where ha , hb , hc are the lengths of the respective altitudes from A, B, C , while I (r, r, r), r being the inradius. It follows that line M I has equation a +(c a) c = 0. Using again exact trillinear coordinates, B (0, hb , 0), bc a while B1 a+2 sin C, 0, b+c sin A , or line BB1 has equation a(b + c a) = c(a + b c) , 2 or point T has (non-exact) trillinear coordinates T (c(a + b c), 2ac, a(b + c a)). Since A (1, 0, 0) in non-exact trillinear coordinates, line AT has equation 2c = (b + c a) , or point P satises b BP BP sin B b+ca = = = , c CP CP sin C 2c CP b BC 3b+ca or BP = b+2 ca , yielding BP = b+ca , and BP = and equivalently BP = P Q, i AS = 2SM . a(b+ca) 3b+ca .

It clearly follows that BQ = 2BP ,

Mathematical Reflections 4 (2010)

29

O167. Prove that in any convex quadrilateral ABCD,


cos BC C D DA 1 AB + cos + cos + cos 2 + (sin A + sin B + sin C + sin D). 4 4 4 4 2

Proposed by Titu Andreescu, University of Texas at Dallas, USA

Solution by Daniel Lasaosa, Universidad P ublica de Navarra, Spain We can write 2 + sin A + sin B = 2 + 2 sin A+B AB AB AB cos 2 + 2 cos = 4 cos2 2 2 2 4 4 cos

AB , 4 with equality i A + B = 180 and simultaneously A = B , ie, equality holds i A = B = 90 . Adding the cyclic permutations of both sides of this resulting inequality, we obtain the proposed inequality with both sides multiplied by 4. The conclusion follows, equality holds in the proposed inequality i ABCD is a rectangle.

Mathematical Reflections 4 (2010)

30

O168. Given a convex polygon A1 A2 . . . An , n 4, denote by Ri the radius of the circumcircle of triangle Ai1 Ai Ai+1 , where i = 2, 3, . . . , n and An+1 is the vertex A1 . Given that R2 = R3 = = Rn , prove that the polygon A1 A2 . . . An is cyclic. Proposed by Nairi Sedrakyan, Armenia

Solution by Raul A. Simon, Chile That the polygon A1 A2 . . . An is convex means that Ai1 Ai Ai+1 is obtuse - if you take the smallest of the two angles formed at the vertex. Therefore, the circumcenter Oi of Ai1 Ai Ai+1 is exterior to it. In fact, all circumcenters Oi lie in a zone Z that is exterior to all triangles Ai1 Ai Ai+1 ( where i = 2, 3, . . . , n and An+1 is the vertex A1 ) and interior to the polygon A1 A2 . . . An . We have then O2 A1 = O2 A2 = O2 A3 = R, and O3 A2 = O3 A3 = O3 A4 = R, etc. We see that O2 is the intersection of two arcs of circle of radius R, centered at A2 and A3 ; and that O3 is determined in exactly the same way. Therefore, since O2 and O3 lie on the same side of A2 A3 , O2 and O3 must coincide. Repeating this reasoning, we nd that all circumcenters must coincide in a unique circumcenter O common to all vertices. The circle O(O, R) is the circumcircle of the polygon; therefore, the latter is cyclic.

Mathematical Reflections 4 (2010)

31

Junior problems

J169. If x, y, z > 0 and x + y + z = 1, nd the maximum of E (x, y, z ) = xy yz zx + + . x+y y+z z+x

Proposed by Dorin Andrica, Babes-Bolyai University, Cluj-Napoca, Romania

Solution by Ercole Suppa, Teramo, Italy By the AMGM inequality, E (x, y, z ) + + = x+y y+z z+x x+y y+z z+x = + + = 4 4 4 x+y+z 1 = = 2 2
(x+y )2 4 (y +z )2 4 (z +x)2 4

Therefore the maximum of E (x, y, z ) is 1 2 , occurring for example if x=y=z= 1 3

Also solved by Arkady Alt, San Jose, California, USA; Michel Bataille, France; Daniel Lasaosa, Universidad P ublica de Navarra, Spain; Perfetti Paolo, Dipartimento di Matematica, Universit` a degli studi di Tor Vergata Roma, Italy; Sayan Mukherjee; Lorenzo Pascali, Universit` a di Roma La Sapienza, Roma, Italy.

Mathematical Reflections 5 (2010)

J170. In the interior of a regular pentagon ABCDE consider the point M such that triangle M DE is equilateral. Find the angles of triangle AM B. Proposed by Catalin Barbu, Vasile Alecsandri College, Bacau, Romania

Solution by Lorenzo Pascali, Universit` a di Roma La Sapienza, Roma, Italy We have that M EA = AED M ED = 108 60 = 48 . Morever, AE = EM implies that 1 EAM = EM A = (180 48 ) = 66 . 2 By symmetry 1 ABM = M BC = ABC = 54 . 2 Finally M AB = EAB DCM = 108 66 = 42 , AM B = 180 M AB ABM = 84 .

Also solved by Daniel Lasaosa, Universidad P ublica de Navarra, Spain; Sayan Mukherjee; Raul A. Simon, Chile; Ercole Suppa, Teramo, Italy; Vicente Vicario Garcia, Huelva, Spain.

Mathematical Reflections 5 (2010)

J171. If dierent letters represent dierent digits, could the addition AXXXU BXXV CXXY +DEXXZ XXXXX be correct? Proposed by Titu Andreescu, University of Texas at Dallas, USA Solution by Francisco Javier Garcia Capitan, Spain We assume that leading digits A, B , C , D and X cannot be 0. Now, A and D are distinct numbers, so A + D 3. In addition, the sum in the fourth column (we refer the columns from right to left) has carrying, thus we have in fact X > 3. The sum in the rst column has carry as well, because if not, we have 4X X mod 10 in the second column, and this is impossible. Call R the carrying of the sum in the rst column. Well have R + 4X = 10R + X adding the second column, thus X = 3R. Since X > 3, it can be X = 6, R = 2 or X = 9, R = 3, but the latter is impossible, since U + V + X + Y <= 6 + 7 + 8 + 9 = 30 and X = 9, R = 3 implies U + V + Y + Z = 39. If we must have X = 6, R = 2, which implies U + V + Y + Z = 26 and some of (1) B+C +E =8 A+D =5 , (2) B + C + E = 18 A+D =4 .

If we take E = 0 and the remaining digits such that {B, C } = {1, 7}, {A, D} = {2, 3} and {U, V, Y, Z } = {4, 5, 8, 9} then (1) holds and we have a solution of the problem. 6 6 6 4 1 6 6 5 7 6 6 8 + 3 0 6 6 9 6 6 6 6 6 Note that the conditions (2) are impossible since they imply that (U + V + Y + Z ) + X + (B + C + E ) + (A + D) =26 + 6 + 18 + 4 = 54 = 45, which is the sums of the ten digits. Also solved by Daniel Lasaosa, Universidad P ublica de Navarra, Spain; Ercole Suppa, Teramo, Italy; Andrea Ligori, Universit` a di Roma Tor Vergata, Roma, Italy.
Mathematical Reflections 5 (2010)

J172. Let P be a point situated in the interior of an equilateral triangle ABC and let A , B , C be the intersectionsof lines AP, BP, CP with sides BC, CA, AB , respectively. Find P such that A B 2 + B C 2 + C A2 = AB 2 + BC 2 + CA 2 . Proposed by Catalin Barbu, Vasile Alecsandri College, Bacau, Romania

Solution by Sayan Mukherjee Let us denote the areas of BA = AC BP C, AP B, BP A = CP A AP C as x, y, z respectively. we have, AA B AA C BP A = CP A AP B y = . AP C z

AA B = AA C

Let AB = BC = CA = a, then BA = relations.

az ay ; and A C = . Similarly we obtain four other y+z y+z

Substituting these values in our rst relation we obtain, y2 z2 x2 z2 x2 y2 + + = + + ; (y + z )2 (z + x)2 (x + y )2 (y + z )2 (z + x)2 (x + y )2 Which, on transposition, is equivalent to z 2 x2 x2 y 2 y2 z2 + + = 0. (y + z )2 (z + x)2 (x + y )2 This relation can be simplied to
cyc

xy = 0, which rewrites into x+y

(x y )(z x)(z y ) = 0. (x + y )(y + z )(z + x)


Mathematical Reflections 5 (2010)

Hence the rst relation actually holds true if and only if x = y or y = z or z = x. So, if P is such that at least two of the triangles AP B, BP C, CP A have the same area, then the rst relation holds good. So, P must lie on any of the three medians of ABC . Also solved by Michel Bataille, France; Daniel Lasaosa, Universidad P ublica de Navarra, Spain; Francisco Javier Garcia Capitan, Spain; Raul A. Simon, Chile; Ercole Suppa, Teramo, Italy;Vicente Vicario Garcia, Huelva, Spain.

Mathematical Reflections 5 (2010)

J173. Let a and b be rational numbers such that |a| Prove that a2 + b2 17. Proposed by Titu Andreescu, University of Texas at Dallas, USA |a2 47 3ab2 | and |b| |b2 52 . 3a2 |

First solution by Michel Bataille, France Let X = a3 3ab2 and Y = 3a2 b b3 . From the hypothesis, we have |X | 47 and |Y | 52. Observing that X + iY = (a + ib)3 (easily checked), we deduce that a2 + b2 = |a + ib|2 = (a + ib)3
2 3

= (X 2 + Y 2 ) 2

2 3

= (X 2 + Y 2 ) 3 (472 + 522 ) 3 .

Since 472 + 522 = 4913 = 173 , it follows that a2 + b2 17. Second solution by Daniel Lasaosa, Universidad P ublica de Navarra, Spain Note that a = b = 0 clearly satises a2 + b2 < 17, therefore a2 3b2 and b2 3a2 are nonzero rationals, since otherwise 3 would be the square of a rational, absurd. Dene x = a2 , y = b2 , hence x(x 3y )2 472 and y (3x y )2 522 . Adding these two inequalities results in (x + y )3 472 + 522 = 173 , or x + y = a2 + b2 17. Note that equality is reached whenever equality holds simultaneously on both inequalities given as condition in the problem statement, for example when a = 1 and b = 4.

Mathematical Reflections 5 (2010)

J174. The incircle of triangle ABC touches sides BC, CA, AB at D, E, F , respectively. Let K be a point on side BC and let M be the point on the line segment AK such that AM = AE = AF . Denote by L and N the incenters of triangles ABK and ACK , respectively. Prove that K is the foot of the altitude from A if and only if DLM N is a square. Proposed by Bogdan Enescu, B.P.Hasdeu College, Buzau, Romania

Solution by Ercole Suppa, Teramo, Italy We begin by proving two lemmas. Lemma 1. The points D, K lie on the circle with diameter LN . Proof. Suppose wlog that c < b. Let I , U , V be the incenter of ABC and the points where the circles (L), (N ) touch the side BC ; let r, r1 , r2 be the inradii of the circles (I ), (L), (N ), as shown in gure.
A

E F I

Denote a = BC , b = CA, c = AB , m = BK , n = KC , x = AK . Since L and N are the incenters of ABK and ACK we have 1 (BKA + AKC ) = 90 2

LKN = LKA + AKN =

In order to prove that LDN = 90 we will show that LD2 + DN 2 = LN 2


2 + U D 2 , N D 2 = r 2 + DV 2 and The Pythagoras theorem yields LD2 = r1 2 2 2 LN 2 = U V 2 + (r1 r2 )2 = U D2 + DV 2 2U D DV + r1 + r2 2r1 r2

(1)

Mathematical Reflections 5 (2010)

Therefore to establish (3) it is enough to show that U D DV = r1 r2 . We clearly have U D = BD BU = DV = DC CV = a+cb m+cx a+xbm = 2 2 2 a+xcn a+bc m+bx = 2 2 2 (2) (3)

From (2) and (3), plugging n = a m into the expression, we obtain U D DV = From the similar triangles BU L (x + a b m)(x c + m) 4 BDI , CV N CDI , it follows that c+mx a+cb b+nx a+bc (5) (6) (4)

LU : ID = BU : BD N V : ID = CV : CD From (5),(6) using the well known formula r2 =

r1 = r r2 = r

(b + c a)(a + c b)(a + b c) 4(a + b + c)

and plugging n = a m into the expression, we obtain r1 r2 = Using (4) and (7) we have U D DV r1 r2 = From the Stewarts theorem we get x2 = mb2 + (a m)c2 am(a m) a ax2 ac2 + a2 m b2 m + c2 m am2 2(a + b + c) (8) (b + c a)(a + b m x)(c + m x) 4(a + b + c) (7)

Finally, plugging x2 into (8), after a boring calculation, we have U D DV r1 r2 = c2 am (m + n a) =0 2(a + b + c)

Thus LD2 + DN 2 = LN 2 and the lemma is proved. Lemma 2. If M is the second intersection point of AK with the circle circumscribed to DKN L, then DM LN and AM = AE = AF .
Mathematical Reflections 5 (2010)

E F F' L X N M K'

Proof. Let the incircle of triangle ABK touches side AB at F . According to Lemma 1 the center of is the midpoint of LN , so the point M lies on the external tangent to the circles (L), (N ). Therefore, because of simmetry of the gure, we have DM LN and AM = AF U D = AF (BD BU ) c+xm a+cb c+mx = + 2 2 2 b+ca = = AF 2 The lemma is proved. Considering now the original problem, from Lemma 1 and Lemma 2 it follows that DLM N is cyclic; LDN = LM N = 90 ; DM LN .
A

E F L N M

Mathematical Reflections 5 (2010)

Therefore DLM N is a square if and only if M D is a diameter of the circumcircle of DLM N , i.e. M KD = 90 (i.e. AK BC ). Also solved by Michel Bataille, France; Daniel Lasaosa, Universidad P ublica de Navarra, Spain; Sayan Mukherjee.

Mathematical Reflections 5 (2010)

10

Senior problems

S169. Let k > 1 be an odd integer such that ak + bk = ck + dk for some positive integers a, b, c, d. k +bk Prove that aa+ b is not a prime. Proposed by Ivan Borsenco, Massachusetts Institute of Technology, USA

Solution by Daniel Lasaosa, Universidad P ublica de Navarra, Spain Note rst that we need to assume that {a, b} = {c, d}, since otherwise we may take k = 3, k bk a = c = 2, b = d = 1, and aa+ +b = 3 is prime. Moreover, as long as {a, b} = {c, d}, the k k k k condition a + b = c + d in the problem is irrelevant, so I will assume that {a, b} = {c, d}. k bk k1 cannot be a prime because k 3, whereas Note also that, if a = b > 1, then aa+ +b = a a = b = 1 is only possible if c = d = 1. We may then assume wlog that a b + 1 2, hence for any odd k 3, ak + bk (a + b)2 a3 + b3 a2 b2 2ab = (a + b)(a2 ab + b2 a b) = = (a + b)(a b 1)2 + b(a + b)(a 2) + (a + b)(a b 1) 0, with equality i k = 3, a = 2 and b = 1, possible only if {c, d} = {1, 2}. We may therefore assume that {a, b} = {c, d}, where moreover wlog a > b and c d, and k bk ck +dk by the previous result, aa+ +b > a + b, and similarly c+d > c + d. Note that, if a = c, then bk = dk , or b = d, in contradiction with our assumption, and similarly if b = d, hence either a > c d > b, or c > a > b > d. In the rst case, note that (a c) ak1 + cak2 + + ck1 = (d b) bk1 + dbk2 + + dk1 , and since ai cj > bi dj because a > b and c d, we have a c < d b, or a + b < c + d. k bk ck +dk ak +bk ck +dk Therefore, aa+ +b > c+d , and if a+b is prime, it cannot divide c+d , hence it divides c + d, or ak + bk (a + b)(c + d) < (c + d)2 < ck + dk , absurd. In the second case, we obtain similarly that a + b > c + d, and if ak bk b)2 ,
ak +bk a+b ak +bk a+b

divides c + d, we have
ck +dk c+ d ,

+ (a + b)(c + d) < (a + contradiction, hence divides a + b, and since a + b > c + d, then a + b 2(c + d) > 2c > a + b, absurd. The conclusion follows.

or c + d divides

Mathematical Reflections 5 (2010)

11

S170. Consider n(n 6) circles of radius r < 1 that are pairwise tangent and all tangent to a circle of radius 1. Find r. Proposed by Catalin Barbu, Vasile Alecsandri College, Bacau, Romania

Solution by Daniel Lasaosa, Universidad P ublica de Navarra, Spain We will assume that the problem statement means that n circles or radius r are all tangent to a given circle of radius 1, and each circle of radius r is tangent to another two circles of radius r, since it is well known that at most four circles can be tangent to each other, and at most three of them would have the same radius, since either one of the four circles contains the other three inside them, or one of them is in the gap formed by the other three. With this assumption, clearly all centers of the circles with radii r are at a distance 1 + r from the center of the circle with radius 1, and at a distance 2r from their closest neighbours of radius r, and by cyclic symmetry, these n centers are at the vertices of a regular n-gon, with sidelength 2r and circumradius 1 + r, hence considering the isosceles triangle with equal sides of length 1 + r and sidelength 2r for the other side, where the dierent angle is 360 , we have n 180 r = (1 + r) sin n , or equivalently r= sin 180 n . 1 sin 180 n

Also solved by Raul A. Simon, Chile.

Mathematical Reflections 5 (2010)

12

S171. Prove that if the polynomial P R[X ] has n distinct real zeros, then for any R the polynomial Q(X ) = XP (X ) + P (X ) has at least n 1 distinct real zeros. Proposed by Dorin Andrica, Babes-Bolyai University, Cluj-Napoca, Romania

Solution by Carlo Pagano, Universit` a di Roma Tor Vergata, Roma, Italy Let be a1 < a2 < < an be the real roots of P and
n

P (x) = Q(x) where Q R[x] has no real roots. Therefore

(x aj )
j =1

R(ai ) = ai P (ai ) + P (ai ) = P (ai ) = Q(ai )


j =i

(ai aj )

and its clear that in the last product has n i 1 negative factors. Since Q(x) has constant sign, it follows that R(ai )R(ai+1 ) < 0 for i = 1, . . . , n 1, and by Intermediate Value Theorem there exists at least a zero of R in (ai , ai+1 ). This means that R has at least n 1 distinct real zeros. Also solved by Arkady Alt, San Jose, California, USA; Michel Bataille, France; Daniel Lasaosa, Universidad P ublica de Navarra, Spain; Vicente Vicario Garcia, Huelva, Spain.

Mathematical Reflections 5 (2010)

13

S172. Let a, b, c be positive real numbers. Prove that a2 b2 (b c) 0. a+b

cyc

Proposed by Titu Andreescu, University of Texas at Dallas, USA

Solution by Perfetti Paolo, Dipartimento di Matematica, Universit` a degli studi di Tor Vergata Roma, Italy Clearing the denominators, the inequality is equivalent to (a4 c2 b + c4 b3 )
cyc cyc

2a3 b2 c

but this follows from the AMGM (a4 c2 b + c4 b3 )/2 a3 b2 c Also solved by Arkady Alt, San Jose, California, USA; Michel Bataille, France; Daniel Lasaosa, Universidad P ublica de Navarra, Spain; Sayan Mukherjee; Ercole Suppa, Teramo, Italy.

Mathematical Reflections 5 (2010)

14

S173. Let fn (x, y, z ) =

(x y )z n+2 + (y z )xn+2 + (z x)y n+2 . (x y )(y z )(x z )

Prove that fn (x, y, z ) can be written as a sum of monomials of degree n and nd fn (1, 1, 1) for all positive integers n. Proposed by Ivan Borsenco, Massachusetts Institute of Technology, USA

Solution by Michel Bataille, France It is easily veried that 1 1 1 + + = 0. (x y )(x z ) (y x)(y z ) (z y )(z x) This allows one to transform fn (x, y, z ) as follows: fn (x, y, z ) = = = = = xn+2 y n+2 z n+2 + + (x y )(x z ) (y x)(y z ) (z y )(z x) xn+2 xn+2 y n+2 z n+2 + + + (y x)(y z ) (z y )(z x) (y x)(y z ) (z y )(z x) y n+2 xn+2 z n+2 xn+2 + (y x)(y z ) (z y )(z x) y n+1 + y n x + + xn y + xn+1 z n+1 + z n x + + xn z + xn+1 + (y z ) (z y ) n +1 n +1 n n n (y z ) + x(y z ) + + x (y z ) , yz

and nally fn (x, y, z ) = (y n +y n1 z + +yz n1 +z n )+x(y n1 +y n2 z + +yz n2 +z n1 )+ +xn1 (y +z )+xn . Thus, fn (x, y, z ) is the sum of all monomials xa y b z c over all triples (a, b, c) of nonnegative integers satisfying a + b + c = n. This answers the rst part of the question. As for the second part, fn (1, 1, 1) is just the total number of monomials in the above sum, that n+2) is, (n + 1) + n + + 2 + 1 = (n+1)( . 2 Also solved by Daniel Lasaosa, Universidad P ublica de Navarra, Spain; Moubinool Omarjee , Paris France; Perfetti Paolo, Dipartimento di Matematica, Universit` a degli studi di Tor Vergata Roma, Italy.

Mathematical Reflections 5 (2010)

15

S174. Prove that for each positive integer k the equation


3 3 2 4 x3 1 + x2 + + xk + xk+1 = xk+2

has innitely many solutions in positive integers with x1 < x2 < < xk+1 . Proposed by Dorin Andrica, Babes-Bolyai University, Cluj-Napoca, Romania

First solution by Arkady Alt, San Jose, California, USA Since 13 + 23 + + k 3 = then by substitution x1 = x, x2 = 2x, . . . , xk = kx, xk+1 = (1 + 2 + + k ) y 2 , xk+2 = (1 + 2 + + k ) y in original equation, we obtain 13 + 23 + + k 3 x3 + (1 + 2 + + k )2 y 4 = (1 + 2 + + k )4 y 4 x3 = ay 4 (k 1) (k + 2) k 2 + k + 2 . Since x = a7 n4 , y = a5 n3 for 4 any positive integer n , then x3 = ay 4 . Hence where a = (1 + 2 + + k )2 1 = x1 = a7 n4 , x2 = 2a7 n4 , . . . , xk = ka7 n4 , xk+1 = (1 + 2 + + k ) a10 n6 , xk+2 = (1 + 2 + + k ) a5 n3 for any positive integer n is a solution to the original equation and obviously, x1 < x2 < < xk+1 . Second solution by the authors For any positive integer n we have the well-known identity: 1 + 2 + + n + (n + 1) + + (n + k ) = that is n(n + 1) 2
3 3 3 3 3

k 2 (k + 1)2 = (1 + 2 + + k )2 4

(n + k )(n + k + 1) 2 (n + k )(n + k + 1) 2
2

+ (n + 1) + + (n + k ) =

+k+1) Consider the positive integers n such that the triangular number tn+k = (n+k)(n is a per2 fect squares. There are innitely many such integers since the relation tn+k = u2 is equivalent to the Pells equation (2n + 2k + 1)2 2u2 = 1. The fundamental solution to this Pell equation is (3, 2), hence all these integers are given by the sequence (ns ), where 2ns + 2k + 1 + us 2 = (3 + 2 2)s ,

for s big enough such that ns 1.


Mathematical Reflections 5 (2010)

16

We can take x1 = ns + 1, , xk = ns + k, xk+1 = It is clear that for s big enough we have ns 1 and family of solutions.

ns (ns + 1) , xk+2 = us . 2 > n + k , hence we get an innite

n(n+1) 2

Also solved by Daniel Lasaosa, Universidad P ublica de Navarra, Spain.

Mathematical Reflections 5 (2010)

17

Undergraduate problems

U169. Sequences (xn )n1 and (yn )n1 are dened by x1 = 2, y1 = 1, and xn+1 = x2 n + 1, yn+1 = xn yn for all n. Prove that for all n 1 651 xn < . yn 250 Proposed by Dorin Andrica, Babes-Bolyai University, Cluj-Napoca, Romania

Solution by Andrea Ligori, Universit` a di Roma Tor Vergata, Roma, Italy The inequality holds for 1 n 4 because x1 /y1 = 2, x2 /y2 = 5/2, x3 /y3 = 13/5, and x4 /y4 = 677/260,. Moreover, for n 5 x2 + 1 xn xn1 1 x4 = n1 = + = + yn xn1 yn1 yn1 yn y4 Hence it suces to show that
k=4 n k=5

1 677 < + yk 260

k=5

1 . yk

1 651 677 1 = . yk 250 260 6500

We have that for n 1


4 8 2 xn x2 n1 xn2 xn3 2
n1

and, yn = xn1 yn1 22 This means that yn 22


n2

yn1 22

n2 +2n3

yn2 22

n2 +2n3 ++1

= 22

n1 1

n1 1

23n = 8n for n 5, and


k=5

k=5

1 yk

1 1 1 1 = < . = 4 k 78 28672 6500 8

Also solved by Arkady Alt, San Jose, California, USA; Daniel Lasaosa, Universidad P ublica de Navarra, Spain.

Mathematical Reflections 5 (2010)

18

U170. Sequences (xn )n1 , (yn )n1 are dened as follows: x1 = , y1 = , with || = | | = 0, and xn+1 = max (xn yn , xn + yn ) , yn+1 = min (xn yn , xn + yn ) , for all n 1. Prove that
n

lim xn = lim yn = .
n

Proposed by Bogdan Enescu, B.P.Hasdeu College, Buzau, Romania

Solution by Michel Bataille, France It is easily seen that (yn ) has no limit (nite or not) if = 0, = 1. We will show the following results: limn xn = limn yn = if = 0 and || = | |, limn xn = limn yn = 0 if = = 0, and otherwise, limn xn = and (yn ) has no limit either nite or innite. 1 For all real numbers a, b, we have max(a, b) = 1 2 (a + b + |a b|) and min(a, b) = 2 (a + b |a b|). It follows that for all n 1, xn+1 = xn + |yn |, yn+1 = xn |yn | (1).

First, suppose || < | | that is, | | < < | |. Then, using (1), x2 = + | |, y2 = | | < 0, x3 = 2| |, y3 = 2 so that x4 = 2(| | + ||), y4 = 2(| | ||), x5 = 4| |, y5 = 4||. An easy induction yields x2n = 2n1 (| | + ||), y2n = 2n1 (| | ||), x2n+1 = 2n | |, y2n+1 = 2n || for all integer n 2. Thus, limn xn = (since limn 2n = and | | > 0) and limn yn = if = 0 while (yn ) has no limit (nite or not) if = 0 (since then limn y2n = and limn y2n+1 = 0). If > | |, with the help of (1) again, we obtain x2n1 = 2n1 , y2n1 = 2n1 | |, x2n = 2n1 (| | + ), y2n = 2n1 ( | |) for all n 2 and limn xn = limn yn = follows. If < | |, similarly, x2n+1 = 2n ||, y2n+1 = 2n | |, x2n+2 = 2n (|| + | |), y2n+2 = 2n (|| | |) for all n 2 and limn xn = limn yn = again. Lastly, in the same way it is readily seen that if and are equal or opposite nonzero real numbers, then limn xn = while (yn ) has no limit (nite or not). Also, we clearly have limn xn = limn yn = 0 if = = 0 and the proof is complete. Also solved by Arkady Alt, San Jose, California, USA; Daniel Lasaosa, Universidad P ublica de Navarra, Spain; Perfetti Paolo, Dipartimento di Matematica, Universit` a degli studi di Tor Vergata Roma, Italy; Andrea Ligori, Universit` a di Roma Tor Vergata, Roma, Italy.

Mathematical Reflections 5 (2010)

19

U171. Let A be a matrix of order n such that A10 = On . Prove that 1 4 1 3 1 2 A + A + A + A + In 4 2 2 is invertible. Proposed by Titu Andreescu, University of Texas at Dallas, USA

Solution by Arkady Alt, San Jose, California, USA 1 1 1 Let f (A) = A3 A3 A In and B = Af (A) . Since for any two polynomial 4 2 2 P (x) and Q (x) and any matrix A holds P (A) Q (A) = Q (A) P (A) then B 10 = (Af (A))10 = A10 f (A)10 = 0n .Hence, 1 4 1 1 A + A3 + A2 + A + I n 4 2 2 In + B + B 2 + + B 9 =

(In B ) In + B + B 2 + + B 9 = In B 10 = In .Thus, In + B + B 2 + + B 9 is 1 1 1 inverse matrix for A4 + A3 + A2 + A + In . 4 2 2 Also solved by Michel Bataille, France; Daniel Lasaosa, Universidad P ublica de Navarra, Spain; Moubinool Omarjee , Paris France; Andrea Ligori, Universit` a di Roma Tor Vergata, Roma, Italy.

Mathematical Reflections 5 (2010)

20

U172. Let f : R R be a strictly increasing invertible function such that for all x R, f (x) + f 1 (x) = ex 1 for all x R. Prove that f has at most one xed point. Proposed by Samin Riasat, University of Dhaka, Bangladesh

Solution by Emanuele Natale, Universit` a di Roma Tor Vergata, Roma, Italy If x0 is a xed point then f (x0 ) = f 1 (x0 ) = x0 and f (x0 ) + f 1 (x0 ) = 2x0 = ex0 1 which has two solutions: 0 and a some c > 0. So, if f has more than one xed point then it has just these two xed points: 0 and c. Since f is a strictly increasing invertible function then f 1 is strictly increasing too. Take x < 0 then f (x) < f (0) = 0, f 1 (x) < f 1 (0) = 0. Moreover f (x) < x otherwise f (x) > x and x > f 1 (x). In both cases e x 1 = f (x) + f 1 (x) < x + 0 = x which is a contradiction because for x < 0 we have that x < ex 1. Therefore c is the only possible xed point. Also solved by Michel Bataille, France; Daniel Lasaosa, Universidad P ublica de Navarra, Spain.

Mathematical Reflections 5 (2010)

21

U173. Let be a real number. Prove that


n1

sin

3+ k=0

2k n 2 cos 2k n

(1)n n sin(n) 2 + 4(1)n sin2 5Fn

n 2

where Fn denotes the nth Fibonacci number. Proposed by Javier Buitrago, Universidad National de Colombia, Colombia Solution by G. C. Greubel, Newport News, USA Let Pn = It is readily clear that Pn = From this we have
n1

sin 3+

2k n 2 cos 2k n

(1)

1 d ln 3 + 2 cos 2 d
n1

2k n

(2)

Pn =
k=0

1 d 2 d

ln 3 + 2 cos
k=0

2k n (3)

1 d = ln {Sn } , 2 d where
n1

Sn =
k=0

3 + 2 cos

2k n

(4)

Taking n = 1 we have
0

S1 =
k=0

[3 + 2 cos (2k )]

= 3 + 2 cos = 5 4 sin2 In the case when n = 2 we have


1

(5)

S2 =
k=0

3 + 2 cos

2k 2

= (3 + 2 cos )(3 + 2 cos( ) = 9 4 cos2 = 5 + 4 sin2 .


Mathematical Reflections 5 (2010)

(6)
22

The two cases of n given here have the general form


2 Sn = 5Fn + 4(1)n sin2

n 2

(7)

Further values of n lead to the same result. Now using (7) in (4) leads to
n1

Pn =
k=0

1 d 2 ln 5Fn + 4(1)n sin2 2 d

n 2 n 2

1 d 2 5Fn + 4(1)n sin2 2Sn d 1 [2(1)n n sin(n)] = 2Sn (1)n n sin(n) = . Sn

(8)

By combining the components of this result leads to the expression


n1 k=0

sin 3+

2k n 2 cos 2k n

(1)n n sin(n) . 2 + 4(1)n sin2 n 5Fn 2

(9)

Also solved by Michel Bataille, France; Daniel Lasaosa, Universidad P ublica de Navarra, Spain.

Mathematical Reflections 5 (2010)

23

U174. Let p be a prime. A linear recurrence of degree n in Fp is a sequence {ak }k0 in Fp satisfying a relation of the form ai+n = cn1 ai+n1 + + c1 ai+1 + c0 ai for all i 0, where c0 , c1 , . . . , cn1 Fp and c0 = 0. (a) What is the maximal possible period of a linear recurrence of degree n in Fp ? (b) How many distinct linear recurrences of degree n have this maximal period? Proposed by Holden Lee, Massachusetts Institute of Technology

Solution by Holden Lee, Massachusetts Institute of Technology (a) pn 1. There are only pn possibilities for n consecutive terms of the sequence. If (0, . . . , 0) appears then nothing else can appear. (Pf. of existence below) (b) Suppose a recurrence has period pn 1. Note that by Vandermondes determinant the n functions f (t) = t where F pn are a basis for the space of functions Z/(p 1)Z Fpn . Then the general term can be written as
k k ak = b1 1 + + bm m . for some m, distinct i F pn , and b1 , . . . , bm Fpn . By plugging into the recurrence, we get that all the i must be roots of the characteristic polynomial. If i has degree di then pdi 1 i = 1. Thus in order for the period to be pn 1 the characteristic polynomial must be irreducible (so the roots have degree n). The roots have the same order; the common order is the least N so that the characteristic polynomial divides xN 1. Again using the fact the f are linearly independent we can show that the period has to be this common order and not less. Hence the characteristic polynomial has to divide the cyclotomic polynomial pn 1 (x). Each factor of it in Fp has degree n, so there are (pn 1)/n valid polynomials. For each of these we can choose a0 , . . . , an1 in pn 1 dierent ways. (Note a nonzero sequence cannot satisfy two recurrences corresponding to two dierent n (pn 1) irreducible characteristic polynomials.) Answer: (p 1) . n

Mathematical Reflections 5 (2010)

24

Olympiad problems

O169. Let a, b, c, d be real numbers such that (a2 + 1)(b2 + 1)(c2 + 1)(d2 + 1) = 16. Prove that 3 ab + bc + cd + da + ac + bd abcd 5. Proposed by Titu Andreescu, University of Texas at Dallas, USA, and Gabriel Dospinescu, Ecole Normale Superieure, France

Solution by Michel Bataille, France Consider the complex number Z = (1 + ia)(1 + ib)(1 + ic)(1 + id). An easy calculation yields: Re(Z ) = 1 (ab + bc + cd + da + ac + bd) + abcd and |Z |2 = (a2 + 1)(b2 + 1)(c2 + 1)(d2 + 1). Now, the hypothesis gives |Z | = 4 so that the inequality |Re(Z )| |Z | writes as |(ab + bc + cd + da + ac + bd abcd) 1| 4 that is, 3 ab + bc + cd + da + ac + bd abcd 5, as required. Also solved by Daniel Lasaosa, Universidad P ublica de Navarra, Spain; Sayan Mukherjee; Pedro H. O. Pantoja, Natal-RN, Brazil.

Mathematical Reflections 5 (2010)

25

O170. Let a and b be positive integers such that a does not divide b and b does not divide a. Prove that there is an integer x such that 1 < x a and both a and b divide x(b)+1 x, where is Eulers totient function. Proposed by Vahgan Aslanyan, Yerevan, Armenia

Solution by Sayan Mukherjee We have, x x(b) 1 is divisible by x and when gcd(b, x) = 1; applying Eulers theorem this is also divisible by b. a a So, letting x = we get, using gcd , b = 1, that gcd(a, b) gcd(a, b) a gcd(a, b) a gcd(a, b)
(b)

1 0 mod

a b gcd(a, b)

mod lcm(a, b).

Therefore the required x is

a since this also satises 1 < x a. gcd(a, b)

Also solved by Daniel Lasaosa, Universidad P ublica de Navarra, Spain; Tigran Hakobyan, Armenia.

Mathematical Reflections 5 (2010)

26

O171. Prove that in any convex quadrilateral ABCD, sin A + 60 3 + sin B + 60 3 + sin C + 60 3 + sin D + 60 3

1 (8 + sin A + sin B + sin C + sin D). 3 Proposed by Titu Andreescu, University of Texas at Dallas, USA

Solution by Perfetti Paolo, Dipartimento di Matematica, Universit` a degli studi di Tor Vergata Roma, Italy We employ the theory of Sch urconcave functions, see A.W.Marshall, I.Olkin, Inequalities: Theory of Majorization and Its Applications, Academic Press, 1979. Lets recall few notions about the theory as exposed in chapters 3 and 4 of the cited book. For any x = (x1 , x2 , . . . , xn ) Rn let x[1] x[2] . . . x[n] denote the components of x in the decreasing order. We write
k k n n

xy

if
i=1

x [i ]
i=1

y[i]

k = 1, 2, . . . , n 1,
i=1

x [ i] =
i=1

y [i ]

When x y we say that x is majorized by y or that y majorizes x. A typical example is 1 1 1 1 1 1 1 1 , ,..., , , . . . , , 0 ( , , 0, . . . , 0) (1, 0, . . . , 0) n n n n1 n1 n 2 2 Two other examples are 2s 2s 2s 2s , , , a1 , a2 , a3 , a4 4 4 4 4 where (a1 , . . . , a3 ) are the sides of a quadrilateral and , , , 2 2 2 2 (A, B, C, D) (, , 0, 0)

where A, B, C, D, are the angles of a quadrilateral. Denition A function : D Rn R is said to be Schurconvex (concave) if for any x D, y D x y = (x) (y ) ((x) (y )) Now let D be a subset of Rn such that: i) x D = x D if the components of x are permutated respect to the components of x. ii) D is convex and has nonempty interior. We have the theorem (the proof is at page 57 of [1]; see also the remark [A.4.a]).

Mathematical Reflections 5 (2010)

27

Theorem Let D be the set {x R4 : 0 x1 a, 0 x2 a, 0 x3 a, 0 x4 a} and let : D R be a continuously dierentiable function. Necessary and sucient condition for to be Schurconvex (concave) are 1) is symmetric 2) (x1 x2 )(x1 x2 ) 0 ( 0) for any x D.

To apply the theorem to our inequality we dene D = {x R4 , 0 xi } and : D R, (A, B, C, D) = sin A+ 3 + sin B+ 3 + sin C + 3 + sin D+ 3 +

1 (8 + sin A + sin B + sin C + sin D) 3 and we show that (A B )(A B ) 0 if A B namely the function is Sch urconvex. This implies that (A, B, C, D) (/2, /2, /2, /2) = 0 and the result is achieved. We have to prove that A B 0 if A B. A B = cos B+ A+ cos (cos A cos B ) 3 3

and its nonnegative since the function cos x is strictly decreasing between x = 0 and x = B + AB and A B > A+ 3 3 = 3 Also solved by Daniel Lasaosa, Universidad P ublica de Navarra, Spain; Sayan Mukherjee.

Mathematical Reflections 5 (2010)

28

O172. Prove that if a 7 7 square board is covered by 38 dominoes such that each domino covers exactly two squares of the board, then it is possible to remove one domino after which the remaining 37 cover the board. Proposed by Nairi Sedrakyan, Yerevan, Armenia

Solution by Daniel Lasaosa, Universidad P ublica de Navarra, Spain Consider a graph with 49 vertices, representing each vertex one of the squares in the board, where two vertices are joined by and edge i their corresponding squares are covered by the same domino. Assume that the board can be covered by 38 dominoes so that no matter which domino is removed, at least one square becomes uncovered. Therefore, no edge in the graph may join two vertices, such that both of them are ends of other edges. Consider two vertices V1 and V2 , joined by an edge. Clearly, one of them (wlog V2 ) cannot have any other edges. If V1 has other vertices joined to it through edges, these must also have no other edges connecting them to further vertices in the graph, or all points that may be reached from V1 through edges of the graph form a tree with root V1 and leaves V2 , V3 , . . . , no other vertices involved. Clearly, no vertex can be joined to more than 4 other vertices, otherwise by Dirichlets principle, two dominoes would cover the same two squares, and one of them could be removed leaving all 49 squares covered. Therefore, the graph may be decomposed in disjoint subgraphs, each one of them with k vertices, such that k 1 of them are joined by an edge to the remaining vertex, no other vertices or edges are present in each subgraph, ie, denoting nk the number of subgraphs with k vertices (where clearly k = 2, 3, 4, 5) we have 2n2 + 3n3 + 4n4 + 5n5 = 49 and n2 + 2n3 + 3n4 + 4n5 = 38. Assume that n5 = 4 d where d 0, hence n2 + n3 + n4 = 7 + d, whereas n2 + 2n3 + 3n4 = 22 + 4d > 3n2 + 3n3 + 3n4 , absurd, hence n5 5. Now, on the board, a graph with k = 5 would represent a cross-shaped pentamino with one square joined to its four neighbours, hence no more than 2 squares on each side of the board, and none of its corners, can be covered by a domino involved in each one of these n5 subgraphs. It follows that the n5 subgraphs cover no more than 33 squares, ie n5 6. Assume that n5 = 5, then 2n2 + 3n3 + 4n4 = 24 and n2 + 2n3 + 3n4 = 18. Note now that 0 = 3 24 4 18 = 2n2 + n3 , and since n2 , n3 0, we have n2 = n3 = 0, n4 = 6. Note however that each corned must be covered by one of the n4 subgraphs because it cannot be covered by one of the n5 subgraphs, and each n4 subgraph is a T-shaped tetramino, hence each one of the four corners of the board must be covered by one of the two squares forming the horizontal bar of the T, leaving one prisoner square, limiting on one side with the side of the board, and on two sides with sides of this T-shaped tetramino. This square must therefore be covered, next to each corner square, by another one of the n4 subgraphs, hence n4 8, contradiction. Therefore n5 = 6. Assume nally that n5 = 6, then 2n2 + 3n3 + 4n4 = 19 and n2 + 2n3 + 3n4 = 14, or 1 = 3 19 4 14 = 2n2 + n3 , or since n2 , n3 0, it follows that n2 = 0, n3 = 1, and n4 = 4. As in the case of n5 = 5, it follows that one of the four corners of the board must be covered by the n3 subgraph, while
Mathematical Reflections 5 (2010)

29

the other three must be covered by n4 subgraphs, yielding n4 6 (two of the n4 subgraphs must be placed on or close to each corner), contradiction again. The conclusion follows. It is possible however to cover the board with n5 = 3 pentaminoes, n4 = 7 tetraminoes and n3 = 2 trominoes (leaving n2 = 0), or the board can be covered by 37 dominoes, such that no domino can be removed without leaving uncovered squares. Also solved by G.R.A.20 Math Problems Group, Roma, Italy.

Mathematical Reflections 5 (2010)

30

O173. Find all triples (x, y, z ) of integers such that x3 + y 3 + z 3 xyz = 2010 max{ 3 x y, 3 y z, 3 z x}. 3 Proposed by Titu Andreescu, University of Texas at Dallas, USA and Gabriel Dospinescu, Ecole Normale Superieure, France

Solution by Daniel Lasaosa, Universidad P ublica de Navarra, Spain Denote u = 3 x y , v = 3 y z and w = 3 z x, hence clearly u3 + v 3 + w3 = 0, which is known to have integral solutions only if at least one of u, v, w is zero, while (x + y + z )(x2 + y 2 + z 2 xy yz zx) x3 + y 3 + z 3 xyz = = 3 3 = (x + y + z )(u6 + v 6 + w6 ) , 6

(x + y + z )(u6 + v 6 + w6 ) = 12060u, where we have assume wlog by cyclic symmetry in the variables that u = max{u, v, w}. Note also that u 0, with equality i u = v = w = 0, since if u 0, then 0 = u3 + v 3 + w3 0, equality must hold, hence u = v = w = 0. We then have two possible cases: If u = 0, then u = v = w = 0, or x = y = z . Note that any x = y = z results in both sides of the proposed equation being zero, hence (x, y, z ) = (r, r, r) is a solution for any integer r. If u > 0, and since one of v, w is necessarily zero, while v 3 + w3 = u3 , then the other one necessarily equals u, ie (u, v, w) is some permutation of (u, 0, u), yielding (x + y + z )u5 = 6030 = 2 32 5 67, or since the RHS is not divisible by any fth power, u = 1, and x + y + z = 6030, where x = y + 1. Now, since one of v, w is zero, either z = y or z = x, yielding respectively 3y + 1 = 6030 or 3y + 2 = 6030, impossible since 6030 is a multiple of 3, but 3y + 1, 3y + 1 are not. It follows that (x, y, z ) is an integral solution of the proposed equation if x = y = z = r for some integer r.

Mathematical Reflections 5 (2010)

31

O174. The point O is considered inside of the convex quadrilateral ABCD of area S . Suppose that K, L, M, N are interior points of the sides AB, BC, CD, DA, respectively. If OKBL and OM DN are parallelograms of areas S1 and S2 , respectively, prove that (a) S1 + S2 < 1.25 S ; sin(2+ ) 4 (b) S1 + S2 < C0 S, where C0 = max0<< . cos 4 Proposed by Nairi Sedrakyan, Yerevan, Armenia

Solution by the authors Without loss of generality we can suppose that points O and D are not on dierent sides of 1 the line AC. Let SABC = a, SACD = b, and SOAC = x. We have SOKB = SOBL = SKLB = S 2 and KB BL SKBL SOKB SOBL = = , SOAB SOBC AB BC SABC frpm which we nd S1 = S1 +
2SOAB SOBC . a

Similarly, we obtain S2 =

2SOAD SOCD ; b

hence

SOAB + SOBC SOAD + SOCD a+x bx S2 + = + = 2a 2a 2b 2b

a+ b a b x. 2 2ab

+ b If a b, then S1 + S2 a a + b = S. If a < b, then the point O can not be 2 outside the parallelogram ABCE, thus x a, so that a+ b a b b + 2ab a S1 + S2 a= . 2 2n 2ab sin(2+ 2aba b+ 2 4) Let a : a + b = C0 . Consequently b = tan , where 0, 4 . Then cos 2b sin(2+/4) 2aba C0 S. When = S1 + S2 b+ = 1, that is C0 1. Thus in all cases 4, cos 2b S1 + S2 C0 S. If the quadrilateral satises the following conditions: AB = BC, AD = sin(20 + SABC 4) CD, tan where C = and ABCO is a parallelogram, then S1 + S2 = C0 S. 0 0 cos 0 < a) To prove the inequality it is sucient to prove that when 0 4 , sin 2 + 4 4 1.25 cos . Indeed, let 0, 4 and cos = 5 , then, if 0 < , then sin(2 + ) 1 = 5 3 2 1 = tan 4 cos . And if 4 , then tan = 4 > 8 ; hence > 8 and

sin 2 + 4

sin 2 + 4

2 31 2 5 = < 1.25 cos . 2 25 2 4

Remark. Using derivatives it is possible to prove that tan 0 =


3

2+1

2 1 = 0.59 . . .

while C0 = 1.11 . . . .

Mathematical Reflections 5 (2010)

32

Junior problems
2 J175. Let a, b (0, 2 ) such that sin a + cos 2b 1 2

sec a and sin2 b + cos 2a

1 2

sec b. Prove that

1 cos6 a + cos6 b . 2 Proposed by Titu Andreescu, University of Texas at Dallas, USA

Solution by Prithwijit De, HBCSE, India We will use the following well-known trigonometric identities (a) sin2 x = 1 cos2 x, (b) cos 2x = 2 cos2 x 1, 1 . (c) sec x = cos x The inequalities can be written as 2 cos2 b cos a cos3 a and 1 2 (1)

1 2 cos2 a cos b cos3 b . 2

(2) . Now by 2

The signs of the inequalities are preserved because cos x is positive when x 0, squaring both sides of (1) and (2) and adding them we get 1 cos6 a + cos6 b . 2

Also solved by Arkady Alt, San Jose, California, USA; Daniel Lasaosa, Universidad P ublica de Navarra, Spain; Perfetti Paolo, Dipartimento di Matematica, Universit` a degli studi di Tor Vergata Roma, Italy; Tigran Hakobyan, Armenia.

Mathematical Reflections 6 (2010)

J176. Solve in positive real numbers the system of equations x1 + x2 + + xn = 1 1 1 1 1 3 x1 + x2 + + xn + x1 x2 xn = n + 1. Proposed by Neculai Stanciu, George Emil Palade Secondary School, Buzau, Romania

Solution by Tigran Hakobyan, Armenia We have 1 1 n2 1 + + + = n2 x1 x2 xn x1 + x2 + + xn 1 x1 x2 xn 1


x1 +x2 ++xn n n

and

= nn .

Thus, n3 + 1 nn + n2 which implies that n 2. If n = 1 we get a contradiction. For n = 2 we get x1 + x + 2 = 1 1 1 1 x1 + x2 + x1 x2 = 9 which is (n, x1 , x2 )
2 2 1 2, 1 3 , 3 , 2, 3 , 3

Also solved by Arkady Alt, San Jose, California, USA; Daniel Lasaosa, Universidad P ublica de Navarra, Spain; Perfetti Paolo, Dipartimento di Matematica, Universit` a degli studi di Tor Vergata Roma, Italy; Lorenzo Pascali, Universit` a di Roma La Sapienza, Roma, Italy.

Mathematical Reflections 6 (2010)

J177. Let x, y, z be nonnegative real numbers such that ax + by + cz 3abc for some positive real numbers a, b, c. Prove that x+y + 2 y+z + 2 z+x 1 + 4 xyz (abc + 5a + 5b + 5c). 2 4

Proposed by Titu Andreescu, University of Texas at Dallas, USA

Solution by the author From the given condition, 3a ax y z + + bc c b x by z 3b + + c ca a x y cz 3c + + . b a ab ax by cz + + bc ca ab

Then 3(a + b + c) hence abc + 5(a + b + c)

x+y y+z z+x + + + c a b

x+y + 2c + c

y+z + 2a + a (y + z ) + 2

2 2(x + y ) + 2

z+x ax by cz + 2b + abc + + + b bc ca ab (z + x) + 4 4 xyz

and the conclusion follows. The equality holds if and only if x + y = 2c2 , y + z = 2a2 , z + x = 2b2 by cz 2 2 2 2 2 2 2 2 2 2 2 2 2 2 2 and ax bc = ca = ab = abc. This implies b c = c a = 2c , c a + a b = 2a , a b + b c = 2b , that is b2 + a2 = c2 + b2 = a2 + c2 = 2, implying a = b = c = 1 and x = y = z = 1. If a = b = c = x = y = z = 1, the equality, as well as the condition of the problem, hold

Mathematical Reflections 6 (2010)

J178. Find the sequences of integers (an )n0 and (bn )n0 such that n 1+ 5 (2 + 5) = an + bn 2 for each n 0. Proposed by Dorin Andrica, Babes-Bolyai University, Cluj-Napoca, Romania First solution by Arkady Alt, San Jose, California, USA Let pn = and qn = for n = 1, 2, . . . . Then 2 + satisfy the same recurrence 5
n

2+

+ 2 2

2+

= pn + qn 5, n = 0, 1, 2, . . . and both obtained sequences xn+1 = 4xn + xn1 , n N (1)

2 2 5

with initial conditions p0 = 1, p1 = 2, q0 = 0, q1 = 1. It is clear that (pn )n0 and (qn )n0 are sequences of nonnegative integers and since 1+ 5 bn bn an + bn = pn + qn 5 an + + 5 = p n + qn 5 2 2 2 b n an + = pn 2 b n = qn 2 an = pn qn , n N {0} bn = 2qn we have that (an )n0 and (bn )n0 are sequences of integers and can be dened independently by recurrence (1) with initial conditions a0 = 1, a1 = 1, b0 = 0, b1 = 2. In explicit form n n n n 2+ 5 2 5 51 2+ 5 + 5+1 2 5 bn = , an = . 5 2 5 Second solution by Daniel Lasaosa, Universidad P ublica de Navarra, Spain 5 by Using Newtons binomial formula, exchanging n n 5 in the rst term results n in exchanging 5 by 5 in the second term, i.e., (2+ 5) +(2 5) = 2 a + b and (2+ 5) (2 5)n = n n 5bn , yielding (2 + 5)n (2 5)n ( 5 1)(2 + 5)n + ( 5 + 1)(2 5)n bn = , an = . 5 2 5
Mathematical Reflections 6 (2010)

The fact that an , bn are integers for all n may be easily proved considering that they are solutions of the recursive equations an = 4an1 + an2 and bn = 4bn1 + bn2 , with initial conditions a0 = a1 = 1 and b0 = 0, b1 = 2. Also solved by Ivan Dinkov Gerganov, P. R. Slaveikov Secondary School, Bulgaria; Lorenzo Pascali, Universit` a di Roma La Sapienza, Roma, Italy; Tigran Hakobyan, Armenia.

Mathematical Reflections 6 (2010)

J179. Solve in real numbers the system of equations 3 3 3 3 (x + y )(y z ) = 3(z x)(z + x ) (y + z )(z 3 x3 ) = 3(x y )(x3 + y 3 ) (z + x)(x3 y 3 ) = 3(y z )(y 3 + z 3 ) Proposed by Titu Andreescu, University of Texas at Dallas, USA

Solution by Lorenzo Pascali, Universit` a di Roma La Sapienza, Roma, Italy Without loss of generality assume that x = 0, then it is clear that y = z = 0. Because the given system is symmetric we can assume that x, y, z = 0. Assume that x = y, then x = z or y = z. If we assume that y = z then the rst equation becomes 4x4 = 6x4 , contradiction. Now x = y = z = 0 and after multiplying all three equations we get 3(x2 xy + y 2 )3(y 2 yz + z 2 )3(z 2 zx + x2 ) =1 (x2 + xy + y 2 )(y 2 + yz + z 2 )(z 2 + zx + x2 ) which can be written as 1 2xy (x y )2 + 3xy 1 2yz (y z )2 + 3yz 1 2zx (z x)2 + 3zx
1 3

1 . 27

It is not dicult to see that each factor of the LHS is greater than diction. So the only possible solution is x = y = z.

which leads to a contra-

Also solved by Arkady Alt, San Jose, California, USA; Daniel Lasaosa, Universidad P ublica de Navarra, Spain; Tigran Hakobyan, Armenia.

Mathematical Reflections 6 (2010)

J180. Let a, b, c, d be distinct real numbers such that 1 1 1 1 + + + = 0. 3 3 3 ab bc cd da Prove that 3 a b + 3 b c + 3 c d + 3 d a = 0. 3 Proposed by Dorin Andrica, Babes-Bolyai University, Cluj-Napoca, Romania

Solution by Anthony Erb Lugo Let 3 a b, 3 b c, 3 c d, 3 d a by w, x, y, z , respectively and let S = w + x + y + z . So that, w3 + x3 + y 3 + z 3 = 0() and 1 1 1 1 + + + = 0() w x y z

Since a, b, c and d are distinct, we can conclude that w, x, y, z = 0, and thus, wxy + wxz + wyz + xyz = wxyz Furthermore, using (*), we have that, S 3 = 6(wxy + wxz + wyz + xyz ) + 3(w2 + x2 + y 2 + z 2 )S S (S 2 3(w2 + x2 + y 2 + z 2 )) = 6(wxy + wxz + wyz + xyz ) Which implies that S = 0, otherwise the right hand side would be 0 which is a contradiction of ( ), and were done. Also solved by Arkady Alt, San Jose, California, USA; Daniel Lasaosa, Universidad P ublica de Navarra, Spain; Lorenzo Pascali, Universit` a di Roma La Sapienza, Roma, Italy; Perfetti Paolo, Dipartimento di Matematica, Universit` a degli studi di Tor Vergata Roma, Italy; Tigran Hakobyan, Armenia; Daniel Campos Salas, Costa Rica. 1 1 1 1 + + + w x y z = 0( )

Mathematical Reflections 6 (2010)

Senior problems S175. Let p be a prime. Find all integers a1 , . . . , an such that a1 + + an = p2 p and all solutions to the equation pxn + a1 xn1 + + an = 0 are nonzero integers. Proposed by Titu Andreescu, University of Texas at Dallas, USA and Dorin Andrica, Babes-Bolyai University, Cluj-Napoca, Romania

Solution by Daniel Lasaosa, Universidad P ublica de Navarra, Spain Let r1 , r2 , . . . , rn be the n nonzero integral roots of the equation given in the problem statement, then the equation rewrites as p(x + r1 )(x + r2 ) . . . (x + rn ) = 0, or all its coecients are nonzero integral multiples of p. In particular, ak is p times the sum of all possible products of k of the ri s, hence p + a1 + a2 + + an = p(1 + r1 )(1 + r2 ) . . . (1 + rn ) = p2 , yielding (1 + r1 )(1 + r2 ) . . . (1 + rn ) = p, where each one of the 1 + ri are integers other than 1, ie, n 1 of them must be equal to 1, and the remaining one must be p when n is odd and p when n is even; in other words, n 1 of the ri s are equal to 2, and the other one is p 1 when n is 1 odd and p 1 when n is even. Now, there are n k1 products of k of the ri such that exactly 1 one of them is not 2, and n products of k of the ri such that all of them are 2, ie, when k n is odd, ak = p n1 n1 (n 1)! (2)k1 (p 1) + (2)k = (2)k1 (pk + k 2n), k1 k (n k )!k !

and when n is even, ak = p n1 n1 (n 1)!(2n + pk k ) (2)k1 (p 1) + (2)k = (2)k1 . k1 k (n k )!k !

Note now that, when n is odd,


n k=1

ak = p

n1

(p + 1)
k1=0

(n 1)! (2)k1 + (n k )!(k 1)!

n k=0

n! (2)k 1 = (n k )!k !

= (p + 1)(1 2)n1 + (1 2)n 1 = p 1, while when n is even,


n k=1

ak = (p 1) p

n1 k1=

(n 1)! (2)k1 + (n k )!(k 1)!

n k=0

n! (2)k 1 = (n k )!k !

= (p 1)(1 2)n1 + (1 2)n 1 = p 1. There can be no other solutions. Also solved by Anthony Erb Lugo; Tigran Hakobyan, Armenia.

Mathematical Reflections 6 (2010)

S176. Let ABC be a triangle and let AA1 , BB1 , CC1 be cevians intersecting at P . Denote by Ka = KAB1 C1 , Kb = KBC1 A1 , Kc = KCA1 B1 . Prove that KA1 B1 C1 is a root of the equation x3 + (Ka + Kb + Kc )x2 4Ka Kb Kc = 0. Proposed by Ivan Borsenco, Massachusetts Institute of Technology, USA

First solution by Arkady Alt, San Jose, California, USA Without loss of generality assume that area of triangle ABC is 1. Let pa , pb , pb be the baricentric coordinates of P, that is pa , pb , pc > 0, pa + pb + pc = 1 and pa P A + pb P B + pc P C = 0. Since AC1 AC1 pb [ACC1 ] = = and then BC1 pa [BCC1 ] BC1 [ACC1 ] = Also, [AB1 C1 ] AB1 pc = = yields [CB1 C1 ] CB1 pa [AB1 C1 ] pc = . [ACC1 ] pc + pa Hence, Ka = [AB1 C1 ] = pb pc . Similarly, (pa + pb ) (pc + pa ) pc pa pa pb , Kc = . (pb + pc ) (pa + pb ) (pc + pa ) (pb + pc ) pb pb [ABC ] = . pa + pb pa + pb

Kb = Let K = [A1 B1 C1 ] , then

K = [ABC ] ([AB1 C1 ] + [BC1 A1 ] + [CA1 B1 ]) pb pc = 1 (Ka + Kb + Kc ) = 1 (pa + pb ) (pc + pa ) cyc = On the other hand, since Ka pb + pc Kb pc + pa Kc pa + pb = , = , = K 2pa K 2pb K 2pc then Ka Kb Kc (pa + pb ) (pb + pc ) (pc + pa ) 1 = = . 3 K 8pa pb pc 4K
9

2pa pb pc . (pa + pb ) (pb + pc ) (pc + pa )

Mathematical Reflections 6 (2010)

Thus, K 2 = 4Ka Kb Kc and, therefore, K = 1 (Ka + Kb + Kc ) K 3 = K 2 (Ka + Kb + Kc ) K 2 K 3 + (Ka + Kb + Kc ) K 2 4Ka Kb Kc = 0.

Second solution by Daniel Campos Salas, Costa Rica Let x, y, z be the areas of the triangles BP C , CP A, AP B , respectively. Its easy to prove that y AC1 AB1 z B1 C = x and C1 B = x . Therefore we have that 1 1 zAC yAB yz (x + y + z ) Ka = AB1 AC1 sin A = sin A = . 2 2 x+z x+y (x + y )(x + z ) The expressions for Kb and Kc are obtained analogously. It follows that KA1 B1 C1 equals (x + y + z ) 1 = zx xy yz (x + y )(x + z ) (y + z )(y + x) (z + x)(z + y ) 2xyz (x + y + z ) . (x + y )(y + z )(z + x)

Finally, note that


3 KA + KA1 B1 C1 (Ka + Kb + Kc ) 1 B1 C1 2 = KA (KA1 B1 C1 + Ka + Kb + Kc ) 1 B1 C1 2

2xyz (x + y + z ) ) (x + y )(y + z )(z + x) 4(xyz )2 (x + y + z )3 = ((x + y )(y + z )(z + x))2 = 4Ka Kb Kc ,

(x + y + z )

which proves that KA1 B1 C1 is a root of the given polynomial. Also solved by Daniel Lasaosa, Universidad P ublica de Navarra, Spain.

Mathematical Reflections 6 (2010)

10

S177. Prove that in any acute triangle ABC, sin B C 5R + 2r A + sin + sin . 2 2 2 4R

Proposed by Titu Andreescu, University of Texas at Dallas, USA

First solution by Arkady Alt, San Jose, California, USA A B C Since r = 4R sin sin sin , the inequality can be rewritten as 2 2 2 sin A B C A B C 5 + sin + sin 2 sin sin sin . 2 2 2 2 2 2 4 ( A)

Inequality (A) is an immediate corollary from more general inequality represented by the following theorem Theorem. Let k be any real number such that k k where k = Then for any , , 0, 2 2 4 2 1.2835. 2+3

, such that + + = the following inequality holds 4 2 12 k . 8 (M )

sin + sin + sin k sin sin sin

Proof. Assuming, due symmetry, that and, denoting = + we obtain , = , where the new variables and satisfy the inequalities 0 < = 2 or equivalently 2 4 4 3 . (1) 2 2 2 Since sin + sin + sin k sin sin sin = 2 sin = 2 sin + cos + sin (1 k sin sin ) 2 2

cos + cos (1 k sin sin ( )) 2 2

then inequality (M) can be equivalently rewritten as 2 sin 12 k cos + cos (1 k sin sin ( )) 2 2 8 (2)

where variables and are subject to the system (1).


Mathematical Reflections 6 (2010)

11

Let for any xed , 4 3 2 , . Indeed, 2 2

h () = 2 sin

cos + cos (1 k sin sin ( )) 2 2 2 and k > = 1. 154 7. We will prove that h () is decreasing on 3

h () = 2 sin on 2

sin

k cos cos 2 2

2 , since sin 0, k > and 2 2 2 3 sin k cos cos sin k cos cos 2 2 2 2 3 1 k sin k cos cos = 6 3 6 2 2 2 1 3 k 1 3 1 < 2 2 2 2 2 3 = 0. and it remains to prove the inequality (3)

Thus, h () h

= 2 sin + cos 1 k sin2 2 2 2 2 sin

12 k + cos 1 k sin2 . 2 2 8

Let t = sin

1 then sin t and (3) is equivalent to 2 8 2 1 kt2 12 k 16kt4 (8k + 16) t2 + 16t (4 k ) 0 8 (1 2t)2 k (2t + 1)2 4 0

2t + 1 2t2

2 because k (2t + 1)2 k 2 sin + 1 = 4. Since equality in (2) occurs if and only if = 8 2 and = = and = then in (M) equality occurs if and only if = = = . 3 6 3 6 4 A B C In particular for k = 2 and k = , replacing (, , ) in (M) with , , , for any acute 3 2 2 2

triangle ABC we, respectively, obtain inequality (A) and inequality sin B C 4 A B C 4 A + sin + sin sin sin sin . 2 2 2 3 2 2 2 3 (G)

(The last one is an inequality due to J. Garfuncel, given in [RAGI] without proof in a private communication.) Remark.
Mathematical Reflections 6 (2010)

12

The original inequality immediately follows from (G). Indeed, sin A B C A B C A B C 4 A B C + sin + sin 2 sin sin sin = sin + sin + sin sin sin sin 2 2 2 2 2 2 2 2 2 3 2 2 2 A B C 2 sin sin sin 3 2 2 2 4 2 A B C 4 2 1 sin sin sin 3 3 2 2 2 3 3 8 5 = . 4

[RAGI]. Mitrinovi c D.S., Pe cari c J. E. , Volenec V. Recent Advances, Geometric Inequality, p.269, inequality 5.10. Second solution by Daniel Lasaosa, Universidad P ublica de Navarra, Spain
A B = , B = . It is well known that Assume wlog that C B A, and denote A+ 4 4 A B C r = 4R sin 2 sin 2 sin 2 , or the proposed problem is equivalent to showing that

sin

A B C A B C 5 + sin + sin 2 sin sin sin . 2 2 2 2 2 2 4

Assume that C is known such that the LHS is minimum, or for that value of C , dene f (x, y ) = sin x + sin y 2 sin x sin y sin where x + y = f A B , 2 2
A+B 2

C , 2

= 90

C 2

is xed. Now, C = 2

f (, ) = f ( , + ) f (, ) = 2 sin (cos 1) + 2 sin2 sin = 4 sin2 2 2 cos2 C sin sin . 2 2

A 2 2 2 Now, if > 0, and since = B < B 4 4 < 45 , we have sin 2 > 0, 2 cos 2 > 2 cos (45 ) > 1. C A+B Moreover, 2 > 4 , since equality would only hold i A = B = C , which is not true because B A+B A+B > 0. Thus, f A , with equality i A = B = 90 C 2, 2 f 4 , 4 2 . It therefore suces to show that, for all 90 > C 60 , we have

5 2u + (1 2u2 ) 2u2 (1 2u2 ) . 4


C C where we have dened u = sin 45 C = 1 2u2 . 4 , and therefore sin 2 = cos 2 45 4 2 2 After some algebra, this last inequality is equivalent to (2u 1) (4u + 8u + 1) 0. Since C 90 > C 60 , we have 45 2 < 45 4 30 , or u > 0. The conclusion follows, equality 1 C holds i u = 2 , ie i 45 4 = 30 , or C = 60 . We conclude that equality holds i ABC is equilateral.

Third solution by Neculai Stanciu, George Emil Palade, Romania


Mathematical Reflections 6 (2010)

13

We have sin A B C 5R + 2r A r 3 + sin + sin 2 sin 1 + + 2 2 2 4R 2 R 2 A+B B+C C +A A+B+C 2 cos + 2 cos + 2 cos cos A + cos B + cos C + 3 cos 2 2 2 3

which is true from Popovicius Inequality for the concave function cos x on the interval 0, 2 . Also solved by Daniel Campos Salas, Costa Rica

Mathematical Reflections 6 (2010)

14

S178. Prove that there are sequences (xk )k1 and (yk )k1 of positive rational numbers such that for all positive integers n and k , n 1+ 5 (xk + yk 5) = Fkn1 + Fkn , 2 where (Fm )m1 is the Fibonacci sequence. Proposed by Dorin Andrica, Babes-Bolyai University, Cluj-Napoca, Romania

Solution by G.R.A.20 Problem Solving Group, Roma, Italy


1 Take xk = 1 2 Lk and yk = 2 Fk (see problem J178) where Ln is the n-th Lucas number. Since k k k k 1 1+ 5 1 5 1 5 1+ 5 Fk = + and Lk = 2 2 2 2 5

it follows that (xk + yk 5) =


n

1+ 5 2

kn

= Fkn1 + Fkn

1+ 5 2

Also solved by Arkady Alt, San Jose, California, USA; Daniel Lasaosa, Universidad P ublica de Navarra, Spain; Daniel Campos Salas, Costa Rica.

Mathematical Reflections 6 (2010)

15

S179. Find all positive integers a and b for which

(a2 +1)2 ab1

is a positive integer.

Proposed by Valcho Milchev, Petko Rachov Slaveikov Secondary School, Bulgaria

Solution by Daniel Campos Salas, Costa Rica Note ab 1 divides (ab 1)(2a2 + ab + 1) = (a2 + ab)2 (a2 + 1)2 . This implies that ab 1 divides a2 (a + b)2 , but ab 1 and a are coprime, from where we conclude that ab 1 divides a+b)2 (a + b)2 . Let k = (ab 1 . Note that it is not possible for a, b to be equal, since it would imply = 4 + a24 to be an integer, and 4 has no divisor of the form a2 1. The equation can 1 also be written in the form a2 (k 2)ab + b2 + k = 0. (1) Note that for every pair of integers (a, b) satisfying the equation k =
(a+b)2 ab1 , 4a2 a2 1

the pair a, a
a2 +k b

2 +k

is an and its permutations are also positive integer solutions (its easy to prove that integer). Iterating this process we nd that every solution is part of a family of solutions. Let (a0 , b0 ) be a solution for any of these families with the minimal sum. We may assume without a2 +k a2 +k a2 +k loss of generality that b0 > a0 . This implies that 0b0 b0 since a0 , 0b0 and 0b0 , a0 are also solutions to the equation. It follows that
2 k b2 0 a0 = (b0 a0 )(a0 + b0 ) a0 + b0 .

This implies that

(a0 +b0 )2 a0 b0 1

= k a0 + b0 , or equivalently, 2 (a0 1)(b0 1). Note that a0 < 3,


(b0 +1)2 b0 1

since a0 < b0 . If a0 = 1, it follows that

is an integer. This implies that


(b0 +2)2 2b0 1

4 b0 1 ,

so it

follows that b0 can be equal to 2, 3 or 5. If a0 = 2, it follows that is an integer. This implies that 2b25 , or equivalently, b can equal 3 or 13, since b > a = 2. 0 0 0 0 1 For (a0 , b0 ) = (1, 2) it follows that k = 9. This initial solution generates the family of solutions given by the pairs, (cn , cn+1 ), n 0, which satisfy, c0 = 1, c1 = 2, cn+2 = 7cn+1 cn , where the recursion holds by (1). The explicit expression for cn is given by the formula cn = 51 2 5 n 7+3 5 5+1 73 5 + 2 2 2 5 4n1 4n1 1+ 5 1 5 1 2 2 5
n

1 5

= F4n1 , where Fn is the n-th term of the Fibonacci sequence dened by F0 = 0, F1 = 1 and Fn+2 = Fn+1 + Fn (abusing of the denition we set F1 = 1).

Mathematical Reflections 6 (2010)

16

For (a0 , b0 ) = (1, 3) it follows that k = 8. This initial solution generates the family of solutions given by the pairs, (dn , dn+1 ), n 0, which satisfy, d0 = 1, d1 = 2, dn+2 = 6dn+1 dn , where the recursion holds by (1). The explicit expression for dn is given by the formula dn = = n 1 n 1 3+2 2 + 32 2 2 2 1 ( 2 + 1)2n + ( 2 1)2n . 2

For (a0 , b0 ) = (1, 5) it follows that k = 9. This initial solution generates the family of solutions given by the pairs, (en , en+1 ), n 0, which satisfy, e0 = 1, e1 = 2, en+2 = 7en+1 en , where the recursion holds by (1). The explicit expression for en is given by the formula en = 5+1 2 5 n 7+3 5 51 73 5 + 2 2 2 5 4n+1 4n+1 1+ 5 1 5 1 2 2 5
n

1 5

= F4n+1 . We may interpret this solution as an extension of the rst one for negative indexes since F4n+1 = F4n1 . For (a0 , b0 ) = (2, 3) it follows that k = 5. This initial solution generates the family of solutions given by the pairs, (fn , fn+1 ), n 0, which satisfy, f0 = 1, f1 = 2, fn+2 = 3dn+1 fn , where the recursion holds by (1). The explicit expression for fn is given by the formula fn = 3+ 5 2 1+ 5 2 3 5 + 2 2n 1 5 + 2
n n

2n

= L2n , where Ln is the n-th term of the Lucas sequence dened by L0 = 2, L1 = 1 and Ln+2 = Ln+1 + Ln . Therefore, we conclude that all the possible pairs of positive integers are of the form (F4n1 , F4n+3 ), (L2n , L2n+2 ) (for all integers n) and (dn , dn+1 ) (dened above), and its permutations, and were done. Also solved by Tigran Hakobyan, Armenia

Mathematical Reflections 6 (2010)

17

S180. Solve in nonzero real numbers the system of equations x4 y 4 = x4 +


121x122y 4xy x+121y 2 2 14x y + y 4 = 122 . x2 +y 2

Proposed by Titu Andreescu, University of Texas at Dallas, USA

Solution by Daniel Lasaosa, Universidad P ublica de Navarra, Spain Note rst that x4 + 14x2 y 2 + y 4 = 4(x2 + y 2 )2 3(x2 y 2 )2 = s4 s2 d2 + d4 , where we have 2 2 dened x + y = s and x y = d, while x4 y 4 = sd(x2 + y 2 ) = sd(s 2+d ) , 4xy = s2 d2 , 2 d2 x2 + y 2 = s + 2 . Therefore, the system may be rewritten as (s4 We can then obtain (243d s)(243s + d) = sd(s2 + d2 )(s2 d2 )(243s + d) = (s4 s2 d2 + d4 )(s2 + d2 )(243d s). Since s2 + d2 > 0 (otherwise x = y = 0, in contradiction with the problem statement), it follows that 243s2 d(s2 d2 ) + sd2 (s2 d2 ) = 243d(s4 s2 d2 + d4 ) s(s4 s2 d2 + d4 ), which after simplication yields s5 = 243d5 , or s = 3d. Substitution in both equations yields d6 = d, or since x = y (if x = y = 0 the LHS of the rst equation would be zero, but the RHS would not), we nd that d5 = 1, ie s = 3 and d = 1 for x = 2, y = 1. These values can be clearly shown to satisfy the system by plugging them into the given equations, and no other solutions exist. Second solution by Arkady Alt, San Jose, California, USA Since x4 + 14x2 y 2 + y 4 and x, y = 0 then x4 + 14x2 y 2 + y 4 x2 + y 2 x2 + y 2 (x y ) 4xy x4 y 4 (x + y ) x4 + 14x2 y 2 + y 4 (x y ) 4xy x2 y 2 (x + y ) x4 + 14x2 y 2 + y 4 (x y ) 4xy x2 y 2 (x + y ) x2 + y 2 = 122x + 121y, 4xy x4 y 4 = 121x 122y sd(s2 + d2 )(s2 d2 ) = 243d s, s2 d2 + d4 )(s2 + d2 ) = 243s + d.

= (122x + 121y ) (x y ) (121x 122y ) (x + y ) = x2 + y 2

= 1 (x y )5 = 1 x y = 1. Let t = x + y then x2 y 2 = t, x2 + y 2 =
Mathematical Reflections 6 (2010)

t1 t1 t2 + 1 , 4xy = t2 1, y = , 121x 122y = 121 2 2 2


18

and the equation x4 y 4 =

121x 122y becomes 4xy

t t4 1 t1 = 121 t t4 1 + t 1 = 242 t5 = 243 t = 3. 2 2 Hence, xy =1 x+y =3 x = 2, y = 1.

Mathematical Reflections 6 (2010)

19

Undergraduate problems U175. What is the maximum number of points of intersection that can appear after drawing in a plane l lines, c circles, and e ellipses? Proposed by Dorin Andrica, Babes-Bolyai University, Cluj-Napoca, Romania

Solution by Andrea Ligori and Emanuele Natale, Universit` a di Roma Tor Vergata, Roma, Italy We note that the intersection of two circles yields 2 points, so the contribution is 2
c 2

the intersection of a line with a circle or an ellipse yields 2 points, so the contribution is 2l(c + e); the intersection of an ellipse with a circle or another ellipse yields 4 points, so the contrie + 4ec; bution is 4 2 the intersection of two incindent lines yields 1 point so the contribution is Therefore the nal formula is 2 l e c . + 4ec + + 2l(c + e) + 4 2 2 2
l 2

It is easy to nd a conguration of l lines, c circles, and e ellipses with such a number of intersection points. Also solved by Daniel Lasaosa, Universidad P ublica de Navarra, Spain.

Mathematical Reflections 6 (2010)

20

U176. In the space, consider the set of points (a, b, c) where a, b, c {0, 1, 2}. Find the maximum number of non-collinear points contained in the set. Proposed by Ivan Borsenco, Massachusetts Institute of Technology, USA

Solution by Daniel Lasaosa, Universidad P ublica de Navarra, Spain It is possible to have 16 non-collinear points, taking for example the following ones: (0, 0, 0), (0, 1, 0), (1, 0, 0), (1, 2, 0), (2, 1, 0), (2, 2, 0), (0, 0, 1), (0, 2, 1), (2, 0, 1), (2, 2, 1), (0, 1, 2), (0, 2, 2), (1, 0, 2), (1, 2, 2), (2, 0, 2), (2, 1, 2). Consider a set of non-collinear points of the form (a, b, c) where a, b, c {0, 1, 2}. Note rst that for each pair of xed values (b, c), there may be at most two values of a such that (a, b, c) is in the set, otherwise if points with a = 0, 1, 2 are in the set, they would be collinear. The same reasoning applies to all pairs of xed values (c, a) and (a, b). It follows that, for each xed value of c, there are at most six pairs of values (a, b) such that (a, b, c) is in the set, and such that no three of them have the same value of a or of b. Assume now that, for a given value of c and two distinct values b1 , b2 of b, the same two values a1 , a2 of a exist such that (a, b, c) is in the set. Calling b3 the third possible value of b, note that neither (a1 , b3 , c) nor (a2 , b3 , c) may be in the set, ie, at most ve points with the aforementioned value of c are in the set. We conclude that, if there are six points with the same value of c, then the pairs of values of a such that (a, b, c) is in the set as b takes its three possible values, must be distinct and therefore must be (0, 1), (1, 2), (0, 2), each pair corresponding to each one of the possible values of b. Finally, if (0, 1, c) and (1, 1, c) are in the set, note that (0, 0, c) cannot be in the set, since neither (0, 2, c) nor (2, 2, c) could be on the set, and similarly for (2, 1, c) by symmetry, ie, if there are six points which have the same value of c, they must be either (0, 0, c), (0, 1, c), (1, 0, c), (1, 2, c), (2, 1, c) and (2, 2, c), or (0, 1, c), (0, 2, c), (1, 0, c), (1, 2, c), (2, 0, c) and (2, 1, c). Assume that it is possible to have 17 non-collinear points. By the previous arguments, exactly two of the planes with constant value of c must contain exactly 6 points, and the remaining plane must contain exactly 5 points. Moreover, both plains containing exactly 6 points must have a dierent combination out of the two mentioned above, since if (a, b, c) belongs in the set for two distinct values of c, and six distinct pairs of values (a, b), then (a, b, c) cannot be in the set for each one of this six pairs of values and the third value of c. It follows that, the plane containing exactly 5 points, cannot have points at (0, 1, c), (1, 0, c), (1, 2, c) or (2, 1, c), since they would be collinear with the corresponding points of the other two planes. Therefore the plane containing exactly 5 points contains at least points (0, 0, c), (1, 1, c) and (2, 2, c), which are collinear, contradiction. There cannot be more than 16 non-collinear points, but we have produced an example of such 16 non-collinear points, hence the maximum number of non-collinear points is 16. Also solved by Andrea Ligori, Universit` a di Roma Tor Vergata, Roma, Italy.

Mathematical Reflections 6 (2010)

21

U177. Let a1 , a2 , . . . , an and b1 , b2 , . . . , bn be integers greater than 1. Prove that there are innitely many primes p such that p divides bi
p1 ai

1 for all i = 1, 2, . . . , n.

Proposed by Gabriel Dospinescu, Ecole Normale Superieure, France

Solution by the author This is a very simple (but rather surprising) application of the following beautiful result. Theorem 1.1. (Nagell) Let f1 , f2 , . . . , fn be nonconstant polynomials with integer coecients. Then for innitely many primes p one can nd integers x1 , x2 , . . . , xn such that p divides fi (xi ) for all i. For the proof, we refer to T. Andreescu, G. Dospinescu, Problems from the Book. Now, consider the polynomials fi (X ) = X ai bi and add to their collection the polynomials ai , where n is the nth cyclotomic polynomial. Pick a prime p > b1 b2 bn + a1 a2 an as in the theorem and xi , yi such that p divides fi (xi ) and ai (yi ). As p does not divide any of the ai s, it is a classical property of cyclotomic polynomials that we must have p 1 (mod ai ). i Moreover, p clearly does not divide xi , since it does not divide bi . Since xa i bi (mod p) and ai 1 (mod p), Fermats little theorem implies that p divides bi
p1 ai

and we are done.

Mathematical Reflections 6 (2010)

22

U178. Let k be a xed positive integer and let Sn = Prove that


(0) (1) Sn + Sn cos (1) + Sn sin

(j )

n j

n j +k

+
2

n j +2k

+ , j = 0, 1, . . . , k 1.

2 2(k 1) (k1) + + Sn cos k k

2 4 2(k 1) (2) (k1) + Sn sin + + Sn sin k k k

= 2 cos

2n

Proposed by Dorin Andrica, Babes-Bolyai University, Cluj-Napoca, Romania

Solution by Arkady Alt, San Jose, California, USA Let Z+ = N {0} and Dj = {j + mk | m Z+ and j + mk n} . Then Sn
k1 (j )

=
p

n p

and

Dj = {0, 1, 2, . . . , n} . Let
j =0 k 1

a=
j =0 k 1

(j ) Sn cos

2j k 2j k
k 1 (j ) Sn cos j =0

b=
j =0

(j ) Sn sin

= cos

2 2 + i sin k k

2 2 + i sin k k

Then k = 1 and
k1

a + ib =
j =0 k 1

(j ) Sn cos

2j +i k

k1 (j ) Sn cos j =0

2j k

=
j =0 k 1

(j ) Sn cos

2j 2j + i sin k k
n p j =0 pDj n

k1 (j ) j Sn

=
j =0

=
n p p=1

=
pDj

n p

p =

p = (1 + )n
n

2 2 n + i sin = 2 cos cos + i sin k k k k k n n = 2 cos cos + i sin . k k k 1 + cos

Mathematical Reflections 6 (2010)

23

Hence, |a + ib| = = 2 cos k 2 cos k k


n n

cos

n n + i sin = 2 cos k k k n = 2 cos cos + i sin k k k

cos
n

+ i sin k k

Therefore, a2 +b2 = 2 cos

2n

Also solved by Daniel Lasaosa, Universidad P ublica de Navarra, Spain; Daniel Campos Salas, Costa Rica.

Mathematical Reflections 6 (2010)

24

U179. Let f : [0, ] R be a continuous function such that f (0) = 0 and f (2x) f (x) + x for all x 0. Prove that f (x) < x for all x [0, ]. Proposed by Samin Riasat, University of Dhaka, Bangladesh

Solution by Perfetti Paolo, Dipartimento di Matematica, Universit` a degli studi di Tor Vergata Roma, Italy Note that f (x) f and after n steps we have x f (x) f n + 2 The limit n yields f (x) lim x f n + 2
n k=1

x x x x x x x x x + f + + f + + + ... 2 2 4 4 2 8 8 4 2
n k=1

x 2n

x 2n

x = lim f n + lim n n 2

n k=1

x =0+x=x 2n

where we have used the continuity of f (x) and f (0) = 0 for writing
n

lim f

x 2n

=0

and we are done. Also solved by Arkady Alt, San Jose, California, USA; Daniel Lasaosa, Universidad P ublica de Navarra, Spain; John Mangual, UCSB; Emanuele Natale, Universit` a di Roma Tor Vergata, Roma, Italy; Tigran Hakobyan, Armenia; Daniel Campos Salas, Costa Rica.

Mathematical Reflections 6 (2010)

25

U180. Let a1 , . . . , ak , b1 , . . . , bk , n1 , . . . , nk be positive real numbers and a = a1 + + ak , b = b1 + + bk , n = n1 + + nk , k 2. Prove that


1

(a1 + b1 x)n1 (ak + bk x)nk dx


0

(a + b)n+1 an+1 . (n + 1)b

Proposed by Dorin Andrica, Babes-Bolyai University, Cluj-Napoca, Romania

Solution by Perfetti Paolo, Dipartimento di Matematica, Universit` a degli studi di Tor Vergata Roma, Italy By concavity of the logarithm we have
k k

ln

(aj + bk x)nj = n
j =1 j =1

nj ln(aj + bj x) n ln n

k j =1

nj (aj + bj x) n

Moreover the monotonicity (increasing) of the logarithm yields k k nj (aj + bj x) n ln (aj + bj x) = ln(a + bx)n n ln n
j =1 j =1

Now exponentiating and integrating we have


1 1

(a1 + b1 x)n1 (ak + bk x)nk dx


0 0

(a + bx)n dx =

(a + b)n+1 an+1 (n + 1)b

concluding the proof. Also solved by Arkady Alt, San Jose, California, USA; Daniel Lasaosa, Universidad P ublica de Navarra, Spain; Daniel Campos Salas, Costa Rica.

Mathematical Reflections 6 (2010)

26

Olympiad problems O175. Find all pairs (x, y ) of positive integers such that x3 y 3 = 2010(x2 + y 2 ). Proposed by Titu Andreescu, University of Texas at Dallas, USA

First solution by the author Write x = du, y = dv with d 1 and u, v relatively prime positive integers. The equation becomes d(u3 v 3 ) = 2010(u2 + v 2 ). Thus u2 + uv + v 2 divides 2010(u2 + v 2 ) and since it is relatively prime to u2 + v 2 , we deduce that u2 + uv + v 2 divides 2010 = 2 3 5 67. We claim that A = u2 + uv + v 2 actually divides 67. It is immediate that A is odd (if not, u, v must be both even). Next, it is easy to see that if 5 divides A, then 5 divides both u, v, a contradiction. Finally, if 3 divides A, we must have u v mod 3 and so u3 v 3 = (u v )A is a multiple of 9. Thus 2010(u2 + v 2 ) is a multiple of 9, which is not the case. Thus A is a divisor of 67 and since u, v 1, we deduce that u2 + uv + v 2 = 67. Clearly u v, then 67 3v 2 , thus v = 4. Considering each case, we deduce that v = 2, u = 7 and so d = 318. Hence there is one solution, x = 7d, y = 2d with d = 318. Also solved by Tigran Hakobyan, Armenia.

Mathematical Reflections 6 (2010)

27

O176. Let P (n) be the following statement: for all positive real numbers x1 , x2 , . . . , xn such that x1 + x2 + + xn = n, x2 x3 x1 n + + + . x1 + 2 x3 x2 + 2 x4 xn + 2 x2 3

Prove that P (n) is true for n 4 and false for n 9. Proposed by Gabriel Dospinescu, Ecole Normale Superieure, France

First solution by the author Let S (x1 , x2 , . . . , xn ) be the left hand side of the inequality. Using Holders inequality, we obtain S 2 (x2 (x1 + 2x3 ) + + x1 (xn + 2x2 )) (x1 + x2 + + xn )3 = n3 . On the other hand, we have x2 (x1 + 2x3 ) + + x1 (xn + 2x2 ) = 3(x1 x2 + x2 x3 + + xn x1 ). Using the fact that x1 x2 + x2 x3 + + xn x1 n whenever x1 + x2 + + xn = n and n 4. The last fact follows from the fact that ab + bc + ca and (a + b + c)2 3

(a + b + c + d)2 . 4 The conclusion follows easily for n 4. Chosing x1 , x2 , x3 , x4 close to n 4 and the other variables n equal and close to 0, one easily obtains that the expression is smaller than for n 9. The 3 conclusion follows. ab + bc + cd + da = (a + c)(b + d) Second solution by Arkady Alt, San Jose, California, USA Let n 4.Then, applying consequentially AM-GM and Cauchy inequalities, we obtain
n cyc

x2 3 (x1 + 2x3 ) 2( n k=1 xk +


n cyc

n cyc

2x2 =2 3 + ( x1 + 2 x3 ) + 2x2 x3 ) = 3n + 3

n cyc

x2 2 3x2 + (x1 x2 + 2x2 x3 )

3 Thus,

2 n k=1 xk ) n cyc (x1 x2

2n2

. n cyc x1 x2

2 n2 x2 . n x1 + 2 x3 3 n + cyc x1 x2
n cyc x1 x2

For n = 3, since x1 + x2 + x3 = 3 we have

(x1 + x2 + x3 )2 = 3. 3

Mathematical Reflections 6 (2010)

28

2 n2 2 Then = n n + x x 3 3 3+ cyc 1 2 If n = 4 then x1 + x2 + x3 + x4 = 4 and (x1 + x3 ) + (x2 + x4 ) 2 2 3 Let


2

n cyc x1 x2 n cyc x1 x2

2 9 3 = 3= . 6 3 3 = (x1 + x3 ) (x2 + x4 )

2 n2 = = 4.Therefore, n 3 n + cyc x1 x2 2 16 16 4 = . n 4 + cyc x1 x2 3 4+4 3 n n 9 and let xk = k , k = 1, 2, ..., n.Then 2 n


n2 k=1

L.H.S. = n 2n n 2n1 Since 1 2 n 3

x1 xk+1 xn = + + xk + 2xk+2 xn1 + 2x1 xn + 2 x2 n 2 n n +2 n 2 4 n 2k+1 = =


n2 k=1

n2 k=1

2k+1 n n + 2 k+2 k 2 2 1 2n2 n +2

n +2 2
n2 k=1

n 2k+1

2k+2 1

n + n +1 2 + 22n 1

1 n 2 = = 1 2 3 21 1 2 n n n 2+1 3 3 and < then L.H.S. < + + . n 2 3 2 2 2 2 1 +2 2n2 Moreover, since n 9 we obtain 3 3 2+1 3 3 1 1+ 3 1 1+ + + < + + <1+ + = 2. 976 < 3, n 9 2 2 2 2 2 2 2 2 2 29 n and , therefore, L.H.S. < 3n < . 3 So, P (n) is false for n 9. 1

2k+2

n2 k=1

< 3 2 2k1 n n n 2+1 , < n +1 2 n 6 2 2n 2 +2 2k+1

n 3

n2 k=1

Mathematical Reflections 6 (2010)

29

O177. Let P be point situated in the interior of a circle. Two variable perpendicular lines through P intersect the circle at A and B . Find the locus of the midpoint of the segment AB . Proposed by Dorin Andrica, Babes-Bolyai University, Cluj-Napoca, Romania

First solution by G.R.A.20 Problem Solving Group, Roma, Italy We can assume, without loss of generality, that P = t [0, 1] and the circle C = {|z | = 1}. Let A = z = x + iy C then B = w = si(z P ) + P C with some s > 0. Hence 1 = |w|2 = (t sy )2 + s2 (x t)2 The midpoint of the segment AB is given by M = (A + B )/2. Now we verify that |M P/2| = In fact, by (1), (2|M P/2|)2 = (x sy )2 + (s(x t) + y )2 = x2 + y 2 + 1 t2 = 2 t2 . Hence the required locus is a circle with center P/2 and radius 2 |P |2 /2. 2 |P |2 /2. (1)

In the general setting, if the circle C has center at P0 and radius R then the locus is a circle with center (P0 + P )/2 and radius 2R2 |P P0 |2 /2. Second solution by the author Let ABCD be a quadrilateral and let M and N be the midpoints of sides AB and CD, respectively. Using the Median Theorem it is easy to prove that the following relation holds : AC 2 + BD2 + BC 2 + DA2 = AB 2 + CD2 + 4M N 2 . Let M be the midpoint of the segment AB and let N be the midpoint of the segment OP , where O is the center of the given circle. Applying the relation above in the quadrilateral ABP O we obtain AP 2 + R2 + BP 2 + R2 = AB 2 + OP 2 + 4M N 2 . It is clear that AP 2 + BP 2 = AB 2 , hence we get 4M N 2 = 2R2 OP 2 , that is 1 2R2 OP 2 . 2 Since the point N is xed, it follows that the desired locus is the circle of center N and radius 1 2 2 2 2R OP . NM =

Also solved by Daniel Lasaosa, Universidad P ublica de Navarra, Spain; Daniel Campos Salas, Costa Rica; Tomas Calderon Gomez, Costa Rica.
Mathematical Reflections 6 (2010)

30

O178. Let m and n be positive integers. Prove that for each odd positive integer b there are innitely many primes p such that pn 1 (mod b)m implies bm1 | n. Proposed by Vahagn Aslanyan, Yerevan, Armenia

Solution by the author


k 1 2 Let b = p 1 p2 pk be the canonical factorization of b. Because b is odd pi > 2 for i = 1, 2, . . . , k. Let P = p1 p2 pk . Consider the sysmte of conguences ()x pi + 1 (mod p2 i ), i = 1, 2, . . . , k. By the Chinese Remainder Theorem the system () has solution. Let that solution 2 be x0 . We have x0 pi + 1 (mod p2 i ) for all i. If x x0 (mod P ), then x is solution of system ().

Claim. If x x0 (mod P 2 ), then from condition xn 1 (mod bm ) it follows that bm1 | n. Proof. Suppose p P is a prime and a is a positive integer. Let vp (a) be the degree of p in the canonical factorization of a. For each i, 1 i k, pi | x0 1. We know that pi is odd, therefore by a well knwon lemma vpi (xn 1) = vpi (x 1) + vpi (n). But x x0 pi + 1 n (mod p2 i ), so vpi (x 1) = 1. Hence vpi (n) = vpi (x 1) 1 mi 1 (m 1)i (because ( m 1) i | n. It is true for all i, and gcd(pi , pj ) = 1 whenever i = j, xn 1 (mod bm )). So pi ( m 1) k i m 1 | n. Now from Dirichlets theorem there are innitely many therefore b = i=1 pi primes p such that p x0 (mod P 2 ). Also solved by Tigran Hakobyan, Armenia.

Mathematical Reflections 6 (2010)

31

O179. Prove that any convex quadrilateral can be dissected into n 6 cyclic quadrilaterals. Proposed by Dorin Andrica, Babes-Bolyai University, Cluj-Napoca, Romania

Solution by Daniel Lasaosa, Universidad P ublica de Navarra, Spain Any convex quadrilateral is dissected into two triangles by either of its diagonals; any concave quadrilateral is dissected into two triangles by exactly one of its diagonals; any crossed quadrilateral is already formed by two triangles joined at one vertex, and where two of the sides of each triangle are on the straight line containing two of the sides of the other. In triangle ABC , let I be the incenter and D, E, F the points where the incircle touches respectively sides BC, CA, AB . Clearly, ABC my be dissected into three cyclic quadrilaterals AEIF , BF ID, CDIE . In triangle ABC , wlog acute at C , consider the circumcenter O, and take a point O on the perpendicular bisector of AB that is closer to AB than O. The circle with center O through A, B leaves C outside, hence it must intersect the interior of segments AC, BC at E, D, or ABDE is cyclic. We may then proceed as follows: write n = 3 + 3u + v , where u 1 is an integer and v {0, 1, 2}. Dissect (any) quadrilateral ABCD in two triangles, then dissect one of them into three cyclic quadrilaterals. If v = 0, dissect the other triangle into one cyclic quadrilateral and one triangle, and if v = 2, dissect again this latter triangle into one cyclic quadrilateral and one triangle. After this procedure, we have dissected the original quadrilateral into 3 + v cyclic quadrilaterals (3, 4, 5 respectively for v = 0, 1, 2) and one triangle. Dissect now this triangle into u triangles (for example dividing one of its sides in u equal parts and joining each point of division with the opposite vertex), and dissect now each one of these u triangles into three cyclic quadrilaterals. We have thus dissected the original quadrilateral into 3 + v + 3u = n cyclic quadrilaterals. Also solved by Daniel Campos Salas, Costa Rica.

Mathematical Reflections 6 (2010)

32

O180. Let p be a prime. Prove that each positive integer n p, p2 divides

n+p 2 p

n+2p 2p

n+p 2p

Proposed by Dorin Andrica, Babes-Bolyai University, Cluj-Napoca, Romania

First solution by G.R.A.20 Problem Solving Group, Roma, Italy Since by Wolstenholme Theorem, for any prime p > 2, ap bp a b
p1

(mod p ),

and
k=1

1 0 k

(mod p),

it is easy to verify by induction with respect to n = n/p p + [n]p 0 that for any prime p [n]p n + ap n/p + a 1 (mod p2 ). 1 + bp k bp b
k=1

Hence by letting x = n/p and y = n+p p


2

[n]p 1 k=1 k

we obtain x+2 x+1 + 2 2 (1 + 2py )

n + 2p n+p 2p 2p

(x + 1)2 (1 + py )2 p2 y 2 (x + 1)2 0

(mod p2 ).

Second solution by Campos Salas, Costa Rica Well prove it rst for p = 2. Its easy to verify that the expression equals 4 divisible by 4.
n+3 4

, and its

Suppose p 3. Note that the greatest power of p that divides (p!)2 and (2p)! is p2 . The following congruences will be taken modulo p2 . Its well-known that 2pp 2. This implies
p)! p!) that (2 2(p 2 , or equivalently p2 4 that p divides p p
2

p2 (p!)2

2p2 (2p)! .

Therefore, the problem is equivalent to prove

(n + i)
i=1 i=1

(n + p + i) +
i=1

(n p + i) 2
i=1

(n + i) .

It would suce to prove that p3 divides


p p p

(n + p + i) +
i=1 i=1 p i=1 (n

(n p + i) 2
i=1 p i=1 (n

(n + i).

Consider the expression ( p 3). Note that


Mathematical Reflections 6 (2010)

+ x + i)

+ i)) modulo x3 (this can be done since

33

(n + x + i)
i=1 i=1

(n + i)
i=1

(n + i) x2 s2 xs1 ,

where sk represents the sum of all the possible products of k dierent terms of the set 1 1 { n+1 , ..., n+ p }. Therefore, modulo p3 we have that
p p p p

(n + p + i) +
i=1 i=1

(n p + i) 2
i=1

(n + i) 2p2 s2
i=1

(n + i).

Were left to prove that p divides s2 p i=1 (n + i). Its clear that this expression, modulo p, is congruent to s2 p!, where s2 represents the sum of all the possible products of 2 dierent terms 1 1 of the set { 1 1 , ..., p }. If a product doesnt have p as one of its factors, then is divisible by p 1 1 when multiplied by p!. Then, it is enough to prove that p divides 1 1 + 2 + ... + p1 which is true by grouping the k -th term and the (p k )-th term, and were done. Third solution by the author According to the Li-Jen-Shu fromula, for any positive integers n and p we have
p

k=0

p k

n + 2p k 2p

n+p p

It follows

p1

k=1

p k

n + 2p k 2p
p k

n+p p

n + 2p n+p . 2p 2p
p 2 k

Considering p a prime, we have relation, we obtain


p1

0 (mod p), hence


2

0 (mod p2 ). From the above

k=1

p k

n + 2p k 2p
2

0(modp2 ),

hence

p2

divides n+p p n + 2p n+p , 2p 2p

and we are done. Also solved by Daniel Lasaosa, Universidad P ublica de Navarra, Spain.

Mathematical Reflections 6 (2010)

34

Laboratorio di Matematica
Foglio n.1 - 15 ottobre 2010

Esercizio 1. Siano A e B matrici n n a coecienti reali. Dimostrare che se A ` e simmetrica e denita non negativa e AB + BA = 0 allora AB = BA = 0. Esercizio 2. Siano A1 , A2 , . . . , An i vertici di un poligono regolare di n lati inscritto in una la circonferenza C di raggio unitario. Dimostrare che per ogni P C 1 n ` e un numero intero e determinarne il valore. Esercizio 3. Un poligono ` e formato da n rettangoli 3 2 uniti lungo una parte del lato lungo e disposti in modo alternato. Tale poligono deve essere ricoperto con tessere rettangolari 2 1 o 1 2. Qui sotto ` e rappresentato un esempio di ricoprimento per n = 5:
n

|P Ak |2
k=1

Sia an il numero di tali ricoprimenti. Dimostrare che 3n an < 4n per ogni n intero positivo. Esercizio 4. Sia x un numero reale non negativo. Calcolare la somma della serie (1)2 2n n=1

Esercizio 5. Dimostrare o confutare che per ogni intero positivo k esiste una potenza di due che in base dieci ha almeno k cifre uguali a 9.

Laboratorio di Matematica
Foglio n.1 - 15 ottobre 2010

Esercizio 1. Siano A e B matrici n n a coecienti reali. Dimostrare che se A ` e simmetrica e denita non negativa e AB + BA = 0 allora AB = BA = 0. Soluzione. Dato che A ` e simmetrica, A ` e diagonalizzabile e tutti i suoi autovalori sono reali. Inoltre, se Av = v allora AB + BA = 0 implica che A(Bv ) = B (Av ) = (Bv ) Siccome A ` e denita non negativa, 0 e 0 A(Bv ), Bv = ||Bv ||2 0.

Cos` ||Bv ||2 =0, ossia = 0 oppure Bv = 0 Sia {v1 , v2 , , vn } una base di autovettori che diagonalizza A con 1 , 2 , , n i relativi autovalori allora A(Bvi ) = i (Bvi ) = 0 e dunque AB = 0 e BA = AB = 0. Si noti che le ipotesi del problema non implicano che A = 0 o B = 0. Ad esempio A= Ik 0 0 0nk e B= 0k 0 0 Ink

con 1 < k < n soddisfano le ipotesi del problema. Esercizio 2. Siano A1 , A2 , . . . , An i vertici di un poligono regolare di n lati inscritto in una la circonferenza C di raggio unitario. Dimostrare che per ogni P C 1 n ` e un numero intero e determinarne il valore. Soluzione. Possiamo supporre senza perdere di generalit` a che i vertici del poligono siano Ak = eik con = 2/n e k = 1, . . . , n. Sia P = ei C allora Quindi 1 n
n k=1 n k=1

|P Ak |2

|P Ak |2 = (ei eik )(ei eik ) = 2 2Re(eiik ). 2 Re ei n


n

|P Ak |2 = 2

eik
k=1

=2

1 ein 2 Re ei n 1 ei

= 2.

Esercizio 3. Un poligono ` e formato da n rettangoli 3 2 uniti lungo una parte del lato lungo e disposti in modo alternato. Tale poligono deve essere ricoperto con tessere rettangolari 2 1 o 1 2. Qui sotto ` e rappresentato un esempio di ricoprimento per n = 5:

Sia an il numero di tali ricoprimenti. Dimostrare che 3n an < 4n per ogni n intero positivo. Soluzione. Gli an ricoprimenti del poligono di ordine n possono essere divisi in due categorie: i tn ricoprimenti che non contengono tessere che attraversano la linea di congiunzione tra i primi n 1 rettangoli e lultimo; i restanti sn ricoprimenti dove tale linea viene attraversata da almeno una tessera. In questo caso una parte del poligono ha un ricoprimento forzato. tn sn

Dato che un singolo rettangolo ha 3 ricoprimenti diversi allora tn = 3an1 per n 2. Inoltre nei ricoprimenti del secondo tipo ogni ricoprimenti della parte non forzata pu` o essere estesa univocamente a un ricoprimento del poligono formato da n 1 rettangoli e quindi sn an1 per n 2. Inne siccome a1 = 3 e per n 2 3an1 = tn an = tn + sn 4an1 si deduce facilmente per induzione che per ogni n intero positivo 3n an 3 4n1 < 4n . Con un ragionamento simile si pu` o dedurre che la successione (an )n1 soddisfa la ricorrenza lineare an+1 = 4an 2an1 e quindi an = (2 + n+1 2) + (2 2)n+1 . 4

Esercizio 4. Sia x un numero reale non negativo. Calcolare la somma della serie (1)2 2n n=1

Soluzione. Sia (ak )k1 la rappresentazione diadica di {x}, la parte frazionaria di x, allora

x = x + {x} = x +

k=1

ak . 2k

Quindi per n 1 si ha che

2n x = (2n x + 2n1 a1 + + 2an1 ) + an +

k=1

ank . 2k

Siccome il numero tra parentesi ` e pari e la somma della serie ` e un numero non negativo minore di 1 (ogni ai {0, 1} e non sono denitivamente uguali a 1) allora (1)2 e quindi (1)2 2n n=1

n n

= (1)an = 1 2an

1 2 an an =12 = 1 2{x}. n 2 2n n=1 n=1

Esercizio 5. Dimostrare o confutare che per ogni intero positivo k esiste una potenza di due che in base dieci ha almeno k cifre uguali a 9. Soluzione. Dimostriamo un risultato pi` u generale: per ogni numero intero N 1 esiste un numero intero m 0 tale che la rappresentazione decimale di 2m inizia con N . Basta far vedere che esistono degli interi a, m 0 tali che m log10 2 a [log10 (N ), log10 (N + 1)). Siccome log10 2 ` e un numero irrazionale, per il teorema di Weyl, la successione ({n log10 2})n1 ` e densa in [0, 1) e dunque esiste un intero n0 1 tale che 0 < {n0 log10 2} < 1 1 (log10 (N + 1) log10 (N )) = log10 (1 + 1/N ). 2 2

Allora ` e possibile trovare un numero intero k0 1 tale che k0 {n0 log10 2} [log10 (N ), log10 (N + 1)) e basta porre m = k0 n0 e a = k0 n0 log10 2. Una soluzione alternativa. Il numero 2 ` e un generatore di Z 5n e quindi esiste an tale che 2an 1 (mod 5n ) (si pu` o prendere an = (5n )/2 = 2 5n1 ). Sia m = 2k + a2k allora 2m 0 22k (mod 22k ) e 2m 22k (mod 52k )

e per il Teorema Cinese dei Resti 2m 22k (mod 102k ). Cos` 2m 22k = 102k 22k (10k 1)10k + (10k 4k ) (mod 102k ). Questo signica che le k cifre decimali che precedono le ultime k sono tutte uguali a 9.

Laboratorio di Matematica
Foglio n.2 - 19 novembre 2010

Esercizio 1. Siano n e k interi tali che 1 k n. Qual ` e il numero medio dei cicli di lunghezza k di una permutazione dei numeri {1, 2, . . . , n}. Esercizio 2. Per a > 1, si consideri la serie S (a) = 1 , an! n=0

(i) dimostrare che S (a) ` e irrazionale per ogni intero a > 1; (ii) esiste un numero reale a > 1 tale che S (a) ` e razionale?

Esercizio 3. Sia f C 2 (R). Provare che


+

|f (x)| dx

12 f (a) 2f |b a|3

a+b 2

+ f (b)

per ogni scelta dei numeri reali distinti a e b. Esercizio 4. Dati tre numeri interi distinti a, b e c dimostrare o confutare che esiste un polinomio P (x) Z[x] che permuta ciclicamente tali numeri ossia che: P (a) = b, P (b) = c e P (c) = a.

Esercizio 5. Sia (n) la funzione che per ogni intero n 1 indica il numero di primi nellintervallo [1, n]. Esistono inniti numeri interi n 2 tali che (n) divide n?

Laboratorio di Matematica
Foglio n.2 - 19 novembre 2010 Esercizio 1. Siano n e k interi tali che 1 k n. Qual ` e il numero medio dei cicli di lunghezza k di una permutazione dei numeri {1, 2, . . . , n}. Soluzione. Sia una permutazione di Sn . Indichiamo con Cyck ( ) linsieme costituito dai numeri i {1, 2, . . . , n} che appartengono a un ciclo di lunghezza k di . Il numero medio n (k ) dei cicli di lunghezza k di una permutazione di Sn ` e dato da n (k ) = 1 n! 1 |Cyck ( )| = k k n!
n

[i Cyck ( )] =
Sn k=1

Sn

1 k n!

[i Cyck ( )]
k=1 Sn

dove [Q] vale 1 se la proposizione Q ` e vera e 0 altrimenti. La somma [i Cyck ( )]


Sn

enumera il numero di permutazioni che hanno il numero i contenuto in un ciclo di lunghezza k ed 1 ` e uguale al prodotto di n k1 (k 1)! (numero di modi di completare il ciclo) per (n k )! (numero di modi di completare la permutazione), ossia (n 1)!. Cos` n (k ) = 1 k n!
n

(n 1)! =
k=1

1 . k

Esercizio 2. Per a > 1, si consideri la serie S (a) = 1 , n! a n=0

(i) dimostrare che S (a) ` e irrazionale per ogni intero a > 1; (ii) esiste un numero reale a > 1 tale che S (a) ` e razionale? Soluzione. (i) Il numero S (a), espresso in base a, ha un sviluppo innito costituito da cifre 0 e 1 dove la distanza tra ln-simo 1 e il precedente ` e uguale a n! (n 1)!. Dato che tale distanza diventa arbitrariamente grande, lo sviluppo non ` e periodico e quindi S (a) non ` e razionale. Una dimostrazione pi` u diretta ` e la seguente. Supponiamo per assurdo che S (a) = p/q con p e q interi positivi. Allora per ogni n0 1

t := q an0 !
n=n0

0 1 1 n0 ! n0 ! = p a q a = intero positivo 1. n! n! a a n=0 +1

Daltra parte, siccome n! n0 ! n n0 , si ha che per n0 sucientemente grande t= q an0 n0 !

1 an!n0 !

q an0 n0 !

1 ann0

n=n0 +1

n=n0 +1

q <1 (a 1)an0 n0 !

in contraddizione con la disequazione precedente. (ii) La serie di potenze

xn! ,
n=0

dominata dalla serie geometrica di ragione x, converge per |x| < 1 a una funzione continua F (x). Per il Teorema dei Valori Intermedi, per a > 1, S (a) = F (1/a) assume tutti i valori nellintervallo (F (0), F (1)) = (0, +). In particolare esistono inniti valori di a > 1 per cui S (a) ` e razionale.

Esercizio 3. Sia f C 2 (R). Provare che


+

|f (x)| dx

12 f (a) 2f |b a|3

a+b 2

+ f (b)

per ogni scelta dei numeri reali distinti a e b. Soluzione. Possiamo supporre che a < b. Sia g C 2 (R) allora per la disuguaglianza di Cauchy-Schwarz
b b b 2

|f (x)| dx
a a

|g (x)| dx
a

f (x)g (x) dx

Sia c = (a + b)/2 e posto g (x) = si ha che


a b

xa bx
2

per x [a, c] , per x [c, b] (b a)3 . 12


c b

|g (x)| dx = Inoltre
b b

f (x)g (x) dx =
a a

g (x) d(f (x)) = [g (x)f (x)]a


a

f (x)g (x) dx
c

f (x)g (x) dx

= 0 f (c) + f (a) + f (b) f (c) = f (a) 2f (c) + f (b). Inne,


+ b

|f (x)| dx
a

|f (x)| dx

12 2 (f (a) 2f (c) + f (b)) . |b a|3

Esercizio 4. Dati tre numeri interi distinti a, b e c dimostrare o confutare che esiste un polinomio P (x) Z[x] che permuta ciclicamente tali numeri ossia che: P (a) = b, P (b) = c e P (c) = a.

Soluzione. Se P (x) Z[x] e P (a) = b allora esiste un polinomio Q(x) Z[x] tale che P (x) b = (x a)Q(x). Quindi m1 := In modo simile si ottiene che m2 := ac Z cb e m3 := ba Z. ac P (x) b cb = ba xa = Q(b) Z.
x =b

Dato che m1 m2 m3 = 1 se ne deduce che |m1 | = |m2 | = |m3 | = 1. Cos` |c b| = |b a| = |a c| > 0 ovvero a, b, c sono tre punti distinti di una retta ognuno equidistante dagli altri due. Tale condizione non pu` o mai essere soddisfatta e dunque abbiamo una contraddizione.

Esercizio 5. Sia (n) la funzione che per ogni intero n 1 indica il numero di primi nellintervallo [1, n]. Esistono inniti numeri interi n 2 tali che (n) divide n? Soluzione. Sia a 2 un intero e consideriamo linsieme Sa = {k 1 : (a k ) k }. Linsieme Sa non ` e vuoto perch e 1 Sa . Inoltre, siccome (n) n/ log n, si ha che lim (a k ) =0 ak

e cos` , denitivamente, (a k )/(a k ) < 1/a ossia (a k ) < k . Quindi linsieme Sa ` e nito. Sia ka := max Sa , allora (a ka ) ka . Se (a ka ) > ka allora (a (ka + 1)) (a ka ) ka + 1 contro la massimalit` a di ka (si noti che la funzione (n) ` e non decrescente ed ` e a valori interi). Quindi (a ka ) = ka e posto na := a ka , si ha che a (na ) = a (a ka ) = a ka = na , ossia (na ) divide na . Dato che na a, per larbitrariet` a della scelta di a, possiamo concludere che esistono inniti na .

Laboratorio di Matematica
Foglio n.3 - 14 gennaio 2010

Esercizio 1. Se si scelgono a caso n punti in un intervallo di lunghezza L, qual ` e il valore medio della somma delle distanze di tutte le possibili coppie di punti? Esercizio 2. Sia f C 1 ([a, b]). Dimostrare che esistono due punti a < x1 < x2 < b tali che f (x1 ) f (x2 ) = f (b) f (a) ba
2

Esercizio 3. Determinare una formula chiusa per la somma multipla x.


A{1,...,n} B {1,...,n} xAB A= B =

Esercizio 4. Sia n = 2010100 . Determinare il numero di elementi dellinsieme Sn = d N : 1 d n, d | n2 , d n .

Esercizio 5. Sia p un numero primo. Dimostrare che per ogni intero n 0 si ha che Lpn Ln (mod p)

dove Ln ` e ln-simo numero di Lucas denito dalla ricorsione L0 = 2, L1 = 1 e Ln = Ln1 + Ln2 per n 2.

1.

Elementi di analisi funzionale Esercizi

http://www.ciram.unibo.it/~barozzi/MI2/PDF/MI2-Cap.1-Ese.pdf

1.1. Spazi vettoriali 1.2. Spazi vettoriali normati


1.2-1. Dimostrare la diseguaglianza triangolare in Cn relativamente alla norma ( esempio 1).
Suggerimento Soluzione. Si ha
n n

Si ha x + y

n k=1

|xk + yk | . . . .

x+y

=
k=1 n

|xk + yk |
k=1 n

(|xk | + |yk |) = + y 1.

=
k=1

|xk | +
k=1

|yk | = x

1.2-2. Dimostrare la diseguaglianza triangolare in Cn relativamente alla norma ( esempio 1).


Suggerimento Soluzione. Si ha

Si parta dal fatto che x + y

. = |xk + yk | per un certo indice k

x+y

= |xk + yk | |xk | + |yk | max |xk | + max |yk | = x


k k

+ y

1.2-3. Per ogni x Rn mostrare che


p

lim x

= x

= max |xk |.
1kn

Suggerimento

Se x = 0 non c` e niente da dimostrare. Altrimenti, supposto, per ssare le idee, che la componente di valore assoluto massimo sia la prima, cio` e |x1 | = x , si pu` o scrivere
n

|xk |
k=1

1/p

= |x1 | 1 +
k=2

xk x1

p 1/p

dove lultima quantit` a entro parentesi tonde ` e compresa tra 1 e n. Soluzione. Proseguendo nel ragionamento iniziato nel suggerimento, si ha
n

1 1+
k=2

xk x1

p 1/p

n1/p ,

dove lultima quantit` a tende a 1 per p . 1.2-4. Vericare che sullo spazio C [a, b] delle funzioni reali continue sullintervallo [a, b] le norme di indici 1 e sono eettivamente tali ( esempio 2).

Capitolo 1. Elementi di analisi funzionale

c 88-08-07923-6

Si tratta di vericare (per entrambe le norme) la diseguaglianza triangolare: per la seconda norma utilizzare il teorema di Weierstrass sullesistenza del massimo di una funzione continua su un compatto. Soluzione. Si ha
Suggerimento
b b b

f +g

=
a

|f (x) + g (x)| dx
a

|f (x)| dx +
a

|g (x)| dx = f

+ g 1.

Sia poi x [a, b] un punto tale f + g f +g

= |f (x) + g (x)|. Si ha
x x

= |f (x) + g (x)| |f (x)| + |g (x)| max |f (x)| + max |g (x)| = = f

+ g

1.2-5. Vericare che per le funzioni costanti sullintervallo [a, b], f (x) = c, si ha f
1

= (b a) f
b

Soluzione. Si ha infatti
b

=
a

|c| dx =
a

dx = (b a) f

1.2-6. Dimostrare che le norme 1 e sono equivalenti sullo spazio vettoriale delle funzioni polinomiali di grado 1 sullintervallo [a, b] (in accordo con la Proposizione 1.2-1), calcolando due costanti che consentano di maggiorare una norma con laltra.
Suggerimento

Sia p( ) un polinomio di grado 1; posto A := |p(a)|, B := |p(b)|, si cominci con losservare che si ha p = max{A, B },
b

=
a

|p(x)| dx =

ba (A + B ) 2 b a A2 + B 2 2 A+B

se p(a) p(b) 0 (in tal caso il graco di |p| ` e un segmento), mentre


b

=
a

|p(x)| dx =

se p(a) p(b) < 0 (in tal caso il graco di |p| ` e una spezzata composta da due segmenti ugualmente inclinati sullasse delle ascisse).

B A A a b a b

Soluzione. Con riferimento alla gura di sinistra, ` e chiaro che si tratta dellarea di un

trapezio con basi A e B e altezza b a. Nel caso della gura di destra, la similitudine tra i due triangoli evidenziati implica che le basi degli stessi triangoli valgono A B (b a), (b a). (A + B ) (A + B ) Si ha poi A + B 2 p , e per A + B > 0 (in caso contrario p ` e identicamente nullo e non c` e niente da dimostrare) si ha A2 + B 2 (A + B )2 , quindi

c 88-08-07923-6

Esercizi

A2 + B 2 A + B. A+B

f1 (x, y ) = x + y

f2 (x, y ) =

x2 + y 2 x+y

1
In conclusione: ba p 1 2 p = (b a) p . 2 Vale il segno di uguaglianza per i polinomi costanti. 1.2-7. Utilizzando le funzioni polinomiali fornite al termine dellesempio 1.2-2, dimostrare che le norme 1 e non sono equivalenti sullo spazio vettoriale delle funzioni polinomiali (senza limitazioni sul grado) sullintervallo [a, b]. Soluzione. Si tratta delle funzioni fn (x) = xn , considerate sullintervallo [0, 1]. Per esse abbiamo trovato fn 1 = 1/(n +1), fn = 1. Dunque non esiste una costante C > 0 tale da aversi p C p 1 per ogni funzione polinomiale p. 1.2-8. Sia V = C (1) [a, b] lo spazio delle funzioni continue su [a, b] assieme alla derivata prima; si considerino le norme f f
1

:= max |f (x)| + max |f (x)|,


axb axb

:= |f (a)| + max |f (x)|.


axb

Dimostrare che esse sono eettivamente due norme e che sono equivalenti.
Suggerimento

Si pu` o osservare (teorema del valor medio) che

f (x) = f (a) + (x a) f ( ), con [a, b], da cui |f (x)| |f (a)| + (b a) max |f (x)|.
axb

Soluzione. Proseguendo nel ragionamento iniziaro nel suggerimento, si ha

max |f (x)| |f (a)| + (b a) max |f (x)|,


x x

da cui

Capitolo 1. Elementi di analisi funzionale

c 88-08-07923-6

|f (a)| + (b a) max |f (x)| + max |f (x)| =


x x

= |f (a)| + (b a + 1) max |f (x)| <


x

< (b a + 1) |f (a)| + max |f (x)| = (b a + 1) f


x

2.

La disuguaglianza f

` e evidente.

1.2-9. Sia V = C [a, b], w una funzione continua e positiva su [a, b]. Dimostrare che la quantit` a f
w

:= max |w(x) f (x)|


axb

` e una norma e dire se essa ` e equivalente alla norma del massimo. Si ha 0 < minx w(x) w(x) maxx w(x); sfruttare lidentit` a f (x) = w(x) f (w)/w(x). Soluzione. Si ha f (x) f (x) f = max |f (x)| = max w(x) max w(x) = x x x w(x) minx w(x) 1 1 = max |w(x) f (x)| = f w. minx w(x) x minx w(x)
Suggerimento

Inversamente f
w

= max |w(x) f (x)| max


x x x x

max w(x) f (x) =


x x .

= max w(x) max |f (x)| = max w(x) f

1.2-10. Siano 1 e 2 due norme equivalenti su V . Sia (xn )nN una successione in V e x V . Vericare che xn x xn x
1

xn x xn x

0 . c1 xn x 1 .

Soluzione. Infatti per due opportune costanti positive c1 e c2 si ha


1

c2 xn x 2 ,

1.2-11. Se V ` e uno s.v.n. reale o complesso, una trasformazione lineare f : V R (o rispettivamente f : V C) si chiama semplicemente un funzionale lineare. Si consideri lo spazio C (1) [0, 1] munito della norma del massimo. Vericare che il funzionale lineare x( ) x (0) non ` e limitato, dunque ( Proposizione 1.2-2) non ` e continuo.
Suggerimento

Scegliere, ad esempio, xn (t) = sin nt.

Soluzione. Per le funzioni suggerite si ha xn (t) = n cos nt, quindi xn (0) = n, mentre

xn

= 1 per ogni n.

1.2-12. Si consideri lo spazio C [0, 1] munito della norma del massimo. Controllare 1 che i funzionali lineari x( ) 0 x(t) dt, e x( ) x(t0 ), per ogni ssato t0 [0, 1], sono continui. Soluzione. Si ha
1 1 1

x(t) dt
0 0

|x(t)| dt
0 .

dt = x

Si ha poi |x(t0 )| maxx |x(t)| = x

1.2-13. Si considerino gli spazi V = C (1) [0, 1] e W = C [0, 1] muniti entrambi della norma del massimo. Controllare che loperatore lineare x( ) x ( ) non ` e continuo da V a W (considerare, ad esempio, le funzioni xn (t) := (sin nt)/ n ). Soluzione. Per le funzioni suggerite si ha xn = 1/ n, xn = n.

c 88-08-07923-6

Esercizi

1.2-14. Si consideri lo spazio V = C [0, 1] munito della norma del massimo. Controllare che loperatore lineare di V in s e che ad x( ) V associa la funzione t t 0 x(s) ds, t [0, 1], ` e continuo.
Soluzione. Posto X (t) :=
t t 0

x(s) ds si ha infatti
t 1 t

= max
t 0 1

x(s) ds max
0 .

|x(s)| ds =
0

|x(s)| ds =

= x

1 x

= x

Dunque loperatore lineare in esame ` e continuo anche da C [0, 1] munito della norma di indice 1 allo stesso spazio munito della norma del massimo. 1.2-15. Si consideri lo spazio C [0, 1] munito della norma del massimo (norma di indice ). Controllare se sono chiusi in tale spazio gli insiemi (alcuni dei quali sono sottospazi) costituiti dalle funzioni x( ) tali che: a) x(0) = 0; b) x(0) = x(1). c) x ` e non negativa; d) lintegrale di x su [0, 1] ` e nullo; e) lintegrale di x su [0, 1] ` e non negativo; f) x ` e derivabile in 1/2 e x (1/2) = 0; g) x ` e costante su [0, 1].
Suggerimento Per f ) si consideri, ad esempio, la successione di funzioni

1 1 2 + t . 2 n 2 Soluzione. Si tratta di vericare se, data una successione (xn ) convergente uniformemente ad una funzione limite x, dal fatto che tutte le xn vericano una delle condizioni dalla a) alla g) segue (o meno) che la stessa condizione ` e vericata dalla funzione limite x. La risposta ` e aermativa tranne nel caso f); infatti la funzione limite della successione suggerita ` e |x 1/2| che non ` e derivabile nel punto 1/2. Si consideri, alternativamente, la successione 1 xn (x) := sin n(t 1/2) t; n essa converge uniformemente alla funzione t t, derivabile nel punto 1/2 con derivata uguale a 1, pur essendo xn (1/2) = 0 per ogni n. xn (t) := 1.2-16. Stesso problema del precedente esercizio per gli insiemi costituiti dalle funzioni x( ) tali che: a) x ` e un polinomio di grado 2; b) x ` e un polinomio di grado esattamente 2; c) x ` e un polinomio.
Suggerimento

Si tenga presente che la convergenza uniforme implica la convergenza puntuale. Scelti tre valori distinti della variabile indipendente, ad esempio t = 0, e una successione di polinomi di grado 2 t = 1/2, t = 1, se xn (t) := an t2 + bn t + cn ` convergente uniformemente su [0, 1], dalla convergenza delle tre successioni n xn (0) = cn , n xn (1/2) = an /4 + bn /2 + cn , n xn (1) = an + bn + cn , dedurre la convergenza delle successioni n an , n bn (oltre a quella della successione n cn ).

Capitolo 1. Elementi di analisi funzionale

c 88-08-07923-6

Per c) si considerino i polinomi di Taylor della funzione seno.


Soluzione. Proseguendo secondo le linee fornite dal suggerimento, si osserva che il

sistema ottenuto si scrive 0 0 1 an xn (0) 1/4 1/2 1 bn = xn (1/2) , 1 1 1 xn (1) cn la cui soluzione ` e data da an 2 4 2 xn (0) bn = 3 4 1 xn (1/2) . 1 0 0 xn (1) cn Dunque la convergenza delle successioni xn (0) , xn (1/2) , xn (1) implica la convergenza delle successioni (an ), (bn ) e (cn ). Dunque la risposta alla domanda a) ` e aermativa. Al contrario la risposta alla domanda b) ` e negativa: la successione dei polinomi n x2 /n converge uniformemente a 0 sullintervallo [0, 1] (di fatto su ogni intervallo compatto). Lo stesso per la domanda c). la successione dei polinomi di Taylor della funzione seno (in breve: la serie di Taylor) converge uniformemente a sin x su ogni intervallo compatto, e la funzione limite non ` e polinomiale. Si tenga presente che se Tn (x) ` e il polinomio di Taylor relativo alla funzione seno (punto iniziale x = 0) per il relativo resto si ha lespressione secondo Lagrange rn (x) = xn+1 f (n+1) ( ) (n + 1)! = |rn (x)| |x|n+1 , (n + 1)!

in quanto le derivate successive della funzione seno sono del tipo sin x, cos x. 1.2-17. Sia limn xn = x nello s.v.n. V ; dimostrare che limn xn = x .
Suggerimento Soluzione. Si ha infatti

Utilizzare la diseguaglianza (3 ). x n x xn x .

1.2-18. Sia limn xn = x nello s.v.n. V ; dimostrare che esiste una costante C per cui xn < C, n. A parole: ogni successione convergente ` e limitata in norma. Soluzione. Il precedente esercizio ci assicura che la successione n xn converge a x . Scelto > 0 esiste n tale che per ogni n > n si ha xn [ x , x + ]. Al di fuori dellintervallo appena considerato restano dunque, al pi` u, le norme dei primi n elementi. baster` a prendere a := min{ x1 , . . . , xn , x }, b := max{ x1 , . . . , xn , x + } per avere un intervallo [a, b] che contiene tutte le norme. 1.2-19. Sia limn xn = x nello s.v.n. complesso V , limn an = a in C; dimostrare che limn an xn = ax. Da an xn ax = an xn an x + an x ax segue ... Soluzione. . . . segue
Suggerimento

an xn ax an xn an x + an x ax = = |an | xn x + |an a| x ; e limitata in quanto convergente, dunque il secondo membro la successione (|an |) ` tende a 0 per n .

c 88-08-07923-6

Esercizi

1.3. Spazi vettoriali con prodotto scalare


1.3-1. Vericare che ponendo in Cn (come in Rn ) (x | y ) := introdotto una denizione scorretta.
Suggerimento
n k=1

xk yk , avremmo

Si consideri il vettore avente tutte le componenti uguali allunit` a immaginaria i. Soluzione. Per il vettore in questione si avrebbe infatti (x | x) = n. 1.3-2. Dimostrare che vale il segno di uguaglianza nella diseguaglianza di CauchySchwarz se e solo se i vettori x e y sono linearmente dipendenti. Se x e y sono vettori non nulli, vericare che vale il segno di uguaglianza nella diseguaglianza triangolare se e solo se x = ty , con t > 0. Soluzione. Con riferimento alla dimostrazione della Proposizione 1.3-2, ` e chiaro cha vale il segno di = nella diseguaglianza di Cauchy-Schwarz se si ha 0 = x + y 2 , con e i due vettori dati sono linearmente = (y | x)/ x 2 , dunque se x + y = 0, cio` dipendenti. Inversamente, sa x = y , allora (x | y ) = y 2 , da cui |(x | y )| = || y y = x y . Perch e valga il segno di = nella diseguaglianza triangolare occorre che in tutti i passaggi della dimostrazione i segni siano sostituiti da =. Questo richiede che valga il segno di = nella diseguaglianza di Cauchy-Schwarz e dunque deve essere x = y . Deve poi essere Re(x | y ) = |(x | y )|, cio` e Re() = ||, dunque > 0. 1.3-3. Dimostrare che in uno s.v. con prodotto scalare il prodotto scalare stesso ` e una funzione continua, nel senso che se xn x, yn y Si utilizzi la diseguaglianza di Cauchy-Schwarz. (vale a dire xn x 0, yn y 0 per n +), allora (xn | yn ) (x | y ).
Suggerimento Soluzione. Si ha

(xn | yn ) (x | y ) = (xn | yn ) (xn | y ) + (xn | y ) (x | y ) = = (xn | yn y ) + (xn x | y ). Dalla diseguaglianza triangolare e da quella di Cauchy-Schwarz segue allora |(xn | yn ) (x | y )| |(xn | yn y )| + |(xn x | y )| xn y n y + xn x y . Il risultato segue allora dalle ipotesi, tenendo conto del fatto che la successione n xn ` e limitata in quanto convergente (v. esercizio 1.2-18). 1.3-4. Dedurre dalle identit` a (9) che, se la norma sullo spazio vettoriale V ` e hilbertiana, cio` e indotta dal prodotto scalare ( | ), allora tale prodotto scalare ` e univocamente individuato dalla norma mediante luguaglianza 1 (x | y ) = x+y 2 xy 2 4 se V ` e reale, dalluguaglianza 1 (x | y ) = x + y 2 x y 2 + i x + iy 2 i x iy 2 4 se V ` e complesso. Soluzione. Basta sviluppare i secondi membri. Si ha, ad esempio, 1 1 x + y 2 x y 2 = [ (x + y | x + y ) (x y | x y )] = 4 4 1 = [ 2(x | y ) + 2(x | y )] = (x | y ). 4

Capitolo 1. Elementi di analisi funzionale

c 88-08-07923-6

1.3-5. Dimostrare che le norme 1 e su Rn , n 2 ( esempio 1.2-1) non vericano lidentit` a del parallelogramma, e dunque non sono norme hilbertiane.
Suggerimento

Prendere x = e1 = (1, 0, 0, . . . , 0), y = e2 = (0, 1, 0, . . . , 0). = 1,

Soluzione. Per i vettori in questione si ha infatti

e1 e1

= e2 = e2

e1 + e2

= e1 e2

= 2;

= e1 + e2

= e1 e2

= 1.

1.3-6. Dimostrare che le norme 1 e su C [0, 1] ( esempio 1.2-2) non vericano lidentit` a del parallelogramma, e dunque non sono norme hilbertiane. Prendere, ad esempio, f (x) = x, g (x) = 1 x. Soluzione. Sia ha f (x) + g (x) = 1, f (x) g (x) = 2x 1, da cui facilmente
Suggerimento

f f

= g = g

= 1,

, f +g

= 1,

f g

= 1/2;

= f +g

= f g

= 1.

1.3-7. Sia V uno spazio vettoriale con prodotto scalare ( | ). Vericare che se V ` e reale, allora u+v
2

= u

+ v

(u|v ) = 0 , Re(u|v ) = 0 .

mentre se V ` e complesso, allora u+v


2 2

= u
2

+ v
2

Costruire due vettori non nulli di Cn per cui valga luguaglianza u+v = u + v ma (u|v ) = 0 (sintende di utilizzare il prodotto scalare canonico e la relativa norma). Soluzione. Si ha infatti u+v
2

= (u + v |u + v ) = u

+ v

+ (u|v ) + (v |u),

dove la somma degli ultimi due addendi vale 2 Re(u|v ) se si tratta di uno spazio complesso, e semplicemente 2(u|v ) se si tratta di uno spazio reale. Per i vettori u = (i, i, . . . , i) e v = (1, 1, . . . , 1) si ha Re(u|v ) = 0 ma (u|v ) = 0. 1.3-8. Utilizzando le stesse funzioni dellesempio 1.2-16, si verichi che lo spazio C [1, 1], R non ` e completo rispetto alla norma indotta dal prodotto scalare (f |g ) := b = a f (x)g (x) dx. Si tenga presente che, se |f (x)| 1, allora |f (x)|2 |f (x)|. Soluzione. Si tratta di dimostrare che la successione di funzioni (fn ) considerata nellesempio citato ` e di Cauchy rispetto alla norma di indice 2. Ora si ha (con i simboli dellesempio)
Suggerimento

fn fm

2 2

= (fn f ) + (f fm ) 2 fn f
1 2 2

2 2

2 2

fn f =
1 1 1 1

+ fm f

2 2

+ fm f

=2
1 1

|fn (x) f (x)|2 dx + |fn (x) f (x)| dx +

|f (x) fm (x)|2 dx =

2 2 + . n m

=2
1

|f (x) fm (x)| dx

Abbiamo sfruttato il fatto che |fn (x) f (x)| 1 e |fm (x) f (x)| 1, dunque le funzioni ai primi membri sono dei rispettivi quadrati. Si ` e utilizzata inoltre la disuguaglianza (A + B )2 2(A2 + B 2 ), dove A, B 0.

c 88-08-07923-6

Esercizi

1.3-9. Utilizzando la diseguaglianza di Cauchy-Schwarz nello spazio C [a, b]; C relativamente alle funzioni x |f (x)| e x 1, dove f ` e una qualsivoglia funzione dello spazio in esame, dimostrare la diseguaglianza f 1 b a f 2 . Soluzione. Si ha infatti b b f 1= |f (x)| dx = |f (x)| 1 dx f 2 1 2 = b a f 2 .
a a

1.4. Proiezioni ortogonali


Esercizi proposti

1.4-P.1. Sia V = Rn munito del prodotto scalare canonico, V1 il sottospazio di V di dimensione 1 generato dal vettore v1 := (1, 1, . . . , 1). Un vettore x = (x1 , x2 , . . . , xn ) n ` e ortogonale a v1 (dunque appartiene a V1 ) se e solo se k=1 xk = 0. Vericare che la proiezione ortogonale di x su V1 ` e data dal vettore con tutte le componenti uguali alla media aritmetica delle componenti di x stesso. 1.4-P.2. Sia V = R8 munito del prodotto scalare canonico e V2 sia il sottospazio di V di dimensione 2 generato dai vettori v1 = ( 1, 1, . . . , 1 ),
8 componenti

v2 = (1, 2, 3, . . . , 8).

V2 ` e costituito da tutti (e soltanto) i vettori di R8 le cui componenti sono in progressione aritmetica: xk xk1 = costante, per k = 2, 3, . . . , 8. Vericare che il procedimento di Gram-Schmidt, applicato ai vettori dati, produce i vettori z1 = v1 , z2 = v2 (9/2)v1 = (1 9/2, 2 9/2, . . . , 8 9/2) dove 9/2 ` e la media aritmetica delle componenti di v2 .
8 6 4 2 1 2 3 4 5 6 7 8 3 2 1 -1 1 -2 -3 2 3 4 5 6 7 8

A sinistra i vettori v1 e v2 , a destra i vettori z1 e z2 .

La proiezione ortogonale di x = (x1 , x2 , . . . , xn ) su V2 si scrive dunque x z1 x z2 z1 + z2 = z1 2 z2 2


8 k=1

xk

z1 +

8 k=1

k xk z2 42

42 essendo il quadrato della norma di z2 . 1.4-P.3. Generalizzare i risultati del precedente esercizio al caso dello spazio Rn , con n qualunque 2. Ora abbiamo v1 = ( 1, 1, . . . , 1 ),
n componenti

v2 = (1, 2, 3, . . . , n),

da cui segue z1 = v1 , z2 = (1 m, 2 m, . . . , n m)

10

Capitolo 1. Elementi di analisi funzionale

c 88-08-07923-6

dove m = (n + 1)/2 ` e la media ritmetica delle componenti di v2 . Il quadrato della norma di z1 vale n, mentre il quadrato della norma di z2 vale n(n2 1)/12. Per eettuare questultimo calcolo occorre ricordare la formula (v. PCAM, pag. 54)
n

k2 =
k=1

n(n + 1)(2n + 1) . 6

1.4-P.4. Consideriamo i polinomi p0 (x) = 1, p1 (x) = x, p2 (x) = x2 nello spazio delle funzioni continue sullintervallo [0, 2], munito del consueto prodotto scalare 2 (f |g ) = 0 f (x)g (x) dx. Applicare ad essi il procedimento di Gram-Schmidt (Prop. 1.4-3), ottenendo i polinomi q0 (x), q1 (x), q2 (x). Si ottiene una base ortogonale del sottospazio P2 costituito dai polinomi di grado 2. Utilizzando tale base, calcolare il polinomio di secondo grado che meglio approssima la funzione f (x) = x3 nello spazio considerato. Suggerimento Utilizzare la Proposizione 1.3-4.)
Soluzione. Si trova

q0 (x) = 1,

q1 (x) = x 1,

q1 (x) = x2 2x + 2/3.

Il polinomio di migliore approssimazione ` e p(x) = c0 q0 (x) + c1 q1 (x) + c2 q2 (x) con ck = (f |qk )/(qk |qk ), k = 0, 1, 2, quindi c0 = 2, c1 = 18/5, c2 = 3,
2

da cui nalmente p(x) = 3x 12/5 x + 2/5.

2.

Elementi di teoria dellintegrazione Esercizi

http://www.ciram.unibo.it/~barozzi/MI2/PDF/MI2-Cap.2-Ese.pdf

2.1. Richiami sullintegrale di Riemann 2.2. La misura di Lebesgue


2.2-1. Ritocchiamo la denizione di pluri-intervallo, convenendo di considerare solN tanto unioni nite di intervalli: P = k=1 Ik , con N naturale positivo ad arbitrio. In conseguenza di tale scelta, deniamo una misura esterna me (E ) := inf m(P ),
P

intendendo che lestremo inferiore venga considerato al variare di P tra i pluriintervalli che coprono E . Una tale misura esterna viene spesso associata ai nomi di G. Peano (1858-1932) e C. Jordan (1838-1922). Se E = [0, 1] Q ( esempio 2.2-1), si trova me (E ) = me [0, 1] \ E = me [0, 1] = 1, dunque E non ` e misurabile nel senso di Peano-Jordan, in quanto non vale lanaloga dellidentit` a (6).
Soluzione. Se P =

con N N , possiamo riscrivere P nella forma P = N = k=1 Ik , con N N , dove gli intervalli Ik sono a due a due disgiunti. Si tenga presente che se lintersezione tra due intervalli non ` e vuota, la loro unione ` e ancora un intervallo. Ora tanto E (insieme dei razionali dellintervallo [0, 1]) quanto [0, 1] \ E (insieme degli irrazionali dellintervallo [0, 1]) sono densi nellintervallo [0, 1], quindi sono conN tenuti in un plurintervallo del tipo P = k=1 Ik solo se [0, 1] ` e contenuto in uno degli intervalli Ik . Questo implica che m(P ) 1. Daltra parte lintervallo [0, 1], di misura 1, contiene tanto E quanto [0, 1] \ E , e questo prova che, per entrambi gli insiemi, la misura esterna vale 1.
N k=1 Ik ,

2.2-2. Vogliamo denire linsieme ternario di Cantor per sottrazioni successive a partire dallintervallo I = [0, 1]. Sia P1 linsieme che si ottiene togliendo da I lintervallo aperto concentrico di ampiezza 1/3: P1 = I \ (1/3, 2/3) = [0, 1/3] [2/3, 1]. Da ciascuno dei due intervalli di cui si compone P1 togliamo lintervallo aperto concentrico di ampiezza pari ad 1/3 dellampiezza dello stesso intervallo. Dunque P2 ` e lunione di 4 intervalli chiusi di lunghezza 1/9, quindi m(P2 ) = 4/9 = (2/3)2 . Proseguiamo sempre col medesimo criterio, togliendo da ciascuno dei 2n intervalli di cui si compone Pn lintervallo aperto concentrico di ampiezza pari a 1/3 dellintervallo stesso. Si trova, per ogni n, m(Pn ) = (2/3)n .

Capitolo 2. Elementi di teoria dellintegrazione

c 88-08-07923-6

Sia P lintersezione di tutti i pluri-intervalli Pn : P := n1 Pn . Vericare che P ` e di misura nulla, anzi ` e tale anche rispetto alla misura di Peano-Jordan considerata nel precedente esercizio.
Soluzione. Chiaramente P Pn per ogni n, dove Pn ` e un pluri-intevallo unione di

2n intervalli disgiunti, di misura (2/3)n . Ma limn (2/3)n = 0, quindi P pu` o essere ricoperta da un pluri-intervallo (unione di un numero nito di intervalli) di misura arbitrariamente piccola. Ci` o corrisponde al fatto che la misura degli intervalli che sono stati rimossi vale 1 2 4 8 1 + + + + ... = 3 9 27 81 3 1 = 3 1+ 23 2 22 + 2 + 3 + ... = 3 3 3 2 n 1 1 = =1 3 3 1 2/3

n0

2.2-3. Vericare che linsieme di Cantor del precedente esercizio pu` o essere identicato con linsieme dei numeri reali dellintervallo [0, 1] che possono essere espressi in base tre senza utilizzare la cifra uno. Ad esempio: 1 = (0.2)3 , 2/3 = (0.2)3 , 1/3 = (0.02)3 .
Soluzione. Consideriamo la rappresentazione ternaria (= in base tre) dei numeri

dellintervallo [0, 1]. Poich e 1/3 si scrive (0.1)3 e 2/3 si scrive (0.2)3 , vediamo che al primo passo, quando rimuoviamo il terzo intermedio dellintervallo [0, 1], vengono tolti i numeri nella cui rappresentazione ternaria compare la cifra 1 nella prima posizione delle parte frazionaria. Se si tiene presente che 1/3 (punto che non viene rimosso) si scrive anche (0.02)3 possiamo dire che vengono rimossi tutti i numeri che presentano 1 nella prima posizione della parte frazionaria. Al secondo passo vengono rimossi tutti i punti che presentano la cifra 1 nella seconda posizione della parte frazionaria e cos` via. In generale, i secondi estremi degli intervalli che costituiscono il pluri-intervallo Pn hanno rappresentazioni ternarie limitate in cui lultima cifra vale 1 mentre le (eventuali) cifre precedenti valgono 0 oppure 2. Ciascuna di tali rappresentazioni equivale a quella che si ottiene srivendo 0 al posto di 1, seguito da una successione di cifre tutte uguali a 2. Ad esempio il numero 7/9 (secondo estremo del terzo intervallo di P2 ) si scrive 7 2 1 = + = (0.21)3 = (0.2022222 . . .)3 = (0.202)3 . 9 3 9 Ci si pu` o chiede quali punti restino nellinsieme di Cantor, oltre agli estremi degli intervalli che costituiscono ciascun Pn . Un esempio ` e fornito dal numero 1/4, la cui rappresentazione ternaria ` e (0.02)3 . Per convincersene basta scrive 4 in base 3, cio` e 4 = (11)3 , ed iniziare la divisione di 1 per 4 facendo i calcoli in base 3. Dopo due passi si sono ottenute le cifre 0 e 2 e nuovamente il resto 1:

c 88-08-07923-6

Esercizi

1 10 100 22 1

11 0.02

Di fatto linsieme di Cantor, pur essendo di misura nulla, ha la stessa cardinalit` a dellintervallo [0, 1], cio` e pu` o essere posto in corrispondenza biunivoca con tale intervallo. Per convincersene basta considerare la corrispondenza che, ad ogni allineamento ternario costituito soltanto dalle cifre 0 e 2, associa lallineamento binario ottenuto scrivendo 1 al posto di 2. Ad esempio, al numero (0.0202020202 . . .)3 = (0.02)3 = 1/4 viene associato il numero (0.0101010101 . . .)2 = (0.01)2 = 1/3. Poich e linsieme di Cantor contiene tutti (e soltanto) i numeri che hanno rappresentazioni ternarie costituite da una parte intera nulla e dalle cifre 0 e 2, si ottengono come corrispondenti tutti gli allineamenti binari con parte intera nulla, dunque tutti i punti dellintervallo [0, 1]. Se ` e la corrispondenza appena considerata, che trasforma linsieme di Cantor C sullintervallo [0, 1], possiamo prolungarla in una funzione dellintervallo [0, 1] su se stesso convenendo che sia costante su ciascuno degli intervalli che costituiscono il complementare dellinsieme C , e precisamente sia uguale al valore che assume in corrispondenza degli estremi dello stesso intervallo. Ad esempio, poich e 1/3 e 2/3 hanno le rappresentazioni (0.022222 . . .)3 e (0.2)3 , a cui corrispondono la rappresentazioni binarie (0.0111111 . . .)2 e (0.1)2 rispettivamente, e queste rappresentano entrambe 1/2, allora la funzione varr` a 1/2 nellintervallo [1/3, 2/3]. La funzione cos` costruita coincide con quella che viene introdotta e studiata nellesempio 2.3-8, dove viene chiamata V . Il suo graco ` e la scala del diavolo. 2.2-4. Dimostrare che, in R2 , ogni segmento con i lati paralleli agli assi coordinati ` e di misura nulla.
Soluzione. Sia L > 0 la lunghezza del segmento in esame; dato > 0 ad arbitrio,

possiamo coprire il segmento con un unico intervallo bidimensionale (un rettangolo) di dimensioni L e /L. 2.2-5. Si consideri in R2 un segmento non parallelo agli assi coordinati; con riferimento alla gura 2.2-7, si verichi che la misura del pluri-intervallo (= pluri-rettangolo) che ricopre il segmento vale (L2 sin cos )/n, dove L ` e la lunghezza del segmento, la misura dellangolo che esso forma con lasse delle ascisse, n il numero di parti uguali in cui lo stesso segmento ` e stato suddiviso.

Figura 2.2-7. La misura del pluri-intervallo in gura si pu` o rendere arbitrariamente piccola prendendo convenientemente grande il numero degli intervalli (= rettangoli) che lo compongono.

Capitolo 2. Elementi di teoria dellintegrazione

c 88-08-07923-6

Se ne deduca che il segmento in questione ` e di misura nulla in R2 . Si dimostri che 2 ogni retta ha misura nulla in R . Soluzione. Ciascuno dei rettangoli in gura ha area L L cos sin n n quindi il plurirettangolo mostrato ha misura (L2 sin cos )/n 0 per n . Quanto al caso di una retta nel piano, possiamo pensarla come unione di uninnit` a numerabile di segmenti di lunghezza L (peraltro arbitraria) e applicare il risultato contenuto nellesercizio seguente. 2.2-6. Dimostrare che lunione di uninnit` a numerabile di insiemi di misura nulla ` e ancora di misura nulla. Soluzione. Sia E = nN En , dove ciascuno degli insieme En ha misura nulla. Scelto > 0, copriamo En con un plurintervallo Pn , unione di uninnit` a numerabile di I ` e una famiglia intervalli In,k di misura complessiva /2n . Allora n=1 n,k k=1 numerabile di intervalli di misura complessiva non superiore a .
+ 2.2-7. Sia f : E R una funzione misurabile, e siano fj e fj due successioni + crescenti di funzioni semplici non negative, convergenti q.o. a f e f rispettivamente. + Vericare che fj + fj ` e una successione crescente di funzioni semplici convergente q.o. a |f |; se questultima ` e sommabile, dalle relazioni + fj (x) dx fj (x) dx + (x) + fj (x) dx fj + (x) + fj (x) dx fj

|f (x)| dx,
E

|f (x)| dx
E

dedurre la sommabilit` a di f + e f e quindi la sommabilit` a di f .


Soluzione.
+ Ciascuna funzione fj + fj ` e semplice in quanto somma di funzioni + semplci; la successione j fj + fj ` e crescente in quanto somma di successioni crescenti. Per ottenere la sommabilit` a di f + e f si pu` o ricorrere al teorema di B. Levi.

2.2-8. Sia f : R+ R una funzione sommabile; dimostrare che se esiste il limite e necessariamente uguale a 0. limx+ f (x), esso ` Suggerimento Sia, per assurdo, limx+ f (x) = > 0; allora per tutti gli x abbastanza grandi, diciamo per x > x , si ha f (x) > /2. Per gli stessi x avremmo
x x x x

f (t) dt =
0 0

f (t) dt +
x

f (t) dt >
0

f (t) dt +

(x x ), 2

quindi per x + . . . .
Soluzione. Proseguendo nel ragionamento iniziato nel suggerimento, nelle ipotesi

ammesse avremmo limx+ 0 f (t) dt = +, in quanto il prodotto /2 (x x ) tende a +, contro lipotesi di sommabilit` a della funzione f . Un ragionamento del tutto analogo vale se limx+ f (x) = < 0. 2.2-9. Si consideri la funzione caratteristica del pluri-intervallo di misura nita ( esercizio 3.2-6) P := n1 [n 1/n2 , n] ; se ne concluda che una funzione sommabile su R+ pu` o essere priva di limite allinnito.
Soluzione. La sommabilit` a della funzione in esame, sia f , equivale alla convergenza

della serie armonica generalizzata 1 , n2


n1

c 88-08-07923-6

Esercizi

vericata in PCAM, Esempio 6.2-2, ed anche nellesercizio citato, dove si dimostra che la somma delle serie vale 2 /6.

Chiaramente la funzione in esame ` e priva di limite per x : comunque si ssi un valore x, esistono valori di x > x tali f (x) = 0, ed altri tali che f (x) = 1.

2.3. Lintegrale di Lebesgue


2.3-1.P. Dedurre dal teorema della convergenza dominata di Lebesgue il seguente enunciato: Sia f (x, t) una funzione denita per x E Rn , t I intervallo di R. Supponiamo f derivabile parzialmente rispetto a t per ogni t e si abbia |ft (x, t)| g (x) q.o. in E con g sommabile su E . Allora d dt f (x, t) dx =
E E

ft (x, t) dx

Soluzione.

Fissiamo t I e scegliamo una successione innitesima hn tale che t + hn I . Posto fn (x) :=

f (x, t + hn ) f (x, t) , hn la tesi signica che


n

lim

fn (x) dx =
E

E n

lim fn (x) dx.

Si pu` o scrivere f (x, t + hn ) f (x, t) f (x, t) = hn t


t=

per un opportuno , dunque per q.o. x si ha |fn (x)! g (x). A questo punto possiamo applicare il teorema della convergenza dominata. Nota. Applicando il teorema appena dimostrato si ha una dimestrazione diretta della Proposizione 6.2-5, relativa alla derivata della trasformata di Fourier.

2.4. Spazi di funzioni sommabili


2.4-1. Vericare che il predicato x ` e irrazionale, ` e vericato q.o. su R. Altrettanto per il predicato x non ` e intero.
Soluzione. Infatti il primo predicato ` e falso in corrispondenza dei numeri razionali,

il cui insieme, in quanto numerabile, ` e di misura nulla. Lo stesso vale per linsieme degli interi in cui ` e falso il secondo predicato. 2.4-2. Vericare che q.o. su R si ha limn en|x| = 0, e precisamente ... .

Capitolo 2. Elementi di teoria dellintegrazione

c 88-08-07923-6

Soluzione. Lunico punto in cui il limite in questione non ` e0` e lorigine.

2.4-3. Sia f : R+ R la funzione f (x) = 1/x (funzione denita q.o. su R+ ); procedendo come nellesempio 2.3-4, vericare che f / L1 [0, 1], f / L1 [1, +), dove abbiamo indicato semplicemente con f la restrizione della stessa funzione agli intervalli [0, 1] e rispettivamente [1, +).
Soluzione. Infatti le successioni
1

n
1/n

1 dx = log n, x

n
1

1 dx = log n x

sono divergenti. 2.4-4. Sia f : R+ R la funzione f (x) = 1/x1+ , > 0. Vericare che f / L1 [0, 1], f L1 [1, +). Ad esempio x 1/x2 ` e sommabile su [1, ), non ` e sommabile su [0, 1].
Soluzione. Una primitiva di f si scrive

F (x) = quindi
1

1 1 , x 1 1 n + per n ,

f (x) dx =
1/n n

f (x) dx =
1

1 1 1 1 n

per n .

2.4-5. Sia f : R+ R la funzione f (x) = 1/x1 , 0 < < 1. Vericare che f L1 [0, 1], f / L1 [1, +). Ad esempio x 1/ x non ` e sommabile su [1, ), mentre ` e sommabile su [0, 1].
Soluzione. Una primitiva di f si scrive F (x) = x /, quindi
1

f (x) dx =
1/n n

1 1 1 1 n 1 n 1 +

per n , per n .

f (x) dx =
1

funzione degli errori

2.4-6. Vericare che la funzione degli errori 2 erf(x) :=


x 0

et dt
2

tende a 1 per x + ( esempio 2.3-6). Dallo sviluppo in serie della funzione integranda, dedurre il seguente sviluppo (utile per x piccolo): 2 erf(x) =

(1)n
0

x2n+1 2 x5 x7 x9 x3 = x + + + ... (2n + 1) n! 3 10 42 216


t2 e 0

Soluzione. Sappiamo dallesempio citato che

dt =

/2. Dallo sviluppo

et =
2

(1)n
0

t2 n n!

segue il risultato integrando termine a termine sullintervallo [0, x].

c 88-08-07923-6

Esercizi

2.4-7. Partendo dalla uguaglianza 2 erf(x) = (2/ ) et dt


0

2 et dt = 1 (2/ )

et dt,
2

dedurre lo sviluppo asintotico per x grande: ex erf(x) = 1


Suggerimento
x
2 2

13 135 1 1 + 2 5 3 7 + ... . x 2 x3 2 x 2 x
2

Applicare per k = 0, 2, 4, . . . lidentit` a (integrazione per parti)


x

et ex k+1 dt = tk 2xk+1 2

et dt. tk+2
2

7 2 4

0.5

1.5

0.5

1.5

2.5

Figura 2.4-3. A sinistra: graco della funzione degli errori e di alcune somme parziali del suo sviluppo in serie (il numero a anco della curva indica il grado della somma corrispondente); a destra alcuni termini dello sviluppo asintotico della stessa funzione (il numero a anco della curva indica quanti termini sono stati presi in considerazione).

Soluzione. Per interpretare correttamente lintegrazione per parti suggerita occorre

scrivere la funzione integranda a primo membro come 2t et 2tk+1


2

e considerare 2t et come la derivata di et . Dopodich e, applicando luguaglianza scritta per k = 0, 2, 4, . . ., i termini integrati forniscono uno dopo laltro gli addendi entro parentesi tonde.
2 2

3.

Serie di Fourier Esercizi

http://eulero.ing.unibo.it/~barozzi/MI2/PDF/MI2-Cap.3-Ese.pdf

3.1. Polinomi di Fourier 3.2. Serie di Fourier: convergenza puntuale


3.2-1. Si dimostri che la serie (v. esempio 3.2-2)
n=0

1 1 1 cos (2n + 1)x = cos x + 2 cos 3x + 2 cos 5x + . . . , 2 (2n + 1) 3 5

` e totalmente, dunque uniformemente, convergente su R. Soluzione. Si ha cos (2n + 1)x 1 = 2 (2n + 1) (2n + 1)2 e la serie che ha come termine generale lultima quantit` a scritta converge in quanto 1 1 2. (2n + 1)2 4n 3.2-2. Data la funzione sommabile f (x) := 1/ x, 0 < x 1 (v. esempio 3.2-3), si determini una funzione g costante a tratti, minorante di f e tale che
1 1

|f (x) g (x)| dx =
0 1 n 0

[f (x) g (x)] dx =

=
0

f (x) dx
k=1

ck (xk xk1 ) <

( Prop. 3.2-1), procedendo nel modo che segue. Si determini x1 > 0 in modo tale che x1 1 dx < /2; x 0 (si determini esplicitamente lintegrale a primo membro). Scelto cos` x1 , si suddivida lintervallo [x1 , 1] in n 1 parti uguali (con n abbastanza grande) e si ponga g (x) := 0, per 0 < x < x1 , g (x) := inf {f (x) | xk1 < x < xk } = 1/ xk per xk1 < x < xk , k = 2, 3, . . . , n. Soluzione. Si ha x1 1 dx = 2 x1 , x 0 dunque basta scegliere x1 < 2 /16 per soddisfare la prima condizione posta.

Capitolo 3. Serie di Fourier

c 88-08-07923-6

Si ha poi, per ogni k = 2, 3, . . . , n,


xk xk1

[f (x) g (x)] dx =

1 1 dx = xk x xk1 xk xk1 = 2[ xk xk1 ] = xk xk xk1 xk xk1 = 2 = xk + xk1 xk xk xk1 = (xk xk1 ) xk ( xk + xk1 ) xk xk1 1 x1 xk xk1 (xk xk1 ) = , 2 n1 2 1 x1 n1
n

xk

quindi
n xk xk1

[f (x) g (x)] dx

xk

xk1 =

k=2

k=2

1 x1 (1 x1 ), n1

dove lultima quantit` a tende a 0 per n . 3.2-3. Dimostrare luguaglianza 2n + 1, se t 0 (mod 2 ), n ikt e = sin (n + 1/2)t , altrimenti. k=n sin (t/2) procedendo per induzione rispetto a n. Soluzione. Luguaglianza da dimostrare sussiste per n = 0, riducendosi in tal caso alluguaglianza 1 = 1. Ammessa la validit` a delluguaglianza per un assegnato n, si ha
n+1 n

eikt =
k=n+1 k=n

eikt + ei(n+1)t + ei(n+1)t =

sin (n + 1/2)t + 2 cos (n + 1)t = sin (t/2) sin (n + 1/2)t + 2 cos (n + 1)t sin (t/2) = = sin (t/2) sin (n + 1 + 1/2)t = . sin (t/2) = Abbiamo utilizzato lidentit` a (formula diWerner) 2 cos sin = sin( + ) sin( ). 3.2-4. Dimostrare che se f : R C ` e periodica di periodo T e sommabile su [0, T ], T a+T allora 0 f (x) dx = a f (x) dx, per ogni a reale. Si spezzi lintegrale sullintervallo [a, a + T ] nella somma di tre integrali sugli intervalli [a, 0], [0, T ], [T, a + T ].) Soluzione. Si ha
Suggerimento
T a a+T T

f (x) dx =
0 0

f (x) dx +
a a+T

f (x) dx +
a+T a

f (x) dx;
a

lultimo addendo si scrive T f (x) dx = 0 f (T + ) d = 0 f ( ) d . Abbiamo eettuato il cambiamento di variabile x = T + e abbiamo sfruttato la periodicit` a di f . 3.2-5. Utilizzare il lemma di Riemann-Lebesgue per dimostrare che si ha, per ogni > 0,

c 88-08-07923-6

Esercizi

n+

lim

Dn (t) dt = 0.

Dedurne che
n+

lim

Dn (t) dt = 1.

Soluzione. Basta osservare che la funzione t 1/ sin(t/2) ` e continua, dunque somma-

bile, sugli intervalli [, ] e [, ]. 3.2-6. Dal risultato 2 = 8


k=1 k=0

1 1 1 = 1 + 2 + 2 + ... (2k + 1)2 3 5

ottenuto nellesempio 3.2-2, dedurre che 1 1 1 2 = 1 + + + . . . = . k2 22 32 6

Detta s la somma della serie considerata, la somma della serie costituita dai termini di indice pari vale s/4: k=1 1/(2k )2 = s/4. Soluzione. Infatti si ha s = s/4 + 2 /8, da cui il valore di s indicato nel testo.
Suggerimento

3.2-7. Se g (x) := | x | ` e la funzione dellesempio 3.2-2, e f (x) := sign(x) ` e la funzione dellesempio 3.2-1, si ha g (x) = f (x) per ogni x = k , con k intero. Se i coecienti di Fourier di g vengono ancora indicati con i simboli an e bn , mentre i coecienti analoghi di f vengono ribattezzati an , bn , vericare che gli sviluppi di Fourier di f = g e di g possono essere dedotti luno dallaltro in base alle formule an = n bn , bn = n an . Soluzione. Basta ricordare (v. esempi citati) che an = bn = 0, an = 0 bn = .

3.2-8. Sia f (x) := 0, < x < 0, f (x) := 1, 0 < x < . Vericare che si ha 1 dt = 1, 0 an = 0, per ogni n > 0, mentre a0 = bn = 1

sin nt dt =
0 k=0

0, per n pari, 2/(n ), per n dispari.

In denitiva f (x) = 1 2 + 2 1 sin (2k + 1)x = (2k + 1)

1 2 1 1 = + sin x + sin 3x + sin 5x + . . . . 2 3 5 Si osservi che la funzione f (x) 1/2 ` e la met` a della funzione sign(x) dellesercizio precedente, e ci` o giustica il risultato che abbiamo ottenuto. Soluzione. Si tratta di calcolare, per n 1, lintegrale

sin nt dt =
0

cos nt n

1 (1)n = n

0, 2/n,

per n pari, per n dispari.

3.2-9. Sia f la funzione periodica dispari che vale 1 per b < x < b, con 0 < b < /2, e si annulla nei restanti punti dellintervallo [0, ]. Vericare il seguente sviluppo in serie di Fourier: 4 cos 3b cos 5b f (x) = cos b sin x + sin 3x + sin 5x + . . . . 3 5

Capitolo 3. Serie di Fourier

c 88-08-07923-6

Si osservi che per b 0 si ottiene lo sviluppo della funzione sign (x) ( Esempio 3.2-1). Soluzione. Si ha 2 2 b bn = f (t) sin(nt) dt = sin(nt) dt = 0 b 2 = [ cos(nb) cos n( b) ]. n Ma le formule di addizione forniscono cos n( b) = cos(n ) cos(nb) = (1)n cos(nb), quindi i coecienti bn sono nulli per n pari, valgono 4 cos(nb)/(n) per n dispari. 3.2-10. Vericare le uguaglianze

t sin kt dt =
0

(1)k+1 ; k 2 (1)k . k2

t cos kt dt =
0

0, per k pari, 2/k 2 , per k dispari;

t2 cos kt dt =
0

Soluzione. Basta fare unintegrazione per parti (per i primi due integrali) e due inte-

grazioni per parti per lutimo integrale. In realt` a, dopo aver eseguito unintegrazione per parti sullultimo integrale ci si riporta al primo dei tre integrali considerati. 3.2-11. Dimostrare che si ha + sin t dt = t 2 0 in base alle seguenti considerazioni: i) lintegrale in questione esiste come integrale generalizzato semplicemente (ma non assolutamente) convergente, in virt` u del criterio di convergenza di Dirichlet: esso aerma che lintegrale generalizzato
x x+

lim

f (t)g (t) dt
0

esiste nito se f ` e dotata di una primitiva limitata (nel nostro caso ` e la funzione cos t) e g ` e monotona e decrescente a 0 allinnito (nel nostro caso ` e la funzione t 1/t); ii) si ha, per ogni n,
0

sin (n + 1/2)t dt = ; 2 sin(t/2) 2

iii) si ha
n+

lim

1 1 2 sin(t/2) t

sin (n + 1/2)t dt = 0

(dimostrare che la funzione entro parentesi tonde tende a 0 per t 0); iv ) inne
0

sin (n + 1/2)t dt = t

(n+1/2) 0

sin x dt. x

Soluzione. Lidentit` a ii)


0

sin (n + 1/2)t dt = ; 2 sin(t/2) 2

` e stata stabilita come una delle propriet` a del nucleo di Dirichlet ( formula (5), par. 3.2). Per la iii) si tratta di applicare il Lemma di Riemann-Lebesgue alla funzione

c 88-08-07923-6

Esercizi

1 1 t 2 sin(t/2) = . 2 sin(t/2) t 2t sin(t/2)

Ora si ha t3 2t sin(t/2) = t2 + O(t3 ), t 0 + O(t4 ), 24 quindi la funzione in esame ` e prolungabile con continuit` a nellorigine, ponendola uguale a 0. Combinando la ii) con la iii) si ottiene t 2 sin(t/2) =
n+

lim

sin (n + 1/2)t dt = . t 2

Inne, per vericare la iv ) basta operare il cambiamento di variabile (n+1/2)t = x.

3.3. Serie di Fourier: convergenza uniforme 3.4. Serie di Fourier: convergenza in media quadratica
3.4-1. Vericare il seguente sviluppo in serie per la funzione f (x) := | cos x|: f (x) = 2 4 + cos 2x cos 4x cos 6x + + ... 13 35 57
/2

Soluzione. Si trova

2 2 | cos x| dx = 0 4 = . Per n 1 si ha analogamente a0 = an = 2

cos x dx
0 /2

cos x dx =

| cos x| cos nx dx =
0

/2

cos x cos nx dx
0 /2

cos x cos nx dx .

Per n = 1 la funzione integranda ammette la primitiva x (x = sin x cos x)/2, da cui segue facilmente a1 = 0. Per n > 1, con due integrazioni per parti si ottiene la primitiva sin x cos nx n cos x sin nx , n2 1 da cui segue 4 (1)n/2 an = (n 1)(n + 1) , per n pari , 0, per n dispari. x 3.4-2. Per la funzione f (x) := max {0, cos x} = (cos x)+ vericare lo sviluppo: f (x) = 1 cos x 2 + + 2 cos 2x cos 4x cos 6x + + ... 13 35 57 .

Fare un calcolo diretto, oppure sfruttare il fatto che (cos x)+ = (cos x + | cos x|)/2. Vericare la convergenza totale, dunque uniforme, della serie scritta; stessa questione per la serie del precedente esercizio. Soluzione. Sommando cos x allo sviluppo precedentemente ottenuto e dividendo per 2, si ottiene quanto richiesto. Si pu` o osservare che tanto la funzione x | cos x| quanto la funzione x (cos x)+ vericano le ipotesi della Proposizione 3.3-1. 3.4-3. Se a > 0 ` e una costante non intera, vericare il seguente sviluppo in serie uniformemente convergente:

Capitolo 3. Serie di Fourier

c 88-08-07923-6

cos ax =

2 sin a

1 + 2a

(1)k
k=1

a cos kx . a2 k 2

Dedurne il seguente sviluppo (in serie non uniformemente convergente) sin ax = 2 sin a (1)k
k=1

k sin kx, a2 k 2

sfruttando i legami tra i coecienti di Fourier di una funzione ed i coecienti analoghi della sua derivata. Soluzione. Sintende che la funzione f ` e 2 -periodica e vale cos ax per |x| . 1

-10 -5 5

10

La gura sopra mostra il caso a = 0.6, mentre quella sotto mostra il caso a = 1.3 1

-5 -10

5 10

-1
Si ha 2 2 2 sin a cos at dt = sin at 0 = 0 a a Per ogni k 1 si ha poi 2 2 ak = cos at cos kt dt = Ik . 0 Lintegrale Ik pu` o essere calcolato mediante le formule di Werner, oppure con due integrazioni per parti. Seguendo questultimo metodo si ha sin kt a Ik = cos at + sin at sin kt dt = k k 0 0 a cos kt a = + cos at cos kt dt = sin at k k k 0 0 a0 = = a sin a da cui segue (1)k 2a sin a (1)k = ak = . 2 2 a k a2 k 2 La funzione f verica le ipotesi della Prop. 3.3-1. Si osservi che ak = O(1/k 2 ). Essendo poi 1 sin ax = (cos ax) , a i coecienti bk dello sviluppo della funzione sin ax si ottengono dai coecienti ak dello sviluppo di cos ax moltiplicandoli per il fattore k/a. Ik = a sin a (1)k a2 + 2 Ik , 2 k k

c 88-08-07923-6

Esercizi

3.4-4. Dallidentit` a 1 (x | y ) := x + y 2 x y 2 + i x + iy 2 i x iy 2 4 valida in ogni s.v. complesso con prodotto scalare, dedurre che se : V1 V2 (V1 , V2 s.v. complessi) ` e unisometria lineare, cio` e (x)
2

= x

per ogni x V1 , allora conserva i prodotti scalari: (x) | (y )


2

= (x | y )1 ,

per ogni x, y V1 . (Gli indici in basso stanno a specicare che si tratta dei prodotti scalari e delle norme negli spazi V1 e V2 rispettivamente.) Soluzione. Lidentit` a precedente, scritta in corrispondenza dei vettori (x) e (y ), fornisce infatti 1 (x) | (y ) 2 = (x) + (y ) 2 (x) (y ) 2 + 4 + i (x) + i(y ) 2 i (x) i(y ) 2 = 1 = (x + y ) 2 (x y ) 2 + 4 + i (x + iy ) 2 i (x iy ) 2 = 1 = x + y 2 x y 2 + i x + iy 2 i x iy 2 = 4 = (x | y )1 3.4-5. Applicando lidentit` a di Parseval allo sviluppo 1 4 1 |x| = cos x + 2 cos 3x + 2 cos 5x + 2 3 5 dedurre luguaglianza 1 4 = (2k + 1)4 96

k0

Con un ragionamento simile a quello svolto nellesercizio 3.2-6, dedurne che 1 4 = 4 n 90

n1

Soluzione. Si ha

2 2

=2
0

x2 dx =

2 3 . 3
2 2

Lidentit` a di Parseval si scrive 2 |a0 | |ak |2 = f + 2


k1

16 + 2 2
2 k0

1 = 2 3 , (2k + 1)4 3

da cui 16 2
k0

1 2 = (2k + 1)4 6

=
k0

1 4 . = (2k + 1)4 96

Si ha poi s :=
k1

1 = k4

k0

1 + (2k + 1)4

k1

1 1 4 + s, = 4 (2k ) 96 16

da cui segue subito il valore fornito per s.

Capitolo 3. Serie di Fourier

c 88-08-07923-6

3.4-6. Applicando lidentit` a di Parseval allo sviluppo 4 1 1 sgn (x) = sin x + sin 3x + sin 5x + 3 5 dedurre luguaglianza 1 2 . = 2 (2k + 1) 8

k0

Soluzione. Si ha

2 2

1 dx = 2. 16 2 1 = 2 (2k + 1)2

Lidentit` a di Parseval si scrive


k1

|bk |2 = f

2 2

k0

da cui segue subito il risultato. 3.4-7. Applicando lidentit` a di Parseval allo sviluppo 1 1 x = 2 sin x sin 2x + sin 3x 2 3 dedurre luguaglianza 1 2 = . n2 6

n1

Soluzione. Si ha

2 2

x2 dx =

2 3 . 3

Lidentit` a di Parseval si scrive 1 2 4 = 3 , 2 n 3


n1

da cui segue subito il risultato. 3.4-8. Applicando lidentit` a di Parseval allo sviluppo 2 1 1 4 cos x 2 cos 2x + 2 cos 3x 3 2 3 dedurre luguaglianza x2 = 4 1 = . 4 n 90

n1

Soluzione. Si ha

2 2

x4 dx =

2 5 . 5 8 4 1 , = n4 45

Lidentit` a di Parseval si scrive 4 4 2 1 + 16 = 5 18 n4 5


n1

16
n1

da cui segue subito il risultato.

3.5 Serie di Fourier: ulteriori risultati


3.5-1. Sia x f (x) una funzione periodica di periodo T , avente coecienti di Fourier ck , k Z. Vericare che la funzione x f (x x0 ) ha coecienti di Fourier eikx0 ck .

c 88-08-07923-6

Esercizi

Soluzione. Per il coeciente di Fourier della funzione in esame, sia k , abbiamo

k =

1 T

T 0

f (x x0 ) eikx dx =

1 ikx0 e T

T 0

f (t) eikt dt = eikx0 ck .

Abbiamo eettuato il cambiamento di variabile x x0 = t e abbiamo tenuto conto del fatto che lintegrale di una funzione T -periodica su un intervallo di lunghezza T ` e invariante rispetto alla collocazione dello stesso intervallo ( esercizio 3.2-4). In particolare si ha |k | = |ck |, per ogni k . 3.5-2. Dedurre dal precedente esercizio che le funzioni x f (x) e x f (x x0 ) hanno lo stesso spettro di ampiezza. 2 Soluzione. Dalle relazioni Ak = a2 k + bk , ck (ak ibk )/2, segue |Ak | = 2|ck |. Dunque lo spettro di ampiezza dipende esclusivamente dai valori assoluti dei coecienti di Fourier. 3.5-3. Sia f una funzione periodica di periodo T , avente coecienti di Fourier ck , k Z. Dimostrare che se f (t) = f (t) (propriet` a di simmetria hermitiana), dunque in particolare se f ` e reale pari, allora i coecienti di Fourier sono reali: ck = ck . Suggerimento Scrivere ck sotto forma integrale e cambiare di segno la variabile dintegrazione. Soluzione. Seguendo il suggerimento si trova ck = 1 T = ck .
T

f (t) eikt dt =
0

1 T

T 0

f (x) eikx dx =

1 T

T 0

f (x) eikx dx =

Esercizi proposti

3.P-1. Calcolare la somma della serie trigonometrica cos nx 2n


n0

ni a partire dalla serie geometrica einx . 2n

n0

Soluzione. La somma della serie trigonometrica ` e la arte reale della somma della

serie geometrica eix 2


n

n0

poich e la ragione di tale serie ha come valore assoluto 1/2, essa converge alla somma 1 2 2 = = . ix ix 1 e /2 2e (2 cos x) i sin x La parte reale della somma calcolata vale 4 2 cos x . 5 4 cos x 3.P-2. Calcolare lo sviluppo in serie di Fourier della funzione periodica di periodo 2 , la cui restrizione allintervallo [, ] vale x per /2 x /2, e vale 0 altrimenti (fare uno schizzo). A quale valore converge la serie di Fourier per x = /2? Soluzione. Poich ef` e una funzione dispari, avremo uno sviluppo del tipo

10

Capitolo 3. Serie di Fourier

c 88-08-07923-6

f (x) =
n1

bn sin nx.

Le condizioni di Dini sono soddisfatte per x = /2, quindi la somma della serie in tale punto vale /4: bn sin(n/2) = /4
n1

cio` e b2k+1 sin((2k + 1)/2) =


k0 k0

b2k+1 (1)k = /4.

()

Abbiamo tenuto conto del fatto che sin(n/2) ` e nullo per n pari, vale (1)k per n = 2k + 1. Per i coecienti bn abbiamo, con unintegrazine per parti, bn = 2
/2

x sin nx dx =
0

2 sin(n/2) cos(n/2) + . n n2 2 (1)k . (2k + 1)2 1 2 = , (2k + 1)2 8

Separando i casi n pari e n dispari, avremo duque b2 k = (1)k+1 , 2k b2k+1 =

Sostituendo nella (*) si trova dunque 2 (1)k (1)k = 2 (2k + 1) 4


k0

k0

risultato gi` a ottenuto (v. esercizio 3.4-6). Si veda anche il notebook http://eulero.ing.unibo.it/~barozzi/MI2/Mathematica/Mathematica-3.2.nb

c 88-08-07923-6

Esercizi

11

Un elenco di sviluppi in serie di Fourier in forma visuale


1. f (x) = 4 cos 3b cos 5b cos b sin x + sin 3x + sin 5x + . . . 3 5
1

-6 -4 -2 2

4 6

-1

La restrizione di f allintervallo [, ] si scrive [b,b] (x) sgn(x), con 0 b < /2. Per b = 0 si ottiene lo sviluppo della funzione segno. La gura seguente mostra i primi cinque polinomi di Fourier di ordine dispari della funzione segno.

2 1 0 -1 -2 5 0 -5 5

2. f (x) =

2 b sin 3b sin 5b + sin b cos x + cos 3x + cos 5x + . . . 2 3 5


1

(esempio 3.5-1)

-6 -4 -2 2

4 6

b b

12

Capitolo 3. Serie di Fourier

c 88-08-07923-6

La restrizione di f allintervallo [, ] si scrive [b,b] (x). 3. f (x) = 4 cos 3x cos 5x cos 7x cos x + + ... 3 5 7
1

-6 -4 -2 2

4 6

-1

La restrizione di f allintervallo [, ] si scrive sgn(/2 |x|). 4. f (x) = 2 sin x + sin 2x sin 3x + + ... 2 3 ( graco monometrico)

3 2 1 -6 -4 -2 -1 -2 -3 2

4 6

La restrizione di f allintervallo [, ] si scrive x sgn(x) . La gura seguente mostra i primi cinque polinomi di Fourier della funzione considerata.

2 0 -2 5 0 -5 5

c 88-08-07923-6

Esercizi

13

5. f (x) =

8 sin 3x sin 5x sin x + + ... 2 32 52

( graco monometrico)

1 -6 -4 -2 -1 2 4 6

La restrizione di f allintervallo [, ] vale x per |x| /2, vale sgn(x) x per /2 |x| . La gura seguente mostra i primi cinque polinomi di Fourier della funzione considerata.

-1 5 0 -5 5
6. f (x) = 1 cos 2b b 1 cos 3b 2 (1 cos b) cos x + cos 2x + cos 3x + . . . + 2 2 22 32
1

-7.5

-5

-2.5

2.5

7.5

La restrizione di f allintervallo [, ] vale (1 |x/b|)+ = max{1 |x/b|, 0}. 7. f (x) = (cos x)+ = max{0, cos x} = 1 cos x 2 cos 2x cos 4x cos 6x = + + + + ... 2 13 35 57 (esempio 3.4-2)

14

Capitolo 3. Serie di Fourier

c 88-08-07923-6

-7.5

-5

-2.5

2.5

7.5

La gura seguente mostra i primi cinque polinomi di Fourier della funzione f .

1 0.5 0 5 0 -5 5

8. f (x) = | cos x| =

2 4 +

cos 2x cos 4x cos 6x + + ... 13 35 57


1

. (esempio 3.4-1)

-7.5

-5

-2.5

2.5

7.5

La gura seguente mostra i primi cinque polinomi di Fourier della funzione f .

0.5 5

-5

4.

Funzioni di variabile complessa Esercizi

[Revisione: dicembre 2007] http://www.ciram.unibo.it/~barozzi/MI2/PDF/MI2-Cap.4-Ese.pdf

4.1. Il campo complesso 4.2. Funzioni di una variabile complessa 4.3. Funzioni olomorfe
4.3-1. Vogliamo vericare che, per ogni z = x + iy C, si ha x + iy n lim 1 + = ex (cos y + i sin y ). n n Si tratta di vericare che il modulo del primo membro tende a ex , il suo argomento tende a y . Utilizzeremo i seguenti risultati (che supporremo noti): a) se (an ) ` e una successione reale che tende al limite a, allora an n lim 1 + = ea ; n n b) limt0 (arctan t)/t = 1. La dimostrazione pu` o essere articolata nei seguenti punti. Si ha 1+ da cui 1+ x + iy n
n

x + iy 2x x2 + y 2 = 1+ + n n n2

1 /2

= 1+

1 x2 + y 2 2x + n n

n/2

dove lultima quantit` a tende a e2x/2 = ex , in virt` u del punto a). Poich e la parte reale di 1 + (x + iy )/n tende a 1, essa ` e positiva per tutti gli n abbastanza grandi, quindi Arg 1 + (x + iy )/n = arctan Ne segue x + iy n y x + iy = n arctan ; = n Arg 1 + n n n+x a questo punto basta fare il limite dellultima quantit` a per n , sfruttando il punto b). arg 1 +
Soluzione. La soluzione ` e gi` a contenuta nel testo. Ci limitiamo ad osservare che

(y/n) y = arctan . (n + x)/n n+x

Arg 1 + e

x + iy n

= Arg

n+x y +i = arctan n n

y n , n n+x

Capitolo 4. Funzioni di variabile complessa

c 978-88-08-12546-0

n arctan

y ny n+x y = arctan . x+n n+x y n+x

4.3-2. La formula di De Moivre ( formula (8) del paragrafo 4.1) si pu` o scrivere cos nt = (c + is)n , avendo posto c := cos t e s := sin t. Supposto n 2, si sviluppi il secondo membro con la formula del binomio e si uguaglino a primo e a secondo membro le parti reali, ottenendo lidentit` a cos nt =
0kn/2

n (1)k cn2k s2k . 2k

Tenendo presente che s2 = 1 c2 , si deduca che cos nt si pu` o scrivere sotto forma di polinomio di grado n nella variabile c: cos nt =
0kn/2

n (1)k cn2k (1 c2 )k =: Tn (c). 2k

Tn ` e pari per n pari, dispari per n dispari. Si verichino le formule: T1 (c) = c, T2 (c) = 2c2 1, T3 (c) = 4c3 3c, T4 (c) = 8c4 8c2 + 1. sev, dal nome del matematico russo I polinomi Tn sono noti come polinomi di Ceby P.L. Ceby sev (1821-1894). Ritroveremo questi polinomi nellesempio 8.1-4.
Soluzione. Limitiamoci a vericare lespressione di T4 :

T4 (c) =
0k2

4 (1)k c42k (1 c2 )k = 2k

4 4 4 2 4 c c (1 c2 ) + (1 c2 )2 = 0 2 4

= c4 6(c2 c4 ) + 1 + c4 2c2 = 8c4 8c2 + 1. 4.3-3. Osservato che luguaglianza x = cos t, per x [1, 1] equivale a t = arccos x, si ottenga luguaglianza Tn (x) = cos(n arccos x), x [1, 1], e da questa le relazioni Tn (1) = 1, Tn (1) = (1)n .
Soluzione.

Basta osservare che arccos 1 = 0, arccos(1) = , e che cos 0 = 1, cos(n ) = (1)n .


1

0.5

-1

-0.5

0.5

-0.5

-1

I polinomi di Ceby sev di grado non superiore a 4. 4.3-4. A partire dalle formule di addizione

c 978-88-08-12546-0

Esercizi

cos (n + 1)t = cos nt cos t sin nt sin t, cos (n 1)t = cos nt cos t + sin nt sin t si deduca la relazione ricorrente cos (n + 1)t = 2 cos nt cos t cos (n 1)t , e da questa la relazione Tn+1 (x) = 2x Tn (x) Tn1 (x). Essa consente il calcolo dei polinomi di Ceby sev, a partire dai termini iniziali T0 (x) = = 1, T1 (x) = x.
Soluzione. Sommando membro a membro le due formule di addizione si ottiene la

formula cos (n + 1)t = 2 cos nt cos t cos (n 1)t ; basta poi ricordare che, ove si ponga t := arccos x, si ha Tn (x) := cos(nt). 4.3-5. Si verichi che la successione 0, cos nt cos mt dt = , 0 /2, n cos nt ` e ortogonale su L2 [0, ] : se n = m, se n = m = 0, se n = m > 0.

Operando il cambiamento di variabile t = arccos x, x [1, 1], si ottengano le relazioni se n = m, 0, 1 Tn (x) Tm (x) se n = m = 0, dx = , 1 x2 1 /2, se n = m > 0. Esse esprimono ortogonalit` sev sullintervallo [1, 1], rispetto a dei polinomi di Ceby alla funzione peso x 1/ 1 x2 .
Soluzione. Per vericare le prime identit` a si pu` o ricorrere alle cosiddette formule di

Werner (PCAM, esercizio 2.4-9): 1 n+m nm cos t + cos t . 2 2 2 Le seconde identit` a si riportano alle prime mediante il cambiamento di variabile x = = cos t. cos nt cos mt = 4.3-6. Vericare, mediante un calcolo diretto, luguaglianza | sin z |2 = sin2 x + sinh2 y. Determinare gli zeri della funzione seno in C. Determinare luguaglianza analoga per la funzione cos z .
Soluzione. Se z = x + iy , si ha iz = y + ix, da cui

sin z =

1 y 1 iz (e eiz ) = e (cos x + i sin x) ey (cos x i sin x) = 2i 2i 1 = i sin x (ey + ey ) + cos x (ey ey ) = 2i = sin x cosh y + i cos x sinh y.

Ne segue | sin z |2 = sin2 x cosh2 y + cos2 x sinh2 y = = sin2 x cosh2 y + (1 sin2 x) sinh2 y = = sin2 x (cosh2 y sinh2 y ) + sinh2 y = = sin2 x + sinh2 y.

Capitolo 4. Funzioni di variabile complessa

c 978-88-08-12546-0

Dunque la funzione seno ` e illimitata in C, in accordo col teorema di Liouville; da sin z = 0 segue sin x = sinh y = 0, dunque y = 0, x = k , k intero. In conclusione: gli zeri della funzione seno in campo complesso sono soltanto gli zeri della stessa funzione in campo reale. In modo analogo si ottiene cos z = cos x cosh y i sin x sinh y, da cui | cos z |2 = cos2 x + sinh2 y .

Esercizi proposti
4.3-P1. Vericare che lunica funzione f olomorfa in C per cui si abbia f = f e f (0) = 1 ` e la funzione esponenziale f (z ) = f (x, y ) = ex (cos y + i sin y ): z C, f (z ) = f (z ) f (0) = 1 = f (z ) = ez = ex (cos y + i sin y )

Soluzione. Se f = u+iv , dalla uguaglianza f (z ) = fx (x, y ) = f deduciamo ux +ivx =

u + iv , dunque ux = u, vx = v . Per ogni ssato y le funzioni x u(x, y ) e x v (x, y ) sono soluzioni dellequazione dierenziale (x) = (x) e pertanto si scrivono u(x, y ) = c1 (y ) ex , u(x, y ) = c2 (y ) ex ,

con c1 e c2 funzioni opportune. Daltra parte le condizioni di Cauchy-Riemann forniscono ux = vy c1 (y ) ex = c2 (y ) ex c1 (y ) = c2 (y ), uy = vx c1 (y ) ex = c2 (y ) ex c1 (y ) = c2 (y ). Finalmente da f (0) = 1 seguono le condizioni iniziali c1 (0) = 1, c2 (0) = 0. In denitiva c1 e c2 sono soluzioni del problema di valori iniziali c1 (y ) = c2 (y ), c2 (y ) = c1 (y ), c1 (0) = 1, c2 (0) = 0 e dunque c1 (y ) = cos y , c2 (y ) = sin y . 4.3-P2. Data la funzione u(x, y ) = 3x + 4y , determinare la funzione v (x, y ) tale che v (0, 0) = 0 e f (x + iy ) = u(x, y ) + iv (x, y ) sia intera. Soluzione. Deve essere vy = ux = 3, quindi v (x, y ) = 3y + g (x), con g funzione da determinare. Daltra parte deve essere anche vx = uy , cio` e g (x) = 4, da cui g (x) = 4x + c. Dunque v (x, y ) = 3y 4x + c, e nalmente, per avere v (0, 0) = 0,c = 0. In denitiva f (z ) = f (x + iy ) = 3x + 4y + i(3y 4x) = 3(x + iy ) 4i(x + iy ) = (3 4i)z.

4.4. Serie di potenze


4.4-1. Dimostrare per induzione la seguente formula per le derivate del prodotto di due funzioni
n

Dn f (z )g (z ) =
k=0

n Dk f (z ) Dnk g (z ). k

Soluzione. Per n = 1 abbiamo la formula per la derivazione del prodotto:

D f (z )g (z ) = Df (z ) g (z ) + f (z ) Dg (z ). Procediamo per induzione rispetto a n; se la formula in questione sussiste per un assegnato n abbiamo

c 978-88-08-12546-0

Esercizi

D(n+1) f (z )g (z ) = D Dn f (z )g (z )
n

=D
k=0 n

n Dk f (z ) Dnk g (z ) = k

=
k=0

n D(k+1) f (z ) Dnk g (z ) + k

k=0

n Dk f (z ) Dnk+1 g (z ). k

Se nella prima delle due sommatorie si pone k + 1 = h k = h 1, e successivamente si scrive ancora k al posto di h si ottiene
n

Dn+1 f (z )g (z ) +
h=1 n

n Dh f (z ) Dn+1h g (z )+ h1 n Dk f (z ) Dn+1k g (z ) + f (z ) Dn+1 g (z ) = k n n + k1 k Dk f (z ) Dn+1k g (z )+

+
k=1 n

= Dn+1 f (z )g (z ) +
k=1

+ f (z ) Dn+1 g (z ) =
n+1

=
k=0

n+1 Dk f (z ) Dn+1k g (z ). k

Abbiamo sfruttato lidentit` a n n n+1 + = . k1 k k 4.4-2. Sia f una funzione olomorfa in un intorno dellorigine e f (z ) = n0 an z n il suo sviluppo di Taylor in tale intorno. Indichiamo simbolicamente con la scrittura f (an ) la corrispondenza tra la funzione f e la successione (an ) dei suoi coecienti di Taylor. Se g ` e una funzione analoga ad f , e g (z ) = n0 bn z n ` e il suo sviluppo di Taylor, vericare che: f +g f f g (an ) + (bn ) := (an + bn ), (an ) := (an ),
n

C, ak bnk .
k=0

(an ) (bn ) :=

Loperazione che alla coppia di successioni (an ), (bn ) associa la successione (an ) (bn ) = a0 b0 , a0 b1 + a1 b0 , a0 b2 + a1 b1 + a2 b0 , . . . , si chiama convoluzione tra le successioni date. Vericare che se tali successioni sono denitivamente nulle (dunque f e g sono polinomi) si ritrova lordinaria moltiplicazione tra polinomi. Suggerimento Si scriva lidentit` a dellesercizio precedente nella forma Dn f (z )g (z ) = n!
n

k=0

Dk f (z ) Dnk g (z ) . k! (n k )!

Soluzione. Lunico cosa da dimostrare ` e la terza formula, relativa al prodotto di due

funzioni. Basta ricordare che ln-esimo coeciente dello sviluppo in serie del prodotto fg ` e [D(n) (f g )(0)]/n! e applicare il risultato dellesercizio precedente. Dunque se f, g , e h sono tre funzioni analitiche in un intorno dellorigine, f (an ), g (bn ) e h (cn ), ed inoltre (cn ) = (an ) (bn ), allora h = f g . 4.4-3. Sia f (z ) = ez1 z , g (z ) = ez2 z , dove z1 e z2 sono due numeri complessi ad arbitrio. Vericare che f (z )g (z ) = ez1 z ez2 z = e(z1 +z2 )z , da cui, ponendo z = 1, ez1 ez2 = ez1 +z2 .

Capitolo 4. Funzioni di variabile complessa

c 978-88-08-12546-0

Soluzione. Con i simboli dellesercizio precedente abbiamo

ez1 z

n z1 n!

ez2 z

n z2 n!

e(z1 +z2 )z

(z1 + z2 )n n!

la convoluzione tra le prime due successioni fornisce la terza successione, come abbiamo vericato nel testo, a pagina 126. 4.4-4. Dalluguaglianza Dz n /n! = z n1 /(n 1)!, valida per ogni n > 0, dedurre che Dez = ez .
Soluzione. Basta osservare che, derivando termine a termine la serie esponenziale
n0

z n /n!, si ottiene nuovamente le serie stessa.

4.4-5. Derivando termine a termine lo sviluppo della funzione 1/(1 z )2 , ottenuto nellesempio 4.4-1, si ottenga lo sviluppo 1 = (1 z )3 (n + 1)(n + 2) n z , 2 |z | < 1.

n0

Si ottenga lo stesso risultato moltiplicando tra loro gli sviluppi delle funzioni 1/(1 z ) e 1/(1 z )2 .
Soluzione. Derivando termine a termine luguaglianza

1 = (1 z )2 = (1 z )2 si ottiene 2(1 z )3 =
n1

(n + 1) z n
n0

(n + 1) n z n1 ,

da cui la formula proposta nel testo, scrivendo n al posto di n 1. Moltiplicando la serie n0 (n + 1) z n per la serie n0 z n si ottiene la serie
n

(k + 1) z k z nk ,
n0 k=0

dunque la serie precedentemente ottenuta, ove si ricordi che la somma dei numeri naturali da 1 a n + 1 vale (n + 1)(n + 2)/2. 4.4-6. Dallo sviluppo in serie geometrica di ragione z 1 (1)n z n , |z | < 1, = 1+z
n0

si deduca lo sviluppo 1 = (1)n (n + 1) z n = 1 2z + 3z 2 4z 3 + . . . , (1 + z )2


n0

prima mediante derivazione e successivamente mediante moltiplicazione della serie di partenza per se stessa.
Soluzione. Limitiamoci alla prima richiesta; derivando la serie data si ottiene

1 = (1 + z )2

(1)n n z n1 ,
n1

da cui il risultato fornito nel testo, scrivendo n al posto di n 1. 4.4-7. Dedurre dalla serie geometrica lo sviluppo 1 = a2 + z 2 (1)n
n0

z 2n , a2n+2

c 978-88-08-12546-0

Esercizi

valido, per ogni ssato a = 0, per |z | < |a|.


Soluzione. Si ha

1 1 1 = 2 = 2 2 2 2 2 a +z a (1 + z /a ) a

(1)n
n0

z 2n . a2n

4.4-8. La successione dei numeri di Fibonacci ` e denita ricorsivamente ponendo F0 := 0, F1 := 1; Fn := Fn1 + Fn2 , n 2. Vericare che la serie Fn+1 z n = 1 + z + 2z 2 + 3z 3 + 5z 4 + . . .
n0

ha raggio di convergenza R = 2/(1 +


Suggerimento

5).

Posto rn := Fn+1 /Fn 1, vericare la formula ricorsiva rn+1 = = 1 + 1/rn , da cui passando il limite . . . .

Soluzione. Scrivendo la formula ricorsiva che genera i numeri di Fibonacci nella

forma Fn+2 := Fn+1 + Fn , si ottiene la relazione rn+1 = 1 + 1/rn , da accoppiare alla condizione iniziale r1 = 1, semplicemente dividendo per Fn+1 . La successione dei e studiata in dettaglio in PCAM, esempio 3.2-3 ed esercizio 3.4-3; per rapporti (rn ) ` la soluzione degli esercizi si veda il le http://eulero.ing.unibo.it/~barozzi/PCAM Complementi/ PCAM Soluzioni 3.pdf. La successione dei termini di indice dispari ` e strettamente crescente, quella dei termini di indice pari ` e strettamente decrescente ed entrambe tendono allo stesso limite. Se L ` e il limite in questione, la relazione rn+1 = 1 + 1/rn per n fornisce 1 1+ 5 L=1+ L2 L 1 = 0 = L = . L 2 Abbiamo tenuto conto del fatto che tutti i rapporti rn sono 1. Il criterio del rapporto applicato alla serie n0 Fn+1 |z |n conduce al limite 1+ 5 lim rn+1 |z | = |z |, n 2 dunque la serie in esame ha raggio di convergenza R = 2/(1 + 5). 4.4-9. Detta s(z ) la somma della serie considerata nel precedente esercizio, |z | < < 2/(1 + 5), dedurre dalla denizione dei numeri di Fibonacci la relazione 1 s(z )(1 z z 2 ) = 1 = s(z ) = . 1 z z2 Vericare che la funzione razionale 1/(1 z z 2 ) ` e olomorfa nel disco |z | < 2/(1+ 5).
Soluzione. Dalle uguaglianze

s(z ) = 1 + z + 2z 2 + 3z 3 + 5z 4 + . . . + Fn+1 z n + . . . , zs(z ) = z 2 s(z ) = z + z 2 + 2z 3 + 3z 4 + . . . + Fn z n + . . . , z 2 + z 3 + 2z 4 + . . . + Fn1 z n + . . . ,

si ottiene la relazione voluta sottraendo la seconda e la terza uguaglianza dalla prima. Il trinomio 1 z z 2 ammette gli zeri (1 5)/2, di cui il secondo ` e il pi` u prossimo allorigine, distando da essa 51 2 . = 2 5+1

Capitolo 4. Funzioni di variabile complessa

c 978-88-08-12546-0

4.5. Integrazione in campo complesso


Nel seguito useremo il simbolo f (z ) dz

per indicare lintegrale di f sul circuito , orientato positivamente. 4.5-1. Sia : t z0 + r eit , 0 t k 2 , con k intero 1, cio` e la circonferenza di centro z0 e raggio r percorsa in senso positivo k volte. Vericare che ( Esempio 4.5-4) 1 1 dz = k. 2i z z0
Soluzione. A parametrizzazione avvenuta si ottiene

1 2i

2k 0

1 1 ir eit dt = r eit 2

2k

dt = k.
0

4.5-2. Sia u(x, y ) una funzione di classe C (1) in un aperto contenente il rettangolo D := [a, b] [c, d] e sia la frontiera di D orientata positivamente rispetto ad esso. Vericare che d u [ u(b, y ) u(a, y ) ] dy = u dy, dx dy = c D x
D

u dx dy = y

b a

[ u(x, d) u(x, c) ] dx = u dx.

Soluzione. Infatti

u dx dy = x

d c a

u dx x

dy =
c

u(x, y )

x=b x=a

dy.

4.5-3. Supponiamo che il dominio D sia descritto nel modo seguente: D := {(x, y ) | a x, 1 (x) y 2 (x)} dove 1 e 2 sono funzioni continue. Se u ` e di classe C (1) in un aperto contenente D, vericare che b u dx dy = [ u(x, 2 (x)) u(x, 1 (x)) ] dx = u dx. D y a Analogamente, se D ` e descritto come D := {(x, y ) | c y d, 1 (y ) x 2 (y )}, dove 1 e 2 sono funzioni continue, allora
D

u dx dy = x u dx dy = y

d c

[ u(2 (y ), y ) u(1 (y ), y ) ] dy = u dy.


b 2 (x) 1 (x) b

Soluzione. Infatti

u dy y

dx =
a

u(x, y )

y =2 (x) y =1 (x)

dx.

4.6. Proprieta ` delle funzioni analitiche


4.6-1. Provare lidentit` a ez1 +z2 = ez1 ez2 , z1 , z2 , C, utilizzando il teorema di unicit` a del prolungamento analitico. Suggerimento Fissare z1 R e considerare le funzioni intere z ez1 +z , z z1 z e e ; esse coincidono . . . .

c 978-88-08-12546-0

Esercizi

Soluzione. . . . sullasse reale, dunque coincidono in tutto il campo complesso.

4.6-2. Utilizzando il teorema fondamentale dellalgebra, mostrare che ogni polinomio p (non costante) trasforma C su se stesso, cio` e il piano complesso ha come immagine mediante p se stesso. Suggerimento Per ogni ssato w, lequazione p(z ) = w equivale a . . . .
Soluzione. . . . p(z ) w = 0, dove p(z ) w ` e ancora un polinomio; dunque il teorema

fondamentale assicura che il primo membro si annulla in almeno un punto.

4.7. Punti singolari. Serie bilatere


4.7-1. Determinare (e classicare) le singolarit` a delle funzioni 1) z , sin z 2) sin z . z2 2

Soluzione. La prima funzione ha una sigolarit` a eliminabile nellorigine e un polo

semplice in ciascuno dei punti zk = k , k intero non nullo; infatti in tali punti il numeratore non si annulla mentre il denominatore presenta uno zero del primo ordine (= zero semplice). Poich e il denomitatore della seconda funzione si scrive (z )(z + ), essa presenta due singolarit` a eliminabili nei punti z = e z = ; in tali punti tanto il numeratore quanto il denominatore presentano zeri semplici. 4.7-2. Considerate le funzioni 1) e1/z ,
2

2)

e1/z , z

3)

sin

1 , z

4)
2

sinh

1 , z

sin z 4 1 ez 1 e1/z 5) , 6) , 7) , 8) z 3 sin , z3 z z z classicare lorigine in quanto singolarit` a per ciascuna di esse.
Soluzione. Esercizio 1. Da

1 1 1 + + ... 2 2 z 2! z 3! z 6 segue che lorigine ` e una singolarit` a essenziale. Esercizio 2. Da e1/z = 1


2

e1/z 1 1 1 1 + + ... = + 2+ 3 z z z 2! z 3! z 4 segue che lorigine ` e una singolarit` a essenziale. Esercizi 3 e 4. Considerazioni analoghe alle precedenti: basta scrivere 1/z al posto di z nelle serie del seno e del seno iperbolico rispettivamente. Esercizio 5. Da sin z 4 z 4 , per z 0, segue che la funzione ha una singolarit` a eliminabile, anzi uno zero del primo ordine nellorigine. Esercizio 6. Poich e ez = 1 z + O(z 2 ), lorigine ` e una singolarit` a eliminabile. Esercizi 7 e 8. Considerazioni analoghe a quelle fatte per gli esercizi 1 e 2: in entrambi i casi si tratta di singolarit` a essenziali. 4.7-3. Calcolare i residui delle funzioni 1 sin z z cos z 1) , 2) , 3) , z2 1 cos z sin z in ciascuna delle loro singolarit` a. 1 5) z sin , z

4) e2/z ,

Soluzione. Esercizio 1. La funzione presenta due poli semplici nei punti z1 = 1,

z2 = 1, con residui rispettivamente 1/2 e 1/2. Si pu` o applicare la formula (3 ) del paragrafo 4.7.

10

Capitolo 4. Funzioni di variabile complessa

c 978-88-08-12546-0

Esercizio 2. La funzione presenta poli semplici in tutti i punti in cui si annulla il denominatore; la formula citata poco sopra consente di aermare che i residui valgono 1. Esercizio 3. La funzione presenta una singolarit` a eliminabile nellorigine. In ciascuno dei punti zk = k , k intero non nullo, c` e un polo semplice con residuo uguale a k . Esercizio 4. Da 2 2 e2/z = 1 + + 2 + . . . z z segue che il residuo nellorigine vale 2. Esercizio 5. Da 1 1 z sin = 1 + ... z 3! z 2 segue che lorigine ` e una singolarit` a essenziale con residuo nullo. 4.7-4. Mediante il metodo dei coecienti indeterminati, dedurre dagli sviluppi delle funzioni seno e coseno lo sviluppo sin z 1 2 tan z = = z + z 3 + z 5 + O(z 7 ). cos z 3 15
Soluzione. Sappiamo che la tangente ` e una funzione dispari, dunque lo sviluppo

a dunque cercato ` e del tipo tan z = a1 z + a3 z 3 + a5 z 5 + O(z 7 ). Sar` z2 z3 z4 z5 + + O(z 6 ) = z + + O(z 7 ). 2 24 6 120 Sviluppando il prodotto a primo membro, ed uguagliando i coecienti delle potenze z , z 3 e z 5 si ottengono le uguaglianze a1 z + a3 z 3 + a5 z 5 + O(z 7 ) 1 a1 = 1, 1 1 a1 + a3 = , 2 6 1 1 1 a1 a3 + a5 = , 24 2 120 da cui segue facilmente il risultato dato nel testo.

4.8. Il teorema dei residui


4.8-1 Calcolare (mediante il teorema dei residui) gli integrali
2

1)
0 2

sin2 x dx = , dx = 2 , 2 1 + sin x dx 4 = , 2 + sin x cos x 15

2)
0 2

3)
0 2

dx = 2 . 2 + sin x + cos x 0 Suggerimento Trasformare ciascun integrale in un integrale sulla circonferenza di centro lorigine e raggio 1. 4)
Soluzione. La tecnica consiste nellutilizzare le formule di Eulero, in modo da scri-

vere un integrale esteso alla circonferenza unitaria che si trasformerebbe nellintegrale proposto se si parametrizzasse la circonferenza stessa. Esercizio 1. Preferiamo indicare con la lettera t la variabile dintegrazione; per il primo integrale, posto z = eit , si ha

c 978-88-08-12546-0

Esercizi

11

sin2 t = quindi
2 0

z 1/z 2i

z 4 + 2z 2 1 , 4z 2

sin2 t dt =

|z |=1

z 4 + 2z 2 1 dz. 4z 3 i

Abbiamo tenuto conto del fatto che da z = eit segue 1 dz = ieit dt = iz dt dt = dz. iz A questo punto possiamo utilizzare il teorema dei residui. Poich e la funzine ntegranda ammette nellorigine un polo del terzo ordine con residuo uguale a i/2, si trova per lultimo integrale il valore 2i(i/2) = . Peraltro lintegrale in esame si calcola agevolmente utilizzando la primitiva t sin t cos t x . 2 Esercizio 2. Si trova 2 dt 4iz dz. 2 = 1 6 z2 + z4 1 + sin t |z |=1 0 Lequazione biquadratica che si ottiene uguagliando a zero il denominatore ammette quattro radici (a due a due tra loro opposte) di cui hanno valore assoluto minore di 1 le radici z1 = 1 2, punti e z2 = z1 . I residui della funzione integranda in tali valgono entrambi i/(2 2), dunque per lintegrale si trova 4i (i/(2 2)) = 2 . Esercizio 3. Si trova 2 dt 4z dz. = 2 4 2 + sin t cos t 0 |z |=1 1 + 8iz + z Con la sostituzione z 2 = s si ` e condotti a studiare lequazione di secondo grado 1 + 8is + s2 = 0, che ammette due radici complesse, di cui s0 = i( 15 4) ha valore assoluto minore di 1. Siamo dunque condotti ad applicare il teorema dei residui relativamente ai poli semplici 1 + i z1 = ei3/4 4 15 = 4 15 2 e z2 = z1 ; poich e i residui in tali poli valgono entrambi i/ 15, si trova il risultato fornito nel testo. Esercizio 4. Si trova 2 dt 1i dz. = 2 + sin t + cos t |z|=1 i + (2 + 2i)z + z 2 0

Il denominatore si annulla in due punti, di cui z1 = (1 + i)( 2/2 1) ha valore assoluto e il residuo della funzione integranda in tale punto vale minore di 1. Poich i/ 2, si ottiene il risultato fornito nel testo.
2

4.8-2 Con la stessa tecnica per precedente esercizio vericare che 1)


0 2

dx 2 , = a + cos x a2 1 dx 2 , = 1 + a sin x 1 a2

a > 1, 0 < a < 1, a > b > 0.

2)
0 2

3)
0

dx 2a = 2 , 2 (a + b cos x) (a b2 )3/2

12

Capitolo 4. Funzioni di variabile complessa

c 978-88-08-12546-0

Soluzione. Indichiamo ogni volta col simbolo f la funzione di variabile complessa che

viene integrata sulla circonferenza unitaria. Esercizio 1. Si ha 2 dx 2i = dz = 2i res(f ; a2 1 a) = 2 a + cos x |z |=1 z + 2az + 1 0 i 2 = 2i = . 2 a 1 a2 1 Esercizio 2. Si ha 2 dx 2 1 a2 1 = dz = 2 i res f ; i = 2 1 + a sin x a |z |=1 az + 2iz a 0 1 2 = 2i = 2 i 1a 1 a2 . Esercizio 3. Si ha 2 dx 4iz = dz = 2 2 2 (a + b cos x) |z |=1 (bz + 2az = b) 0 a2 b2 a ia = 2i res f ; = = 2i 2 b (a b2 )3/2 2a = 2 . (a b2 )3/2 4.8-3 Utilizzando il teorema dei residui vericare che 1 1 2 1) dx = dx = , 2+x+1 2x+1 x x 3 1 1 4 2) dx = dx = , 2 + x + 1)2 2 x + 1)2 ( x ( x 3 3 2 x 3) dx = , 2 )2 (1 + x 4 0 2 x 4) dx = , 2 + 4)2 (x2 + 9) ( x 200 0 2 dx 5) = , 6+1 x 3 dx 6) = , 4 + x2 + 1 x 3 (e 1) sin x 7) dx = , 2) x (1 + x e cos x 8) dx = . 2 1 + x e
Soluzione. In ciascuno degli esercizi dal numero 1 al numero 6 indichiamo con la

lettera f la funzione di variabile complessa che si ottiene scrivendo z C al posto di x R. In base al lemma del grande cerchio si hanno i seguenti risultati. Esercizio 1. i 1 2 3 )/2 = 2i = . dx = 2 i res f ; ( 1 + i 2+x+1 x 3 3 e un Esercizio 2. A dierenza dellesercizio precedente, il punto z0 = 1/2 + i 3/2 ` polo doppio per la funzione integranda, dunque il relativo residuo si calcola con la formula fornita dalla Proposizione 4.7-1, ottenendo 1 2i 4 dx = 2i res f ; (1 + i 3 )/2 = 2i = . 2 2 3 3 3 3 (x + x + 1)

c 978-88-08-12546-0

Esercizi

13

Esercizio 3. Trattandosi di una funzione pari, possiamo calcolare lintegrale su tutta la retta reale. Abbiamo i x2 dx = 2i res(f ; i) = 2i = . 2 )2 (1 + x 4 2 Esercizio 4. Trattandosi di una funzione pari, possiamo calcolare lintegrale su tutta la retta reale. Abbiamo x2 13i 3i dx = 2i [res(f ; 2i) + res(f ; 3i)] = 2i + = 2 + 4)2 (x2 + 9) ( x 200 50 = . 100 Esercizio 5. Il polinomio z 6 + 1 ammette, nel semipiano Im(z ) > 0, i tre zeri 3 3 1 1 i/6 i/2 i5/6 z1 = e = = i, e = + i , z2 = e +i . 2 2 2 2 Per ogni k = 1, 2, 3, si ha il residuo 1 zk zk rk = 5 = 6 = , 6zk 6zk 6 quindi lintegrale da calcolare vale i i 2 2i (r1 + r2 + r3 ) = (z1 + z2 + z3 ) = 2i = . 3 3 3 Esercizio 6. La funzione integranda ammette, nel semipiano Im(z ) > 0, i due zeri 3 3 1 1 i/3 i2/3 z1 = e = +i = +i , z2 = e , 2 2 2 2 con residui, rispettivamente, 1 1 i i r 1 = 1 + , r2 = 1 ; 4 4 3 3 ne segue il risultato proposto nel testo. Esercizio 7. In questo caso non ` e conveniente considerare la funzione sin z f (z ) = , z (1 + z 2 ) in quanto tale funzione ` e illimitata (in valore assoluto), tanto nel semipiano Im(z ) > 0 quanto nel semipiano Im(z ) < 0. Consideriamo piuttosto la funzione f (z ) = eiz , z (1 + z 2 )

la cui restrizione allasse reale vale cos x + i sin x . x(1 + x2 ) Integriamo tale funzione sul circuito Cr,R mostrato in gura.

Tenendo conto del fatto che a funzione x cos x/(x(1 + x2 )) ` e dispari, dopo aver fatto il limite per R e r 0, e applicato i lemmi di 4.8-3 e 4.8-4, si trova

14

Capitolo 4. Funzioni di variabile complessa

c 978-88-08-12546-0

sin x dx i res(f ; 0) = 2i res(f ; i), x(1 + x2 )

cio` e

sin x dx = , 2 x(1 + x ) e

da cui il risultato proposto nel testo. Esercizio 8. Analogamente a quanto abbiamo fatto nellesercizio precedente, conviene considerare la funzione eiz f (z ) = . 1 + z2 Integrando sul circuito CR che si ottiene concatenando il segmento [R, R] con la semicirconferenza di centro lorigine e raggio R contenuta nel semipiano Im(z ) > 0, applicando il Lemma di Jordan si ottiene

cos x e1 dx = 2i res(f ; i) = 2i = . 2 1+x 2i e

4.8-4 Vericare luguaglianza


+ 0

dx 1 = , 1 + xn n sin(/n)

n2

mediante il teorema dei residui, utilizzando il circuito che congiunge lorigine con R > 1, successivamente con Re2i/n attraverso un arco di cerchio, e di nuovo con lorigine. Utilizzare il risultato ottenuto per calcolare lintegrale dx 3 = . 3 x +8 18 0
Soluzione. Sia In lintegrale che vogliamo calcolare e sia f (z ) := 1/(1 + z n ); il

e un polo semplice per f circuito specicato contiene soltanto il punto z0 = ei/n che ` con residuo 1 z0 1 n1 = nz n = n z0 . nz0 0 Il segmento che congiunge lorigine col punto Re2i/n si parametrizza nella forma t e2i/n t,
R

0 t R,

dunque lintegrale ad esso relativo vale dt = e2i/n In . n 1 + t 0 In denitiva, applicando il teorema dei residui e facendo tendere R a +, tenendo conto del lemma del grande cerchio si ottiene 2i i/n In (1 e2i/n ) = , e n da cui 2i ei/n /n 2i In = = = . n 1 e2i/n n ei/n ei/n sin(/n) e2i/n In particolare si ottiene dx = , 2 1 + x 2 0 calcolabile elementarmente utilizzando la funzione arcotangente. Lulteriore integrale proposto si riconduce al calcolo di I3 , osservando che I2 =
+

c 978-88-08-12546-0

Esercizi
0

15

dx 1 = x3 + 8 8

dx . (x/2)3 + 1

4.8-5 Vericare luguaglianza ex/3 2 dx = x 1 + e 3 utilizzando la poligonale che congiunge, nellordine, i punti R, R + 2i, R + 2i, R, R, con R > 0. Dimostrare che gli integrali estesi ai due segmenti paralleli allasse immaginario tendono a 0 per R . Soluzione. Allinterno del circuito suggerito la funzione f (z ) = ez/3 /(1+ ez ) ammette il polo semplice z0 = i , con residuo ei/3 = ei/3 . ei La retta y = 2 ammette la parametrizzazione t x + 2i, r R, quindi lintegrale di f relativo ad essa vale ei2/3 I , dove I ` e lintegrale che vogliamo calcolare. Per R si ottiene dunque I (1 ei2/3 ) = 2i ei/3 , da cui ei/3 2i 2 = i/3 = = sin(/3) 1 ei2/3 e ei/3 3. Resta da dimostrare che gli integrali estesi ai due segmenti paralleli allasse immaginario tendono a 0 per R . Per il segmento di estremi R e R + 2i si ha la parametrizzazione t z (t) = R + 2i t, 0 t 1, dunque tale integrale vale I = 2i ei(2/3)t dt. R i2t 0 1+e e Il valore assoluto del numeratore della funzione integranda vale 1, mentre il valore assoluto del denominatore ` e maggiore o uguale a |eR ei2t ||1| = eR 1. In denitva il valore assoluto dellintegrale in esame non supera 2eR/3 /(eR 1). 2i eR/3
1

Esercizi proposti
4.8-P1. Calcolare + x I= dx. 2 x +1 0 Soluzione. Consideriamo la funzione z f (z ) = 2 , zC z +1 e integriamola sul circuito Cr,R gi` a utilizzato per lesercizio 4.8-3, numero 7. Per la funzione 0, per z = 0, z= |z | ei/2 , per z = 0, dove arg(z ), scegliamo una determinazione dellargomento che collochi la semiretta di discontinuit` a della stessa funzione nel semipiano Im(z ) < 0, ad esempio (/2, 3/2]. Con tale scelta f ` e olomorfa in un aperto contenente il circuito Cr,R . Il teorema dei residui fornisce ei/4 1+i z f ( z ) dz = 2 i res( f ; i ) = 2 i lim = 2i = . z i 2 z 2 i 2 Cr,R Per r 0 lintegrale di f sulla semicirconferenza di raggio r tende a 0 (Lemma del piccolo cerchio), e lo stesso accade per lintegrale sulla semicirconferenza di raggio

16

Capitolo 4. Funzioni di variabile complessa

c 978-88-08-12546-0

R, per R (Lemma del grande cerchio). Per r 0 e R lintegrale di f sullintervallo [r, R] fornisce lintegrale I che vogliamo calcolare; per x < 0, posto t := |x| = x, si ha inne r r R t t f (x) dx = i dt = i dt iI. 2+1 2+1 t t R R r Dunque 1+i I (1 + i) = 2 = I= . 2

4.8-P2. Calcolare 3 x sin x2 I= dx. x4 + 1 0


Soluzione. Consideriamo la funzione

z 3 eiz f (z ) := 4 ; z +1 essa possiede quattro poli semplici, di cui uno nel primo quadrante: 1 z0 = ei/4 = (1 + i), 2 avendo ivi come residuo 1/4e. Valutiamo lintegrale di f sul circuito ottenuto concatenando: il segmento 1 che congiunge lorigine col punto z = R sullasse reale (R > 1); il quarto di cerchio 2 (contenuto nel primo quadrante) che congiunge il punto z = R col punto z = iR; il segmento 3 che congiunge il punto z = iR con lorigine. Applicando il teorema dei residui si trova f (z ) dz +
1 2

f (z ) dz +
3

f (z ) dz =

2i . 4e

()

Ammettiamo per un istante che lintegrale su 2 tenda a 0 per R ; lintegrale su 1 tende a


0

x3 cos x2 dx + i x4 + 1

x3 sin x2 dx. x4 + 1

Lintegrale su 3 (si tenga conto del verso di percorrenza e si ponga z = iy ) tende a


0

y 3 cos y 2 dy + i y4 + 1

y 3 sin y 2 dx. y4 + 1

Indicando ovunque la variabile dintegrazione con la lettera x, la (*), per R , fornisce 3 x sin x2 2i 2i dx = , 4+1 x 4e 0 quindi lintegrale da calcolare vale /(4e). Lintegrale su 2 si pu` o parametrizzare ponendo z = Reit , 0 t /2. Esso diventa
/2

iR
0

R3 ei3t eiR (cos 2t+i sin 2t) dt. R4 ei4t 1

Prendiamo il valore assoluto e sfruttiamo a denominatore la diseguaglianza triangolare nella forma |z1 z2 | |z1 | |z2 | ; si trova che lintegrale scritto non supera, in valore assoluto,

c 978-88-08-12546-0

Esercizi

17

R4 4 R 1

/2 0

eR

/2 sin 2t

dt < 2
0

eR

sin 2t

dt.

Abbiamo sfruttato il fatto che la funzione R R4 /(R4 1) ` e monotona decrescente per R > 1, e tende a 1 per R , dunque ` e certamente minore di 2 per R abbastanza grande (ad esempio per R > 2). Per q. o. ssato t [0, /2], la funzione integranda nellultimo integrale tende a 0 per R , e, al variare di R, le funzioni integrande sono maggiorate dalla funzione continua (dunque sommabile) g (t) = e sin 2t . Il teorema di Lebesgue della convergenza dominata assicura la liceit` a del passaggio al limite sotto il segno di integrale.

5.

La trasformata di Laplace Esercizi

Aggiornamento: febbraio 2003 http://www.ciram.unibo.it/~barozzi/MI2/PDF/MI2-Cap.5-Ese.pdf

5.1. Introduzione alla trasformata di Laplace 5.2. Proprieta ` della trasformata di Laplace
5.2-1. Consideriamo la funzione limitata f (t) = (sin t)/t ( esempio 2.2-5) e indichiamo con F (s) la sua L-trasformata. In base alla Prop. 5.1-2 si verichi che 1 F (s) = L[tf (t)](s) = L[sin t](s) = . 1 + s2 Se ne deduca, per s reale positivo, F (s) = c arctan s, e nalmente per s , in base alla Prop. 5.1-1:

F (s) =
0

est

sin t dt = arctan s. t 2

Soluzione. Da

1 , 1 + s2 segue F (s) = c arctan s per ogni s con Re(s) > 0. In particolare, la formula scritta vale per ogni s reale positivo. Daltra parte, per s , F (s) deve tendere a 0, quindi la costante c vale necessariamente /2. F (s) = 5.2-2. La funzione integralseno ` e stata denita nellesempio 2.2-5 come la primitiva della funzione f (t) = (sin t)/t, nulla nellorigine: sin t dt. t 0 Dedurre dalla Proposizione 5.2-3 che la L-trasformata dellintegralseno ` e Si(x) := F (s) 1 = arctan s . s s 2 Per poter applicare il teorema del valore nale, occorre sapere che la funzione integralseno tende ad un limite nito per x + (in realt` a tale limite ` e gi` a stato calcolato nellesempio 4.8-5, ma vogliamo ricalcolarlo in modo indipendente). Per ottenere questo risultato, si ssi a > 0, e si osservi che, per ogni b > a, si ha G(s) =
a b sin t sin t sin t dt = dt + dt; t t t 0 0 a si integri per parti il secondo integrale e si dimostri che esso tende ad un limite nito per b + (la funzione x (cos x)/x2 ` e sommabile sullintervallo [a, +) in quanto maggiorata in valore assoluto da 1/x2 ). b x

Si(b) =

Capitolo 5. La trasformata di Laplace

c 88-08-07923-6

Finalmente si applichi il teorema del valore nale alla funzione integralseno ottenendo lim Si(x) = lim s G(s) = lim arctan s = . x+ s0 s0 2 2 Soluzione. Lo schema della soluzione ` e contenuto nellenunciato. Eseguiamo lintegrazione per parti suggerita; si ha
b a

sin t cos a cos b dt = + t a b

b a

cos t dt; t2

laddendo (cos b)/b tende a 0 per b +, lultimo integrale tende allintegrale della funzione sommabile t (cos t)/t2 su [b, +). 5.2-3. Sia h (t) := [0,h] (t)/h, con h > 0, come nellesempio 5.1-3. Abbiamo osservato che lintegrale di h vale 1 quale che sia h. Sia fh (t) la primitiva di h (t) nulla per t 0. Calcolare fh e la relativa trasformata. A che cosa tende fh (t) per h 0 ?

1 h

1
Figura 5.2-8. Integrando un impulso unitario si ottiene una rampa unitaria.

Soluzione. Per la primitiva fh (t) si trova (vedi gura a destra)

0, per t 0 t/h, per 0 < t h 1, per 1 t . Per h 0, fh (t) tende alla funzione nulla per t 0, uguale a 1 per t > 0, dunque la funzione limite ` e il gradino unitario (o funzione di Heaviside) a meno del valore nellorigine. Per la trasformata di fh (t) abbiamo, con unintegrazione per parti, fh (t) = ehs ehs 1 ehs + + = s hs2 s 0 h 1 ehs = hs2 per ogni s con parte reale positiva. Poich e ehs = 1 hs + h2 s2 /2 + . . . , per h 0 la trasformata calcolata tende a 1/s.
h

(t/h) est dt +

est dt =

5.2-4. Sia f (t) = [0,1] (t), cio` e la funzione uguale a 1 nellintervallo [0, 1] e nulla fuori dello stesso intervallo. Vericare che t, per 0 t 1, (f f )(t) = 2 t per 1 t 2, 0, altrimenti. Tracciare un graco della convoluzione ottenuta. Soluzione. La funzione f ( ) ha come supporto lintervallo [0, 1], mentre la funzione f (t ) ha come supporto lintervallo [t 1, t]. Dunque la convoluzione ` e nulla quando t ` e esterno allintervallo [0, 2], in quanto, per tali valori, i supporti delle due funzioni in questione sono tra loro disgiunti. Per t [0, 1] essa vale
t

d = t,
0

c 88-08-07923-6

Esercizi

mentre per t [1, 2] essa vale


2

d = 2 t.
t

-1

5.2-5. Sia h (t) := [0,h] (t)/h, con h > 0, come nellesercizio 5.2-3. Si verichi che (f h )(t) ` e uguale alla media integrale della funzione f sullintervallo [t h, t]. Se f ` e una funzione continua su R, a che cosa tende (f h )(t) per h 0 ? Soluzione. Infatti, poich e il supporto della funzione h (t ) ` e lintervallo [t h, t], si trova (f h )(t) = 1 h
t

f ( ) d.
th

Se f ` e continua, la media integrale calcolata coincide col valore di f in un punto opportuno [t h, t], e dunque tende a f (t) per h 0. 5.2-6. Con i simboli dellesercizio precedente, calcolare esplicitamente (H h )(t) e tracciarne il graco. Si suggerisce di fare calcoli separati per 0 t h e per t h. Vericare che la convoluzione ottenuta ` e continua su R. Confrontare con quanto visto nellesercizio 5.2-3. Soluzione. Utilizzando il risultato del precedente esercizio, si riconosce subito che la convoluzione richiesta ` e nulla per t < 0, e vale 1 per t > h. Per t [0, h], si ha 1 t d = . h h 0 In denitiva si ottiene la rampa fh (t) considerata nellesercizio 5.2-3. (H h )(t) = 1 5.2-7. Stesso problema dellesercizio precedente per (t+ h )(t). Vericare che la convoluzione ottenuta ` e di classe C (1) (R). Soluzione. Sempre in base al risultato dellesercizio 5.2-5, abbiamo cha la convoluzione richiesta ` e nulla per t 0. Per 0 < t h essa vale 1 t t2 d = , h 0 2h mentre per t h essa vale 1 h
t t

d =
th

1 2 h 2

t th

=t

h . 2

0, per t 0 t2 per 0 < t h (t+ h )(t) = 2h , h t , per t > h. 2 Come si vede, non solo la convoluzione calcolata ` e continua, ma anche la sua derivata prima ` e continua: 0, per t < 0 t (t+ h ) (t) = , per 0 < t < h h 1, per t > h.

In denitiva

Capitolo 5. La trasformata di Laplace

c 88-08-07923-6

5.3. Le funzioni beta e gamma di Eulero


5.3-P1. Dedurre dalla relazione (x + 1) = x(x), la relazione (x + n + 1) = x(x + 1)(x + 2) . . . (x + n) (x) per ogni naturale n.
Soluzione. Basta procedere per induzione rispetto a n.

5.4. Inversione della trasformata di Laplace


5.4-P1. Mediante il teorema di convoluzione, determinare lantitrasformata di Laplace per ciascuna delle funzioni F0 (s) = (s2 1 , + 1)2 F1 (s) = (s2 s , + 1)2 F2 (s) = (s2 s2 . + 1)2

Soluzione. Poich e 1/(s2 + 1) e s/(s2 + 1) sono la trasformate di (sin t)+ e (cos t)+

rispettivamente, le antitrasformate richieste sono, nellordine, f0 (t) = (sin t)+ (sin t)+ , f2 (t) = (cos t)+ (cos t)+ . Per t 0 si trova
t t t

f1 (t) = (sin t)+ (cos t)+ ,

f0 (t) =
0

sin sin(t ) d = sin t


0

sin cos d cos t


0

sin2 d =

sin2 t t sin t cos t = sin t cos t = 2 2 1 = (sin t t cos t). 2 In modo analogo si trova 1 1 f1 (t) = t sin t, f2 (t) = (t cos t + sin t). 2 2

5.5. Equazioni differenziali ordinarie


5.5-P1 Utilizzare la trasformata di Laplace per risolvere il problema di valori iniziali y + 2y + y = 1 con le condizioni y (0) = 1, y (0) = 1.
Soluzione. Si trova

Y (s) =

1 s s2 . s(s + 1)2

Poich e abbiamo un polo semplice per s1 = 0, e un polo doppio in s2 = 1, avremo una decomposizione in fratti semplice del tipo (si veda la Prop. 4.7-1) Y (s) = 1 s s2 B C A + = + . s(s + 1)2 s s + 1 (s + 1)2

La costante A ` e semplicemente il residuo di Y (s) nel polo s1 = 0, quindi si calcola immediatamente A = lim sY (s) = 1.
s0

Luguaglianza 1 s s2 1 B C = + + 2 s(s + 1) s s + 1 (s + 1)2

c 88-08-07923-6

Esercizi

si scrive anche 3 2s B C = , + (s + 1)2 s + 1 (s + 1)2 dopodich e si possono calcolare i valori B = 2 e C = 1 col metodo dei coecienti indeterminati o, pi` u semplicemente, osservando che 3 2s = 2(s + 1) 1. In denitiva 1 2 1 Y (s) = s s + 1 (s + 1)2 da cui y (t) = 1 (2 + t)et per t 0. 5.5-P2 Determinare le risposte impulsive per ciascuna delle equazioni dierenziali lineari a coecienti costanti: 1. y + y = 0; 2. y + y + y = 0; 3. y y + y = 0; 4. 5. y y = 0; y + 2y + y = 0. 1 1 ; P3 (s) = 2 ; s2 + s + 1 s s+1 1 1 . P5 (s) = 2 = s + 2s1 (s + 1)2 P2 (s) =

Soluzione. Si tratta di anti-trasformare le funzioni

P1 (s) =

1 ; s2 + 1 1 P4 (s) = 2 ; s 1

Si trovano, nellordine, le soluzioni 3 2 y2 (t) = et/2 sin t 2 3 3 2 t/2 3. y3 (t) = e sin t 2 3 4. y4 (t) = (sinh t)+ ; 5. y4 (t) = (tet )+ . 2. 1. y1 (t) = (sin t)+ ;

Si osservi il legame tra il comportamento delle risposte impulsive calcolate e la collocazione, nel piano complesso, degli zeri dei polinomi caratteristici.

6.

La trasformata di Fourier Esercizi

http://www.ciram.unibo.it/~barozzi/MI2/PDF/MI2-Cap.6-Ese.pdf

6.1. Introduzione alla trasformata di Fourier


6.2-1. Sia f una funzione sommabile e f ( ) := R eix f (x) dx la sua F-trasformata. Utilizzare il lemma di Riemann-Legesgue ( Prop. 3.2-1) per dimostrare che
| |

lim f ( ) = 0.

Se (n ) ` e una successione convergente a 0 , vericare che limn f (n ) = f (0 ) (continuit` a di f ( )) sfruttando il teorema della convergenza dominata di Lebesgue ( Prop. 2.3-1).
Suggerimento
r

Per dimostrare che lim|| f ( ) = 0, si osservi che, scelto > 0, si pu` o trovare in corrispondenza un r > 0 tale che |f (x)| dx +
r

|f (x)| dx < /2.

Per concludere che f ( ) ` e innitesima allinnito, applicare il lemma di Riemannr Legesgue ( Prop. 3.2-1) allintegrale r eix f (x) dx. Soluzione. Riprendendo il ragionamento indicato nel suggerimento, si ha eix f (x) dx =
R r r r

eix f (x) dx +

r r

eix f (x) dx +
r

eix f (x) dx,

da cui, prendendo i valori assoluti,


R

eix f (x) dx /2 +

eix f (x) dx .

Ma il lemma di Riemann-Legesgue ci assicura che lultimo integrale scritto si pu` o rendere in valore assoluto minore di /2 a patto che | | sia abbastanza grande. Per vericare che
n

lim

ein x f (x) dx =
R R

ei0 x f (x) dx

basta osservare che le funzioni fn (x) = ein x f (x) sono tutte maggiorate in valore assoluto dalla funzione sommabile |f (x)|, dunque ` e applicabile il teorema di Lebesgue. 6.2-2. Con una tecnica simile a quella mostrata nellesempio 6.1-3, calcolare le seguenti trasformate: 1 2 F 2 ( ) = ei /2 e( 3/2)|| , x x+1 3 F x4
1 ( ) = e||/ 2 cos(/ 2) + sin(| |/ 2) . +1 2

Capitolo 6. La trasformata di Fourier

c 88-08-07923-6

Ritrovare il primo risultato combinando la trasformata della funzione x 1/(x2 + a2 ) ( Tabella 6.2-1) con la Prop. 6.2-2. Dedurre dallultimo risultato le trasformate delle funzioni x x2 , . x4 + 1 x4 + 1 Soluzione. La funzione eiz f (z ) = 2 z +z+1 presenta due poli semplici nei punti z1,2 = 1/2 i 3/2, con residui r1 , 2 = dunque 1 F 2 ( ) = x +x+1 2i r1 = 2i r2 =
2 3 2 3

e(

i 3

3/2+i/2)

ei /2 e( e e

3/2)

i /2 ( 3/2)

per < 0, , per > 0.

Alternativamente, poich e sappiamo che la trasformata di 1/(a2 + x2 ) ` e a|| e , a>0 a utilizzando la Proposizione 6.2-2 troviamo che la trasformata di 1 1 = 2 x2 + x + 1 (x + 1/2) + ( 3/2)2 ` e ancora 2 ei /2 e( 3/2)|| . 3 Per quanto riguarda la funzione x 1/(x4 + 1), si pu` o procedere in modo analogo a quello che abbiamo seguito per lesercizio precedente, osservando che la funzione eiz z4 + 1 presenta nel semipiano Im(z ) > 0 i due poli semplici 1+i 1 + i z1 = ei/4 = , z2 = ei3/4 = 2 2 e nel semipiano Im(z ) < 0 i due poli semplici 1 i 1i z3 = ei5/4 = , z4 = ei7/4 = . 2 2 In ciascun polo zk il residuo vale f (z ) = rk = eizk zk = eizk , 3 4zk 4

4 in quanto zk = 1. Dunque per > 0 si ha 1 i F 4 ( ) = 2i(r3 + r4 ) = [z3 eiz3 + z4 eiz4 ] = x +1 2 i /2 = e (1 + i) cos(/ 2) + i sin(/ 2) + 2 2 = + (1 i) cos(/ 2) i sin(/ 2) = = e/ 2 cos(/ 2) + sin(/ 2) . 2

c 88-08-07923-6

Esercizi

Scrivendo | | al posto di e tenendo conto della parit` a del coseno, si ottiene il risultato. Per calcolare poi la trasformata delle funzioni x/(x4 + 1) e x2 /(x4 + 1) si pu` o applicare la Proposizione 6.2-5. Si trova, ad esempio, per la trasformata della prima funzione (che ` e reale e dispari): ie||/ 2 sin(/ 2). 6.2-3. Generalizzare la prima funzione del precedente esercizio, mostrando che, per ogni reale e per ogni > 0, si ha F 1 ( ) = ei e || . (x )2 + 2 eiz (z )2 + 2 e( i) . 2i

Soluzione. Basta osservare che la funzione

f (z ) =

presenta poli semplici nei punti z1,2 = i , con residui r1 , 2 =

6.2-4. Vericare che la funzione eax , per x 0, f (x) = 0, altrimenti, con a > 0, ha come F -trasformata 1/(a + i ). Dedurne le trasformate delle funzioni x xf (x), x x2 f (x), in generale x xn f (x), per ogni naturale n. Soluzione. Si ha e(a+i)x 1 f ( ) = ei xeax d = = . ( a + i ) ( a + i ) 0 0 Per calcolare le trasformate di xf (x), x2 f (x), . . . , si pu` o applicare la proposizione 6.2-5, ottenendo 1 F [xf (x)] ( ) = if ( ) = , (a + i )2 F x2 f (x) ( ) = if ( ) = e in generale F x2 f (x) ( ) = n! . (a + i )n+1 2 , (a + i )3

Pi` u semplicemente si pu` o osservare che la trasformata di Laplace del segnale xn + essendo n!/sn+1 , per Re(s) > 0, la restrizione di tale trasformata alla retta a + i , R, ci fornisce la F -trasformata del segnale f (x) = (xn eax )+ , secondo quanto abbiamo osservato a pag. 244 del testo. 6.2-5. Si consideri limpulso unitario 1 , se |x| h/2 1 ph (x) := [h/2,h/2] (x) = h h 0, altrimenti. Vericare, mediante un calcolo diretto, che 1 1 |x| , se |x| h + 1 |x| h (ph ph )(x) = th (x) := = h 1 h h 0, altrimenti.

Capitolo 6. La trasformata di Fourier

c 88-08-07923-6

Ritrovare lo stesso risultato utilizzando la trasformata di Fourier, sfruttando le trasformate calcolate negli esempi 6.1-1 e 6.2-5. Soluzione. La funzione ph ( ) ha come supporto lintervallo [h/2, h/2], la funzione ph (x ) ha come supporto lintervallo [x h/2, x + h/2]; lintersezione tra tali supporti ` e dunque , se x < h, [h/2, x + h/2], se h x 0, [h/2, h/2] [x h/2, x + h/2] = se 0 x h, [x h/2, h/2], , se h < x.

h h/2

h/2 x

h h/2

h/2

h x

h h/2

h/2

h h/2

h/2

Figura 6.E-1. Mutue posizioni dei supporti delle funzioni ph ( ) e ph (x ).

Di conseguenza, poich e il prodotto ph ( ) ph (x ) vale 1/h2 nei punti dellintersezione appena calcolata, mentre vale 0 nei restanti punti, si ha 0, se x < h, h+x , se h x 0, 2 (ph ph )(x) = h h x , se 0 x h, h2 0, se h < x. Questo ` e precisamente il risultato annunciato; salvo le notazioni. Si osservi che la convoluzione appena calcolata si scrive anche, utilizzando i simboli dellesempio 6.25, gh (x)/h2 .

-1

-1

-1

Figura 6.E-2. Da sinistra a destra: graci degli impulsi x ph (x), x gh (x) e x gh (x)/h2 = (ph ph )(x), con h = 0.7.

Daltra parte ph ha come F -trasformata (v. esempio 6.1-1)

c 88-08-07923-6

Esercizi

2 sin(h/2) h dunque ph ph ha come F -trasformata 4 h2 sin(h/2)


2

che sappiamo essere la F -trasformata di gh (x)/h2 . 6.2-6. Con i simboli del precedente esercizio, vericare che per x < h/2 0, (ph H )(x) = x/h + 1/2, per h/2 x h/2 1, per x h/2; 0, (x + h)2 /(2h2 ), (th H )(x) = (x2 + 2hx + h2 )/(2h2 ), 1, 0, (3h + 2x)2 /(8h2 ), (th ph )(x) = 3/4 x2 , (3h 2x)2 /(8h2 ), per per per per x < h h x 0 0xh x h.

per |x| > 3h/2 per 3h/2 x h/2 per h/2 x h/2 per h/2 x 3h/2.

H (x) ` e la funzione di Heaviside. Si osservi che th ph = th th th . Vericare che (th H )(x) e (th ph )(x) sono funzioni di classe C (1) (R). Soluzione. Si osservi che, per ogni funzioni sommabile f , si ha
x

(f H )(x) = Pertanto

f ( )H (x ) d = 0, ph ( ) d =

f ( ) d.

per x < h/2 h 1 x 1 = + , per h/2 x h/2 2 h h 2 per x h/2. per x < h per h x 0 per 0 x h per x h.

(ph H )(x) = Analogamente

x+ 1,

0, (x + h)2 /(2h2 ), x (th H )(x) = th ( ) d = (x2 + 2hx + h2 )/(2h2 ), 1,


x

Si tenga conto che, per h x 0, si ha th ( ) d = 1 h2


x

( + h) d,
h

mentre per 0 x h si ha
x

th ( ) d =

1 1 + 2 h2

(h ) d.
0

Se le funzioni e ph e th vengono considerate come densit` a di due varibili aleatorie continue, le funzioni ph H e th H sono le rispettive funzioni di ripartizione (o funzioni di distribuzione; in inglese: cumulative distribution functions).

Capitolo 6. La trasformata di Fourier

c 88-08-07923-6

0.5

0.5

-2

-1

-2

-1

Figura 6.E-3. Graco della funzione (ph H )(x) (a sinistra), della funzione (th H )(x) (a destra); in gura h = 0.8. I graci non sono monometrici.

Il calcolo per la convoluzione th ph ` e analogo, tenendo conto del fatto che th ( ) ha come supporto lintervallo [h, h], mentre ph ( ) ha come supporto lintervallo [h/2, h/2]. 6.2-7. Siano f e g due funzioni con i supporti contenuti, rispettivamente, negli intervalli [a, b] e [c, d], vale a dire f ` e nulla fuori di [a, b], g ` e nulla fuori di [c, d]; vericare che f g ha il supporto contenuto nellintervallo [a + c, b + d]: (supp f [a, b]) (supp g [c, d])
Suggerimento

supp (f h) [a + c, b + d].

Per ogni ssato x, il supporto della funzione g (x ) ` e contenuto nellintervallo [x d, x c]. Soluzione. La funzione integranda della convoluzione, f ( )g (x ), ` e identicamente nulla per quei valori di x per i quali gli intervalli [a, b] e [x d, x c] sono disgiunti, dunque per x c < a x < a + x, x d > b x > b + d.

6.2-8. Siano f e g due impulsi unitari, nel senso che si tratta di due funzioni aventi come supporto un intervallo limitato e integrale uguale a 1: R f (x) dx = R g (x) dx = = 1. Vericare che f g ` e ancora un impulso unitario. Suggerimento Utilizzare la trasformata di Fourier per vericare che lintegrale di f g vale 1. Soluzione. In base al precedente esercizio f g ` e ancora una funzione a supporto compatto. Posto h = f g , si ha poi R h(x) dx = h(0) = f (0) g (0) = 1.
Soluzione.

6.2-9. Vericare che la convoluzione tra due funzioni reali pari ` e ancora reale pari. Con gli stessi simboli del precedente esercizio, si ha che h( ) ` e una funzione reale pari in quanto prodotto delle funzioni reali pari f ( ) e g ( ). Ne segue che anche h ` e reale pari in virt` u della formula di dualit` a (v. formula (4 ) a pag. 244): h ( ) = 2h( ).

Peraltro ` e immediato che la convoluzione tra due funzioni reali sia reale, mentre non ` e altrettanto immediato che la convoluzione tra due funzioni pari sia ancora pari. Occorre un calcolo: se f e g sono pari, allora da (f g )(x) = R f ( )g (x ) d segue (f g )(x) = =
R R

f ( ) g (x ) d = f ( ) g (x + ) d =

(f e g sono pari) (t = ) f (t) g (x t) dt = (f g )(x).

f (t) g (x t) dt =

c 88-08-07923-6

Esercizi

0.5

0.5

-2

-1

-2

-1

Figura 6.E-4. A sinistra il graco della funzione (p1 t1 )(x) ( esercizio 6.2-6), a destra il graco della funzione (t1 t1 )(x) ( esercizio 6.2-10). La prima funzione ` e di classe C (1) (R), (2) la seconda ` e di classe C (R). I graci non sono monometrici.

6.2-10. Coi simboli dei precedenti esercizi, vericare che (t1 t1 )(x) ` e la funzione pari che, per x 0, vale 3 2 per 0 x 1 x /2 x + 2/3, 3 2 (t1 t1 )(x) = x /6 + x 2x + 4/3, per 1 x 2 0, per 2 x. Si osservi che t1 t1 = p1 p1 p1 p1 . Vericare che (t1 t1 )(x) ` e una funzione di classe C (2) (R); si tratta di una cosiddetta funzione spline cubica. Soluzione. In base al precedente esercizio ci basta calcolare (t1 t1 )(x) per x 0. La gura seguente mostra le mutue posizioni della funzioni t1 ( ) e t1 (x ) a seconda che sia 0 x 1, 1 x 2 ed inne 2 x.

-1

x 1

-1

-1

2x

Figura 6.E-5. Mutue posizioni delle funzioni t1 ( ) e t1 (x ).

Se 0 x 1, allora
0 x

(t1 t1 )(x) = =

x1 1 x

(1 + )(1 x + ) d +
0

(1 )(1 x + ) d +

(1 )(1 + x ) d,

Capitolo 6. La trasformata di Fourier

c 88-08-07923-6

mentre se 1 x 2, allora
1

(t1 t1 )(x) =

x1

(1 )(1 x + ) d,

da cui segue il risultto con un calcolo elementare. Basta poi derivare due volte le espressioni ottenute negli intervalli 0 x 1, 1 x 2 e 2 x, per vericare che tanto le derivate prime quanto le derivate seconde si raccordano con continuit` a nei punti di ascissa intera. 6.2-11. Vericare, mediante un calcolo diretto, che la convoluzione della funzione
2 1 ex /2 2

(densit` a di una variabile aleatoria normale standard) con se stessa ` e ( esempio 6.2-7) 2
2 1 ex /4 .

Suggerimento

La convoluzione in questione si scrive


2

1 2

et
R

/2 (xt)2 /2

dt =

ex /2 2
2

e(t
R

xt)

dt;

utilizzare il completamento del quadrato, cio` e lidentit` a t2 xt = (t x/2)2 x2 /4. Soluzione. Proseguendo in base al suggerimento si trova 1 2 et
R
2

/2 (xt)2 /2

dt =

ex

e 2

/2 x2 /4 R

e d =
2

2 1 ex /4 dt, 2

avendo operato il cambiamento di variabile = t x/2.

Esercizi proposti
6.2-P1. Calcolare la trasformata di Fourier della funzione reale e pari x f (x) che vale cos x per |x| /2 e vale 0 nei restanti punti dellasse reale. Soluzione. Trattandosi di un segnale reale pari, la F -trasformata si riduce ad una coseno-trasformata
/2 /2

f ( ) =
/2

cos(x) cos x dx = 2
0

cos(x) cos x dx.

Lultimo integrale pu` o essere calcolato con due integrazioni per parti, oppure trasformando lespressione cos(x) cos x mediante le cosiddette formule di Werner. Seguendo la prima strada si ha
/2

I=
0

cos(x) cos x dx = sin x cos(x) cos x sin(x)


/2 0

/2 0

/2

+
0 /2

sin x sin(x) dx =

= cos(/2) +

+
0

cos x cos(x) dx =

= cos(/2) + 2 I. Si trova dunque 2 cos(/2) , 1 2 che ` e ancora una funzione reale pari. Si osservi che la trasformata ottenuta ha singolarit` a eliminabili nei punti = 1, in quanto tali valori sono zeri semplici tanto del numeratore quanto del denominatore. f ( ) =

c 88-08-07923-6

Esercizi

Alternativamente si pu` o osservare che f (x) = [/2,/2] cos x, e quindi, essendo nota la F -trasformata di [/2,/2] , si pu` o applicare la formula fornita nellesempio 6.2-3. Si trova dunque lespressione 1 2 sin[(/2)( 1)] 2 sin[(/2)( + 1)] f ( ) = + . 2 1 +1 Applicando le formule di addizione alle funzioni a numeratore si ottiene poi 1 1 2 cos(/2) f ( ) = cos(/2) + = . 1 +1 1 2 6.2-P2. Calcolare la trasformata di Fourier della funzione reale e dispari x f (x) che vale sin x per |x| e vale 0 nei restanti punti dellasse reale. Soluzione. Lesercizio pu` o essere risolto con la stessa tecnica usata nel precedente. Si trova la trasformata (puramente immaginaria e dispari) 2 i sin( ) f ( ) = . 2 1 6.2-P3. Mediante un calcolo diretto calcolare la F -trasformata della funzione f (x) = = [0,a] (x) (impulso di ampiezza unitaria e durata a > 0). Ottenere poi la trasformata delle funzioni x [0,a] (x) e x [0,a] (x) [a,0] (x) in base al fatto che queste sono rispettivamente la parte pari e la parte dispari della funzione x 2[0,a] (x). Soluzione. Per la funzione data si trova la trasformata a eix 1 eia sin(a ) cos(a ) 1 f ( ) = eix dx = = = +i . i i 0 Per la funzione x [0,a] (x) si trova dunque la trasformata 2 sin(a ) , mentre per la funzione x [0,a] (x) [a,0] (x) si trova la trasformata 2 Re[f ( )] = cos(a ) 1 4 sin2 (a/2) =i . Abbiamo sfruttato lidentit` a 1 cos t = 2 sin2 (t/2). Si confronti con gli esempi 6.1-1 e 6.2-2. 2i Im[f ( )] = 2i 6.2-P4. Sia f una funzione sommabile e f ( ) = X ( ) + iY ( ) la sua F -trasformata, con X e Y reali. Dmostrare che se X ` e pari e Y ` e dispari, allora |f ( )| ` e pari e Arg[f ( )] ` e sostanzialmente dispari, nel senso che, tranne gli eventuali valori di per cui f ( ) ` e reale negativa (X ( ) < 0) (Y ( ) = 0) , si ha Arg[f ( )] = Arg[f ( )].
Soluzione. Infatti

|f ( )|2 = X ( )2 + Y ( )2 = X ( )2 + Y ( )

= X ( )2 + Y ( )2 .

Si osservi poi che, nelle ipotesi ammesse, X ( ) + iY ( ) ` e il coniugato di X ( ) + iY ( ), e dunque ha come argomento principale lopposto dellargomento principale di questultimo, a meno che esso non sia reale negativo: X ( ) + iY ( ) = X ( ) iY ( ) = X ( ) < 0, quindi Arg[f ( )] = Arg[f ( )] = . 6.2-P5. Utilizzare la tecnica mostrata nellesercizo 6.2-8 per dimstrare che la convoluzione tra due densit` a di probabilit` a (funzioni non negative con integrale unitario su R) ` e ancora una densit` a di probabilit` a. Soluzione. A quanto mostrato nella soluzione dellesercizio 6.2-8 basta aggiungere losservazione che la convoluzione tra due funzioni non negative ` e ancora una funzione non negativa.

7.

Distribuzioni Esercizi

[Revisione: ottobre 2004]

http://www.ciram.unibo.it/~barozzi/MI2/PDF/MI2-Cap.7-Ese.pdf

7.1. Il concetto di distribuzione


7.1-1. Consideriamo la funzione f (x) := (e1/x )+ = e1/x , 0, per x > 0 , altrimenti.

Si dimostri che essa ` e di classe C () (R), vericando innanzitutto che limx0+ f (x) = 0, e successivamente che una relazione analoga vale per ciascuna delle derivate. Suggerimento Si dimostri, procedendo per induzione, che per x > 0 ciascuna delle derivate della funzione f si pu` o scrivere come prodotto pn (1/x) f (x), dove pn ` e un polinomio opportuno nella variabile 1/x. Soluzione. Osserviamo innanzitutto che se p ` e un qualsivoglia polinomio si ha
x0+

lim p(1/x)f (x) = lim

t+

p(t) = 0, et

(t := 1/x)

come si riconosce applicando la regola di LH opital tante volte quant` e il grado di p. Ci` o posto, chiaramente la proposizione sussiste per n = 0 ponendo p0 = 1. Se per un assegnato n si ha Dn f (x) = pn (1/x) f (x), con pn polinomio opportuno, allora Dn+1 f (x) = pn (1/x) avendo posto 1 1 pn (1/x) 2 . x2 x Abbiamo utilizzato la relazione f (x) = (1/x2 ) f (x). pn+1 (1/x) := pn (1/x) 7.1-2. In base al risultato del precedente esercizio, vericare che la funzione (x) := e1/(x 0,
2

1 1 f (x) pn (1/x) 2 f (x) = pn+1 (1/x) f (x) x2 x

1)

per |x| < 1, altrimenti,


R

gi` a ripetutamente considerata, appartiene allo spazio D(R). Si scelga c := consideri la funzione x 1 (t) dt. v (x) := c

(x) dx e si

Si verichi che v ` e una funzione crescente, di classe C () (R), nulla per x 1, uguale a 1 per x 1. Si esamini la funzione v (x) := v (x/), con > 0. Soluzione. La funzione ` e di classe C () (R) in quanto composta mediante funzioni della stessa classe. Quanto alla v , essa ` e crescente in quanto primitiva della funzione non negativa . La funzione v (x) := v (x/) ` e qualitativamente simile alla v , ma ` e strettamente crescente

Capitolo 7. Distribuzioni

c 88-08-07923-6

0.5

Figura 7.E-1. La funzione v (x) ` e crescente, nulla per x 1, uguale a 1 per x 1.

-2

-1

nellintervallo [, ], mentre vale 0 per x e vale 1 per x . Il suo graco si ottiene da quella di v comprimendo questultimo di un fattore 1/ nel senso dellasse delle ascisse. 7.1-3. Dato lintervallo [a, b], con a < b, si verichi che la funzione (i simboli sono quelli del precedente esercizio) v (x) := v (x a) v (b x) ha come supporto lintervallo [a , b + ] e vale 1 nellintervallo [a + , b ] (si suppone che sia 2 < b a). Soluzione. Basta osservare che il graco di v (x a) si ottiene da quello di v (x) traslandolo della quantit` a a, mentre il graco di v (b x) si ottiene da quello di v (x) traslandolo della quantit` a b e poi simmetrizzandolo rispetto alla parallela allasse delle ordinate passante per il punto di ascissa b.

1 1

1 1

Figura 7.E-2. Dallaltro al basso: graci delle funzioni v (x a), v (b x) e del loro prodotto v (x a) v (b x), per a = 0.5, b = 3, = 0.3.

7.1-4. Si consideri la funzione f (x) := (1 |x 1|)+ e se ne tracci il graco. In base a quanto visto nellesempio 7.1-7, si verichi che la successione fk (x) := kf (kx) tende a (x) per k , mentre la stessa successione tende puntualmente a 0 per ogni x reale. Si riveda lesempio 2.1-3 con hn = n.

c 88-08-07923-6

Esercizi

Figura 7.E-3. Graco della funzione f (x) := (1 |x 1|)+ .

Soluzione. Per ogni x 0, si ha fk (x) = 0 per ogni k, mentre, per ogni ssato x > 0, si ha fk (x) = 0 per ogni k 2/x, in quanto la funzione fk ha come supporto lintervallo [0, 2/k]. Per quanto riguarda la convergenza alla distribuzione , si tratta semplicemente di un caso particolare di quanto dimostrato nellesempio 7.1-7, scrivendo k al posto di .

Esercizi proposti
7.1-P1. Se u(x) ` e la funzione gradino unitario, u(x)[0,+) (x), vericare che la successione fn (x) := u(x n) tende a 0 in L1 loc (R). Soluzione. Scelto ad arbitrio lintervallo [a, b], risulta fn (x) = 0 su [a, b] per tutti gli n > b. 7.1-P2. Sia fn (x) := [0,n] (x); vericare che fn tende a u(x) in L1 loc (R). Soluzione. Scelto ad arbitrio lintervallo [a, b], risulta fn (x) = u(x) su [a, b] per tutti gli n > b. 7.1-P3. Posto fn (x) := 0, nx, 1, per x 0 per 0 x 1/n, per x 1/n,

vericare che fn tende a u(x) in L1 loc (R). Soluzione. Le funzioni fn e u coincidono fuori dellintervallo [0, 1/n]. Su un qualunque intervallo [a, b] che contanga, in tutto o in parte, lintervallo [0, 1/n] si ha
b 1/n 1/n

|fn (x) u(x)| dx


a 0

|fn (x) u(x)| dx =


0

(1 nx) dx =

1 . 2n

7.1-P4. Dalla relazione eikx 0 (in D (R)) per k , stabilita nellesempio 7.1-5, dedurre che sin2 (kx) 1/2 utilizzando le formule di Eulero. Soluzione. Si ha sin2 (kx) = eikx eikx 2i
2

ei2kx + ei2kx 1 1 + . 4 2 2

7.1-P5. vericare che la successione k sin(kx) (che tende a 0 nel senso delle distribuzioni) non tende a 0 in L1 loc (R). Soluzione. Si ha, per esempio
k=n1 (k+1)/n

| sin(kx)| dx =
0 k=0 k/n

| sin(kx)| dx = n

2 = 2. n

Capitolo 7. Distribuzioni

c 88-08-07923-6

Figura 7.E-4. Graco della funzione x | sin(5x)| sullintervallo [0, ].

7.2. Operazioni sulle distribuzioni


7.2-1. Si consideri la distribuzione f (x) := (x k); per ogni funzione test v si k= ha f (x), v (x) = k= v (k), dove la somma indicata contiene al pi` u un numero nito di termini diversi da 0, quelli corrispondenti agli interi k che appartengono al supporto di v . Si verichi che f ` e periodica di periodo 1: f (x) = f (x + 1). Soluzione. Si ha f (x +1), v (x) = f (x), v (x 1) = k= v (k 1); ma lultima somma, scrivendo h al posto di k 1 e poi ancora k al posto di h, si scrive ancora v (k). In k= conclusione: tanto f (x), v (x) quanto f (x + 1), v (x) forniscono la somma dei valori che la funzione test v assume nei punti di ascissa intera appartenenti al proprio supporto. 7.2-2. Vericare che la distribuzione v.p.(1/x) ( esempio 7.1-8) ` e dispari. Soluzione. Si ha 1 1 v.p. , v (x) = v.p. , v (x) = x x
r r

v (x) v (0) dx x

dove r ` e abbastanza grande perch e lintervallo [r, r] contenga il supporto di v . Ponendo t = x, e successivamente indicando ancora con x la variabile di integrazione lultima quantit` a si scrive
r r

v (t) v (0) dt = t

r r

v (x) v (0) 1 dx = v.p. , v (x) . x x

7.2-3. Vericare che se f ` e una distribuzione pari, essa si annulla in corrispondenza di ogni funzione test v dispari, e viceversa. Soluzione. Sia f pari, dunque f (x) = f (x). Allora per ogni funzione test v si ha f (x), v (x) = f (x), v (x) = f (x), v (x) ; ma se v ` e dispari, v (x) = v (x), dunque lultima quantit` a` e uguale a f (x), v (x) . In conclusione: f (x), v (x) = f (x), v (x) = f (x), v (x) = 0.

7.2-4. Vericare che se f ` e una distribuzione pari, f ` e dispari e viceversa. Soluzione. Utilizzeremo: (a) la composizione di una distribuzione f con la funzione x x (simmetria rispetto allorigine); (b) la derivata di una distribuzione. Si ha f (x), v (x) = f (x), v (x) = = f (x), v (x)
a b b a

= f (x), v (x) = (f ` e pari)

= f (x), v (x) = f (x), v (x) = = f (x), v (x) .

c 88-08-07923-6

Esercizi

Dunque f (x) = f (x). 7.2-5. Se f ` e una distribuzione e una funzione di classe C () , vericare la formula ( f ) = f + f . Soluzione. Per ogni funzione test v si ha: ( f ) , v = f, v analogamente f, v = f, v , f , v = f , v = f, ( v ) = = f, v f, v , da cui il risultato sommando membro a membro. 7.2-6. Consideriamo la famiglia di funzioni localmente sommabili x , a = 0. fa (x) := 2 a + x2
2

= f, v ;

a = 1/3 a = 2/3

a=1

-2

-1

Figura 7.E-5. La famiglia di funzioni fa (x) := x/(a2 + x2 ) tende alla distribuzione v.p. (1/x) per a 0.

-1

Dimostrare che 1 . x Suggerimento Si proceda, in modo analogo a quanto visto nellesempio 7.2.-2, osservando che, per ogni funzione test v con supporto contenuto nellintervallo [r, r], si pu` o scrivere
a0

lim fa (x) = v.p.

fa (x), v (x) =
r r

x v (x) dx = a2 + x2

r r

x v (x) v (0) dx = a2 + x2

=
r

v (x) v (0) x2 dx. a2 + x2 x

Nellultimo integrale scritto si passi al limite sotto il segno di integrale applicando il teorema della convergenza dominata di Lebesgue ( Prop 2.3-1). Soluzione. Proseguendo nel ragionamento suggerito, basta osservare che la famiglia di funzioni x v (x) v (0) x2 a2 + x2 x

e sommabile, in quanto continua, ammette la maggiorante x [v (x) v (0)]/x , che ` sullintervallo [r, r].

Capitolo 7. Distribuzioni

c 88-08-07923-6

2 1 -2 -1 -1 -2 1 2 -2 -1

2 1 1 -1 -2 2

Figura 7.E-6. A sinistra viene mostrato il graco di una funzione test v , a destra il graco della funzione x (v (x) v (0)/x, sovrapposto a quello della funzione dispari x v (0)/x.

Esercizi proposti
7.2-P1. Utilizzare lesercizio 7.2-5 per calcolare la distribuzione x (x). Soluzione. Sappiamo che x (x) = 0, dunque 0 = x (x) = (x) + x (x) = x (x) = (x).

Daltra parte un calcolo diretto fornisce, per ogni funzione test v , x (x), v (x) = (x), x v (x) = (x), x v (x) =

= (x), v (x) (x), x v (x) = (x), v (x) .


k (k ) (x) = xn /(n k)! per ogni 7.2-P2. Per la funzione f (x) := xn + /n!, vericare che f + (n+1) k n, mentre f (x) = (x).

Soluzione. La funzione f ha derivate continue no allordine n 1, essendo f (n1) (x) = x+ , funzione che, a sua volta, ha come derivata q.o. la funzione x x0 + , e questultima coincide q.o. col gradino unitario u(x). 7.2-P3. Consideriamo la famiglia di funzioni 1 h (x) := ([h/2,h/2]) (x), h che sappiamo tendere alla di Dirac per h 0. In precedenza (v. esercizio 6.2-5 e seguenti) e una abbiamo usato anche il simbolo ph (x) per la stessa funzione. Si verichi che, se f ` funzione polinomiale di primo grado, f (x) = ax + b, allora (h f )(x) = f (x), h > 0. h2 , 12

e una funzione polinomiale di secondo grado, Analogamente, se f (x) = ax2 + bx + c ` (h f )(x) = f (x) + a h > 0.

Soluzione. Per ogni funzione f sommabile si ha (h f )(x) = 1 h


x+h/2

f (t) dt,
xh/2

cio` e (h f )(x) ` e la media integrale di f sullintervallo [x h/2, x + h/2]. Se f ` e un polinomio di primo grado, si trova subito
x+h/2

(at + b) dt = a
xh/2

t2 2

x+h/2 xh/2

+ b = axh + bh = f (x) h.

Geometricamente: il sottograco di f , relativamente allintervallo [x h/2, x + h/2], ` e un trapezio di area f (x) h. Se poi f (x) ` e un polinomio di secondo grado, scriviamo f (x) = aq (x) + p1 (x) dove q (x) = x2 e p1 (x) ` e il polinomio di primo grado bx + c: Avremo che

c 88-08-07923-6

Esercizi

(h f )(x) = a(h q )(x) + (h p1 )(x) = a(h q )(x) + p1 (x), per quanto gi` a sappiamo sulla convoluzione di h con un polinomio di primo grado. In denitiva si tratta di vericare che (h q )(x) = x2 + Infatti 1 h
x+h/2

h2 . 12 h2 . 12

t2 dt =
xh/2

1 3 t 3h

x+h/2 xh/2

= x2 +

7.2-P4. Con gli stessi simboli dei precedenti esercizi, si dimostri che (h sin)(x) = Soluzione. Si ha
x+h/2

sin(h/2) sin x, h/2

(h cos)(x) =

sin(h/2) cos x. h/2

sin t dt = cos t
xh/2

x+h/2 xh/2

= cos(x h/2) cos(x + h/2) = 2 sin x sin(h/2),

in quanto le formule di addizione forniscono cos(x h/2) = cos x cos(h/2) sin x sin(h/2). Geometricamente: il graco della funzione (h sin)(x) ` e il graco della funzione seno, ridotto in ampiezza del fattore sin(h/2)/(h/2); si osservi che tale fattore tende crescendo a 1 per h 0. Calcolo analogo per la funzione coseno. 7.2-P5. Con gli stessi simboli del precedente esercizio, vericare che se f (x) = |x|, allora (h f )(x) =

|x|,
h

per |x| > h/2,

2 x + h , per |x| h/2.

e una funzione di classe C (1) (R). Vericare che (h f )(x) ` Soluzione. Per |x| > h/2 la restrizione di f allintervallo [x h/2, x + h/2] ` e un polinomio di primo grado, dunque il risultato segue dal precedente esercizio. Se poi |x| h/2, si ha
x+h/2 0 x+h/2

|t| dt =
xh/2 xh/2

t dt +
0

t dt = h 2
2

t2 2

0 xh/2

t2 2

x+h/2

=
0

1 2

x+

h 2

+ x+

= x2 +

h2 . 4

1
Figura 7.E-7. Graco della convoluzione (h f )(x) per h = 0.5.

-2

-1

Capitolo 7. Distribuzioni

c 88-08-07923-6

Per quanto riguarda la derivata della convoluzione calcolata, basta osservare che, per |x| < h/2, tale derivata vale 2x/h; essa tende a 1 per x 1 , e tende a 1 per x 1+ .

7.3. Distribuzioni temperate Esercizi proposti


7.3-P1. Dimostrare che se una successione di funzioni vk (x) D(R) converge a 0 nel senso della Denizione 7.1-2, cio` e nel senso proprio dello spazio D(R), essa converge a 0 anche nel senso dello spazio S (R). Soluzione. Nelle ipotesi ammesse tutti i supporti delle funzioni vk sono contenuti in un opportuno intervallo [r, r], dunque per ogni coppia di numeri naturali p e q si ha max |xp Dq vk (x)| = max |xp Dq vk (x)| rp vk
xR |x|r

0.

7.3-P2. Dimostrare che se una successione di funzioni vk (x) converge a 0 in S (R), lo stesso vale per la successione p(x)vk (x), quale che sia il polinomio p. Soluzione. Basta dimostrare laermazione nel caso in cui p ` e un monomio, diciamo xn con n naturale. Si tratta di dimostrare che, per ogni p, q N, la successione di funzioni k xp Dq [xn vk (x)] tende uniformemente a 0 in R. Ora si ha
q

xp Dq [xn vk (x)] = xp
h=0

q Dh xn Dqh vk (x) = h

min(q,n)

c(n, q, h) xp+nh Dqh vk (x),


h=0

e dove c(n, q, h) indica un coeciente dipendente da n, q e h. Infatti Dh xn vale 0 se h > n, ` un monomio di grado n h in caso contrario. Dunque
min(q,n)

xp Dq [xn vk (x)]

h=0

|c(n, q, h)| xp+nh Dqh vk (x)

0.

7.3-P3. Dimostrare direttamente, in base alla denizione di F -trasformata di una distribuzione temperata, che la trasformata di (x x0 ) ` e eix0 . Soluzione. Si ha F [ (xx0 )]( ), v ( ) = (xx0 ), v (x) = v (x0 ); poich e abbiamo indicato con la lettera x largomento della trasformata della funzione test v , abbiamo per tale trasformata unespressione del tipo R eix v ( ) d . Dunque (x x0 ), v (x) = R eix0 v ( ) d , quindi la trasformata richiesta coincide con eix0 . e una funzione limitata, e come tale ` e (pi` u esattamente: induce) Si osservi che eix0 ` una distribuzione temperata. 7.3-P4. Nota la trasformata di Fourier della funzione cos 2x, calcolare la trasformata delle funzioni sin2 x e cos2 x sfruttando le formule 1 cos 2x 1 + cos 2x sin2 x = , cos2 x = . 2 2 Alternativamente, nota la trasformata della funzione eikx , con k reale, ottenere le stesse trasformate utilizzando le formule di Eulero. Soluzione. La trasformata di cos 2x vale [ ( + 2) + ( 2)]; troveremo dunque per la trasformata di sin2 x 1 1 1 [2 ( ) ( + 2) ( 2)] = [ ( ) ( + 2) ( 2)]. 2 2 2 Alternativamente, da sin2 x = eix eix 2i
2

ei2x + ei2x 1 + 4 2

c 88-08-07923-6

Esercizi

segue lo stesso risultato, in quanto la trasformata di ei2x ` e 2 ( 2). 7.3-P5. Abbiamo dimostrato (v. pag. 285, esempio 7.3-6) che se una distribuzione temperata f ` e tale che xf (x) = 0, allora f (x) = c (x) con c costante. Dimostrare, analogamente, che se f ` e tale che x2 f (x) = 0 allora f (x) = c0 (x) + c1 (x), con c0 e c1 costanti opportune. In breve: la distribuzioni temperate per cui x2 f (x) = 0 sono tutte (e soltanto) quelle del tipo f (x) = c0 (x)+ c1 (x). Il risultato si estende per induzione al caso generale: xn f (x) = 0 se e solo se f ` e una combinazione lineare della e delle sua derivate no allordine n 1. Soluzione. Da x(xf (x)) = 0, segue xf (x) = c (x). F -trasformando si trova i f ( ) = c, cio` e f ( ) = c i, da cui f ( ) = c i + c0 = c1 i 1 + c0 1 = c1 ( ) + c0 ( ), avendo posto c1 = c, da cui nalmente segue il risultato anti-trasformando. 7.3-P6. Calcolare la trasformata di f (x) = [a,a] (x) a partire dal fatto che f (x) = = 0 + (x + a) (x a) = (x + a) (x a). Soluzione. Trasformando lultima uguaglianza si ottiene i f ( ) = eia eia . Le distribuzioni che risolvono lequazione scritta sono tutte (e soltanto) quelle del tipo eia eia + c ( ), i

e continua, si ha necesma poich e f ( ), in quanto trasformata di una funzione sommabile, ` sariamente c = 0, dunque f ( ) = 2 sin(a ) .

7.3-P7. Calcolare la trasformata della funzione f (x) = x [1,1] (x), a partire dal fatto che la sua derivata ` e esprimibile mediante la funzione [1,1] (x) e la delta di Dirac. Soluzione. Si trova f (x) = [1,1] (x) (x + 1) (x 1), distribuzione che ha come trasformata sin sin 2 ei ei = 2 cos . Ragionando come nel precedente esercizio si trova f ( ) = 2 sin cos . i 2

Si osservi che la trasformata ` e puramente immaginaria e dispari, in quanto f ` e reale dispari. Tale trasformata si annulla nellorigine in quanto sin cos = O( 3 ), per 0. Suggeriamo di ricalcolare direttamente la trasformata in esame come 2i
1 0

sin(x) x dx.

7.3-P8. Calcolare la F -trasformata della funzione f (x) := (1 |x|)+ (v. esempio 6.2-5) a partire dal fatto che f (x) = (x + 1) 2 (x) + (x 1). Soluzione. Si ha Df (x) = (1,0) (x) (0,1) (x), da cui f (x) = 0+ (x +1) 2 (x)+ (x 1). Trasformando lultima uguaglianza si ottiene 2 f ( ) = ei 2 + ei = 2(1 cos ). per quanto sappiamo dal precedente esercizio 7.3-P5, sono soluzione dellequazione scritta tutte (e soltanto) le distribuzioni 2 1 cos + c0 ( ) + c1 ( ); 2 sin2 (/2) 1 cos = 4 = 2 2
2

ma poich ef ` e sommabile, la sua trasformata ` e una funzione continua, dunque f ( ) = 2 sin(/2) /2 .

10

Capitolo 7. Distribuzioni

c 88-08-07923-6

7.3-P9. Calcolare la F -trasformata della funzione f (x) := (1 x2 )+ a partire dal fatto che f (x) = 2(1,1) (x) + 2 (x + 1) + 2 (x 1). Ritrovare lo stesso risultato mediante un calcolo diretto. Soluzione. Si osservi che si ha Df (x) = 2x (1,1) (x), da cui f (x) = 2(1,1) (x) + 2 (x + 1) + 2 (x 1).

1 1

1 -1 1 -1

Figura 7.E-8. A sinistra il graco della funzione f (x) := (1 x2 )+ , a destra il graco della derivata prima.

Trasformando lultima uguaglianza si ottiene 2 f ( ) = 4 sin sin + 2ei + 2ei = 4 + 4 cos .

Ragionando come nel precedente esercizio si trova f ( ) = 4 sin cos . 3

Si osservi che, poich e sin t = t t3 /6 + O(t5 ) mentre cos t = 1 t2 /2 + O(t4 ), il numeratore ha uno zero del terzo ordine nellorigine, al pari del denominatore. Poich` e la funzione data ` e reale e pari, la sua trasformata si riduce allintegrale
1

2
0

(1 x2 ) cos(x) dx,

che pu` o essere calcolato con due integrazioni per parti. 7.3-P10. Si vuole calcolare la F -trasformata della funzione f (x) := x [0,1] (x), che vale x per x [0, 1] e 0 altrimenti. Eseguire innanzitutto un calcolo in base alla denizione, poi ritrovare lo stesso risultato calcolando la derivata seconda di f (nel senso delle distribuzioni) e trasformando questultima. Soluzione. Si trova, con un calcolo diretto, f ( ) = ei (1 + i ) 1 2

La derivata di f ` e esprimibile mediante [0,1] e : f (x) = [0,1] (x) (x 1). Unulteriore derivazione fornisce f (x) = (x) (x 1) (x 1),

c 88-08-07923-6

Esercizi

11

da cui F [f ]( ) = 1 ei i ei . Poich ef` e sommabile si trova nalmente f ( ) dividendo per 2 , ottenendo nuovamente il risultato iniziale. Si osservi che da ei = 1 i 2 /2 + O( 3 ) segue ei (1 + i ) = 1 + 2 /2 + O( 3 ) quindi lim0 f ( ) = 1/2. 7.3-P11. Calcolare la F -trasformata della funzione cos x deducendola da quella della funzione sin x in base alla regola sulla trasformata della derivata. Suggerimento Sfruttare il fatto che ( 1) ( 1) = 0.

8.

Applicazioni Esercizi

[Aggiornamento: febbraio 2005]

http://newton.ciram.unibo.it/~barozzi/MI2/PDF/MI2-Cap.8-Ese.pdf

8.1.

Problemi ai limiti per equazioni differenziali omogenee

8.1-1. Dimostrare che lequazione dierenziale p0 (x) y (x)+ p1 (x) y (x)+ p2 (x) y (x) = = 0 pu` o essere posta nella forma autoaggiunta (1) moltiplicandola per unopportuna funzione = 0.
Suggerimento. Sia f (x) la funzione che funge da moltiplicatore; allora devessere

f (x) p0 (x) = f (x)p1 (x) . . . . Si ottiene unequazione dierenziale lineare del primo x ordine la cui soluzione ` e f (x) = exp x0 [p1 (t)/p0 (t)] dt /p0 (x).
Soluzione. Non ` e restrittivo suppporre p0 (x) > 0. Luguaglianza (f p0 ) = f p1 si

scrive anche p1 po f = f p0 equazione dierenziale la cui soluzione ` e f (x) = c exp p1 (t) p0 (t) dt = p0 (t) x0 x p1 (t) p0 (x) = c exp dt log p ( t ) p 0 (x0 ) x0 0 x c p0 (x0 ) p1 (t) = exp dt p0 (x) x0 p0 (t)
x

dq cui segue la soluzione indicata nel suggerimento scegliendo opportunamente la costante c. 8.1-2. Trovare autovalori ed autofunzioni del problema di valori ai limiti y + y = 0, y (0) = 0, y ( ) = 0.
Soluzione. Per > 0, poniamo = 2 =

; lequazione in esame ammette la soluzione generale y (x) = c1 cos( x) + c2 sin( x); dunque y (x) = c1 sin( x) + c2 cos( ) = 0. La condizione y (0) = 0 diventa c2 = 0; la condizione y ( ) = 0 si scrive dunque 2k + 1 cos( ) = 0 = + k = , k N. 2 2 Abbiamo dunque gli autovalori k = (2k + 1)2 /4, k N. Per = 0 lequazione in esame ammette la soluzione generale y (x) = c1 + c2 x, con derivata y (x) = c2 . La condizione y (0) = 0 implica c2 = 0; la condizione y ( ) =0 implica c1 = 0. Dunque 0 non ` e un autovalore.

Capitolo 8. Applicazioni

c 88-08-07923-6

Per < 0, poniamo = 2 = soluzione generale y (x) = c1 e x + c2 e x ; le condizioni ai limiti conducono al sistema c1 + c2 = 0 c1 e c2 e = 0

||; lequazione in esame ammette la

che ammette soltanto la soluzione nulla. Dunque non esistono autovalori negativi. 8.1-3. Trovare autovalori ed autofunzioni del problema di valori ai limiti y + y = 0, y (0) = 0, y ( ) = 0.
Soluzione. Per > 0, poniamo = 2 =

; lequazione in esame ammette la soluzione generale y (x) = c1 cos( x) + c2 sin( x); dunque y (x) = c1 sin( x) + c2 cos( ) = 0. La condizione y (0) = 0 diverta c2 = 0; la condizione y ( ) = 0 si scrive sin( ) = 0 =k = k 2 , k N .

Si hanno dunque gli autovalori k = k 2 , k N , con le corrispondenti autofunzioni yk (x) = cos(kx). Per = 0 lequazione in esame ammette la soluzione generale y (x) = c1 + c2 x, con derivata y (x) = c2 . Abbiamo dunque la condizione c2 = 0, e lautofunzione y0 (x) = 1. Per < 0, poniamo = 2 = ||; lequazione in esame ammette la soluzione generale y (x) = c1 e x + c2 e x ; le condizioni ai limiti conducono al sistema c1 c2 = 0 c1 e c2 e = 0

che ammette soltanto la soluzione nulla. Dunque non esistono autovalori negativi.

Esercizi proposti
8.1-P.1. Vericare che la successione sin kx, k N , ` e ortogonale nello spazio L2 [0, ]. Suggerimento Utilizzare lidentit` a sin sin = (1/2)[cos( + ) cos( )]. Alternativamente, sfruttare le formule di Eulero tenendo conto del fatto che, per n intero, si ha , se n = 0 se n = 2k , k N , einx dx = 0, 0 2i/n, se n = 2k + 1, k N.
Soluzione. Si tratta di calcolare, per h, k N , lintegrale

sin(hx) sin(kx) dx =
0

1 2

[cos((h + k )x) cos((h k )x)] dx.


0

Se h = k , una primitiva della funzione integranda ` e sin((h + k )x) sin((h k )x) , h+k hk da cui cui segue subito lannullamento dellintegrale in questione. Se poi h = k si ottiene

c 88-08-07923-6

Esercizi

1 [cos(2hx) 1] dx = . 2 2 0 0 Le analoghe relazioni di ortogonalit` a se h = k, 0, se h = k = 0, cos(hx) cos(kx) dx = , 0 /2, se h = k > 0 sin2 (hx) dx = sono state vericate nellesercizio 4.3-5.

8.2

Polinomi ortogonali

8.2-1. Mediante ripetuta integrazione per parti mostrare che, per ogni coppia di numeri naturali n e m, si ha
b a

(b x)m (x a)n (b a)m+n+1 dx = ; m! n! (m + n + 1)! 2 4 . . . 2n (2n)!! =2 . 1 3 . . . (2n + 1) (2n + 1)!!

in particolare (a = 1, b = 1, m = n):
1 1 m

(1 x2 )n dx = 2

Suggerimento

Una primitiva di (x a)n /n! ` e (x a)n+1 /(n + 1)!; la derivata di m1 (b x) /m! ` e (b x) /(m 1)!.
1 1 2 Pn (x) dx =

8.2-2. Per stabilire la formula (5 ) si tratta di calcolare 1 [(2n)!!]2


1 1

Dn (x2 1)n Dn (x2 1)n dx.

Con unintegrazione per parti mostrare che


1 1

Dn (x2 1)n Dn (x2 1)n dx =


1 1 1

= Dn1 (x2 1)n Dn (x2 1)n


1

Dn1 (x2 1)n Dn+1 (x2 1)n dx =

Dn1 (x2 1)n Dn+1 (x2 1)n dx.

Cos` proseguendo si trovi, dopo n integrazioni per parti,


1 1

Dn (x2 1)n Dn (x2 1)n dx =


1

= (1)n

Dnn (x2 1)n Dn+n (x2 1)n dx =


1 1

= (1)n (2n)! =2

(x2 1)n dx = (2n)!

(1 x2 )n dx =

(2n)! (2n)!! , (2n + 1)!!

Suggerimento

dove si ` e utilizzato il risultato del precedente esercizio. Poich e (x2 1)n = (x 1)n (x + 1)n ha uno zero di ordine n tanto in 1 quanto in 1, le derivate di tale polinomio di ordine inferiore a n si annullano in tali punti. Si sfrutti il fatto che la derivata di ordine 2n di (x2 1)n coincide con la derivata del termine direttivo x2n . 8.2-3. Combinare i risultati dei due precedenti esercizi per ottenere nuovamente la formula (4) relativa alla norma di Pn . 8.2-4. Si consideri nuovamente la rappresentazione integrale

Capitolo 8. Applicazioni

c 88-08-07923-6

Pn (x) :=

(s2 1)n 1 ds 2n 2i (x) (s x)n+1

che abbiamo utilizzato per dimostrare la Proposizione 8.2-3. Si verichi che scegliendo come circuito (x) la circonferenza di centro x e raggio 1 x2 (v. gura 8.2-6), essa si scrive 2 1 n Pn (x) := x + i 1 x2 sin d. 2 0 2 i Suggerimento Dalla parametrizzazione s := x + 1 x e , 0 2 , della circonferenza (x), segue ds = i 1 x2 ei d, s x = 1 x2 ei , e s2 1 = x2 + (1 x2 ) ei2 + 2x 1 x2 ei 1 = = (1 x2 ) ei2 + 2x 1 x2 ei (1 x2 ) = = 1 x2 ei 2x + 1 x2 (ei (ei ) = = 2 1 x2 ei x + i 1 x2 sin .
Soluzione. Basta proseguire i calcoli iniziati nel suggerimento, tenendo conto del

fatto che s x =

1 x2 ei .
1

x + i 1 x2 s x
1

x i 1 x2

Figura 8.2-6. Per ogni x [1, 1] il punto x + i 1 x2 sin appartiene al segmento di estremi x + i 1 x2 , x + i 1 x2 , contenuto nel disco di centro lorigine e raggio 1.

8.2-5. Dedurre dalla rappresentazione integrale ottenuta nel precedente esercizio la disuguaglianza |Pn (x)| 1 per ogni naturale n e per ogni x [1, 1]. Si riveda, in proposito, la gura 8.1-1. Suggerimento Vericare che |x + i 1 x2 sin | 1 per x [1, 1], calcolando il quadrato del valore assoluto in esame.
Soluzione. Si ha

|x + i 1 x2 sin |2 = x2 + (1 x2 ) sin2 x2 + 1 x2 = 1. 8.2-6. Dedurre dalla formula ricorsiva (18) che i coecienti dei polinomi di Ceby sev sono interi. Discorso analogo per i polinomi di Hermite, a partire dalla formula (25). 8.2-7. Dimostrare che il polinomio di Ceby sev Tn (x), per n 1, si annulla nei punti xk = cos (2k + 1) , 2n k = 0, 1, 2, . . . , n 1,

Soluzione. Si ha Tn (xk ) = cos(n arccos(xk )) = cos[(2k + 1) /2] = 0.

8.3.

Problemi ai limiti per equazioni differenziali non omogenee La funzione di Green

8.3-1. Siano y1 e y2 due soluzioni dellequazione dierenziale lineare omogenea (py ) + qy = 0, che si pu` o anche scrivere py = p y + qy . Dimostrare che per il relativo wronskiano

c 88-08-07923-6

8.3

Equazioni differenziali non omogenee

y1 (x) y2 (x) w(x) = y1 (x) y2 (x)


Suggerimento

si ha p(x) w(x) = costante, in quanto p(x) w(x) = 0. Si tratta di dimostrare che pw = p w. Ora y1 (x) y2 (x) w (x) = y1 (x) y2 (x) + y1 (x) y2 (x) y1 (x) y2 (x) = y1 (x) y2 (x) y1 (x) y2 (x) .

Si moltiplichino entrambi i membri per p, scrivendo py1 e py2 nella seconda riga dellultimo determinante, e si sfruttino le uguaglianze pyk = p yk + qyk , k = 1, 2.
Soluzione. Proseguendo secondo quanto suggerito si trova

y1 pw = py1

y2 = py2

y1 p y1 + qy1

y2 p y2 + qy2

= p

y1 y1

y2 y2

= p w.

Si tenga presente che un determinante con due righe uguali ` e nullo. 8.3-2. Dedurre dal precedente esercizio che, se p ha segno costante sullintervallo di denizione [a, b], ad esempio p(x) > 0, allora o il wronskiano w ` e identicamente nullo, oppure ` e anchesso di segno costante. La prima alternativa si verica se e solo se y1 e y2 sono linearmente dipendenti. 8.3-3. Si consideri il problema ai limiti dellesempio 8.3-1, cio` e y = f , y (0) = = y (1) = 0. Si scriva esplicitamente la formula risolutiva (8) e si verichi che, in corrispondenza dei termini noti f indicati nella tabella seguente, si hanno le soluzioni indicate a anco di ciascuno. f (x) y (x) f (x) y (x) 1 x2 (x x2 )/2 (x x4 )/12 x x3 (x x3 )/6 (x x5 )/20

Potete fare una congettura sulla soluzione corrispondente al termine noto f (x) = xn ?
Soluzione. Scegliendo come soluzioni dellequazione omogenea y1 (x) = x e y2 (x) =

1 x, il cui wronskiano vale 1, si ottiene la formula risolutiva


x 1

y (x) = (1 x)
0

f ( ) d + x
x

(1 ) f ( ) d.

Per f (x) = xn essa fornisce la soluzione x xn+2 . (n + 1)(n + 2) 8.3-4. Relativamente allo stesso problema ai limiti del precedente esercizio, calcolare le soluzioni corrispondenti ai termini noti f (x) = sin(x), f (x) = cos(x), f (x) = ex . In ciascun caso vericare la soluzione ottenuta.
Soluzione. Si trovano, nellordine, le soluzioni sin(x)/ 2 , (cos(x) + 2x 1)/ 2 ,

1 + (e 1) x ex .

8.3-5. Vericare che tutte le soluzioni calcolate nei due precedenti esercizi si presentano come somma di una soluzione dellequazione omogenea e di una soluzione dellequazione non omogenea. 8.3-6. Si consideri il problema ai limiti y = f , y (0) + y (0) = 0, y (1) + y (1) = 0. Dopo aver aver vericato che il corrispondente problema per lequazione omogenea

Capitolo 8. Applicazioni

c 88-08-07923-6

ammette soltanto la soluzione nulla, si costruisca la funzione di Green, si scriva la formula risolutiva (8) e si verichi che, in corrispondenza dei termini noti f indicati nella tabella seguente, si hanno le soluzioni indicate a anco di ciascuno. f (x) 1 x2 y (x) (x2 + 3x 3)/2 (x4 + 5x 5)/12 f (x) x x3 y (x) (x3 + 4x 4)/6 (x5 + 6x 6)/20

Potete fare una congettura sulla soluzione corrispondente al termine noto f (x) = xn ?
Soluzione. La soluzione generale dellequazione omogenea y

= 0 si scrive y (x) = c1 + c2 x, quindi le condizioni ai limiti diventano c1 + c2 = 0, c1 + 2c2 = 0, sistema che ammette soltanto la soluzione nulla. Per soddisfare separatamente le condizioni ai limiti possiamo scegliere y1 (x) = 1 x (cio` e c1 = 1, c2 = 1) e y2 (x) = 2 x (cio` e c1 = 2, c2 = 1). Poich e il wronskiano vale 1, la formula risolutiva si scrive
x 1

y (x) = (x 2)
0

(1 ) f ( ) d + (x 1)
x

(2 ) f ( ) d.

Relativamente al termine noto f (x) = xn essa fornisce la soluzione y (x) = xn+2 + (n + 3)x (n + 3) . (n + 1)(n + 2)

8.4.

Lequazione del calore

8.4-1. Vericare che le soluzioni dellequazione del calore che sono indipendenti dal tempo sono tutte (e soltanto) quello del tipo u(x, t) = ax + b.
Soluzione. Se u ` e indipendente da t, u(x, t) = u(x), lequazione del calore diventa

uxx = 0. 8.4-2. Vericare che i polinomi del tipo u(x, t) = ax2 + bx + c + 2at (in particolare i polinomi di grado 1 nella sola x) sono soluzioni dellequazione del calore ut = uxx .
Soluzione. si trova ut = uxx = 2a.

8.4-3. Sotto quale condizione sui coecienti e le due funzioni et cos(x) e et sin(x) sono soluzioni dellequazione del calore ut = uxx ?
Soluzione. Sotto la condizione = 2 .

8.4-4. Vericare che la funzione 1 x2 u(x, t) = exp 4t t soddisfa lequazione del calore ut = uxx per t > 0.
Soluzione. Si trova

ut (x, y ) =

x2 1 3/2 1 5/2 2 + t x exp t ; 2 4 4t

1 x2 ux (x, y ) = t3/2 x exp , 2 4t uxx (x, y ) = x2 1 3/2 1 5/2 2 + t x exp t . 2 4 4t

8.4-5. Vericare che la funzione

c 88-08-07923-6

8.4

Lequazione del calore

1 x2 + y 2 exp t 4t soddisfa lequazione del calore ut = uxx + uyy per t > 0. u(x, y, t) =
Soluzione. Si trova

ut (x, y ) = t2 +

1 3 2 x2 + y 2 t (x + y 2 ) exp ; 4 4t

1 x2 + y 2 ux (x, y ) = t2 x exp , 2 4t uxx (x, y ) = e analogamente uyy (x, y ) = x2 + y 2 1 2 1 3 2 t + t y exp . 2 4 4t x2 + y 2 1 2 1 3 2 t + t x exp , 2 4 4t

8.4-6. Applicare il metodo di separazione delle variabili alla soluzione del problema ut = uxx , x [0, L], t > 0, u(x, 0) = u0 (x), ux (0, t) = ux (L, t) = 0, corrispondente al caso di una sbarra conduttrice i cui estremi sono isolati.

4 3 2 1 0 2 1.5 1 0.5 0

Figura 8.4-5. Soluzione approssimata del problema posto nellesercizio 8.4-6, con i dati L = 2, u0 (x) = 4x x3 .

Vericare che si ottiene la soluzione sotto la forma a0 n n u(x, t) = an cos + x exp 2 L L n>0 dove an = (2/L)
L 0

t ,

u0 (t) cos(n/L t) dt. Quanto vale il limite limt u(x, t)?

Soluzione. Procediamo in modo analogo a quanto fatto nel testo a pagine 320 321.

Posto u(x, y ) = X (x)T (t), sostituendo nellequazione uxx = ut si perviene alle due equazioni dierenziali X = X, T = kT. Le condizioni ai limiti per la prima equazione, cio` e X (0) = X (L) = 0, possono essere e la soluzione generale soddisfatte solo per k 0, dunque k = 2 , con 0. Poich dellequazione X + 2 X = 0 si scrive X (x) = c1 cos(x) + c2 sin(x), da cui X (x) = c1 sin(x) + c2 cos(x),

Capitolo 8. Applicazioni

c 88-08-07923-6

ponendo X (0) = 0 si ottiene c2 = 0, dunque X (x) = c1 cos(x). La condizione X (L) = 0, cio e sin(L) = 0, implica che i valori accettabili per sono n , n = 0, 1, 2, . . . . n = L Possiamo dunque scrivere una soluzione dellequazione del calore che verica le condizioni ux (0, t) = ux (L, t) = 0 nella forma u(x, t) = a0 + 2 an cos
n1

n n x exp L L

t .

Per soddisfare la condizione iniziale u(x, 0) = u0 (x) possiamo prolungare per parit` a la funzione u0 sullintervallo [L, L]: x [0, L] : u0 (x) := u0 (x), dopodich e la condizione iniziale si scrive a0 n u(x, 0) = an cos + x = u0 (x); 2 L
n1

essa viene soddisfatta scegliendo come an i coecienti di Fourier di u0 . Per t + la funzione u(x, t) tende ad a0 /2.

8.6.

Il metodo delle serie di potenze


(1 x2 ) y xy + y = 0,

8.6-1. I polinomi di Ceby sev sono autofunzioni dellequazione dierenziale e pi` u precisamente Tn corrisponde allautovalore n = n2 . Procedendo come nellesempio 8.6-5, si utilizzi il metodo delle serie di potenze per dedurre la formula ak+2 = n2 k 2 ak (k + 1)(k + 2)

per il calcolo dei coecienti di Tn . Sapendo che il termine direttivo di Tn vale 2n1 , ponendo k = n 2h si ottenga il seguente schema an := 2n1 , an2h := (n 2h + 1)(n 2h + 2) an2(h1) , 4h (n h) h = 1, 2, . . . , n/2 .

8.6-2. I polinomi di Laguerre sono autofunzioni dellequazione dierenziale x y + (1 x) y + y = 0, e pi` u precisamente Ln corrisponde allautovalore n = n. Utilizzando il metodo delle serie potenze si deduca la formula ak+1 = nk ak , (k + 1)2

che, unita alla condizione iniziale a0 = 1, consente il calcolo ricorsivo dei coecienti di Ln . 8.6-3. I polinomi di Hermite sono autofunzioni dellequazione dierenziale y 2x y + y = 0, e pi` u precisamente Hn corrisponde allautovalore n = 2n. Procedendo come nei precedenti esercizi, si ottenga la formula ak+2 = 2(n k ) ak . (k + 1)(k + 2)

c 88-08-07923-6

8.4

Lequazione del calore

Si ponga poi k = n 2h, e si scriva uno schema ricorsivo che consenta il calcolo dei coecienti del polinomio Hn a partire dal coeciente direttivo dn = 2n .

Esercizi proposti
8.6-P.1. Si chiama equazione ipergeometrica (v. Appendice 8-E) lequazione dierenziale x(1 x) y + [c (a + b + 1) x] y ab y = 0, dove a, b e c sono parametri reali. Dimostrare che il metodo delle serie di potenze conduce alla formula ricorsiva (a + k )(b + k ) ak+1 = ak , (1 + k )(c + k ) e pertanto, se si pone a0 = 1, alla serie ipergeometrica 1+
k1

a(a + 1) . . . (a + k 1) b(b + 1) . . . (b + k 1) k x . k ! c(c + 1) . . . (c + k 1)

Se si introduce il simbolo (x)k := x(x + 1)(x + 2) . . . (x + k 1), x R, k N , con la convenzione (x)0 := 1, allora si pu` o scrivere la serie ipergeometrica nella forma F (a, b, c; x) =
k0

(a)k (b)k k x . k ! (c)k

Si dimostri che il raggio di convergenza della serie scritta ` e 1. Si osservi che k ! = (1)k .
Soluzione. Posto y (x) =

trova x(1 x)
k2

k0

ak xk , sostituendo nellequazione ipergeometrica si k ak xk1 ab


k1 k0

k (k 1) ak xk2 + [c (a + b + 1) x]

ak xk = 0

uguaglianza che pu` o essere riscritta successivamente


k2

k (k 1) ak xk +
k2

k (k 1) ak xk1 + kak xk1 + (a + b + 1)


k1 k1

+c
k0

kak xk ab
k0

ak xk = 0,

k (k 1) ak xk +
k0 k

k (k + 1) ak+1 xk + k ak xk ab
k0 k0

+c
k1

(k + 1) ak+1 x + (a + b + 1)

ak xk = 0.

Uguagliando a 0 il coeciente di xk si trova ak+1 (k + 1)(c + k ) = ak [k (k 1) + k (a + b + 1) + ab] da cui segue la formula ricorsiva indicata. Per calcolare il raggio di convergenza della serie ipergeometrica possiamo calcolare il limite ak lim k ak+1 a patto che esso esista. Si rivedano le pagine 136 e 137 del testo. Ma il limite del rapporto indicato esiste e vale 1 in quanto esso si scrive (1 + k )(c + k ) . (a + k )(b + k ) 8.6-P.2. Vericare le identit` a:

10

Capitolo 8. Applicazioni

c 88-08-07923-6

F (1, 1, 1; x) = 1 + x + . . . + xn + . . . =

1 , 1x

F (n, 1, 1; x) = (1 x)n .

Soluzione. Per a = b = c = 1 il coeciente di xk vale

(1)k (1)k = 1. (1)k (1)k Per a = n, con n naturale e b = c = 1, il coeciente di xk vale (n)k n(n + 1) . . . (n + k 1) = = k! k! n n(n 1)(n 2) . . . (n k + 1) = (1)k . = (1)k k k! 8.6-P.3. Calcolare la serie di Taylor della funzione f (x) = arctan x: f (x) = arctan x =
k0

ak xk

sapendo che a0 = 0 e si ha 1 f (x) = 1 + x2

(1 + x2 ) f (x) = 1.
k1

Soluzione. Si ha innanzitutto a0 = f (0) = 0. Si ha poi f (x) =

k ak xk1 .

Sostituendo nellultima equazione scritta si ha, in un primo tempo, k ak xk1 +


k1 k1

k ak xk+1 = 1,

uguaglianza che si pu` o riscrivere (k + 1) ak+1 xk +


k0 k2

(k 1) ak1 xk = 1.

Abbiamo dunque, per k = 0, a1 = 1, per k = 1, a2 = 0 e, per k 2, la formula ricorsiva k1 ak+1 = ak1 . k+1 Dunque tutti i coecienti ak di indice pari sono nulli (in accordo col fatto che la funzion arcotangente ` e dispari), mentre si ha 1 1 3 1 a3 = a1 = , a5 = a3 = , . . . ; 3 3 5 5 e in generale a2k+1 = (1)k /(2k + 1), cio` arctan x = x x3 x5 x7 + + ... . 3 5 7

8.6-P.4. Calcolare la serie di Taylor della funzione f (x) = ln(1 + x): f (x) = ln(1 + x) =
k0

ak xk

sapendo che a0 = 0 e si ha 1 f (x) = 1+x

(1 + x) f (x) = 1.
k1

Soluzione. Si ha innanzitutto a0 = f (0) = 0. Si ha poi f (x) =

k ak xk1 = k k0 (k + 1) ak+1 x . Sostituendo nellultima equazione scritta si ha dunque [(k + 1) ak+1 + k ak ] xk = 1,


k0

c 88-08-07923-6

8.4

Lequazione del calore

11

da cui segue, per k = 0, a1 = 1, e, per k > 0, la formula ricorsiva k ak+1 = ak . k+1 Dunque 1 1 1 1 2 3 1 a2 = , a3 = = , a4 = = , . . . ; 2 3 2 3 4 3 4 k+1 in generale ak = (1) /k , cio` e ln(1 + x) = x x2 x3 x4 + + ... . 2 3 4

8.6-P.5. Calcolare la serie di Taylor della funzione f (x) = ln[(1 + x)/(1 x)]: 1+x f (x) = ln = ak xk 1x
k 0

sapendo che a0 = 0 e si ha 2 f (x) = (1 x)2


Soluzione. Posto f (x) =
k1

(1 x2 ) f (x) = 2. ak xk , si trova

(1 x2 )
k1

kak xk1 = 2,

uguaglianza che si riscrive successivamente kak xk1


k1 k1

kak xk+1 = 2, (k 1) ak1 xk = 2.


k2

(k + 1) ak+1 xk
k0

Uguagliando a primo e a secondo membro i termini di grado 0 e 1 si ottiene a1 = 2, 2a2 = 0 = a2 = 0; per ogni k 2, uguagliando i termini di grado k si ottiene k1 k + 1) ak+1 = (k 1) ak1 = ak+1 = ak1 . k+1 Ne segue che tutti i coecienti di indice pari sono nulli, mentre si ha 1 2 3 2 a3 = a1 = , a5 = a3 = , 3 3 5 5 e, in generale, a2k+1 = 2/(2k + 1). In conclusione f (x) = ln 1+x =2 1x x2k+1 . 2k + 1

k0

8.6-P.6. Con riferimento ai precedenti esercizi, vericare che 1+x ln(1 + x) = x F (1, 1, 2; x), ln = 2x F (1/2, 1, 3/2; x2 ). 1x arctan x = x F (1/2, 1, 3/2; x2 ), arcsin x = x F (1/2, 1/2, 3/2; x2 ). k! (1)k xk+1 = (k + 1)!

Soluzione. Occupiamoci della prima uguaglianza. Si ha

x F (1, 1, 2; x) = x
k0

(1)k (1)k (1)k xk = (1)k (2)k


k+1 2

k0

=
k0

(1)k

x x x3 =x + + ... . k+1 2 3

12

Capitolo 8. Applicazioni

c 88-08-07923-6

8.6-P.7. Risolvere per serie il problema di valori iniziali (1 + x2 )y = 2xy , y (0) = 1, vericando che la soluzione ` e un polinomio.
Soluzione. Posto y (x) =

ak xk , si trova y (x) = tuendo nellequazione considerata


k0

k1

kak xk1 , quindi sosti-

kak xk1 +
k1 k 1

kak xk+1 = 2
k0

ak xk+1 ,

uguaglianza che si pu` o scrivere in forma equivalente (k + 1)ak+1 xk +


k0 k2

(k 1)ak1 xk 2
k1

ak1 xk = ()

= a1 + (2a2 2a0 ) x +
k2

[(k + 1)ak+1 (k 3) ak1 ] xk = 0.

Si ha innanzitutto a0 = y (0) = 1; uguagliando a 0 i coecienti dei termini di grado 0 e 1 nella serie a primo membro di (*) si ha a1 = 0, 2a2 2a0 = 0 = a2 = 1, mentre uguagliando a 0 il coeciente di xk , per k 2, si ha la formula ricorsiva k3 ak1 . k+1 Da essa segue che tutti i coecienti di indice dispari sono nulli, e poich e (ponendo k = 3) si ha a4 = 0, sono nulli anche tutti i coecienti di ordine pari a partire da a4 stesso. Ne segue che la soluzione ` e y (x) = 1 + x2 , la cui correttezza ` e subito vericata. ak+1 = 8.6-P.8. Generalizzare il precedente esercizio risolvendo per serie il problema di valori iniziali (1 + x2 )y = 2 p x y , y (0) = 1, con p reale, vericando che a0 = 1, i coecienti ak sono nulli per k dispari e, per ogni k > 0, a2k = p(p 1) . . . (p k + 1) = k! p , k

quindi, ricordando la serie binomiale, y (x) = (1 + x2 )p . 8.6-P.9. Risolvere per serie il problema di valori iniziali y + xy + y = 0, y (0) = 1, y (0) = 0, vericando che la soluzione ` e esprimibile mediante la funzione esponenziale: y (x) = e(x) , con funzione opportuna..
Soluzione. Posto y (x) =

ricorsiva an+2 =

n0

an xn , si trova a0 = 1, a1 = 0, e, per n 0, la formula

an . n+2 Dunque per i coecienti di indice dispari si ha a2n+1 = 0 e per quelli di indice pari a2n = 1 1 = n . (2n)!! 2 n! 1 x2 n! 2
n

Ne segue che la soluzione si scrive y (x) =


n0

confrontando con la serie esponenziale si riconosce che y (x) = exp(x2 /2).

8.7.

Le funzioni di Bessel

8.7-P.1. Risolvere per serie lequazione dierenziale xy + y + y = 0, x R. Mostrare che la soluzione ottenuta con la condizione iniziale y (0) = 1 ` e esprimibile, per x 0, mediante la funzione di Bessel J0 nella forma J0 ((x)), con funzione opportuna.
Soluzione. Posto y (x) =
k0

ak xk ,si trova

c 88-08-07923-6

8.7

Le funzioni di Bessel

13

k (k 1) ak xk1 +
k2 k1

kak xk1 +
k0

ak xk = 0.

Nella prima somma lindice pu` o partire da 1, quindi le prime due somme si compattano nellunica somma k1 k 2 ak xk1 che pu` o riscriversi k0 (k + 1)2 ak+1 xk . In conclusione abbiamo luguaglianza [(k + 1)2 ak+1 + ak ] xk = 0,
k0

da cui la formula ricorsiva ak ak+1 = (k + 1)2 Da a0 = 1, segue dunque 1 a1 = 1, a2 = 2 , 2 (1)k k x . (k !)2 (1)k (k !)2
2k

a3 =

1 , (2 3)2

in generale ak = (1)k /(k !)2 , quindi y (x) =


k0

Ora J0 ((x)) =
k0

(x) 2

2 espressione e si scelga che coincide con la precedente a patto che sia (x) /4 = x, cio` (x) = 2 x.

Operatori differenziali nel piano e nello spazio Esercizi proposti


1. Vericare che la funzione u(x, y ) = log(x2 + y 2 ) ` e soluzione dellequazione di Laplace 2 u = uxx + uyy = 0, in tutti i punti del piano distinti dallorigine, cio` e per x2 + y 2 > 0.
Soluzione. Infatti

ux =

x , x2 + y 2

uy =

y , x2 + y 2 uyy = x2 y 2 . (x2 + y 2 )2

uxx =

x2 + y 2 2x2 y 2 x2 = , (x2 + y 2 )2 (x2 + y 2 )2

2. Vericare che la funzione u(x, y, z ) = 1/ x2 + y 2 + z 2 , ` e soluzione dellequazione di Laplace 2 u = uxx + uyy + uzz = 0, in tutti i punti dello spazio distinti dallorigine, cio` e per x2 + y 2 + z 2 > 0.
Soluzione. Se P ` e il punto di coordinate (x, y, z ), introduciamo la funzione r(x, y, z ) =

x2 + y 2 + z 2 ; essa ` e semplicemente la norma euclidea del vettore di posizione OP : r(x, y, z ) = OP


2

Si ha facilmente: x y z r x = , r y = , rz = . r r r Allora per la funzione u = 1/r si trova ux = rx x = 3, r2 r uxx = 1 3xrx 1 3x2 + = + , r3 r4 r3 r5

14

Capitolo 8. Applicazioni

c 88-08-07923-6

e analogamente uyy = In denitiva: 2 u = 3 3(x2 + y 2 + z + 2) 3 3r2 + = 3 + 5 = 0. 3 5 r r r r 1 3y 2 + 5 , 3 r r uzz = 1 3z 2 + 5 . 3 r r

3. Si consideri la famiglia di funzioni, da R3 a C, u(x, y, z ) := eax+by+cz , con a, b e c costanti complesse non tutte nulle. Si chiede quale relazione debbano soddisfare tali costanti anch e u sia soluzione dellequazione 2 u = uxx + uyy + uzz = 0.
Soluzione. Si trova subito 2 u = (a2 +b2 +c2 ) u, quindi la condizione ` e a2 +b2 +c2 = 0.

Esistono innite terne di valori che soddisfano tale condizione: ad esempio si possono scegliere a e b reali ad arbitrio, e ricavare c = i a2 + b2 . Le scelte a = 3, b = 4, c = 5i sono ammissibili, fornendo la funzione u(x, y, z ) = e3x+4y+iz . Scegliendo parte reale e parte immaginaria di u si ottengono le due funzioni armoniche reali u1 (x, y, z ) = e3x+4y cos(5z ), u2 (x, y, z ) = e3x+4y sin(5z )

You might also like